Tổng hợp 20 đề đề xuất kì thi duyên hải và đồng bằng bắc bộ môn Hóa học khối 11 năm 2017 có đáp án

Page 1

ĐỀ THI HSG DUYÊN HẢI BẮC BỘ MÔN HÓA HỌC

vectorstock.com/3687784

Ths Nguyễn Thanh Tú eBook Collection DẠY KÈM QUY NHƠN OLYMPIAD PHÁT TRIỂN NỘI DUNG

Tổng hợp 20 đề đề xuất kì thi duyên hải và đồng bằng bắc bộ môn Hóa học khối 11 năm 2017 có đáp án WORD VERSION | 2021 EDITION ORDER NOW / CHUYỂN GIAO QUA EMAIL TAILIEUCHUANTHAMKHAO@GMAIL.COM

Tài liệu chuẩn tham khảo Phát triển kênh bởi Ths Nguyễn Thanh Tú Đơn vị tài trợ / phát hành / chia sẻ học thuật : Nguyen Thanh Tu Group Hỗ trợ trực tuyến Fb www.facebook.com/DayKemQuyNhon Mobi/Zalo 0905779594


ĐỀ THI MÔN HÓA HỌC LỚP 11 Thời gian làm bài:180 phút (Đề thi gồm có 04 trang)

HỘI CÁC TRƯỜNG THPT CHUYÊN KHU VỰC DUYÊN HÀI VÀ ĐỒNG BẰNG BẮC BỘ TRƯỜNG THPT CHUYÊN BIÊN HÒA, T. HÀ NAM

ĐỀ THI ĐỀ XUẤT Câu 1: Tốc độ phản ứng – Cân bằng hóa học 1. Phân hủy nhiệt của 1,3-diphenylpropane tạo ra các sản phẩm chủ yếu là toluene và styrene cùng với các sản phảm phụ ethylbenzene và những hydrocarbon khác. Người ta đề nghị cơ chế sau đây cho phản ứng này.: k1

(1) (chậm)

PhCH 2CH 2CH 2 Ph → PhCH 2 * + PhCH 2CH 2 * S

1

2

k2

PhCH 2CH 2 * + PhCH 2CH 2CH 2 Ph → PhCH 2CH 3 + Ph CH CH 2CH 2 Ph

(2)

3

k3

PhCH 2 * + PhCH 2CH 2CH 2 Ph → PhCH3 + Ph CH CH 2CH 2Ph

(3)

P1

k4

Ph CHCH 2CH 2 Ph → PhCH=CH 2 + PhCH 2 *

(4)

P2

a. Áp dụng sự gần đúng nồng độ dừng đối với gốc 2 hãy rút ra định luật tốc độ cho phản ứng tạo thành ethylbenzene. b. Tìm tỉ số các nồng độ dừng của các gốc 1 và 3? Ngoài ra, hai gốc tự do có thể kết hợp với nhau với hằng số tốc độ kR được xem là không đổi với các gốc khác nhau:. kR

R1* + R 2 * → R1R 2 c. Vì sao trong các phương trình nồng độ dừng của 1 và 2 có thể bỏ qua tốc độ của các phản ứng này? d. Tìm định luật tốc độ của sự hình thành toluene, xác định bậc phản ứng. Biểu diễn năng lượng hoạt động hóa của phản ứng chung qua các năng lượng hoạt động hóa của các phản ứng sơ cấp. 2. Iot là một nguyên tố vi lượng quan trọng trong cuộc sống và là nguyên tố nặng nhất mà các cơ thể sống cần được cung cấp mỗi ngày. Ở nhiệt độ cao cân bằng giữa I2(k) và I(k) được thiết lập. Bảng sau ghi lại áp suất đầu của I2(k) và áp suất chung khi hệ đạt đến cân bằng ở nhiệt độ khảo sát. T (K) 1073 1173 P(I2) (atm) 0.0631 0.0684 Pchung (atm) 0.0750 0.0918 a. Tính ∆H°, ∆G°và ∆S°ở 1100 K. (Cho rằng ∆H° và ∆S° đều không phụ thuộc nhiệt độ trong khoảng nhiệt độ khảo sát.)


b. Tính phần mol của I(k) trong hỗn hợp cân bằng trong trường hợp trị số Kp lúc này bằng một nửa áp suất chung. c. Biết I2(k) và I(k) đều là khí lý tưởng. Tính năng lượng phân ly liên kết của I2 ở 298 K. Câu 2: Cân bằng trong dung dịch điện li Dung dịch X gồm Na2S 0,010M, KI 0,060M, Na2SO4 0,050M. 1. Tính pH của dung dịch X. 2. Thêm dần Pb(NO3)2 vào dung dịch X cho đến nồng độ 0,090M thì thu được kết tủa A và dung dịch B. a) Cho biết thành phần hoá học của kết tủa A và dung dịch B. b) Tính nồng độ các ion trong dung dịch B (không kể sự thuỷ phân của các ion, coi thể tích dung dịch không thay đổi khi thêm Pb(NO3)2). Câu 3: Điện hóa học Cho các giá trị thế khử Eo(Ag+/Ag) = 0,80 V; Eo(AgI/Ag) = –0,15 V; Eo(Au3+/Au+) = 1,26 V; Eo(Fe2+/Fe) = –0,037 V; Eo(Fe3+/Fe) = 0,44 V. 1. a. Lập sơ đồ pin xác định tích số tan của AgI. Viết phương trình mỗi điện cực và phản ứng xảy ra trong pin. b. Tính độ tan (S) của AgI trong nước ở 25oC. 2. Thiết lập pin có sự oxi hóa Fe2+ thành Fe3+, khử Au3+ về Au+. Tính Kcb và ∆Eopin. Câu 4: Nitơ – Photpho, Cacbon – Silic và hợp chất 1. a. Nguyên tố phi kim X phản ứng với Cl2 cho chất lỏng không màu A (tonóng chảy = -94 oC). Chất A tác dụng với Cl2 dư trong dung môi CCl4 khan cho B (chất rắn màu trắng, tothăng hoa = 160 oC). Cho biết khối lượng phân tử của B bằng 1,516 lần khối lượng phân tử của A. Hãy xác định A, B và vẽ cấu trúc phân tử của chúng. b. Một hợp chất dị vòng của X với cấu trúc phẳng do J. Liebig và F. Wohler tổng hợp từ năm 1834 được tạo thành từ NH4Cl với một chất pentacloro của X có công thức phân tử là (NXCl2)3, sản phẩm phụ của phản ứng này là một khí dễ tan trong nước và phản ứng như một axit mạnh. Hợp chất vô cơ này có tính chất khác thường khi bị đun nóng: nó sôi ở 256oC khi bị đun nóng nhanh, nếu đun nóng chậm nó bắt đầu nóng chảy ở 250oC, làm nguội nhanh chất lỏng này ta được một chất tương tự cao su. Hãy viết phương trình phản ứng, xác định cấu tạo của chất (NXCl2)3 và giải thích tính chất đặc biệt này 2. Năm 1851, S. Cannizzarro và F. Close đã tổng hợp thành công một chất mới B. Cuối thế kỉ 19, một phương pháp tổng hợp B trong công nghiệp được ứng dụng và phát triển. Các chất D và F được sử dụng như nguyên liệu đầu:

Chất Tonc (oC) Tos (oC) Tỉ khối Dung môi hòa tan A -6 12,7 1,1860 CCl4, C6H6... B 44 140 1,0729 H2O, C2H5OH, (C2H5)2O... C 132,7 1,3350 H2O, C2H5OH Xác định A đến F, viết phương trình phản ứng Câu 5: Phức chất, trắc quang 1. Ion phức bis(terpyridyl)coban(II) tồn tại một phần ở trạng thái spin cao, một phần ở trạng thái spin thấp phụ thuộc vào các ion liên kết trực tiếp với nguyên tử trung tâm ClO4-/Cl-/NCS-/Br-. a. Cho biết ba dạng hình học có thể có của phức b. Dựa vào thuyết trường phối tử hãy vẽ giản đồ obitan cho các trường hợp phức spin cao và thấp c. Tính momen từ (M.B) của các phức trên


d. Xét các phức sau: [Co(CN)6]3-, [Co(CO3)2(NH3)2]-, [Co(CO3)3]3- and [Co(NO2)6]3-. Màu của các ion phức này sẽ là: xanh, vàng, cam và da trời (không nhất thiết là phải ở cùng dạng với các phức trên). Hãy cho biết tên của từng phức và xác định màu của chúng: 2. Hệ số hấp thụ phân tử gam của anilin (C6H5NH2) ở bước sóng 280 nm là 1,48.103 mol.l-1.cm-1. Dạng proton hóa của anilin C6H5NH3+ không hấp thụ ở bước sóng này. Độ truyền quang của anilin 2,10-4 M trong cuvet l =20 mm cũng ở bước sóng này là 0,92. Tính pH của dung dịch chứa C6H5NH3+ Cho biết: Ka (C6H5NH3+) = 10-4,8 Câu 6: (2 điểm) Quan hệ giữa hiệu ứng cấu trúc và tính chất Cho các công thức cấu tạo sau: C(CH CHF)2 HC CH2F A

Me Et

1 2 O

3 O B

4 5

Me

Me

OH

O Me O2N

O C

Me

O

O2N

OH D

COOH

N N H

NH2 E

1. Hãy viết công thức các đồng phân lập thể ứng với cấu tạo A 2. Ứng với công thức cấu tạo B có bao nhiêu đồng phân lập thể, vì sao? Dùng các kí hiệu thích hợp để chỉ rõ cấu hình mỗi đồng phân đó 3. Hãy viết cơ chế phản ứng để giải thích vì sao C và D khi tương tác với dung dịch NaOH thì đều tạo natri 3-metyl-4-nitrobenzoat 4. Chỉ rõ trạng thái lai hóa của từng nguyên tử N trong cấu tạo E và ghi giá trị pKa (ở 25oC): 1,8; 6,0; 9,2 vào từng trung tâm axit trong công thức tương ứng với E và giải thích Câu 7: (2 điểm) Hiđrocacbon Một số hiđrocacbon X có công thức phân tử C10H16 và có các tính chất sau: tác dụng với H2 (dư)/Ni ở 120oC cho C10H22; tác dụng với Br2/CCl4 cho C10H16Br6; 1 mol X tác dụng với ozon rồi thủy phân khử (nhờ Zn/HCl) hoặc thủy phân oxi hóa (nhờ H2O2) đều cho 2 mol một sản phẩm hữu cơ duy nhất Y có công thức phân tử là C5H8O; không thể tách X và Y thành các đối quang được. Hãy xác định công thức lập thể có thể có của X và viết phương trình hóa học xảy ra với một trong số các công thức tìm được của X Câu 8: (2 điểm) Xác định cấu trúc + đồng phân lập thể + danh pháp Hợp chất A có công thức phân tử CHON, ở thể khí độ dài liên kết CN bằng 121 pm, CO bằng 117 pm. A tan trong nước tạo thành dung dịch axit với Ka = 1,2.10-4. Trong dung dịch đặc A tự biến đổi thành X có vòng 6 cạnh với độ dài liên kết CN bằng 135 pm. Cho biết độ dài liên kết trung bình ở các hợp chất như sau Liên kết C-C C-N C-O C=N C=O C≡N Cacbon oxit Độ dài, pm 154 147 143 130 123 116 112 1. Xác định công thức cấu tạo của A ở thể khí 2. Dùng công thức cấu tạo viết phương trình phản ứng điện li của A trong dung dịch nước và giải thích vì sao lực axit của nó lớn hơn của axit axetic 3. Viết phương trình phản ứng tạo thành và công thức cấu tạo của X Câu 9: (2 điểm) Cơ chế phản ứng Đề xuất cơ chế để giải thích quá trình tạo ra sản phẩm của các phản ứng sau 1.

2.


3.

4.

Câu 10: (2 điểm) Tổng hợp các hợp chất hữu cơ Erythronolit B là chất đầu của quá trình sinh tổng hợp của chất kháng sinh erythromixin. Để tổng hợp erythronolit B người ta tiến hành phân tích tổng hợp lùi và thấy rằng cần xuất phát từ tiền chất A’. Tiền chất A’ có công thức

Rất lý thú là A’ được tổng hợp từ nguyên liệu đớn giản là 2,4,6-trimetylphenol theo sơ đồ sau: OH

NaOMe, PhMe

H2O2, NaOH,

Br

F

Br2, KBr H2O

1. LiOH, H2O 2. CrO3, H2SO4 axeton

BH3,THF,0oC

A

G

N

n-Bu3SnH, AIBN toluen,

75oC

MeCOOOH EtOAc, 55oC

O

H

CrO3, H2SO4

B

0oC

axeton

Al/Hg, THF/H2O

I

o

0C N

S

S

Ph3P, THF

N

C

H2, Ni-Ra o

DMF, -20 C

A'

Br2, KBr

D

BzCl Py

[E]

THF

H2O

K

KOH, H2O

L

LDA, THF, -78oC MeI, HMPA

M


TRƯỜNG THPT CHUYÊN BIÊN HÒA ĐÁP ÁN ĐỀ ĐỀ XUẤT

THI CHỌN HỌC SINH GIỎI KHU VỰC MỞ RỘNG NĂM HỌC 2016- 2017 MÔN THI: HÓA HỌC LỚP 11.

Câu Đáp án 1. a. 1 d [ 2] = 0 = k1[S] − k2 [S][ 2] dt r = k2 [S][2] = k1[S] 1. b. d [1] = 0 = k1[S] − k3[S][1] + k4 [3] dt d [3] = 0 = k2 [S][2] + k3[S][1] − k4 [3] = k1[S] + k3[S][1] − k4 [3] dt Giai đoạn một chậm nhất, vì thế k1[S] << k3 [S][1] , khi bỏ qua k1[S] ta có:

Điểm 0,25

0.25

k3[S][1] = k4 [3] [1] k = 4 [3] k3[S]

1. c. Vì tốc độ sinh ra các gốc tự do là nhỏ, nồng độ của các gốc nhỏ và tốc độ lan truyền mạch tỉ lệ bậc nhất với nồng độ gốc sẽ lớn hơn tốc độ kết hợp của các gốc (là đại lượng tỉ lệ 0.25 bậc 2 với nồng độ gốc). Người ta gọi sự gàn đúng này gọi là sự gần đúng mạch dài ( nhiều giai đoạn phát triển mạch xảy ra trước khi các gốc kết hợp với nhau). 1. d. Tốc độ của hình thành các gốc phải bằng tốc độ kết hợp của chúng. Vì nồng độ

PhCH 2 * phải lớn hơn nhiều so với các gốc khác nên chỉ cần quan tâm đến tốc độ kết hợp của hai gốc benzyl:


d [ R] = 0 = 2k1[S] − 2k R [1]2 dt k1[S] kR

[1] = r = k3[1][S] =

k11 / 2 k3[S]3 / 2 k R1 / 2

Bậc toàn phần của phản ứng là 1,5. Hằng số tốc độ hiệu dụng (biểu kiến): 0,25

1/ 2

k k k = 1 1 / 23 kR Năng lượng hoạt động hóa:

E=

E1 E E + E3 − R ≈ 1 + E3 , 2 2 2

(Vì năng lượng hoat động hóa của sự kết hợp các gốc coi là bằng không). 2. a. Có cân bằng: P(I2)o – x

I2(k) ⇌

2I(k)

2x

Ở thời điểm cân bằng: P(I2)cb = P(I2)o – x Như vậy Pchung = P(I2)o + x

Ở 1073K x = 0,0750 – 0,0631 = 0,0119 atm P(I)cb = 2x = 0,0238 atm P(I2)cb = 0,0631 – 0,0119 = 0,0512 atm

K=

P( I )cb 2 = 0, 01106 P( I 2 )cb

Tính tương tự cho thời điểm 1173K thu được K = 0,04867

0,25

Từ đây ta có:

ln

K1173 ∆H o  1 1  o = −   ⇒ ∆H = 155, 052 J = 155 kJ K1073 R  1073 1173 

Như vậy ở 1100K ta có

ln

K1100 ∆H o  1 1  = −   ⇒ K1100 = 0, 0169 K1073 R  1073 1100 

∆Go = -RTlnK = 37248,8J ∆G = ∆H - T∆S ⇒∆S = 107,1 J.K-1 b. Có cân bằng:

I2(k)

2I(k)

P(I2)o – x

2x

0,25


Ở thời điểm cân bằng: P(I2)cb = P(I2)o – x Như vậy Pchung = P(I2)o + x

KP =

4 x2 1 1 = P = [ P( I 2 )o + x ] ⇒ P ( I 2 )o = 3x P ( I 2 )o − x 2 2

Vậy Pchung = 4x và P(I2)cb = 2x. Tức phần mol I(k) lúc cân bằng là 0,50

0,25

c. Với khí lý tưởng đơn nguyên tử Cv, m= CV,m + R = 3/2R + R = 5/2R Với khí lý tưởng hai nguyên tử Cv,m= CV,m+ R = 7/2R

∆Ho298 = Năng lượng liên kết I – I Do sự thay đổi nhiệt độ trở nên đáng kể ∆T = -802K

∆Ho298

= ∆Ho1100 + Cp∆T = 155052 + (298 – 1100) x (2 x 2,5 – 3,5)R = 145,050 kJ = 145 kJ

0,25

x2 = 10 −1,1 → x 2 + 0,0794 x − 10 −3,1 = 0 0,01 − x

2

Na2S → 2 Na+ + S20,01 0,01 KI → K+ + I0,06 0,06 + Na2SO4→ 2Na + SO420,05 0,05 S2+ H2O ⇌ HS- + OHKb(1) = 10-1,1 SO42- + H2O ⇌ H SO4- + OH- Kb(2) = 10-12 Kb(1)>> Kb(2) nên cân bằng (1) quyết định pH của dung dịch: S2- + H2O ⇌ HS- + OHK = 10-1,1 [ ] (0,01 -x) x x -3 -3 → x = 8,94. 10 →[OH ] = 8,94.10 → pH = 11,95 a) Tính pH của dung dịch

(1) (2)

0,5 -7,8

PbS : S = 10 -26 = 10 −13 PbI 2 : 3 10 −7, 6 / 4 = 10 −2,7 PbSO 4 : S = 10 = 10 −3,9 b. Pb2+ + S2- → PbS ↓ (Ks-1) = 1026. 0,09 0,01 0,08 Pb2+ + SO42→ PbSO4↓ (Ks-1) = 107,8. 0,08 0,05 0,03 Pb2+ + 2 I→ PbI2 (Ks-1) = 107,6. 0,03 0,06 Thành phần hỗn hợp: ↓A : PbS , PbSO4 , PbI2 Dung dịch B : K+ 0,06M Na+ 0,12M 2+ 22Ngoài ra còn có các ion Pb ; SO4 ; S do kết tủa tan ra. Độ tan của Bởi vì độ tan của PbI2 là lớn nhất nên cân bằng chủ yếu trong dung dịch là cân bằng tan của 1 PbI2. 2+ PbI2↓= Pb + 2I Ks 2+ -47 -3 Do đó [Pb ] = 10 = 2 x 10 M và [I-] = 4.10-3M.


[SO42-] =

[S2-] =

3

10−7,8 2 × 10

−3

=

5. 10−5,8 = 7,9.10−6M << [Pb2+]

=

5. 10−24<< [Pb2+]

10−26 −3

2 × 10 Các nồng độ SO42-, S2- đều rất bé so với nồng độ Pb2+, như vậy nồng độ Pb2+ do PbS và 0,5 PbSO4 tan ra là không đáng kể nên cách giải gần đúng trên là hoàn toàn chính xác. 0 0 a. Vì EAg+ /Ag > E - , pin điện có sơ đồ: I /AgI

(-) Ag, AgI(r) | Ag+(aq),I-(aq) | Ag(r) (+) Phản ứng ở cực âm: Ag(r) + I-(aq) AgI(r) + e + Phản ứng ở cực dương: Ag (aq) + e Ag(r) Phản ứng xảy ra trong pin: Ag+(aq) + IAgI(r) (1) I TAgI = aAg+ aq .aI- (aq) = K Trong đó T là tích số tan, a là hoạt độ, K là hằng số cân bằng của phản ứng (1). Eopin = Eo(+) – Eo(-) = 0,80 – (-0,15) = 0,95 V ∆Go = -F.Eopin = -0,95F = -RTlnK lgK = 0,95/0,059 = 16 K = 1016 -16 TAgI = 10 b. TAgI = s2; trong đó s là độ tan của AgI trong nước nguyên chất s = (TAgI)1/2 = 10-8 mol/L

1

2. Trước tiên cần tính thể khử chuẩn của cặp Fe3+/Fe2+ Fe3+ + 3e Fe Eo(1) = -0,037 V, ∆Go(1) = -3FEo(1) 2+ Fe + 2e Fe Eo(2) = -0,440 V, ∆Go(2) = -2FEo(2)

Fe3+ + e Fe2+Eo 3 = == 3Eo 1 - 2Eo 2 = 0,77V F F EoAu3+ /Au+ >EoFe3+ /Fe2+ nên pin điện có sơ đồ: (-) Pt | Fe3+(aq) 1M, Fe2+(aq) 1M || Au3+(aq) 1M, Au+(aq) 1M | Pt Phản ứng ở cực âm: Fe2+(aq) – e Fe3+(aq) 3+ Phản ứng ở cực dương Au (aq) + 2e Au+(aq) Phản ứng trong pin: Au3+(aq) + 2Fe2+(aq) Au+(aq) + 2Fe3+(aq) o o o Epin = EAu3+ /Au+ - EFe3+ /Fe2+ = 0,49 V -∆Go 3

∆Go 1 - ∆Go 2

(+)

2×0,49

4

1

∆Gopư = -RTlnK = -2FEopin = 0,49 V → K = 10 0,059 = 3,98.1016 Đặt công thức đơn giản nhất của A, B là XCln và XClm ta có M X + m.35, 45 = 1,516 ⇒ M X = 68, 70.( m − 1, 516n) M X + n.35, 45 thay các giá trị nguyên m, n sẽ có m 2 3 4 4 5 5 5 6 n 1 1 1 2 1 2 3 1 MX 33.25 101.95 170.65 66.50 239.35 135.20 31.05 308.05 X (loại) (loại) (loại) (loại) (loại) (loại) P (loại) Đáp số phù hợp m =5, n = 3, Mx = 31,05 tương ứng với X là P Cl

A là PCl3B là PCl5 Cl

P

b. 3NH4Cl + 3PCl5 →Cl(NPCl2)3 + 12HCl

Cl Cl

Cl P Cl

6 2 203.90 (loại)

6 3 99.75 (loại)

0,5 Cl


Khi đun nóng mạnh → chất nóng chảy không bị gãy vòng Khi đun nóng nhẹ → Vòng bị bẻ gãy thành các phân tử polime có hệ liên hợp pi

0,5

2. A: ClCN, B: H2NCN, C: (NH2)2CO, D: CaC2, E: CaCN2 Phương trình o

−10 C KCN + Cl2  → KCl + ClCN ClCN + 2NH3 → NH4Cl + H2NCN o

70 C H2NCN + H2O  → (NH2)2CO o

5

3000 C CaCO3 + 4C  → 3CO + CaC2 CaCl 2 → CaC2 + N2  C + CaCN2 1100o C CaCN2 + CO2 + H2O → CaCO3 + H2NCN 1. a.Bát diện b. Ta có

1 0,25

eg

eg

t2g

t2g spin thấp

spin cao

0,25

c. Phức spin thấp: µ mag = µ B 3 = 1.73µ B spin cao: µ mag = µ B 15 = 3.87µ B

0,25

d. Ta có bảng: Công thức [Co(CN)6]3[Co(NO2)6]3[Co(CO3)3]3[Co(CO3)2(NH3)2]-

Tên gọi Hexaxianocobantat(III) Hexa-N-nitritocobantat(III) Tricacbonatocobantat(III) Dicacbonatodiamincobantat(III)

2. Gọi anilin = An D = -lgT = -lg0,92 = 0,0362 L = 20 mm = 2 cm D 0, 0362 D = εlC = εl[An] → = = 1, 22.10−5 M εl 2.1, 48.103 Theo cân bằng: C6H5NH2 + H2O C6H5NH3+ + OH-

Màu sắc Vàng Cam Xanh Xanh da trời

Kb = 10-9,2

[C6 H 5 NH 3+ ][OH − ] [C H NH 2 ].10−9,2 = K b = 10−9,2 → [OH − ] = 6 5 [C6 H 5 NH 2 ] [C6 H 5 NH 3+ ]

(1)

0,25


6

Mà [C6H5NH2] + [C6H5NH3+] = 2.10-4 → [C6H5NH3+] = 2.10-4 – 1,22.10-5 = 1,878.10-4 M Thay các giá trị vào (1) ta có 1,22.10−5 .10−9,2 [OH − ] = = 10−10,39 → [H + ] = 10→3,61 → pH = 3,61 1,878.10−4 1.

1 1/8.4

2. (0,5 điểm) B có 3C bất đối, không có mặt phẳng và tâm đối xứng nên có 8 đồng phân lập thể. (0,25 điểm) (Cấu hình viết gọn trong bảng, ví dụ với B1 là (1R)-(2R)-(4R)) (0,25 điểm) B1 B2 B3 B4 B5 B6 B7 B8 C1 R S R R S R S S C2 R S R S R S R S C4 R S S R R S S R 3. C, C1, D và D1 là những đồng phân hỗ biến, xúc tác kiềm làm thuận lợi sự hỗ biến đó HO

O

O

O

O

O

OH-/-H2O Me C

NO2

O

O

H2O/-OHMe

Me NO2 O

Me

NO2 O

O

NO2

C1

O

O

O

O

O

H2O/-OHMe

NO2

Me

NO2

Me

NO2 D1

Me D

NO

Xuất phát từ C, C1, D hoặc D1 qua phản ứng chuyển vị benzylic rồi tự mất nước đều chuyển thành hợp chất thơm bền vững, đều dẫn đến cùng một sản phẩm, ví dụ:

0,25

0,25 4. – Nguyên tử N trong nhóm NH ở trạng thái lai hóa sp2, cặp electron chưa chia ở obitan p xen phủ với 5 obitan p khác tạo thành hệ thơm được lợi về mặt năng lượng nhưng mất tính bazơ Nguyên tử N thứ hai ở trạng thái lai hóa sp2, cặp electron chưa chia ở obitan sp2 không tham


gia vào hệ thơm nên còn tính bazơ Nguyên tử N trong nhóm NH2 ở trạng thái lai hóa sp3

- Nhóm NH3+ là axit liên hợp của nhóm NH2, nhóm NH+ là axit liên hợp của nhóm Nsp2 Bazơ càng mạnh thì axit liên hợp càng yếu, vì thế giá trị 9,2 là của nhóm NH3+ còn 6,0 là của nhóm NH+, nhóm COOH có tính axit mạnh nhất nên có giá trị là 1,8

0,25

0,25 7

X có (π+v) = 3 X + H2  → C10H22 (π+v) = 0 ⇒ X có liên kết bội và vòng 3 cạnh (nếu có) X + Br2  → C10H16Br6 (π+v) = 0 1.O3 X  → Y (C5H8O (π+v) = 2) 2.Zn / HCl 1.O3 Y (Y không có đối quang) X → 2.H 2 O 2

⇒ Y có liên kết đôi C=O và 1 vòng 3 cạnh X có 2 vòng 3 cạnh và liên kết đôi C=C Các công thức cấu tạo của X (mỗi công thức được 0,25 điểm)

0,25

0,25.5

Viết phương trình hóa học của với 1 đồng phân (ví dụ với X1): 0,25 điểm

8

1. A có công thức phân tử CHNO Ở thể khí dC≡N (116 pm) < dCN(A) (121 pm) < dC=N (130 pm) dC≡O (112 pm) < dCO(A) (117 pm) < dC=O (123 pm) Liên kết CN và CO trong A vừa có một phần liên kết đôi, một phần liên kết ba

CTCT của A ở thể khí 2. Ka(A) > Ka(CH3COOH) là một axit có oxi - Trong nước, A nằm cân bằng với đồng phân là axit có oxi

1


Anion sinh ra bền vững, cấu trúc thẳng nên khả năng solvat hóa tốt nên lực axit của A lớn hơn CH3COOH - CH3COOH khi phân li tạo ra anion CH3COO- có khả năng solvat hóa kém, có nhóm kị nước 0,5 nên kém bền, làm giảm tính axit của CH3COOH 3. Trong dung dịch đặc A chuyển thành Y có dạng vòng 6 cạnh dC=N (130 pm) < dCN(A) (135 pm) < dC-N (147 pm) Liên kết CN trong Y có một phần liên kết đơn, một phần liên kết đôi CTCT của Y

9

1. O

O

COOH

O

O

N Me

H O

N

H

Me

O

H

N

H

H

Me

N O

OAc H

Me

O

N

O

O

Me

O

O H

CH2 -CH3COOH OCOCH3 O

Me

O

-CH3COOH O

CH2

O N

O

O

0,5 0,5

N

Me CH2 N

O

Me

2. MeO

0,5 C2H5 MeO

O H

O

benzen

MeO

MeO

O

H

OMe

OMe OH

OMe OH

OMe OH

MeO

O

OH

OMe MeO

OMe

MeO MeO

O

OH MeO

O

OMe

OH OMe

OMe

3.

0,5


4.

0,5 H

N Ns

N

O

Bn

H N

O N Ns

Bn

Bn N C

O

BnHN

O

O N Ns

N Bn O

-H+ +H+

10

Ns

N

H

NHBn N Bn O

HO

-H+ +H

+

O

-H

+H+

O NHBn

H

N Bn NHNs

O

+H+

Đúng công thức chất A và E được 0,25 điểm Công thức các chất còn lại từ A đến O được 0,15 điểm

N Bn NsHN

N Ns

N B O

CONHBn -H+

O

+

O

NHBn N Bn

NsHN

N Ns N Bn

O

BnHN

Bn NH O

O


O

OH

O

O

BH3,THF,0oC

NaOMe, PhMe

CrO3, H2SO4 axeton

H2O2, NaOH, 0oC

Br A O

Br

C

O

Br

Br2, KBr

CH2OH

B

O

KOH, H2O

H2O

THF

O O

Br2, KBr H2O

D O

O

n-Bu3SnH, AIBN

O

toluen,

O

75oC

Al/Hg, THF/H2O

O

O I OBz

OBz

H2, Ni-Ra

BzCl Py

O

LDA, THF, -78oC MeI, HMPA

O

BzO

O

M

L

N

MeCOOOH

2. CrO3, H2SO4 axeton BzO

EtOAc, 55 C O BzO COOH

S

N BzO

O O O S

OH O

,

S

O Ph P, THF 3 O O

o

N

O OBz

OBz

OBz

O

BzO

O

K

1. LiOH, H2O

O

HO

H

OH

HO

0oC

O

O

DMF, -20 C

O

O

G

o

COOH

F

COO-

[E] O

Br

COOH O

A'

N


HỘI CÁC TRƯỜNG CHUYÊN VÙNG DUYÊN HẢI VÀ ĐỒNG BẰNG BẮC BỘ TRƯỜNG THPT CHUYÊN HẠ LONG TỈNH QUẢNG NINH

ĐỀ THI MÔN HÓA HỌC – KHỐI 11 NĂM 2017 Thời gian làm bài: 180 phút (Đề này có 06 trang, gồm 10 câu)

ĐỀ THI ĐỀ XUẤT Câu 1: (2,0 điểm) Tốc độ phản ứng – Cân bằng hóa học 1. Trong môi trường axit, ion hidrosulfit bị ion hidrocromat oxi hóa. Các nghiên cứu chỉ ra phương trình tốc − 2 độ phản ứng: v = k[ HCrO−4 ][ HSO3 ] [ H + ] .Phản ứng thực hiện trong dung dịch đệm ở pH=5, nồng độ − −4 − − ban đầu các ion, [ HCrO4 ]o = 10 M; [ HSO3 ]o = 0,1M thì sau 15s, nồng độ ion HCrO4 giảm một nửa. −

Nếu vẫn thực hiện trong dung dịch đệm ở pH=5 nhưng thay nồng độ ban đầu [HCrO4 ]o = 0,01M và − − [ HSO3 ]o = 0,015M. Tính thời gian để HCrO4 giảm còn 12,5% lượng ban đầu. 2. Sự nhiệt phân etanal tiến hành theo cơ chế đã được đơn giản hóa như sau: k1 CH 3CHO  → CH 3* + HCO*

(1)

k2 CH 3* + CH 3CHO  → CH 4 + CH 3CO*

(2)

k3 CH 3CO*  → CH 3* + CO

(3)

k4 HCO*  → H * + CO

(4)

k5 CH 3CHO + H *  → H 2 + CH 3CO*

(5)

k6 2CH 3*  → C2 H 6

(6)

a) Dùng phương pháp gần đúng các trạng thái ổn định của hợp chất trung gian để tìm công thức tính nồng độ của các dạng HCO*, H*, CH3* và CH3CO*. b) Hãy tìm qui luật về tốc độ hình thành metan, etan, hydro và CO theo nồng độ etanal. c) Theo cơ chế trên, etanal có hai cách phân hủy. Đối với mỗi cách, hãy tìm bậc của etanal. 3. Cho một bình kín dung tích 22,4 lít chứa sẵn 1 mol rắn A và 0,55 mol khí B. Đun nóng bình đến 2730C và dừng lại khi áp suất của bình giữ ổn định ở 2,9 atm. a. Tính áp suất riêng phần của từng khí trong hỗn hợp cân bằng b. Chuyện gì sẽ xảy ra nếu ban đầu chỉ có 0,1 mol rắn A? Biết các cân bằng xảy ra trong bình như sau:  → C( k ) + D( k ) (1) K P = 6 A( r ) + B( k ) ←  1  → E( k ) + D( k ) (2) K P = 9 C( k ) + B( k ) ←  2 5 Câu 2: (2,0 điểm) Cân bằng trong dung dịch điện li 1. Có hai dung dịch A chứa H2C2O4 0,1M và dung dịch B chứa Na2C2O4 0,1M. Tính pH và nồng độ ion C2O42- có trong dung dịch A và B. 2. Thêm Fe(NO3)3 (tinh thể) vào dung dịch A và dung dịch B để đạt nồng độ (ban đầu) là 1,0.10-4M. Giả thiết thể tích dung dịch thay đổi không đáng kể. Hãy cho biết có xuất hiện kết tủa Fe(OH)3 không? Chứng minh. 3. Tính phần mol của phức Fe(C2O4)33- trong dung dịch A. Cho các giá trị:


Hằng số tạo thành tổng hợp của phức Fe3+ với C2O42- là β1 = 1,0.108; β2 = 2,0.1014; β3 = 3,0.1018; KW = 10-14. Hằng số phân ly axit của H2C2O4 là Ka1 = 0,05; Ka2 = 5.10-5. Tích số tan của Fe(OH)3 Ks = 2,5.10-39. Câu 3: (2,0 điểm) Điện hóa học Trộn 10ml dd Ag+ 0,01M với 10ml dd NH3 0,12M thu được dd A. Trộn 10ml dd Ag+ 0,02M với 10ml dd CrO 24 − 0,02M được hỗn hợp B. Ghép điện cực Ag nhúng trong dd A với điện cực Ag nhúng trong hỗn hợp B thành pin 1. Pin 2 được ghép bởi điện cực hiđro nhúng trong dd NH4HSO4 0,01M và điện cực hiđro nhúng trong dd (NH4)2S 0,05M. a) Cho biết anot, catot của mỗi pin? Tính suất điện động và viết sơ đồ pin của 2 pin trên? d) Mắc xung đối pin 1 và pin 2. Hãy viết quá trình xảy ra trong 2 pin sau khi mắc xung đối. Từ đó cho biết có thể dùng NH3 làm thuốc thử để hòa tan Ag2CrO4 không?( không căn cứ vào hằng số cân bằng) Cho lg β Ag ( NH )+ = 7, 24; pK s ( Ag2CrO4 ) = 11,89; pK a ( HSO− ) = 2; pK a ( NH + ) = 9, 24; pK a ( H 2 S ) = 7, 02;12,90 3 2

4

4

Câu 4: (2,0 điểm) Nhóm N – P, nhóm C - Si 1. Một nguyên tố X có khả năng phản ứng với canxi cho chất Y. Mặt khác X tan được trong dung dịch kiềm tạo ra một hợp chất A và khí B đều có chứa nguyên tố X. A phản ứng với clorua vôi thu được một kết tủa C. Kết tủa này sẽ chuyển thành Y khi xử lý với nhôm ở nhiệt độ cao. Hòa tan chất Y trong dung dịch HCl loãng thu được B. Biết rằng khi xử lí C với SiO2 và than cốc thu được X, còn trong trường hợp không có than cốc thu được D. D tan được trong cả dung dịch axit loãng và kiềm loãng. Lập luận xác định cấu trúc các chất chưa biết và viết các phương trình phản ứng xảy ra. 2. Một hợp chất A rất độc có màu vàng được sinh ra khi trộn hai axit đặc B và C. Sản phẩm phụ của phản ứng là khí E màu vàng và nước. Mặc dù hai axit đã được biết đến từ rất lâu, nhưng mãi đến năm 1831 thì người ta mới quan sát thấy phản ứng tạo thành A. Một cách khác để thu được A là tiến hành phản ứng giữa hai khí E và D, quá trình này không sinh ra sản phẩm phụ. Tuy nhiên ở nhiệt độ cao phản ứng xảy ra theo chiều nghịch. Khí D được tạo thành khi cho kim loại chuyển tiếp I phản ứng với axit loãng C. Nếu dùng axit đặc thì thay vì D thu được một khí M màu đỏ nau. Trong cả hai trường hợp đều thu được một muối duy nhất J màu xanh da trời. Khí M phản ứng với nước thu được hai axit C và F. Axit F khi phản ứng với axit G thu được hợp chất H, hợp chất này khi phản ứng với axit B cho hai chất A và G. Hợp chất H có thể xem như anhydrit của hai axit này. Khi cho B phản ứng với KMnO4 thu được một lượng nhỏ khí E. Khí này được sử dụng để tổng hợp chất K bằng cách cho phản ứng với muối NaCN. Nhiệt phân Hg(CN)2 thu được L là một chất có tính đối xứng. Khi cho axit G phản ứng với BaCl2 thu được kết tủa trắng. Khí D chuyển thành khí M khi tiếp xúc với không khí. Xác định các chất chưa biết. Câu 5: (2,0 điểm) Phức chất, trắc quang 1. Một hợp chất của Cr được tổng hợp. Sự phân tích nguyên tố cho thấy thành phần của nó là: Cr 27,1%; C 25,2%; H 4,25% khối lượng, phần còn lại là oxi. a. Công thức thực nghiệm của hợp chất này là gì? b. Nếu công thức thực nghiệm gồm 1 phân tử nước, những phối tử khác là gì? Trạng thái số oxi hóa của Cr? c. Nghiên cứu tính chất từ cho thấy rằng hợp chất này là nghịch từ, giải thích tính chất từ của hợp chất này như thế nào? Vẽ cấu trúc của hợp chất này. 2. Từ kim loại M (Z<37) thực hiện các sơ đồ chuyển hóa sau: 0

150 ,15 atm M + nCO  →A 0

t A + 4 KOH  → B + C + 2 H 2O

a) Xác định B. Biết một hợp phần trong B có cấu trúc tứ diện, cacbon chiếm 19,512% khối lượng B. b) Cho A phản ứng với dixyclopentadien, đun nóng thu được D và hỗn hợp khí Y (CO, H2) có dY/H2 = 85/7. Ở điều kiện thường, D là tinh thể màu đỏ tím đậm, dễ dàng hòa tan trong các dung môi hữu cơ phân cực vừa phải như clorofom, pyridin, ít tan trong các dung môi không cực (như CCl4), không tan trong nước. Sản


phẩm của sự khử D bằng kim loại kiềm hoặc hidrua được sử dụng rộng rãi vì khả năng dễ ankyl hóa, axyl hóa của nó. Viết các đồng phân tương ứng của D. 3. Một phương pháp đơn giản để đo nồng độ của ozon trong khí quyển mặt đất được tiến hành như sau. Cho bọt không khí đi qua dung dịch nước đã axit hóa có chứa iot và ozon trong khí quyển sẽ oxi hóa iotdua thành triiodua theo phản ứng chưa cân bằng sau: O3(k) + I-(dd) + H+(dd) I3- (dd) + O2(k) + H2O (1) Sau khi phản ứng kết thúc, nồng độ triiodua được xác định bằng máy đo quang phổ UV-Vis tại 254 nm. Tiến hành thí nghiệm, như sau: Sục bọt không khí trong 30 phút vào 10mL dung dịch nước chứa KI dư tại điều kiện khí quyển như sau: áp suất = 750 torr, nhiệt độ 298K, tốc độ dòng = 250 mL.phút-1. Độ hấp thụ của dung dịch I3- tạo thành đo được trong tế bào có độ dày l = 1,1 cm khi sử dụng một máy trắc quang có trang bị tế bào quang điện. Điện trở của tế bào quang điện tỷ lệ nghịch với cường độ của ánh sáng. Trị số điện trở (của tế bào quang điện) khi bị chiếu bới chùm sáng đi qua cuvet trống và qua cuvet chứa mẫu hòa tan là 12,1 kΩ và 19,4 kΩ . Hệ số hấp thụ mol của I3- hòa tan được xác định là ε = 2,4.105 M −1.cm−1 a. Tính số mol ozon trong mẫu trong không khí. b. Giả thiết rằng các khí được sử dụng là khí lý tưởng. Tính nồng độ theo ppb của ozon có mặt trong mẫu không khí. Câu 6: (2,0 điểm): Quan hệ giữa hiệu ứng cấu trúc và tính chất 1.Cho các công thức cấu tạo sau: C(CH=CHF)2 CH CH2 F

Me 1 2 4 5 Me 3 Et O O Me O

(A)

(B)

Me

OH O

O 2N (C)

Me

O

COOH

N OH

O 2N

N H

(D)

NH2 (E)

a) Hãy vẽ công thức các đồng phân lập thể ứng với cấu tạo A. b) Ứng với công thức cấu tạo B có bao nhiêu đồng phân lập thể, vì sao? Dùng các kí hiệu thích hợp để chỉ rõ cấu hình của mỗi đồng phân đó. c) Hãy viết cơ chế phản ứng để giải thích vì sao C và D khi tương tác với dung dịch NaOH thì đều tạo thành natri 3-metyl-4-nitrobenzoat. d) Hãy chỉ rõ trạng thái lai hóa của từng nguyên tử N ở cấu tạo E và ghi giá trị pKa (ở 25 oC): 1,8; 6,0; 9,2 vào từng trung tâm axit trong công thức tương ứng với E, giải thích. 2. Trong số các chất sau đây, chất nào thuộc loại hợp chất thơm, không thơm, phản thơm, vì sao? Chất nào tác dụng được với kali kim loại trong đietyl ete? Viết phương trình phản ứng và giải thích. EtOOC

O

EtOOC

COOEt (F)

(G )

(H )

(I)

(J )

Câu 7: (2,0 điểm): Hidrocacbon 1. Dewar đã đề nghị cấu trúc C cho benzen, cấu trúc này gồm hai vồng xiclobutan ngưng tụ với nhau: Để làm giảm sức căng vòng, C dễ bị chuyển vị dưới tác dụng của nhiệt tạo thành hợp chất đơn vòng D. a. Xác định công thức cấu tạo và hóa lập thể của D. Gọi tên D theo danh pháp IUPAC, sử dụng kí hiệu lập thể E/Z.


b. Điều thú vị là dù có sức căng rất lớn nhưng hợp chất C không chuyển hóa ngay thành hợp chất E (gọi là cấu trức Kekule) vốn rất bền. Cho C phản ứng với một axit vô cơ thu được chất E. Đề xuất một cơ chế phản ứng cho chuyển hóa này. c. Dự đoán công thức cấu tạo của hợp chất F (C8H8O4) biết F có khả năng phản ứng với Chì tetraaxetat để tạo thành C. d. Ladenburg cũng đề nghị một cấu trúc khác của benzen, sau này được biết đến với tên gọi Prismane H. Cấu trúc này cố gắng giải thích cho sự hình thành một sản phẩm thế mono với benzen và ba đồng phân thế 2 lần của benzen. Quang phân G tạo thành một lượng nhỏ H. Xác định công thức cấu tạo của H. Xác định CTCT dạng Ladenburg của tất cả các đồng phân đibrombenzen.

e. Một cấu trúc khác được đề nghị cho benzen là benzvalene I. I được điều chế theo sơ đồ sau, hãy xác định các chất J,K trong chuỗi tổng hợp đó:

f. Trimetylenxiclopropan N là một đồng phân khác của benzen, có thể tổng hợp được từ chất L theo sơ đồ sau, xác định chất M,N. Vẽ cấu trúc các đồng phân lập thể của L. Dự đoán công thức cấu tạo của sản phẩm O tạo thành từ phản ứng Diels-alder giữa N và axetilen.

2. Phân từ hidrocacbon G (C8H8) chỉ gồm các liên kết đơn và chỉ chứa cacbon bậc ba tương đương nhau. Cho chất G phản ứng với H2/Pd ở điều kiện khắc nghiệt, thu được chất H (C8H10) và chất I (C8H14). Trong phân tử của I. Tỉ lệ số nguyên tử C bậc hai và bậc ba là 3:1. Chất G phản ứng với CBr4/NaOH 50% thu được chất K (C8H7Br) và L (C8H6Br2). Đồng phân hóa G bằng AgClO4 thu được X. Trong phân từ X chỉ chứa cacbon bậc ba và chia làm ba nhóm tương đương nhau. Xác định cấu trúc các chất G,H,I,K,L và X. Câu 8: (2,0 điểm): Xác định cấu trúc Hai lacton thơm A, B có công thức phân tử (C10H10O4) đều tan trong dung dịch NaOH loãng nhưng không tan trong dung dịch NaHCO3. Cả A và B cho phản ứng màu với dung dịch FeCl3. Khi cho A phản ứng CH3I/K2CO3 tạo ra chất C (C11H12O4). Biết C chứa ba nhóm metyl không giống nhau, trong đó có một nhóm metyl liên kết với vòng thơm. Xử lí C với BCl3 để tách loại một nhóm metyl tạo ra D là một đồng phân mới của A. D có một nhóm hidroxi tạo liên kết hidro nội phân tử. Cho chất E (2-metyl-1,3-dihidroxibenzen) phản ứng với MeI/K2CO3 tạo F(C9H12O2), F được khử bằng Li/NH3 lỏng có mặt 2-metylpropan-2-ol cho một ddien đối xứng và không liên hợp G. Tiếp tục cho G phản ứng KNH2/NH3 lỏng tạo ra một sản phẩm H. Thực hiện phản ứng ozon phân H rồi xử lí tiếp thu được nhiều


sản phẩm trong đó có metyl 2-oxopropanoat. Thực hiện phản ứng giữa H với dimetylbut-2-indioat, đun nóng tạo K(C15H20O6), tiếp tục đun nóng K để loại eten tạo được một este thơm L. Thủy phân L trong môi trường bazo rồi axit hóa dung dịch tạo thành M (C11H12O6), đun nóng M trong chân không tạo ra N (C11H10O5). Khử N bằng NaBH4 trong DMF tạo C và một lacton P đồng phân, P cũng có thể thu được nhờ metyl hóa B. Hãy biện luận xác định cấu tạo các chất từ A đến P Câu 9: (2,0 điểm): Cơ chế phản ứng 1. Khi đun nóng hợp chất A sẽ chuyển thành B,C,D. Phản ứng được thúc đẩy khi chiếu xạ. Hãy xác định cấu trúc của B,C,D và dung cơ chế phản ứng để giải thích các sản phẩm này. to

O

2

C4H9Cl B

Cl

+

C3H6O C

+

C17H15ClO D

2. Người ta đã điều chế 2-phenylxiclo [3.2.2] azin từ 2,6-lutidin qua ba giai đoạn: 1) 2-phenyl-5-metylpirocolin được tạo thành với hiệu suất 86% từ 2,6-lutidin và phenaxyl bromua: PhCOCH2Br

500c

NaHCO3

48 h

H2O N

N

2-phenyl-5-metyl pirocolin Ph

2) Xử lí 2-phenyl-5-metylpirocolin bằng phenyl liti trong ete ở -300C. 3) Cộng hợp liên tiếp với N,N-đimetylfomamit và đun nóng trong thời gian ngắn với axit axetic sẽ thu được sản phẩm với hiệu suất 45%. Viết cơ chế phản ứng xảy ra trong quá trình tổng hợp này. Hãy dẫn ra cấu trúc cộng hưởng quan trọng nhất phù hợp với chính công thức xiclo [3.2.2]azin. So sánh tính bazo của sản phẩm với anilin, so sánh khả năng nitro hóa với pirol và piridin. Nhóm nitro vào vị trí nào dễ nhất? Công thức của 2-phenylxiclo [3.2.2] azin:

N

Ph

Câu 10: (2,0 điểm): Tổng hợp các hợp chất hữu cơ 1. Thromboxan (C21H34O4) là một loại thuốc làm co mạch và là một tác nhân làm tăng mạnh huyết áp. Dược phẩm thromboxane A2 được tổng hợp theo sơ đồ sau đây:


Xác định cấu trúc các chất chưa biết, tên phản ứng sử dụng ở giai đoạn chuyển B thành C và đề nghị một cơ chế hợp lí cho phản ứng chuyển D thành E. 2. Huperzine A là một hợp chất được sử dụng trong điều trị chứng mất trí nhớ cũng như rối loạn đường máu. Các nhà khoa học Trung Quốc đã tổng hợp A theo sơ đồ sau. Hãy xác định cấu tạo các chất chưa biết trong sơ đồ. Biết G là đimetyl este có công thức C12H15NO5

**********************************

HẾT **********************************

HƯỚNG DẪN CHẤM ĐỀ ĐỀ XUẤT HÓA 11 (15 trang) Câu 1: (2,0 điểm) Tốc độ phản ứng – Cân bằng hóa học Phần Đáp án 1 −5 a) [ H + ] = 10 M = const. −

[ HSO3 ]o

≫ [ HCrO−4 ]o → [ HSO3− ]o = const. −

− 2

v = k[ HCrO4 ][ HSO3 ] [ H + ] −

− 2

+

= k′ [ HCrO4 ] ( k′ = k[ HSO3 ] [ H ] ) −

→ Phản ứng bậc nhất theo HCrO4 .

k′ =

ln 2 −1 (s ) 15

PTHH: 2HCrO4− + 3HSO3− + 5H +  → 2Cr 3+ + 3SO24− + 5H 2O −

Theo phương trình: Tỉ lệ HCrO 4 và HSO3 là 2:3. −

Lại có: [ HCrO 4 ]o : [ HSO3 ]o = 2:3. → Tại mọi thời điểm trong quá trình phản ứng luôn có:

3 [HSO3− ] = [HCrO−4 ] . 2 −

− 2

v = k[ HCrO 4 ][ HSO3 ] [ H + ]

Điểm 0,5


2

3 −  − = k.[H ]. [HCrO 4 ]  .[HCrO 4 ] 2  9 + − 3 = k[H ].[HCrO 4 ] 4 +

− 3

= k 2 .[HCrO 4 ] +

+

2

Có: k ′ = k.[H ].0,1 → k.[H ] = 100k ′.

→ k2 =

9 ln 2 .100. = 10,4 ( M−2.s−1) 4 15

1 d[HCrO −4 ] d[HCrO −4 ] Lại có: v = − . →− = 2v = 2.k 2 .[HCrO 4− ]3 2 dt dt Tích phân 2 vế:

 1 1 1 −   = 2k 2 t 2  [HCrO −4 ]2 [HCrO −4 ]o2  1  82 1  ⇔  −  = 2.10,4.t 2  0,012 0,012  → t = 15144,23(s). 2

0,75

a/ 0,25 điểm

d  HCO* 

Theo (1) và (4):

Theo (4) và (5): Theo d CH 3*  dt

dt d  H *  dt

= k1[CH 3CHO] − k4 [HCO* ] = 0 ⇒ [HCO* ] =

= k4 [HCO* ] − k5 [H* ][CH3CHO] = 0 ⇒ [H* ] =

(1),

(2),

k1 [CH 3CHO] k4

k4 [HCO* ] k = 1 k5 [CH3CHO] k5

(3)

(6):

= k1[CH 3CHO] − k 2 [ CH 3CHO ][CH*3 ] + k3 [CH 3CO* ] − 2k6 [CH*3 ]2 = 0

Theo (2), (3) và (5):

d CH 3CO* 

= k2 [CH*3 ][CH3CHO ] − k3 [ CH3CO* ] + k5 [H* ][CH 3CHO ] = 0

dt Cộng hai biểu thức trên, rồi thế giá trị nồng độ của H* và HCO* vào ta được 2k1[CH 3CHO] − 2k6 [CH*3 ]2 = 0 ⇒ [CH*3 ] =

⇒ CH 3CO  = *

k2 CH 3*  + k5  H * 

b) 0,25 điểm Tốc độ hình thành:

k3

k1 [CH 3CHO]1/2 k6

[CH 3CHO ] =

k2 k3

k1 k 3/2 [CH 3CHO] + 1 [CH 3CHO] k6 k3


d [CH 4 ] dt

= k2 [CH*3 ][CH 3CHO ] = k2 .

d [C2 H 6 ] dt d [H2 ]

dt

d [CO ] dt

k1 3/2 [CH 3CHO ] k6

= k 6 [CH *3 ]2 = k1[CH 3CHO ]

= k5 [H* ][CH 3CHO] = k1[CH 3CHO] = k3 [CH 3CO* ]+k4 [HCO* ] = 2k1[CH 3CHO]+k2

k1 [CH 3CHO]3/2 k6

c) 0,25 điểm Etanal có hai cách phân hủy:

2CH 3CHO → C2 H 6 + H 2 +2CO

(a)

CH 3CHO → CH 4 + CO

(b)

+ sự hình thành etan và hydro là bậc một đối với etanal ⇒ phản ứng (a) là phản ứng bậc nhất + sự hình thành metan là bậc 3/2 ⇒ phản ứng (b) là phản ứng bậc 3/2 + CO được hình thành từ 2 phản ứng nên nồng độ của CO phụ thuộc vào nồng độ của etanal theo bậc 1 và bậc 3/2.

3

0,75

a.( 0,5đ) Ta có cân bằng sau ở 2730C:

 → C( k ) + D( k ) (1) K P = 6 A( r ) + B( k ) ←  1  → E( k ) + D( k ) (2) K P = 9 C( k ) + B( k ) ←  2 5 n 0 RT Tại thời điểm ban đầu: PB = B = V 0

22, 4 .546 273 = 1,1 atm 22, 4

0,55.

Tại thời điểm cân bằng: PB + PC + PD + PE = 2,9 atm Nhận xét: trong quá trình phản ứng, lượng chất B mất đi bằng với lượng chất D tạo thành. Hay nói cách khác, tổng lượng B và D tại thời điểm cân bằng với lượng ban đầu của chất B Vậy ta được : PB + PD = PB0 = 1,1 atm Mặt khác: PE PD K P2 PC PB PE KP 6 10 = = 2 ⇒ PE = 2 PC2 = PC2 = PC2 9 K P1 PC PD PC K P1 3 5 PB

Từ các phương trình trên ta có được: 10 PC + PC2 = 2,9 − 1,1 = 1,8 3 ⇒ PC = 0, 6 atm ⇒ PE = 1, 2 atm

Tính PB và PD dựa vào K1:

PB PC 0, 6 = = = 0,1 và PB + PD = 1,1 ⇒ PB = 0,1 atm; PD = 1, 0 atm PD K1 6

Tính lại để biết rằng phản ứng theo chiều thuận có kết thúc trước khi đạt cân bằng hay không Từ 2 cân bằng (1) và (2) ta nhận thấy: Lượng chất A mất đi bằng tổng lượng chất C tạo thành, mà tổng lượng chất C này chuyển hóa một phần vào E nên có thể kết luận rằng: nên phản ứng (1) theo


chiều thuận chưa kết thúc khi đạt trạng thái cân bằng. b)(0,25đ) Nếu nA = 0,1 mol thì lúc này ∆n A > n A nên phản ứng (1) theo chiều thuận đã kết thúc trước khi đạt trạng thái cân bằng. Lúc này áp suất bình tại thời điểm cân bằng sẽ khác đi. Lưu ý: Do đề bài không yêu cầu tính lại áp suất bình nên chỉ cần có câu kết luận trên là được trọn điểm, nếu thí sinh có tính toán lại mà không kết luận cũng không được tính điểm Câu 2: (2,0 điểm) Cân bằng trong dung dịch điện li Phần Đáp án 1 Tính pH và nồng độ ion C2O42- có trong dung dịch A

Ta có:

H2C2O4 HC2O4

-

H+ + HC2O4+

H +

Đ iể m 1đ

Ka1 = 0,05

C2O42-

Ka2 = 5,0.10

(1) -5

(2)

Vì Ka1 >> Ka2 nên (1) là cân bằng chính. Gọi C là nồng độ ban đầu của A.

K a1 =

[H + ][HC2 O4- ] [H + ]2 = [H 2 C2 O4 ] CA - [H + ]

[H + ]2 + K a1[H + ] - K a1CA = 0

⇒ [H+] = 0,05M ⇒ pH = 1,3.

[H + ][C2 O42- ] K a2 = [HC2 O-4 ] và [H + ] = [HC2 O-4 ] = 0,05M Mà ta có: ⇒ [C2 O 2-4 ] = K a2 = 5.10−5 M Tính pH và nồng độ ion C2O42- có trong dung dịch B Ta có: H2O + C2O42H2O + HC2O4-

HO- + HC2O4HO- + H2C2O4

Kb1=2,0.10-10 (1) Kb2=2,0.10-13 (2)

Vì Kb1 >> Kb2 nên (1) là cân bằng chính.

[OH - ] = K b1C = 2,0.10-10 .0,1 = 4,5.10-6 M

⇒ pH = 8,7 ⇒ 2

[C 2 O 42- ] = 0,1M

Chứng minh dung dịch A không có kết tủa Fe(OH)3: Có các cân bằng:

Fe3+ + C2O42-

FeC2O4+

Fe3+ + 2C2O42-

Fe(C2O4)2-.

Fe3+ + 3C2O42-

Fe(C2O4)23-

Bảo toàn nồng độ ion Fe3+ ta có:

CFe3+ = [Fe3+ ] + [FeC2O4+ ] + [Fe(C2O4 )2- ] + [Fe(C2O4 )33- ] Mà

[FeC 2 O 4 + ] = β1[Fe3+ ][C2 O 2-4 ]

[Fe(C2 O 4 ) 2 - ] = β 2 [Fe3+ ][C2 O 42- ]2

[Fe(C2 O 4 )33- ] = β3 [Fe3+ ][C2 O 2-4 ]3 Suy ra


CFe3+ = [Fe3+ ] + β1[Fe3+ ][C2O42- ] + β2 [Fe3+ ][C2O2-4 ]2 + β3[Fe3+ ][C2O2-4 ]3 [Fe3+ ] = ⇒

=

CFe3+

1 + β1[C2 O ] + β 2 [C2 O2-4 ]2 + β3 [C2 O2-4 ]3 24

1,0.10−4 = 1,1.10−10 M 8 −5 14 −5 2 18 −5 3 1 + 1,0.10 .5,0.10 + 2,0.10 .(5,0.10 ) + 3,0.10 .(5,0.10 )

Mà [OH-]A = 2.10-13M ⇒

[Fe3+].[OH-]3 = 9,1.10-49 < Ks

⇒ không có kết tủa Fe(OH)3 ở dung dịch A. * Chứng minh dung dịch B có kết tủa Fe(OH)3: Fe3+ + C2O42-

Có các cân bằng:

FeC2O4+

Fe3+ + 2C2O42-

Fe(C2O4)2-.

Fe3+ + 3C2O42-

Fe(C2O4)23-

Cách 1: Bảo toàn nồng độ ion Fe3+ ta có:

CFe3+ = [Fe3+ ] + [FeC2O4+ ] + [Fe(C2O4 )2- ] + [Fe(C2O4 )33- ] + 3+ 2Mà [FeC 2 O 4 ] = β1[Fe ][C2 O 4 ]

[Fe(C2 O 4 ) 2 - ] = β 2 [Fe3+ ][C2 O 42- ]2

[Fe(C2 O 4 )33- ] = β3 [Fe3+ ][C2 O 2-4 ]3 Suy ra

CFe3+ = [Fe3+ ] + β1[Fe3+ ][C2O42- ] + β2 [Fe3+ ][C2O42- ]2 + β3[Fe3+ ][C2O42- ]3 [Fe3+ ] = ⇒

=

CFe3+

1 + β1[C2 O ] + β 2 [C2 O2-4 ]2 + β3 [C2 O2-4 ]3 24

1, 0.10−4 = 3,3.10−20 M 8 14 2 18 3 1 + 1, 0.10 .0,1 + 2, 0.10 .(0,1) + 3, 0.10 .(0,1)

Cách 2: Cân bằng chủ yếu:

Fe3+ + 3C2O42 − Fe(C2O4 )33− 10-4

0,1

10-4

Xét cân bằng ngược lại

Fe(C2O4 )33− Fe3+ + 3C2O42− 10-4 -x

Cân bằng:

x

0,1 + 3x

3

x.(0,1) = (3.1018 ) −1 vì x < 10-4 nên 3x << 0,1 −4 10 − x → x = [Fe3+] = 3,33.10-20 M →

Mà [OH-]A = 4,5.10-6M ⇒ [Fe3+].[OH-]3 = 3,0.10-36 > Ks = 2,5.10-39

⇒ có kết tủa Fe(OH)3 ở dung dịch B.


3. Phần mol của phức Fe(C2O4)33- trong dung dịch A. Phần mol của Fe(C2O4)33- được tính như sau:

x=

=

[Fe(C2 O4 )33- ] β3[Fe3+ ][C2O42- ]3 = CFe3+ [Fe3+ ] + β1[Fe3+ ][C2O2-4 ] + β2 [Fe3+ ][C2O2-4 ]2 + β3[Fe3+ ][C2O2-4 ]3

β3[C2O 42- ]3 1 + β1[C2 O 42- ] + β 2 [C2 O 2-4 ]2 + β3 [C2 O 42- ]3 =

3, 0.1018.(5, 0.10−5 )3 = 0, 43 1 + 1, 0.108.5, 0.10−5 + 2, 0.1014.(5, 0.10−5 ) 2 + 3, 0.1018.(5, 0.10−5 )3

Vậy phần mol của Fe(C2O4)33- bằng 0,43.

Câu 3: (2,0 điểm) Điện hóa học a • Xét pin 1: +

+ Dung dịch A: NH3: 0,05M; Ag(NH3) 2 : 0,005M.

Ag ( NH 3 )2+ ⇌ Ag + + 2 NH 3 0,005 0,005 – x

0,05 0,05 + 2x

x

(0,05 + 2 x)2 .x = 10−7,24 0,005 − x

⇒ x = 1,15.10−7 = [Ag+]A

2−

+ Dung dịch B: CrO 4 : 0,005M; Ag2CrO4

Ag 2CrO4 ⇌ 2 Ag + + CrO42− 2x

0,005 0,005 + x

⇒ (2 x)2 .(0,005 + x) = 10−11,89 ⇒ x = 8,02.10−6 = [Ag+]B Vì [Ag+]A < [Ag+]B nên trong pin 1: Ag/dd A là anot; Ag/dd B là catot Và E pin1 = Ec − Ea = 0,109(V ) Sơ đồ pin 1:

(−)Ag / NH3 0,05M

//

+

Ag(NH3) 2 0,005M Xét pin 2:

2−

CrO 4 0,105M / Ag (+) Ag2CrO4 −

+

+ Dung dịch NH4HSO4 0,01M. So sánh pK của HSO 4 và của NH 4 , suy ra, tính H+ theo cân bằng −

của HSO 4 ⇒ [H+](1) = 6,18.10-3 M + Dung dịch (NH4)2S 0,05M.

NH 4+ + S 2− → NH 3 + HS − 0,1 0,05 0,05 0 Xét các cân bằng:

0,05

0,05

K = 1012,9 - 9,24 >>

0,5


NH 4+ ⇌ NH 3 + H +

(1)

K=10−9,24

HS − ⇌ S 2− + H +

(2)

K=10−12,9

H 2O ⇌ OH − + H +

(3)

K=10−14

NH 3 + H 2O ⇌ NH 4+ + OH −

(4)

K=10−4,76

HS − + H 2O ⇌ H 2 S + OH −

(5)

K=10−6,98

So sánh (1), (2), (3) thì bỏ qua cân bằng (2), (3) So sánh (3), (4), (5) thì bỏ qua cân bằng (3), (5). Vậy cân bằng (1) và (4) quyết định pH của dung dịch, coi đây là hệ đệm +

Nồng độ của NH3 và NH 4 bằng nhau nên pH = 9,24 ⇒ [H+](2) = 10-9,24 M Vì [H+](1) > [H+](2) nên: Đ/c H2 nhúng trong dung dịch NH4HSO4 là catot Đ/c H2 nhúng trong dung dịch (NH4)2S là anot Và Epin2 = 0,416(V) Sơ đồ pin 2: +

(-) Pt(H2) / NH 4 0,05M

0,75

+

//

NH 4 0,01M / Pt(H2) (+) −

NH3 0,05M

HSO 4 0,01M

b

HS 0,05M Khi mắc xung đối, pin 2 sẽ là nguồn điện, pin 1 sẽ là bình điện phân( do Epin1 < Epin2) Vì vậy: + Đối với pin 2:

0,75

catot : 2 HSO4− + 2e → 2 SO42− + H 2 anot : H 2 + 2 NH 3 → 2 NH 4+ + 2e −

+

2−

Và quá trình phóng điện: HSO4 + NH 3 → NH 4 + SO4 + Đối với pin 1( là bình điện phân)

Catot : Ag ( NH 3 ) +2 + 1e → Ag + 2 NH 3 Anot : 2 Ag + CrO42− → Ag 2CrO4 + 2e +

Và quá trình nạp điện: 2 Ag ( NH 3 ) 2 + CrO4 → Ag 2CrO4 + 4 NH 3 (*) Quá trình (*) không tự diễn biến nên quá trình ngược lại sẽ là tự diễn biến. Vì vậy, có thể dùng NH3 để hòa tan kết tủa Ag2CrO4. Câu 4: (2,0 điểm) Nhóm N – P, nhóm C - Si

1

. Do X phản ứng được với Ca nên X là một phi kim. Trong dung dịch kiềm X hòa tan sinh ra một 0,1 muối tan và một khí. Nguyên tố x có mặt trong cả hai thành phần ấy. Trong hợp chất khí tồn tại liên kết X - H. Như vậy chỉ có thể có ba khả năng là Silan, Photphin và amoniac. X sinh ra khi cho than cốc tác dụng với muối C (Có chứa X) và SiO2 nên X chỉ có thể là photpho. Các phản ứng xảy ra như sau: 0,1x9 P4 + 3NaOH + 3H2O 3NaH2PO2 + PH3 P4 + 6Ca 2Ca3P2 2NaH2PO2 + 4CaOCl2 Ca3(PO4)2 + CaCl2 + 2NaCl + 4HCl 2Ca3(PO4)2 + 10C + 6SiO2 6CaSiO3 + 10CO+ P4 3Ca3(PO4)2 + 16Al 3Ca3P2 + 8Al2O3 Ca3P2 + 6HCl 3CaCl2 + PH3 2Ca3(PO4)2 + 6SiO2 6CaSiO3 + P4O10


P4O10 + 6H2O 4H3PO4 P4O10 + 12NaOH 4Na3PO4 + 6H2O

2

A: NOCl B: HCl C:HNO3 D:NO E:Cl2 F:HNO2 G:H2SO4 H:NOHSO4 I:Cu J:Cu(NO3)2 K:CNCl L:(CN)2 M:NO2 Tối đa được 1 điểm, thiếu một chất trừ 0,1đ. Làm được dưới 4 chất không được điểm nào

Câu 5: (2,0 điểm) Phức chất, trắc quang 1

0,75 a. CTTN: CrC4H8O5. b.Phức có dạng là [Cr(CH3COO)2(H2O)]. Do nhóm axetat có điện tích là 1- nên số oxi hóa của Cr ở đây là +2.n b c. Cr+2 có cấu hình e là d4. Đối với các phối tử trường yếu thì phức trên phải là phức spin cao. Tuy nhiên kết quả thu được hợp chất này nghịch từ vì nó tồn tại dưới dạng cấu trúc như sau:

2

a) A có CTTQ: M(CO)n Từ phương trình phản ứng: M(CO)n + 4KOH B + C + 2H2O C phải là muối K2CO3 B: K2[M(CO)n-1] mà [M(CO)n-1]2- có cấu trúc tứ diện n-1=4 hay n=5 Trong B: %C=19,512% M=56 (Fe) Vậy B là K2[Fe(CO)4] b) D là Fe2(CO)4(C5H5)2

3

Từ biểu thức định luật lambe-beer, ta có: A = -log T = -log(Imẫu/Icuvet trổng)=log(Rmẫu/Rcuvet trổng)=log(19,4/12,1)=0,205 [I3-]=A/b=0,205/(240000 M-1.cm-1)(1,1 cm)=7,76.10-7 M Số mol O3=Vmẫu.[I3-]=(0,01 L)(7,76.10-7 mol/L)=7,76.10-9 mol Số mol của mẫu không khí=PV/RT=P(tlấy mẫuF)/RT

0,25

0,5

0,5


=(750torr)(30min)(0,250L/min)/(62,4torr.L.mol-1.K-1)(298K)=0,302 mol Nồng độ O3 (ppb)=(7,76.10-9mol/0,302mol).109=25,7

Câu 6: (2,0 điểm): Quan hệ giữa hiệu ứng cấu trúc và tính chất 1

1,5

a) Hãy vẽ công thức các đồng phân lập thể ứng với cấu tạo A. 0,25 điểm: F

F

F

F

F F

F

F

F (A2)

(A1)

F F F

(A3)

(A4)

b) Ứng với công thức cấu tạo B có bao nhiêu đồng phân lập thể, vì sao? Dùng các kí hiệu thích hợp để chỉ rõ cấu hình của mỗi đồng phân đó. 0,25 điểm B có 3C bất đối, không có mặt phẳng và tâm đối xứng nên có 8 đồng phân lập thể. ví dụ: Cấu hình của B1 như chỉ ra trong bảng, viết gọn là (1R)-(2R)-(4R).

B1

B2

B3

B4

B5

B6

B7

B8

C1

R

S

S

R

S

R

R

S

C2

R

S

R

S

S

R

S

R

C4

R

S

R

S

R

S

R

S

Et

Me 1

O

2

O

3 4

5

Me

O Me

c) Hãy viết cơ chế phản ứng để giải thích vì sao C và D khi tương tác với dung dịch NaOH thì đều tạo thành natri 3-metyl-4-nitrobenzoat. 0,25 điểm: C , C1 , D và D1 là những đồng phân hỗ biến, xúc tác kiềm làm thuận lợi cho sự hỗ biến đó: HO

O

O

O

O

O

O

-

H2O/- OH

OH /- H2O

O

-

( C1 ) Me ( C)

Me NO2

Me

Me NO2

NO2

NO2 O

O

O

O

O

( D1 ) O

( D) O

OH

H2O/- OH-

Me

Me

Me NO2

NO2

Me NO2

NO2

0,25 điểm: Xuất phát từ C , C1 , D hoặc D1 qua phản ứng chuyển vị benzylic rồi tự mất nước đều chuyển thành hợp chất thơm bền vững, đều dẫn đến cùng một sản phẩm, ví dụ:


O

OH

O

O

O

O OH

OHMe (D)

Me

NO2

O

COOH

HO

COO-

COO H - H 2O

Me NO2

Me NO2

Me

Me NO 2

NO 2

NO 2

d) Hãy chỉ rõ trạng thái lai hóa của từng nguyên tử N ở cấu tạo E và ghi giá trị pKa ở 25 oC: 1,8; 6,0; 9,2 vào từng trung tâm axit trong công thức tương ứng với E, giải thích. 0,5 điểm: sp 2 N

6,0

COOH sp 3 N sp 2 H

COOH 1,8

H N

NH 2

N

(E)

NH 3 9,2

H 2

- Nguyên tử N nhóm NH ở trạng thái lai hóa sp , cặp e chưa chia ở obitan p xen phủ với 5 obitan p khác tạo thành hệ thơm được lợi về mặt năng lượng nhưng “mất” tính bazơ. - Nguyên tử N thứ hai ở trạng thái lai hóa sp2, cặp e chưa chia ở obitan sp2 không tham gia vào hệ thơm nên còn tính bazơ. - Nguyên tử N nhóm NH2 ở trạng thái lai hóa sp3.

+

- Nhóm NH3 là axit liên hợp của nhóm H2Nsp3 , nhóm NH+ là axit liên hợp của nhóm Nsp2. - Bazơ càng mạnh thì axit liên hợp càng yếu, vì thế giá trị 9,2 là thuộc nhóm NH3+ còn giá trị 6,0 thì thuộc nhóm NH+.

2

0,5 Khong tho'm: F, G, H, J

Tho'm: (I)

EtOOC EtOOC

+ 2K

O

EtOOC +K

COOEt

(K+)2

EtOOC

O K+

+ 1/ 2H2 COOEt

EtOOC EtOOC

O COOEt

EtOOC

O

EtOOC

COOEt

1 0 eπ 2

Câu 7: (2,0 điểm): Hidrocacbon 1

a.

b.

Mỗi ý 0,2 0,2x6 =1,2


C

+

H+

-H+ H

+

E

H

c.

H COOH COOH H F d.

Br

Br

H e.

_

Li+ J

Br

Br

Br

.. CHCl K

f.

N

Br


2

Br

G

I2

I1

H Br

Tổng 0,8. Sai một chất trừ 0,1

K Br

Br

Br Br

L1

L3

L2

X

Br

Câu 8: (2,0 điểm): Xác định cấu trúc Có 11 chất: sai 1 chất trừ 0,2 điểm. Không biện luận trừ 0,5 điểm A,B có k =6 và là một lacton vòng thơm, tan trong NaOH, không tác dụng với NaHCO3, phản ứng màu với FeCl3 nên có nhóm -OH phenol. A phản ứng với MeI/K2CO3 tạo C vậy C đã thêm một nhóm metyl nên OH phenol chuyển thảnh Ome. C có ba nhóm metyl không giống nhau, có một nhóm liên kết trực tiếp với vòng benzen. Phản ứng ozon phân H thu được nhiều sản phẩm trong đó có metyl-2-oxo-propanoat nên H là:

Xét phản ứng của E ta có các chất F,G, H như sau: OMe

OH

OMe

OMe

Me

Me

Me

Me

HO

HO

MeO

MeO

(E)

(F)

(G)

(H)


C loại một nhóm metyl tạo D, D có một nhóm hidroxi tạo liên kết hidro nội nên C có nhóm cacbonyl cạnh nhóm -Ome. Do đó C là (II) , P là (I), từ đó xác định được cấu tạo của B và D Metyl hóa B được P nên có hai cấu tạo B thỏa mãn là B1, B2

A là đồng phân của D nên A khác D ở vị trí nhóm hidroxi và metoxi, cấu tạo phù hợp A:

Câu 9: (2,0 điểm): Cơ chế phản ứng


1

0,5

2

1,5

Sản phẩm có tính bazo yếu hơn anilin vì cặp đôi electron tự do trên N bị giải tỏa nhiều hơn. Nó có khả năng phản ứng mạnh hơn piridin nhưng kém pirol. Phản ứng thế xảy ra như vị trí mũi tên:

Câu 10: (2,0 điểm): Tổng hợp các hợp chất hữu cơ 1 .


Giai đoạn chuyển từ B thành C là giai đoạn chuyển vị Johnson -Claisen và gỡ bỏ nhóm bảo vệ. Cơ chế giai đoạn chuyển D thành E là:

s

2


HỘI CÁC TRƯỜNG CHUYÊN VÙNG DUYÊN HẢI VÀ ĐỒNG BẰNG BẮC BỘ TRƯỜNG THPT CHUYÊN HOÀNG VĂN THỤ TỈNH HÒA BÌNH

ĐỀ THI MÔN HÓA HỌC - KHỐI 11 NĂM 2016- 2017 Thời gian làm bài: 180 phút (Đề này có 10 câu; gồm 04 trang)

ĐỀ THI ĐỀ XUẤT Câu 1. (2 điểm) Tốc độ phản ứng – Cân bằng hóa học 1. Tại 200C, phản ứng: H2 (khí) + Br2 (lỏng a) Tính Kp của phản ứng: H2(khí) + Br2(khí)

2 HBr (khí) (1) có hằng số cân bằng Kp= 9,0 .1016. 2 HBr(khí) (2) tại 20OC và áp suất p Br2 (k ) = 0, 25 atm.

b) Hãy cho biết sự chuyển dịch cân bằng hoá học của phản ứng (2) nếu giảm thể tích bình phản ứng ở hai trường hợp: - Trong bình không có Br2 (lỏng). - Trong bình có Br2 (lỏng). 2.Khi ®o vo cña ph¶n øng : I- + OClCl- + OI- diÔn ra ë 25oC . Trong dung dÞch cã pH cè ®Þnh vµ c¸c nång ®é I- , OCl- kh¸c nhau ng-êi ta thu ®-îc c¸c kÕt qu¶ sau : [I-]o

( 10-3 mol.l-1 ) -

-3

-1

-5

-1

[OCl ]o ( 10 mol.l )

1

1

1

1, 1

1,3

1

1,2

1,4

1

1

v ( 10 mol.l ) 6,1 7,3 8,5 6,7 7,9 Chøng minh r»ng c¬ chÕ sau ®©y phï hîp víi c¸c d÷ kiÖn thùc nghiÖm : k1 OCl + H2O HOCl + OH( nhanh ) k-1 k2 HOI + Cl( chËm ) HOCl + Ik3 HOI + OH H2O + IO( nhanh )

Câu 2. (2 điểm) Cân bằng trong dung dịch điện li 1. Axit photphoric là một axit ba chức. Nếu chuẩn độ một dung dịch H3PO4 0,1M với NaOH 0,1M. Tính pH tại các thời điểm sau: a) Giữa điểm bắt đầu và điểm tương đương thứ nhất. b) Tại điểm tương đương thứ hai. Cho K1 = 7,1.10-3 K2 = 6,2.10-8 K3 = 4,4.10-13 2. Một dung dịch chứa 530 milimol thiosunfat và một lượng chưa xác định kali iodua. Chuẩn độ dung dịch này với bạc nitrat, đã dùng 20 milimol bạc nitrat trước khi bắt đầu vẩn đục. Tính số mol KI. Biết thể tích phản ứng là 200ml. Ag(S2O3)23Ag+ + 2S2O32- (aq) Kd = 6.10-14 AgI Ag+ (aq) + I- (aq) Ksp = 8,5.10-17 Câu 3. (2 điểm) Điện hóa học Cho giản đồ Latime của các hợp chất của nitơ:


HNO2

1,00

NO

1,50

N2O

1,77

N2

-1,87

NH3OH+

?

NH4+

0,275

Trên các mũi tên là các thế khử chuẩn. Thí dụ, thế khử chuẩn của phản ứng HNO2 + H+ + e → NO + H2O , E0 = 1,00 V. Các hợp chất được xếp theo chiều tăng dần số oxi hóa của nitơ. 1. Điền giá trị của thế khử chuẩn vào vị trí có dấu chấm hỏi (?). 2. Trị số của thế khử chuẩn (tìm được ở trên) sẽ bằng bao nhiêu nếu dung dịch có pH = 3,00 và nhiệt độ 298o K. 3. Cân bằng phương trình phản ứng hình thành N2 từ HNO2 và NH4Cl, dung dịch có pH = 0 và tính hằng số cân bằng của phản ứng này ở 298o K. Câu 4. (2 điểm) Nhóm phi kim

1. Một nguyên tố X có thể tạo được nhiều oxit axit. Lấy muối natri của axit có chứa X phân tích thì thấy: Muối Na (%) X(%) O (%) 1 32,4 21,8 45,1 2 36,5 24,6 38,1 3 20,7 27,9 50,5 4 26,1 5,2 36,4 Xác định công thức phân tử, viết công thức cấu tạo của các muối trên. 2. Y cấu tạo nên từ hidro và nitơ. Khối lượng của 2,462 lít khí Y ở 27,30C và 0,5 atm bằng khối lượng của 1,68 lít oxi đo ở 54,60C và 0,8 atm. a. Xác định công thức phân tử và tên của Y. Viết công thức Lewis của Y. Cho biết trạng thái lai hóa của N trong Y? Từ đó mô tả cấu trúc không gian của Y. b. Thực nghiệm cho biết trong nước Y có khả năng điện li 2 nấc tạo thành dung dịch có tính bazơ. Giải thích tạo sao Y có tính bazơ. Viết các nấc điện li của Y? c. Viết phương trình phản ứng của Y tác dụng lần lượt với: dung dịch HCl, dung dịch kali hipoclorit, Dung dịch KMnO4 trong H2SO4 loãng. 3. Nêu hiện tượng và viết phương trình phản ứng xảy ra khi tiến hành các thí nghiệm sau:( không cần tính toán) a. Thêm dần dung dịch NaNO2 vào dung dịch I2 (trong dung dịch KI) cho đến dư. Thêm từng giọt dung dịch HCl vào dung dịch thu được cho đến khi dung dịch chuyển màu. b. Dung dịch A có chứa các ion S2-0,01M; Cl- 0,01M. Cho dung dịch AgNO3 từ từ vào dung dịch A, sau đó thêm tiếp KCN từ từ đến dư vào dung dịch thu được. Cho T (AgCl) = 10-10; T (Ag2S) = 10-19,7; hằng số bền của phức Kb ([Ag(CN)2]-) = 10-21,1. Câu 5. (2 điểm) Phức chất Axit A gồm 3 nguyên tố. Cho một thanh kim loại vào bình chứa 75,0 gam dung dịch nước của axit A 16,4% . Kết quả thu được 0,672 dm3 (đktc) khí hiđro và dung dịch chứa một muối. Khi thêm một lượng dư AgNO3 vào dung dịch thì có 25,81 gam kết tủa trắng tạo thành. Chất kết tủa này chứa 75,26% Ag về khối lượng. Không có sự thay đổi về khối lượng của thanh kim loại trong quá trình phân tích.

1. Hãy xác định kết tủa trắng và công thức axit A. 2. Thanh kim loại làm bằng kim loại gì? 3. Viết tất cả các phương trình phản ứng đã xảy ra. Câu 6. (2 điểm) Quan hệ cấu trúc – tính chất


1. Hợp chất H tham gia vào thành phần của một số alkaloid. Chất H được tổng hợp theo sơ đồ sau: C H3

O

O

O

xt

C H 2C F 3

o

A

t ,p

1 . L iA lH 4 2 . H 2O

B

1 . O sO 4 2 . N aIO 4

C (C 9 H 1 4 O 4 )

C H3

D

PCC

E

1. C sO H 2 . M e O H /H + pH = 4

I , Ph P

2 3 F im id a z o l (C 1 1 H 1 8 O 4 )

G

P h 3P = C H 2

1 . B H 3 .T H F 2 . H 2O 2, N a O H 3. Z n/ A cO H

D

H

Xác đinh công thức cấu tạo của các hợp chất từ A đến H. Biết C không chứa vòng 6 cạnh.

2. Metyl da cam là chất chỉ thị màu axit-bazơ có công thức: 1

(H3C)2N

2

3

N N

SO3Na

So sánh tính bazơ của các nguyên tử N? Giải thích. .

Câu 7. (2 điểm) Hiđrocacbon Tecpenoit lµ c¸c hîp chÊt cã m¹ch cacbon gåm c¸c ®¬n vÞ isopren liªn kÕt ®Çu-®u«i víi nhau. Geraniol vµ nerol ®Òu lµ tecpenoit vµ lµ hai ®ång ph©n lËp thÓ cña rîu bËc mét cã hai liªn kÕt ®«i trong ph©n tö. Khi cã mÆt axit, geraniol khÐp vßng t¹o thµnh -tecpineol, cßn nerol còng khÐp vßng cho cïng s¶n phÈm nhng x¶y ra nhanh h¬n nhiÒu. 1. ViÕt c«ng thøc cÊu tróc, tªn IUPAC cña nerol, geraniol vµ gi¶i thÝch v× sao nerol dÔ khÐp vßng h¬n? 2 .-tecpineol khi läai níc t¹o thµnh tecpinolen. X¸c ®Þnh cÊu tróc cña tecpinolen 3 .Trong m«i trêng axit, tecpinolen ®ång ph©n hãa t¹o thµnh ba ®ång ph©n dien m¹ch mét vßng gäi chung lµ tecpinen. Trong sè nµy, chØ cã s¶n phÈm chÝnh lµ -tecpinen cho ph¶n øng Diels-Alder. Tuy nhiªn c¶ ba ®ång ph©n t¸c dông víi hidro clorua d ë nhiÖt ®é thÊp cho cïng mét s¶n phÈm. a.ViÕt c«ng thøc cÊu tróc vµ tªn IUPAC cña -tecpinen. b.ViÕt ph¬ng tr×nh ph¶n øng cña -tecpinen víi anhidrit maleic (anhidrit butendioic) c.ViÕt ph¬ng tr×nh ph¶n øng cña -tecpinen víi anhidrit maleic (anhidrit butendioic) Câu 8. (2 điểm) Xác định cấu trúc 1) Hợp chất A( C7H10O4) không tác dụng với H2/Pd, to. A bị thủy phân trong môi trường axit đun nóng cho B (C4H8O2), A tác dụng với LiAlH4, sau đó thủy phân trong môi trường H+ thu được C (C5H10O3), C bị oxi hóa bởi K2Cr2O7/ H2SO4 thu được D (C5H6O5). Trong môi trường H+/to, D chuyển thành E (C3H6O), C tác dụng với H2/Ni cho F không quang hoạt. Xác định công thức cấu tạo từ A-F 2)Hợp chất A (C11H17NO3) không quang hoạt, không tan trong môi trường trung tính và kiềm nhưng dễ tan trong môi trường axit loãng. A có hai nguyên tử H linh động, A phản ứng với Ac2O tạo B(C13H19NO4) trung tính. A phản ứng với MeI dư sau đó thêm AgOH, sản phẩm thu được C có công thức là C14H25NO4. Đun nóng chất này thu được Me3N và D(C11H14O3) trung tính. D phản ứng với O3 thu được HCHO và E . Andehit thơm E phản ứng HI tạo sản phẩm chứa 3 nhóm –OH mà chúng không tạo được liên kết hidro nội phân tử bền vững. a) Xác định các chất chưa biết b) Từ E và các hợp chất vô cơ, hãy điều chế chất A Câu 9. (2 điểm) Cơ chế


1.Enamin có thể được tạo thành khi cho anđehit hoặc xeton phản ứng với amin bậc hai có xúc tác axit. Xiclohexanon phản ứng với piroliđin tạo ra enamin H theo sơ đồ sau: H N

+

H

+

N

H

O

Đề xuất cơ chế giải thích quá trình tạo thành enamin H. 2. Viết công thức cấu tạo của A và trình bày cơ chế của hai giai đoạn phản ứng.

b.

A

to 3,3

B

CCl4

tBuONa (C6H6)

C

FeCl3

D

KOH EtOH

Cl

HOOC

Cl

E

Viết công thức cấu tạo của B, C, D và trình bày cơ chế phản ứng B → C và C → D.

Câu 10. (2 điểm) Tổng hợp hữu cơ 1. Bupivacain (C18H28N2O) là amit của axit 1-butylpiperiđin-2-cacboxylic với 2,6-đimetylanilin ở dạng S được dùng làm thuốc gây tê cục bộ. Tổng hợp (S)-bupivacain từ 2-metylpiriđin và các hóa chất cần thiết khác. 2. Cho s¬ ®å chuyÓn ho¸ c¸c chÊt sau:

Cl O

(AlCl3)

+

- HCl

H2SO4

A

B -HO 2 HO

HOO O X

+

Y

X

O

+ Cl

Cl (MnO4- )

X

D1 D2

(A, B, X, Y, D1 , D2 lµ c¸c hîp chÊt h÷u c¬) ViÕt c«ng thøc cÊu t¹o cña A, B, X, Y, D1 , D2


HỘI CÁC TRƯỜNG CHUYÊN VÙNG DUYÊN HẢI VÀ ĐỒNG BẰNG BẮC BỘ TRƯỜNG THPT CHUYÊN HOÀNG VĂN THỤ TỈNH HÒA BÌNH

ĐỀ THI MÔN HÓA HỌC - KHỐI 11 NĂM 2016- 2017 Thời gian làm bài: 180 phút (Đề này có 10 câu; gồm 04 trang)

ĐÁP ÁNĐỀ THI ĐỀ XUẤT Câu 1. (2 điểm) Tốc độ phản ứng – Cân bằng hóa học Hướng dẫn

Câu1 1a.

Đ iể m

a) Phản ứngH2 (k) + Br2 (lỏng) ⇌ 2 HBr (k)

(1) có Kp1 =

(a) còn phản ứng: H2 (k)+ Br2 (k) ⇌ 2 HBr (k)

(2)

Kp 2 =

(3) có Kp3 = PBr2 ( k ) (c)

Khi tổ hợp (1) với (3) ta có cân bằng (2): H2 (k)+ Br2 (lỏng) ⇌ 2 HBr (k) Br2 (l) ⇌ Br2 (k) (1) – (3): H2 (k)+ Br2 (k) ⇌ 2 HBr (k)

(1) (3) (2)

Kp1 9, 0.1016 = = 3, 6.1017 (atm) Kp3 0, 25

Khi giảm thể tích bình phản ứng nghĩa là tăng áp suất riêng phần của khí trong hệ. Xét Q =

2b.

0,25

2 PHBr (b) PH2 .PBr2

Xét cân bằng Br2 (lỏng) ⇌ Br2 (k)

Kp 2 =

2 PHBr PH2

2 PHBr PH2 .PBr2

0,25

0,25

Trường hợp 1: Không có brom lỏng trong bình: Phản ứng (2) có tổng số mol khí trước và sau phản ứng bằng nhau (∆n = 0) nên sự thay đổi áp suất 0,25 đó không dẫn tới chuyển dịch cân bằng (2). Trường hợp 2: Có brom lỏng trong bình: áp suất riêng phần của các khí H2, HBr tăng; trong lúc đó áp suất riêng phần của Br2 khí lại không đổi do còn Br2 lỏng. Theo (d), vì số mũ của PHBr lớn hơn số mũ của PH2 nên sự tăng áp suất nói trên dẫn đến sự tăng Q và cân bằng (2) chuyển dịch theo chiều nghịch. Gi¶ sö ph¶n øng cã bËc : v = k . [I-]x . [OCl-]y XÐt 3 thÝ nghiÖm ®Çu ta cã : v = k . [1]x . [OCl-]y = k .[OCl-]y , gi¶ sö y=1 Víi thÝ nghiÖm 1 : k1 =6,1 . 10-2 s-1 Víi thÝ nghiÖm 2 : k2 =6,08 . 10-2 s-1 Víi thÝ nghiÖm 3 : k3 =6,07 . 10-2 s-1 XÐt 2 thÝ nghiÖm sau : v = k . [I-]x . [1]y = k . [I-]x , gi¶ sö x = 1 Víi thÝ nghiÖm 4 : k4 =6,09 . 10-2 s-1 Víi thÝ nghiÖm 5 : k5 =6,08 . 10-2 s-1 0,5


Ta thÊy : k1 ~ k2 ~ k3 ~ k4 ~ k5 , gi¶ thiÕt lµ ®óng . VËy , ph-¬ng tr×nh ®éng häc lµ : v = k . [I-] . [OCl-] XÐt c¬ chÕ , giai ®o¹n chËm quyÕt ®Þnh tèc ®é ph¶n øng :

v = k2. [HOCl].[I-] =

k1 . k2 .[I-].[OCl-].[H2O]

k-1 . [OH-] Do pH cè ®Þnh nªn [OH ] cè ®Þnh , dung dÞch rÊt lo·ng , H2O kh«ng tham gia vµo ph-¬ng tr×nh ph¶n øng , [H2O] = 1000 / 18 = 55,3 VËy , v = k . [I-] . [OCl-] víi : k1 . k2 .[H2O] k = k-1 . [OH-] Câu 2. (2 điểm) Cân bằng trong dung dịch điện li -

Câu 2 1

Hướng dẫn a) Tại điểm bắt đầu và điểm tương đương thứ nhất ta có dung dịch đệm H3PO4 và H2PO4- và [H3PO4] = [H2PO4-] H3PO4 H+ + H2PO4[ H 3 PO4 ] [H+] = K1 → [H+] = K1 = 7,1.10-3 → pH = 2,15 − [ H 2 PO4 ] b) Tại điểm tương đương thứ hai có HPO42H2PO4H+ + HPO42- K2 = 6,2.10-8 (1) HPO42H+ + PO43- K3 = 4,4.10-13 (2) Tổ hợp (1) và (2) → [H+] =

2

K 2 .K 3 = 6,2.10 −8.4,4.10 −13 = 1,7.10-10

0,25

Đ iể m

0,5

0,5

→pH = 9,77. Ag(S2O3)23Kd-1 = 1,667.1013 Ta có: Ag+ + 2S2O32- (aq) → 20 milimol 20 → 2.20 Ta thấy hằng số tạo phức rất lớn nên hầu hết Ag+ thêm vào sẽ tạo phức với S2O32-. [Ag(S2O3)23-] = 20/200 = 0,1M → Số mol S2O32- tự do là: 530 – 2.20 = 490 mmol 0,5 [S2O32-] = 490/200 = 2,45M + Tính nồng độ Ag tự do: Ag(S2O3)23Ag+ + 2S2O32- (aq) Kd = 6.10-14 Kd =

[ Ag + ].[S 2 O32− ]2 = 6.10-14 →[Ag+] = 10-15 3− [ Ag ( S 2 O3 ) 2 ]

Khi bắt đầu vẩn đục ta có: Ag+ + I- → AgI (r ) → [I-] =

K sp

[ Ag + ]

=

8,5.10 −17 = 8,5.10-2M −15 10

→ số mol của KI = 8,5.10-2. 200 = 17 mmol

0,5

Câu 3. (2 điểm) Điện hóa học Câu3

Hướng dẫn

Điểm


1.

E 0 (NH3OH + /NH +4 ) = n1 = 1

n2 = 3

n 2 . E 0 (N 2 /NH +4 ) - n1 . E 0 (N 2 /NH3OH + ) n3

0,5

n3 = 2

E 0 (NH 3OH + /NH +4 ) =

3 . 0,275 V - (-1,87 V) = 1,35 V 2

2. NH3OH+ + 2e + 2H+ → NH4+ + H2O RT 8,314 J / K . mol . 298 K E = E0 ln (H + )-2 = 1,35 V ln(1,0 . 10-3 )-2 = 1,18 V nF 2 . 96,5 . 104 C/mol 3. HNO2 + NH4Cl → N2 + 2 H2O + HCl HNO2 + 3 H+ + 3e → 1/2 N2 + H2O ∆G01 NH4+ → 1/2 N2 + 4 H+ + 3 e ∆G02 HNO2 + NH4Cl → N2 + 2 H2O + HCl ∆G0 = ∆G01 + ∆G02 ∆G01 = (-n1E0(HNO2 / NO) – n2E0(NO / N2O) – n3E0( N2O / N2)) . F n1 = n2 = n3 = 1 ∆G01 = (- 1,00 V – 1,50 V – 1,70 V) . 9,65 . 104 C/mol = -405,3 kJ/mol ∆G02 = nFE0(N2 / NH4+) n=3 0 4 ∆G 2 = 3 . 9,65 . 10 C/mol . 0,275 V = 79,6 kJ/mol ∆G0 = ∆G01 + ∆G02 = -405,3 kJ/mol + 79,6 kJ/mol = -326 kJ/mol

K=e

- ∆G 0 RT

=e

- (-326.103 J/mol) 8,314 J/K.mol . 298K

1,0

= 1,4.1057

Câu 4. (2 điểm) Nhóm phi kim Câu14 1.

Hướng dẫn

. Các muối đều có: %mNa+ %mX + %mO < 100% muối có H. - Muối 1: Gọi SOX của X là a. BT SOX: 1.32,4/23 + a.21,8/X + (-2).45,1/16 + 1.0,7/1 = 0 Mà a = 1; 2; 3; 4; 5; 6; 7 chỉ a = 5 X = 31: P thỏa mãn. Muối 1: Na : P : O : H = 2:1:4:1 Na2HPO4.

Đ iể m

0,125

0,125 -

Muối 2: Na2HPO3. 0,125

-

Muối 3: Na2H2P2O7

0,5


O

ONa P OH

-

0,125

HO

O P

ONa

O

Muối 4: NaH2PO2. NaO

H

P

O

H

2

a.MX = 32. Gọi X là NxHy N2H4: hidrazin. H H N

N H H

N: lai hóa sp3. tứ diện ghép đôi: N(1) là tâm của tứ diện gồm các đỉnh là H(1); H(2); N(2) và có đôi e chưa liên kết của N(1). N(2) là tâm tứ diện gồm các đỉnh H(3); H(4); N(1) và có đôi e chưa liên kết của N(2). b. Mỗi nguyên tử N trong hidrazin còn một đôi e chưa liên kết do đó có khả năng nhận proton tính bazơ và phân li theo 2 nấc: N2H4 + H2O ⇄ N2H5+ + OH-. N2H5+ + H2O ⇄ N2H62+ + OH-. c. N2H4 + HCl N2H5Cl hay N2H4 + 2HCl N2H6Cl2. N2H4 + 2KClO 2KCl + N2 + 2H2O 5N2H4 + 4KMnO4 + 6H2SO4 4MnSO4 + 2K2SO4 + 5N2 + 16H2O.

0,5

0,5

0,5 3.

a. Dung dịch I2 (trong KI) có màu nâu, khi thêm NaNO2 dung dịch mất màu do: NO2- + I3- + H2O NO3- + 3I- + 2H+. Thêm HCl dung dịch chuyển lại màu nâu đồng thời có khí không màu thoát ra sau đó lại hóa nâu trong không khí do: NO2- + H+ HNO2. 2HNO2 + 2I- + 2H+ 2NO + I2 + 2H2O. NO + ½ O2 NO2. b. Có kết tủa màu đen xuất hiện trước, sau đó đến kết tủa màu trắng. 2Ag+ + S2- Ag2S. Ag+ + Cl- AgCl.

0,25


Khi thêm KCN vào AgCl tan tạo phức, còn lại màu đen do Ag2S không tan trong KCN: AgCl + 2KCN K[Ag(CN)2] + KCl

0,25

Câu 5. (2 điểm) Phức chất Câu1 1a.

Hướng dẫn

Đ iể m

1. AgnX

%(Ag)=

n × A r (Ag) = 0,7526 n × A r (Ag) + A r (X)

=>

A r (X) = 35,46n

Kết quả thích hợp là : n = 1 và X : Cl. Kết tủa trắng là AgCl. Axit A chứa H, Cl và nguyên tố chưa biết Y.

N(Cl) = N(AgCl) = 25,81 g ×

1 mol = 0,18 mol 143,3 g

N(H) = 2N(H 2 ) = 2 × 0,672 dm 3 ×

1 mol = 0,06 mol 22,4 dm 3

Vì N(H) : N(Cl) = 1 : 3, A : HxYyCl3x Phản ứng tổng quát : m HxYyCl3x + 3x M = mx/2 H2 + 3x MClm + my Y

M(A) = 75 g × 0,164 × Nếu x = 1, A r (Y) = Nếu y = 1, Y : Tc,

x = 205x g/mol 0,06 mol

1,0

205 - 1,008 - 3 × 35,45 97,642 = y y Nếu y = 2, Y : Ti,

...

Cả hai kết quả đều thích hợp với giả thiết Nếu x = 2, A r (Y) = Nếu y = 1, Y : Pt,

2 × 97,642 195,284 = y y Nếu y = 2, Y : Tc,

Tc, ... A : H2[PtCl6] (Kết quả khác không phù hợp với đề bài).

2b.

2.

6M + mH2[PtCl6] = 6MClm + mH2↑ + mPt↓

Nếu y = 3, Y :


Vì không có sự thay đổi khối lượng của thanh kim loại:

m(M) = m(Pt) =

1 195,08 g × 0,06 mol × = 5,8524 g 2 1 mol 1 = 32,51m g/mol 6/m×0,03 mol

M(M) = 5,8524 g × Nếu m = 2, M : Zn 3.

0,5

(Giá trị khác không phù hợp với đề bài)

3Zn + H2[PtCl6] = 3ZnCl2 + H2↑ + Pt↓

0,5

ZnCl2 + 2AgNO3 = Zn(NO3)2 + 2AgCl

Câu 6. (2 điểm) Quan hệ cấu trúc – tính chất Câu16 1

Hướng dẫn

Me

O

Me O

O

OH

C H 2C F 3

O

O

Me

Me

A

B

Điểm

O O

CHO

HO

O HO

C

OH

D

I

1,0

O O COOM e

O O E

2.

O COOMe

O COOMe

F

OH

H

G

Cặp electron không liên kết của N(1) có hiệu ứng +C vào vòng benzen nên N(1) có mật độ electron thấp hơn so với 2 nguyên tử nitơ còn lại. Trong các công thức cộng hưởng ta nhận thấy N(3) có mật độ electron cao nhất do điện tích âm được nằm trên nó trong khi N(2) thì không. N(3) có mật độ điện tích âm cao nhất, nó dễ dàng nhận H+ nhất so với 2 nguyên tử nitơ còn lại. Do đó N(3) có tính bazơ mạnh nhất. (hay hs có thể vẽ hiệu ứng (A, E)) O

O N

N N

1

S ONa O

N

N N

A

N

B

N N

E

S ONa O

O S ONa O

N

O S ONa O

N N

D

1,0


Câu 7. (2 điểm) Hiđrocacbon Câu1 7 1

Hướng dẫn

3 2

HO

2

Đ iể m

3

HO

Nerol (Z)-3,7-dimetylocta-2,6-dien-1-ol

Geraniol (E)-3,7-dimetylocta-2,6-dien-1-ol

Nerol cã cÊu h×nh Z t¹i liªn kÕt ®«i 2,3 nªn c¸c nguyªn tö tham gia ph¶n øng khÐp vßng ë gÇn nhau h¬n lµm cho ph¶n øng diÔn ra nhanh:

0,75

+ 1. HO

H+

2. -H2O

- H+ +

OH2 + OH2

nerol hoÆc

α-tecpineol

2.

0,25 -H2O

tecpinolen OH

3.

a. Trong m«i trêng axit, tecpinolen ®ång ph©n hãa t¹o thµnh ba ®ång ph©n dien m¹ch mét vßng gäi chung lµ tecpinen. Trong sè nµy, chØ cã s¶n phÈm chÝnh lµ tecpinen cho ph¶n øng Diels-Alder. Tuy nhiªn c¶ ba ®ång ph©n t¸c dông víi hidro clorua d ë nhiÖt ®é thÊp cho cïng mét s¶n phÈm. ViÕt c«ng thøc cÊu tróc vµ tªn IUPAC cña -tecpinen V× -tecpinen tham gia ph¶n øng Diels-Alder nªn ph¶i lµ mét dien liªn hîp. C«ng thøc cÊu t¹o phï hîp cña -tecpinen: 0,5

α-tecpinen

0,25 b. ViÕt ph¬ng tr×nh ph¶n øng cña -tecpinen víi anhidrit maleic (anhidrit butendioic)


O +

O O

0,25

O

O

O

α-tecpinen

c. ViÕt c«ng thøc cÊu tróc cña hai tecpinen cßn l¹i?

Câu 8. (2 điểm) Xác định cấu trúc Câu8 1

Hướng dẫn O

O A

2b.

O

OH

OH OH

O

O

B

C

Đ iể m 1,0

COOHCOOH

O O

O

D

E

OH F

a. E là andehit thơm , E phản ứng HI thu được sản phẩm có 3 nhóm –OH không tạo liên kết hidro nôi phân tử nên:

C nhiệt phân tạo D nên C có hai công thức là:

A không quang hoạt nên C không quang hoạt nên công thức cấu tạo của C là 1 Vậy A và B là:

0,5


0,5

b. Sơ đồ tổng hợp A từ E ( coi E là R-CHO) Câu 9. (2 điểm) Cơ chế Câu 9 1

Hướng dẫn

Đ iể m 1,0

H

O

O +H

..

HN

HO NH

+

H2O N

N

-H2O

H

+

-H3O

H2O

2

3

b. 3 2

EtO

O 1

2

to

CCl4

1

3,3

FeCl3

A

EtO

B

Cl EtO

O

C

tBuONa (C6H6)

CCl3

1,0

KOH

Cl CCl3

O

N

EtOOC

CCl3 D

EtOH

HOOC

Cl E

Cl

Câu 10. (2 điểm) Tổng hợp hữu cơ Câu 10 1

Hướng dẫn

Đ iể m

1.

1,0


N

CH3

H2O

SOCl2

H2/Ni

KMnO4 N

N

COOH

COOH

H

H

N+

COCl

H (S)-Bupivacain

NH2 H3C

CH3

H

C4H9Br

N

N H

H

CH3 piridin

O CH3

CH3

N

N

Tách 2 ®èi quang b»ng axit (+)-tactric

C4H9 O CH3

2b.

H CH2

CH2 CH

O A

OH

H

B

X 1,0

OH

HO H

H H

H

OH

HO

D1

D2


HDC ĐỀ THI MÔN HÓA HỌC – KHỐI 11 NĂM 2017 Thời gian làm bài: 180 phút (HDC này có 13 trang, gồm 10 câu)

HỘI CÁC TRƯỜNG CHUYÊN VÙNG DUYÊN HẢI VÀ ĐỒNG BẰNG BẮC BỘ TRƯỜNG THPT CHUYÊN LÀO CAI

ĐỀ THI ĐỀ XUẤT Câu 1(2 điểm): Tốc độ phản ứng – Cân bằng hóa học 1. Để nghiên cứu động học của phản ứng

2[Fe(CN)6]3− + 2I−

2[Fe(CN)6]4− + I2 (*).

Người ta đo tốc độ đầu của sự hình thành iot ở 4 hỗn hợp dưới đây. Các hỗn hợp ban đầu không chứa iot. c([Fe(CN)6]3−) mol/L

c(I−) mol/L

c([Fe(CN)6]4−) mol/L

Tốc độ đầu mmol.L−1. h−1

Thí nghiệm 1

Hỗn hợp 1

1

1

1

1

Thí nghiệm 2

Hỗn hợp 2

2

1

1

4

Thí nghiệm 3

Hỗn hợp 3

1

2

2

2

Thí nghiệm 4

Hỗn hợp 4

2

2

1

16

Trong trường hợp tổng quát, tốc độ phản ứng được biểu thị bởi phương trình:

dc(I 2 ) = k.ca([Fe(CN)6]3−).cb(I−).cd([Fe(CN)6]4−).ce(I2) dt Xác định giá trị của a, b, d, e và hằng số tốc độ phản ứng k. 2. Cơ chế sau đây đã được đề xuất cho phản ứng (*):

[Fe(CN)6]3− + 2 I−

4− k1 [Fe(CN) + I2− 6]

[Fe(CN)6]3− + I2−

[Fe(CN)6]4− + I2

k−1

k2

(1)

(2)

a. Trong 2 phản ứng trên, phản ứng nào diễn ra nhanh, phản ứng nào diễn ra chậm? b. Chứng minh rằng cơ chế trên phù hợp với phương trình biểu diễn tốc độ phản ứng tìm được ở 1.


HDC 1.

Từ thí nghiệm 1 và 2

a=2

Từ thí nghiệm 1 và 3

d = -1

Từ thí nghiệm 1 và 4

b=2 e=0 0,5

dc(I 2 ) = k.c2([Fe(CN)6]3−).c2(I−).c−1([Fe(CN)6]4−).c0(I2) dt Thí nghiệm 1: 1.10−3 mol.L−1. h−1 = k . 1 mol2.L−2.1 mol2.L−2.(1 mol.L−1) −1 K = 1.10−3 mol−2.L2. h−1 2.

k1

[Fe(CN)6]3− + 2 I−

[Fe(CN)6]4− + I2−

[Fe(CN)6]3− + I2−

4− 2 [Fe(CN) + I2 6]

k−1

k

(1) 0,5 (2)

(1) diễn ra nhanh; (2) diễn ra chậm. Nếu chấp nhận như vậy thì chứng minh được rằng cơ chế đề xuất phù hợp với định luật tốc độ rút ra từ thực nghiệm.

0,5

Chứng minh: Từ cân bằng (1) được xác lập rất nhanh:

c([Fe(CN)6 ] ) × c(I 2 ) k1 = k -1 c([Fe(CN)6 ]3- ) × c2 (I - ) 4-

0,5

-

k1 c([Fe(CN)6 ] ) × c (I ) × k -1 c([Fe(CN)6 ]4- ) 3-

c(I2−) =

2

-

(a)

Phản ứng (2) diễn ra chậm:

dc(I ) 2 = k2 × c([Fe(CN)6]3−) × c(I2−) (b) dt Thay (a) vào (b) ta được:

dc(I ) c 2 ([Fe(CN)6 ]3- ) × c 2 (I- ) 2 = k1 × k2 × c([Fe(CN)6 ]4- ) k -1 dt Kết quả này phù hợp với kết quả thực nghiệm. Nếu phản ứng (1) là chậm, phản ứng (2) nhanh:

dc(I ) 2 = k2.c([Fe(CN)6]3−).c(I2−) và c(I2−) = k1.c([Fe(CN)6]3−).c2(I−) dt dc(I ) 2 = k1.k2.c2([Fe(CN)6]3−).c2(I−). Kết quả này không phù hợp với kết quả thực nghiệm. dt


Câu 2(2 điểm): Cân bằng trong dung dịch điện li Axit HIn trong nước phân ly như sau: HIn (màu 1) H+ + In− (màu 2) Đo mật độ quang của dung dịch HIn 5,00.10−4 M trong NaOH 0,1 M và trong HCl 0,1 M ở bước sóng 485 nm và 625 nm với cuvet 1,00 cm. Trong dung dịch NaOH 0,1 M A485 = 0,052 A625 = 0,823 A625 = 0,176 Trong dung dịch HCl 0,1 M A485 = 0,454 1. Tính hệ số hấp thụ mol của In− và HIn ở bước sóng 485 và 625 nm. 2. Tính hằng số phân ly axit của HIn, nếu trong dung dịch đệm pH 5,00 chứa một lượng nhỏ chất chỉ thị có mật độ quang là 0,472 ở 485 nm và 0,351 ở 625 nm 3. Tính pH của một dung dịch chứa một lượng nhỏ chất chỉ thị. Biết mật độ quang là 0,530 ở 485 nm và 0,216 ở 625 nm. 4. Mật độ quang của một dung dịch HIn 2,00.10−4 M tại 485 và 625 nm (cuvet 1,25 cm) là bao nhiêu nếu dung dịch được đệm ở pH bằng 6,00? 5. Chuẩn độ 25,00 mL dung dịch axit hữu cơ tinh khiết yếu HX với chất chỉ thị là phenolphtalein tới điểm cuối chuẩn độ thì cần 24,20 mL dung dịch NaOH chuẩn. Khi thêm đúng 12,10 mL dung dịch NaOH vào 25,00 mL dung dịch axit HX, trong dung dịch có một lượng nhỏ chất chỉ thị HIn, mật độ quang đo được là 0,306 ở 485 nm và 0,555 ở 625 nm (cuvet 1,00 cm). Tính pH của dung dịch và Ka cho axit yếu. HDC Trong dung dịch NaOH, chất chỉ thị chỉ tồn tại chủ yếu ở dạng In−; Ttrong dung dịch axit, chất chỉ thị chỉ tồn tại chủ yếu ở dạng HIn−. Theo định luật Beer: 1. Vì cuvet đều có bề dày dd là 1cm nên A = εi.[i]. In In − Trong dung dịch NaOH: A485 = ε 485 .C = 0,052 → ε 485 = 104 In In A625 = ε 625 .C = 0,823 → ε 625 = 1646

− Trong dung dịch HCl:

HIn HIn A485 = ε 485 .C = 0,454 → ε 485 = 908

A625 = ε

HIn 625

.C = 0,176 → ε

HIn 625

0,25

= 352

2. Ở pH = 5. HIn In .[HIn] + ε 485 .[In−] = 908. [HIn] + 104 .[In−] = 0,472 A485 = ε 485 HIn In A625 = ε 625 .[HIn] + ε 625 .[In−] = 352. [HIn] +1645 .[In−] = 0,351 Giải hệ phương trình tìm được: [HIn] = 5,078.10−4 [In−] = 1,047.10−4. 1, 047.10−4.10−5 → Ka = = 2,06. 10−5 5, 078.10−4 3. Tính pH: HIn In A485 = ε 485 .[HIn] + ε 485 .[In−] = 908. [HIn] + 104 .[In−] = 0,530 HIn In A625 = ε 625 .[HIn] + ε 625 .[In−] = 352. [HIn] +1645 .[In−] = 0,216 Giải hệ phương trình tìm được: [HIn] = 5,83.10−4 [In−] = 6,57.10−6. 5,83.10−4.2, 06.10−5 → [H+] = = 1,82. 10−3 → pH = 2,738 −6 6,57.10 4. Ở pH = 6,000 10−6 [HIn] = 2.10−4 . −6 = 9,26 . 10−6 (M) 10 + 10−4,686 10−4,686 [In−] = 2.10−4 . −6 = 1,9074 . 10−4 (M) −4,686 10 + 10 HIn In A485 = ε 485 .[HIn] + ε 485 .[In−] = 908. 9,26 .10−6 + 104 . 1,9074 . 10−4 = 0,028

0,25

0,5


HIn In A625 = ε 625 .[HIn] + ε 625 .[In−] = 352. 9,26 .10−6 +1645. 1,9074 . 10−4 = 0,330 5. − Chuẩn độ tới đổi màu phenolphtalein cần 24,20 mL dung dịch bazơ → khi thêm 12,10 mL thì dung dịch chứa HX và X− với hai lượng tương đương. [HX ] → Áp dụng công thức có thể suy ra pKa: pKa = pH + lg − = pH. [X ] 0,5 − − HIn In A485 = ε 485 .[HIn] + ε 485 .[In ] = 908. [HIn] + 104 .[In ] = 0,306 HIn In A625 = ε 625 .[HIn] + ε 625 .[In−] = 352. [HIn] +1645 .[In−] = 0,555 Giải hệ phương trình tìm được: [HIn] = 3,06.10−4 [In−] = 2,72.10−4. 3, 06.10−4.2, 06.10−5 → [H+] = = 2,3175. 10−3 → pH = 4,635 → pKa (HX) = 4,635. 2, 72.10−4

Câu 3(2 điểm): Điện hóa học Cho giản đồ quá trình khử - thế khử: quá trình khử diễn ra theo chiều mũi tên, thế khử chuẩn được ghi trên các mũi tên và đo ở pH = 0. 1. Tính E 0x và E 0y . 2. Dựa vào tính toán, cho biết Cr(IV) có thể dị phân thành Cr3+ và Cr(VI) được không? 3. Viết quá trình xảy ra với hệ oxi hóa – khử Cr2 O72- /Cr3+ và tính độ biến thiên thế của hệ ở nhiệt độ 298 K, khi pH tăng 1 đơn vị pH. 4. Phản ứng giữa K2Cr2O7 với H2O2 trong môi trường axit (loãng) được dùng để nhận biết crom vì sản phẩm tạo thành có màu xanh. Viết phương trình ion của phản ứng xảy ra và cho biết phản ứng này có thuộc loại phản ứng oxi hóa – khử hay không? Vì sao? Ghi số oxi hóa tương ứng trên mỗi nguyên tố. 0 Cho: E Cr O2- /Cr3+ = 1,33 V; Hằng số khí R = 8,3145 J.K–1.mol–1; Hằng số Farađay F = 96485 C.mol–1. 2 7

HDC 1. Từ giản đồ ta có: 3.(-0,744) = -0,408 + 2 E 0y → E 0y = -0,912 (V) 0,55 + 1,34 + E 0x – 3.0,744 = 6.0,293 → E 0x = +2,1 (V) 2. Cr(IV) có thể dị phân thành Cr3+ và Cr(VI) khi ∆G0 của quá trình < 0. 2Cr(IV) + 2 e → 2Cr3+ (1) E10 = E 0x = 2,1 V → ∆G10 = -n E10 F = - 2.2,1.F 0, 55 + 1, 34 Cr(VI) + 2 e → Cr(IV) (2) E 02 = = 0,945 (V) → ∆G 02 = -n E 02 F = 2 2.0,945.F Từ (1) và (2) ta có: 3Cr(IV) → 2Cr3+ + Cr(VI) ∆G 30 ∆G 30 = ∆G10 - ∆G 02 = - 2.(2,1 - 0,945).F < 0 → Vậy Cr(IV) có dị phân.

3.

Cr2 O72- + 14H+ + 6e E1 = 1,33 +

0,5

0,5

2Cr3+ + 7H2O

RT [Cr2 O 2-7 ].(10-pH )14 ln 6.F [Cr 3+ ]2

RT [Cr2 O72- ].(10-(pH + 1) )14 ln 0,5 6.F [Cr 3+ ]2 8,3145 . 298 b. Độ biến thiên của thế: E 2 - E1 = .14ln10-1 = -0,138 (V). 6 . 96485 4. +6 -2 +1 -1 +1 +6,-2/-1 +1 -2 0,5 + 2Cr2 O7 + 4H2O2 + 2H → 2CrO5 + 5H2O Phản ứng trên không phải là phản ứng oxi hóa-khử vì số oxi hóa của các nguyên tố không thay đổi trong quá trình phản ứng. Trong CrO5, số oxi hóa của crom là +6 và của oxi là -2, -1 do peoxit CrO5 có cấu trúc: E 2 = 1,33 +


Câu 4(2 điểm): Nhóm N – P, nhóm C – Si 1. Sục khí (A) vào dung dịch (B)) có màu nâu vàng thu được chất rắn (C)) màu vàng và dung dịch (D). Khí (X) có màu vàng lục tác dụng với khí (A A) tạo ra (C) và (F). Nếu (X) tác dụng vớii khí (A) ( trong nước tạo ra (Y) và (F), rồi thêm BaCl2 vào dung dịịch thì có kết tủa trắng. Khí (A) tác dụng vớii dung dịch d chất (G) là muối nitrat kim loại tạo ra kết tủa (H)) màu đen. Đốt cháy (H) bởi oxi ta được chất lỏng ng (I) ( màu trắng bạc. Xác định A, B, C, F, G, H, I, X, Y và viết vi phương trình hóa học của các phản ứng. 2. Cho một dung dịch X chứaa các ion: Na+, Cl-, Br-, CO 23 − , HCO3− , SO32 − , SO 24− , NO3− . Trình bày phương pháp hóa học để nhận biết từng ng ion trong dung ddịch X. HDC 1. A: H2S; B: FeCl3; F: HCl; G: Hg(NO3)2; I: Hg; X: Cl2; Phương trình hóa học của các phản ứng ng : H2S + 2FeCl3 → 2FeCl2 + S + 2HCl Cl2 + H2S → S + 2HCl 4Cl2 + H2S + 4H2O → 8HCl + H2SO4

C: S; H: HgS; Y: H2SO4 (1) (2) (3)

BaCl2 + H2SO4 → BaSO4 ↓ + 2HCl

(4)

H2S + Hg(NO3)2 → HgS ↓ + 2HNO3

(5)

0

HgS + O2 → Hg + SO2 2. Sơ đồ nhận biết: t

(6)

0,5

0

daây Pt, t

löû a maø u vaønng

Na+

1,0

(X) + (CH3COO)2Ba dö

(Na+, Cl-, Br-, HCO3-, NO3-, Ba2+, CH3COO-) (BaCO3, BaSO3, BaSO4) + CH3COOH dö (Na+, Cl-, Br-,NO3-, Ba2+, CH3COOH,, CH3COO-) CO2 + Cu +

-

-

2+

2+

-

(Na , Cl , Br ,Ba , Cu , CH3COOH, CH3C COO ) NO +AgNO O3 dö NO 2 (AgCl, AgBr) NO3+NH3 ddö

HCO3-

BaSO4

+ HCl dö (CO2, SO2) dd Br2 dö

SO42(SO2: mm) SO32-

CO2 +Ca(OH)2 dö keát tuûa

CO32[Ag(NH3)2]Cl +HNO3 AgCl: traéng -

Cl

A AgBr: vaø ng BBr

0,5


Phương trình phản ứng xảy ra: CO 23 − + Ba2+ → BaCO3 SO32 − + Ba2+ → BaSO3 SO 24− + Ba2+ → BaSO4

BaCO3 + 2HCl → BaCl2 + CO2 + H2O BaSO3 + 2HCl → BaCl2 + SO2 + H2O BaSO4 + 2HCl → không phản ứng SO2 + Br2 + 2H2O → 2HBr + H2SO4 CO2 + Ca(OH)2 → CaCO3 + H2O CH3COOH + NaHCO3 → CH3COONa + CO2 + H2O 3Cu + 2NaNO3 + 8CH3COOH → 3(CH3COO)2Cu + 2NO + 2CH3COONa + 4H2O 2NO + O2 → 2NO2 NaBr + AgNO3 → AgBr + NaNO3 NaCl + AgNO3 → AgCl + NaNO3 AgCl + 2NH3 → [Ag(NH3)2]Cl [Ag(NH3)2]Cl + 2HNO3 → AgCl + 2NH4NO3 Câu 5(2 điểm): Phức chất 1. Người ta đã tổng hợp được [NiSe4]2- , [ZnSe4]2- và xác định được rằng phức chất của Ni có dạng hình vuông phẳng, của Zn có dạng hình tứ diện đều. Hãy đưa ra một cấu tạo hợp lí cho mỗi trường hợp trên và giải thích. 2. Phức chất [PtCl2(NH3)2] được xác định là đồng phân trans-. Nó phản ứng chậm với Ag2O cho phức chất [PtCl2(NH3)2(OH2)2]2+(kí hiệu là X). Phức chất X không phản ứng được với etylenđiamin (en) khi tỉ lệ mol phức chất X : en = 1 : 1. Hãy giải thích các sự kiện trên và vẽ (viết) cấu tạo của phức chất X. 3. [Ru(SCN)2(CN)4]4– là ion phức của ruteni, được kí hiệu là P. a. Viết công thức Lewis của phối tử thioxianat SCN–. b. Cho biết dạng lai hóa của Ru trong P. Mô tả sự hình thành ion phức theo thuyết VB (Valence Bond). Giải thích tại sao trong P, liên kết được hình thành giữa Ru và N của phối tử SCN– mà không phải là giữa Ru và S. Cho biết phức có tính thuận từ hay nghịch từ, vì sao? HDC 1. Niken có mức oxi hoá phổ biến nhất là +2; kẽm cũng có mức oxi hoá phổ biến nhất là +2. Selen có tính chất giống lưu huỳnh do đó có khả năng tạo thành ion polyselenua Se 22− hay [-Se— Se-]2-. Cấu tạo vuông phẳng của phức chất [NiSe4]2- là do cấu hình electron của ion Ni2+ cho phép sự lai hoá dsp2. Cấu tạo tứ diện đều của phức chất [ZnSe4]2- là do cấu hình electron của Zn2+ cho phép sự lai hoá 0,5 sp3. Tổng hợp của các yếu tố trên cho phép đưa ra cấu tạo sau đây của 2 phức chất: Se

Se

Se

Ni Se

Zn Se

Se

Se

Se

trong đó ion điselenua đóng vai trò phối tử 2 càng. 2. [PtCl2(NH3)2] (1) là đồng phân trans- đòi hỏi phức chất phải có cấu tạo vuông phẳng: Cl │


H3N—Pt—NH3 │ Cl

(1)

Phản ứng của (1) với Ag2O: Trans-[PtCl2(NH3)2] + Ag2O + H2O → Trans-[PtCl2(NH3)2(H2O)2]2+ + 2OH- Etylenđiamin là phối tử hai càng mạch ngắn. Khi phối trí với các ion kim loại nó chỉ chiếm 2 vị trí phối trí cạnh nhau (vị trí cis). Hiện tượng en không thể phản ứng với 0,5 [PtCl2(NH3)2(H2O)2]2+ theo phản ứng: 2+ 2+ [PtCl2(NH3)2(H2O)2] + en → [PtCl2(NH3)2(H2O)2en] + 2H2O chứng tỏ rằng 2 phân tử H2O nằm ở 2 vị trí trans đối với nhau. Như vậy công thức cấu tạo của phức chất phải là: -

H2O Cl NH3

Pt

NH3

Cl

0,5

H2O

3a. Tổng số electron để xây dựng công thức Lewis cho SCN– là 6 + 4 + 5 + 1 = 16. Công thức Lewis của SCN– là:

S C N 0 0 3b. Ru có cấu hình electron [Kr]4d 5s 5p , là ion trung tâm trong phức bát diện. Vì CN– là phối tử trường mạnh nên ở phân lớp 4d6 của Ru2+ có sự ghép đôi tất cả các electron, tạo ra 2 AO 4d trống. Do đó xảy ra sự lai hóa d2sp3 để tạo 6AO lai hóa hướng tới 6 đỉnh của 1 hình bát diện. Các phối tử (L) sử dụng cặp electron tự do của nguyên tử N gửi vào các 0,5 obitan lai hóa đó để tạo các liên kết cho nhận giữa phối tử và ion Ru2+. 2+

6

d2sp3 [Ru(SCN)2(CN)4]4-

4d6

L

5s ...

5p

L

So với S, N có độ âm điện lớn hơn và bán kính nguyên tử nhỏ hơn, do đó mật độ điện tích âm trên nguyên tử N sẽ lớn hơn, ái lực phản ứng với ion dương Ru2+ lớn hơn, vì vậy trong phức chất P, liên kết phức được hình thành giữa Ru và N mà không phải là giữa Ru và S. Phức P có tính nghịch từ vì trong ion phức không có electron độc thân.

Câu 6(2 điểm): Quan hệ cấu trúc – tính chất 1. Hãy sắp xếp các chất sau theo chiều tăng dần tính bazơ, giải thích ? NH2

O 2N

H 2N

NO 2

NH2

NH2

NH 2

NH2

O N

2 (I) (II) (III) (IV)CN (V) NO 2 (VI) CN NO 2 2. Cho 4 axit: CH3CH2COOH (A); CH3COCOOH (B); CH3COCH2COOH (C) + CH3CH( NH3)COOH (D). Sắp xếp A, B, C, D theo trình tự tăng dần tính axit. Giải thích. HDC

NO 2


1. Sắp xếp: I < VI < V < IV < III < II 0,5 Giải thích: Tính bazơ của N càng giảm khi có mặt các nhóm có hiệu ứng –C càng mạnh. Hiệu ứng –C của NO2 > CN. - Xiclopentadienyl chỉ có hiệu ứng –I. –I làm giảm tính bazơ kém hơn –C - Các hợp chất I và VI đều có 2 nhóm NO2 ở vị trí meta so với nhóm NH2 gây ra hiệu ứng 0,5 không gian làm cản trở sự liên hợp–C của nhóm NO2ở vị trí para nhiều hơn nhóm CN ở vị trí para. Do đó hiệu ứng – C nhóm CN ở vị trí 4 > nhóm NO2 ở vị trí 4 0,5 2. Tính axit tăng dần: A < C < B < D Giải thích: O

O O CH3CH2

C O

+I

CH3 H

C

CH2

O

O

CH3 CH

C H

NH3

O

H

-I m¹nh

-I

A

0,5

C

B

C

D

Câu 7(2 điểm): Hiđrocacbon 1. Các nhà khoa học đầu những năm 1900 cho rằng chỉ cần có hệ liên hợp trong một vòng kín thì phân tử đó có tính thơm.Xiclooctatetraen (A) đã làm sụp đổ nhận định này. Nó lần đầu tiên được điều chế bởi nhà hóa học người Đức Richard Willstater năm 1911. a. Vẽ công thức của xiclooctatetraen, nó có phẳng không? Nếu không, hãy vẽ các dạng tồn tại của nó. b. Xiclooctatetraen tác dụng với 2 đương lượng Kali tạo ra dianion. Viết cấu tạo của dianion này và cho biết tính thơm của nó? Dianion này phản ứng với 2 mol axeton cho 2 sản phẩm C và D. Cho biết công thức cấu tạo của C và D. c. Xiclooctatetraen phản ứng với m-CPBA tạo ra chất E,xử lí E với axit Brontest thu được chất G (C8H8O) có phản ứng với thuốc thử Tollens. Xác định công thức cấu tạo các chất chưa biết. d. Hoàn thành dãy phản ứng sau biết K là chất trung gian.

2. Một ankin D quang hoạt chứa 89,52% C. Hợp chất D có thể bị hidro hóa/xúc tác tạo n- butylxiclohexan. Xử lí D với EtMgBr không thấy thoát khí. Hidro hóa D trên xúc tác Pd/C tronng quinolin( chất đầu độc xúc tác) và xử lí sản phẩm với O3/H2O2 cho axit tricacboxylic quang hoạt E (C8H12O6). Chất E khi đun nóng tách một phân tử nước và tạo F. Viết công thức cấu tạo các chất chất HDC 1.a.

Xiclooctatetraen không phẳng và tồn tại ở 2 dạng: ghế và thuyền. b. Dianion thơm do thỏa mãn các quy tắc Hulkel: các e thảo mãn quy tắc e = 4n + 2, phẳng, có hệ e liên hợp kín. c. Cơ chế.

0,25 0,25


0,5

0,5

OH

H

OH

CHO

0,5 G

d.

2. Câu 8(2 điểm): Xác định cấu trúc

HDC

1. Xác định cấu tạo các chất và hoàn thành sơ đồ phản ứng trên? 2. Viết cơ chế từ C D?


COOMe

HN COOH

COOH

S

S

NH

A

N

COOMe

COOt-Bu

S

N

N

COOt-Bu

N

N3

COOMe

S

E

N

F

CHO

O

NH

S

N

H

0,5

N

H H

COOt-Bu

0,5 COOt-Bu

D Cl3CH2COOC

H S

S

COOt-Bu

O COOMe

COOMe

C

B

HO

N

MeOOC

COOt-Bu

H S

N

CHO

COOt-Bu

0,5

I

0,5

Cơ chế từ C sang D:

Câu 9(2 điểm): Cơ chế

1.

2.


3.

4. HDC

1. 0,5

0,5

2.

0,5

3. 0,5


4.

Câu 10(2 điểm): Tổng hợp hữu cơ 1. Từ β-metylnaptalen điều chế A :

2.Từ phenol và các chất hữu cơ khác hãy tổng hợp thuốc giảm đau Profofol B( được cho là thủ phạm gây ra cái chết của vua nhạc pop Michael Jackson):

HDC O

O

PCC t

1.

O

1,O3

0

2,H2O2 O


O

EtONa EtOH

O

1,0

COOEt

O

O

NH2OH

H+

NOH

1,0 LiAlH4 NH

NH

2. OH

O 1,NaOH

OH

O

t0

1,NaOH

2,Crotyl clorua

2,Crotyl clorua

OH 1,RCO3H 2,H 3O+

OHC

CHO NaBH4

SP

3,NaIO4 *********************************

HẾT **********************************

TRƯỜNG ĐẠI HỌC SƯ PHẠM HÀ NỘI TRƯỜNG THPT CHUYÊN ĐHSP ĐỀ NGUỒN KÌ THI HỌC SINH GIỎI VÙNG DUYÊN HẢI VÀ ĐỒNG BẰNG BẮC BỘ NĂM 2017 MÔN THI: HÓA HỌC LỚP 11 Thời gian làm bài: 180 phút (không kể thời gian phát đề) Câu 1: (2 điểm) Động học (Cơ chế phản ứng)-Cân bằng hóa học Tốc độ phản ứng thủy phân trong môi trường axit:


không phụ thuộc [H+] được mô tả bởi phương trình động học:v =

có hằng số

tốc độ: k = 2,4.10-4 (250C) và k = 8,1.10-4 (350C) đối với phức chất dạng cis; k = 3,5.10-4 (250C) và k = 1,5.10-4 (350C) đối với phức chất dạng trans. a. Mô tả sự thủy phân Co(en)2Cl22+ bằng cơ chế 2 giai đoạn ( sử dụng các hằng số tốc độ k1, k-1, k2) và chứng minh cơ chế đưa ra là phù hợp với thực nghiệm v= b. Vẽ các cấu trúc có thể có của tiểu phân trung gian trong cơ chế của Trans-Co(en)2Cl22+. c. Tính năng lượng hoạt hóa EA cho cis- và trans- Co(en)2Cl2+. Tính entanpi ∆H#, entropi ∆S# cho sự hình thành phức hoạt động của cis- Co(en)2Cl22+.

Hướng dẫn giải a. Cơ chế

Áp dụng phương trình nồng độ dừng cho Co(en)2Cl22+ ta có →

Từ đó rút ra v =

, Nếu

thì v=

Học sinh có thể làm theo cách hằng số cân bằng vẫn cho điểm tối đa. b. Cấu trúc của tiểu phân trung gian của Trans

c. Năng lượng hoạt hóa Đồng phân Trans: Đồng phân Cis: ;

(đpcm)


Câu 2: (2 điểm) Cân bằng dung dịch điện li Dung dịch X gồm Na2S 0,010M, KI 0,060M, Na2SO4 0,050M. a.Tính pH của dung dịch X. b. Thêm dần Pb(NO3)2 vào dung dịch X cho đến nồng độ 0,090M thì thu được kết tủa A và dung dịch B, không kể sự thuỷ phân của các ion, coi thể tích dung dịch không thay đổi khi thêm Pb(NO3)2. i. Cho biết thành phần hoá học của kết tủa A và dung dịch B. ii. Tính nồng độ các ion trong dung dịch B. iii. Nhận biết các chất có trong kết tủa A bằng phương pháp hoá học, viết các phương trình phản ứng (nếu có). c.Axit hoá chậm dung dịch X đến pH = 0, coi thể tích dung dịch không thay đổi. Thêm FeCl3 cho đến nồng độ 0,10M. i. Tính thế của cực platin nhúng trong dung dịch thu được so với cực calomen bão hoà (Hg2Cl2/2Hg,2Cl-). ii. Biểu diễn sơ đồ pin, viết phương trình phản ứng xảy ra tại các điện cực và phản ứng tổng quát khi pin hoạt động. Cho : axit có H2S pK1 = 7,00, pK2 = 12,90; HSO4- có pKa = 2,00; Tích số tan của Ks(PbS) = 10-26 ; Ks(PbSO4) = 10-7,8 ; Ks(PbI2) = 10-7,6. Eo Fe3+/Fe2+ = 0,77 V ; Eo S/H2S = 0,14V ; Eo I2/2I- = 0,54V ; Ecal bão hoà = 0,244V

Hướng dẫn giải a) Tính pH của dung dịch

Na2S → 2 Na+ + S20,01

0,01

KI → K+ + I0,06 Na2SO4

0,06

2Na+ + SO42-

0,05 S2-

HS- + OH-

+ H2O

0,05

SO42- + H2O ⇌ H SO4- + OH-

Kb(1) = 10-1,1

(1)

Kb(2) = 10-12

(2)

Kb(1) >> Kb(2) nên cân bằng (1) quyết định pH của dung dịch: S2-

[]

H2O ⇌

+

HS- + OH-

K = 10-1,1

x2 = 10 −1,1 → x 2 + 0,0794x − 10 −3,1 = 0 0,01 − x (0,01 -x) x x

→x

= 8,94. 10-3

b)

Pb2+

+

→ S2-

[OH-] = 8,94.10-3

PbS ↓

pH = 11,95

(Ks-1) = 1026.


0,09

0,01

0,08 Pb2+

SO42-

+

0,08

PbSO4 ↓

(Ks-1) = 107,8.

PbI2

(Ks-1) = 107,6.

0,05

0,03 Pb2+

2 I-

+

0,03

0,06

Thành phần hỗn hợp: ↓A :

PbS , PbSO4 , PbI2

Dung dịch B :

K+ 0,06M

Na+ 0,12M

Ngoài ra còn có các ion Pb2+ ; SO42- ; S2- do kết tủa tan ra. PbI 2 : 3 10 −7, 6 / 4 = 10 −2,7

PbSO 4 : S = 10

-7,8

= 10 −3,9

PbS : S = 10 -26 = 10 −13

Độ tan của Bởi vì độ tan của PbI2 là lớn nhất nên cân bằng chủ yếu trong dung dịch là cân bằng tan của PbI2.

= Pb2+ +

PbI2↓

2I-

Ks

Do đó [Pb2+] = 10-47 = 2 x 10-3M và

[I-] = 4.10-3M.

10−7,8 [SO42-] =

=

5. 10−5,8 = 7,9.10−6M << [Pb2+]

=

5. 10−24 << [Pb2+]

2 × 10−3 10−26 [S2-] = 2 × 10−3 Các nồng độ SO42-, S2- đều rất bé so với nồng độ Pb2+, như vậy nồng độ Pb2+ do PbS và PbSO4 tan ra là không đáng kể nên cách giải gần đúng trên là hoàn toàn chính xác.

− Nhận biết các chất có trong kết tủa A: PbS; PbSO4; PbI2. Cho kết tủa hoà tan trong NaOH dư : ↓ PbS không tan, có màu đen. Dung dịch có PbO22-, SO42-, I-, OHPbSO4 + 4 OHPbI2

+

4 OH-

→ →

PbO22- + SO42- + 2 H2O

PbO22- + 2 I-

+

2 H2 O

Nhận ra ion SO42-: cho BaCl2 dư: có kết tủa trắng BaSO4, trong dung dịch có PbO22-, OH-, Ba2+, I-. Nhận ra I-, Pb2+: axit hoá dung dịch bằng HNO3 dư sẽ có kết tủa vàng PbI2 xuất hiện: OH- + H+

→ H2O

PbO22- + 4 H+ → Pb2+ + 2H2O

Pb2+

+ 2 I- → PbI2↓

c) Axit hoá dung dịch X: S2- + 2H+ → H2S (C H2S = 0,010 < S H2S nên H2S chưa bão hoà, không thoát ra khỏi dung dich)


Phản ứng:

2 Fe3+ + H2S → 2 Fe2+ + 0,1

0,01

0,08

0,02

2 Fe3+ +

2I- →

2 Fe2+

0,08

0,06

0,02

0,02

0,08

S + 2 H+

K=1021

0,02 +

K=107,8

I2 0,030

Thành phần trong dung dịch: Fe3+ 0,020 ; Fe2+ 0,080 ;I2 0,030M ;H+ 0,02M E Fe3+/Fe2+ = 0,77 + 0,059 lg 0,02/0,08 = 0,743V (cực dương) Ecal = 0,244V ( cực âm) Epin = E+ − E− = 0,743 − 0,244 = 0,499V Sơ đồ pin: Hg Hg2Cl2 Phản ứng: −

2 Hg

+ 2x Fe

3+

2 Hg + 2 Fe

Fe3+, Fe2+

KCl bh 2 Cl-

+ + 3+

= Hg2Cl2

+ 2e

2+

e +

Pt

= Fe -

2 Cl =

Hg2Cl2↓

Câu 3: (2 điểm) Điện hóa học Ắc quy Chì-Axit được sử dụng khá phổ biến trong xe hơi, phản ứng khi phóng điện là

Sự phụ thuộc của thế khử chuẩn E0 vào nhiệt độ được biểu diễn bằng đồ thị dưới đây

a. Viết sơ đồ pin khi ắc quy phóng điện. b. Tính ∆G0, ∆H0, ∆S0 của phản ứng trên ở 298K. c. Tính thế điện cực E của phản ứng ở 50C tại pH=5 và nồng độ ion sunfat là 0,10M. Hướng dẫn giải


a. Sơ đồ pin: Catot (+) PbO2(r) │H2SO4(dd)│Pb(r) (-) Anot b.

,

tại 298K có Eo = 2,05 (V).

; c. E = Eo – (RT/nF)lnQ = 1,64 (V) + (0,00138V/K).278(K) Thay số: E = 1,42 (V) Câu 4: (2 điểm) N-P, C-Si và hợp chất Hydro mới sinh là một tác nhân khử có hiệu quả nhất. Xử lý một lượng natri nitrit bằng hỗn hống natri kim loại cho ra một muối X có 43,38% natri và 26,43% nitơ về khối lượng. Một sản phẩm khác của phản ứng này là natri hydroxit. Để tránh sự làm bẩn sản phẩm cuối này thì quá trình tổng hợp được tiến hành trong khí quyển trơ như môi trường nitơ hay argon. a) Xác định công thức muối X. b) Vẽ công thức Lewis anion của muối X. c) Nếu phản ứng được tiến hành trong không khí thì những tạp chất nào có thể sinh ra? d) Viết phương trình tổng hợp muối X. e) Tương tác giữa X với cacbon dioxit sinh ra một chất khí. Viết phương trình phản ứng. Hướng dẫn giải a) Na2N2O2 – natri hyponitrit. b) Theo lý thuyết thì anion N2O 22− thể có đồng phân cis, trans. Thực nghiệm đã chứng minh được rằng ion này chủ yếu ở dạng trans. O N

O

N

O

N

N

O

N O

N

N O

O

N O

c) Sản phẩm có thể chứa tạp chất NaNO3, NaNO2, Na2CO3, NaHCO3. d) 2NaNO2 + 4Na + 2H2O → Na2N2O2 + 4NaOH e) Na2N2O2 + CO2 → Na2CO3 + N2O Câu 5: (2 điểm) phức chất, trắc quang a) Phức chất A là phức chất của Pt(II), có cấu trúc vuông phẳng, chứa Pt, NH3 và Cl với % khối lượng lần lượt là 65,00%; 11,33% và 23,67%. Viết công thức cấu trúc của phức chất thỏa mãn điều kiện trên, biết phân tử khối của A nhỏ hơn 1000.


b) Khi cho phᝊc chẼt cis-[Pt(NH3)2Cl2] (cẼu trĂşc vuĂ´ng pháşłng) tĆ°ĆĄng tĂĄc váť›i tetrametylen Ä‘iamin (tetraen) ngĆ°áť?i ta tĂĄch ra Ä‘ưᝣc máť™t phᝊc chẼt 2 nhân B cᝧa Pt(II) cĂł cĂ´ng thᝊc [Pt2Cl4(NH3)2(tetraen)] (B). Káşżt quả tháť­ nghiᝇm cho thẼy chẼt B cĂł khả năng khĂĄng táşż bĂ o ung thĆ° cao. Viáşżt cĂ´ng thᝊc cẼu tấo, mĂ´ tả sáťą hĂŹnh thĂ nh liĂŞn káşżt (theo thuyáşżt VB) trong phᝊc chẼt B. c) Khi nghiĂŞn cᝊu cĆĄ cháşż phản ᝊng tháşż pháť‘i táť­ trong phᝊc chẼt vuĂ´ng pháşłng ngĆ°áť?i ta nháş­n thẼy ráşąng táť‘c Ä‘áť™ thay tháşż pháť‘i táť­ X báť&#x;i máť™t pháť‘i táť­ Y cháť‹u ảnh hĆ°áť&#x;ng cᝧa pháť‘i táť­ Z (Z náşąm áť&#x; váť‹ trĂ­ trans so váť›i X). Náşżu Z cĂ ng hoất Ä‘áť™ng vᝠảnh hĆ°áť&#x;ng trans thĂŹ X cĂ ng linh Ä‘áť™ng vĂ cĂ ng dáť… báť‹ tháşż. L

Z Pt L

L

Z

+ Y

+

Pt

X

X

L Y Tᝍ K2[PtCl4], viáşżt sĆĄ Ä‘áť“ Ä‘iáť u cháşż phᝊc chẼt cis vĂ phᝊc chẼt trans -[Pt(C 2H4)(2-ampy)Cl2] (trong Ä‘Ăł 2-ampy

lĂ 2-aminopiriÄ‘in). Biáşżt thᝊ táťą ảnh hĆ°áť&#x;ng trans cᝧa cĂĄc pháť‘i táť­: C2H4 > Cl- > 2-ampy. Trong phᝊc chẼt trĂŞn, nguyĂŞn táť­ kim loấi trung tâm Pt liĂŞn káşżt váť›i pháť‘i táť­ 2-aminopiriÄ‘in qua nguyĂŞn táť­ nitĆĄ nĂ o? Giải thĂ­ch.

Cho biết: CẼu hÏnh electron cᝧa Pt: [Xe]4f145d96s1

HĆ°áť›ng dẍn giải a) Tᝉ lᝇ Pt : NH3 : Cl =

,

âˆś

, ,

âˆś

,

≈ 1 âˆś 2 âˆś 2

CĂĄc phᝊc chẼt A cĂł cĂ´ng thᝊc phân táť­ lĂ [Pt(NH3)2Cl2]n. (M = 300n) VĂŹ kháť‘i lưᝣng phân táť­ cᝧa phᝊc chẼt nĂ y nháť? hĆĄn 1000 u nĂŞn n = 1 hoạc n = 2 hoạc n = 3. - Náşżu n = 1: cĂ´ng thᝊc phân táť­ lĂ [Pt(NH3)2Cl2], cĂł 2 Ä‘áť“ng phân cis vĂ trans. NH3

Cl

NH3

Cl

Pt

Pt

Cl

H3N

NH3

Cl

- Náşżu n = 2: cĂ´ng thᝊc phân táť­ lĂ Pt2(NH3)4Cl4, cĂł 2 Ä‘áť“ng phân: [Pt(NH3)4][PtCl4]; cis trans [Pt(NH3)3Cl][PtCl3(NH3)]. 2+ NH3

H3N

;

Pt NH3

Cl

Cl

Cl

H3N

Cl

Cl

Pt H3N

+

2-

Pt H3N

- Náşżu n = 3: cĂ´ng thᝊc phân táť­ lĂ Pt3Cl6(NH3)6: cĂł 2 Ä‘áť“ng phân: [Pt(NH3)4][PtCl3(NH3)]2; [Pt(NH3)3Cl]2[PtCl4]. (váş˝ tĆ°ĆĄng táťą nhĆ° trĂŞn). b) cis-[Pt(NH3)2Cl2] + H2N-(CH2)4-NH2 CẼu trĂşc báť n cᝧa B:

-

[Pt2Cl4(NH3)2(tetraen)] (B)

NH3

Cl Pt

NH3

Cl

Cl


NH3

Cl

Cl

H3N

Pt

Pt

Cl

NH2-CH2CH2CH2CH2-H2N

Cl

-VĂŹ chẼt Ä‘ầu cĂł cẼu hĂŹnh cis nĂŞn trong phᝊc chẼt B, 2 nguyĂŞn táť­ Cl cĹŠng áť&#x; váť‹ trĂ­ cis, phân táť­ tetrametilen

Ä‘iamin lĂ m cầu náť‘i giᝯa 2 nguyĂŞn táť­ trung tâm Pt. -MĂ´ tả sáťą hĂŹnh thĂ nh liĂŞn káşżt (theo thuyáşżt VB) trong phᝊc chẼt B: Pt2+ [Xe]5d8. lai hĂła dsp2

lai hĂła dsp2 5d

6s

Cl-

NH3 NH2-CH2-CH2-CH2-CH2-NH2 Cl-

Cl-

Cl-

6s

6p

6p

5d

NH3

c) SĆĄ Ä‘áť“ phản ᝊng Ä‘iáť u cháşż phᝊc chẼt cis:

K2[PtCl4] K[Pt(2-ampy)Cl3] cis-[Pt(C2H4)(2-ampy)Cl2] 2-

-

Cl

Cl

+ 2-ampy

Pt Cl

+ C2H4

Pt

Cl

Cl

C2H4

Cl

Cl

Cl

Pt Cl

Am

Am

VĂŹ Cl- cĂł ảnh hĆ°áť&#x;ng trans mấnh hĆĄn 2-ampy nĂŞn nguyĂŞn táť­ Cl áť&#x; váť‹ trĂ­ trans váť›i Am khĂł báť‹ tháşż, phản ᝊng tháşż tiáşżp theo C2H4 tháşż vĂ o nguyĂŞn táť­ Cl áť&#x; váť‹ trĂ­ cis so váť›i Am tấo ra Ä‘áť“ng phân cis. - SĆĄ Ä‘áť“ phản ᝊng Ä‘iáť u cháşż phᝊc chẼt trans:

K2[PtCl4] K[Pt(2-ampy)Cl3] trans-[Pt(C2H4)(2-ampy)Cl2] 2-

-

Cl

Cl

Cl

+ C2H4

Pt

Cl

Am

Cl

+ 2-ampy

Pt

Pt

Cl Cl C H VĂŹ C2H4 cĂł ảnhClhĆ°áť&#x;ng trans mấnh hĆĄn Cl- nĂŞn nguyĂŞn táť­CCl Cl 2H4áť&#x; váť‹ trĂ­ trans so váť›i C2H4 dáť…2báť‹4 tháşż, phản ᝊng

tháşż tiáşżp theo 2-ampy tháşż vĂ o nguyĂŞn táť­ Cl áť&#x; váť‹ trĂ­ trans so váť›i C2H4 tấo ra Ä‘áť“ng phân trans. Trong phᝊc chẼt trĂŞn, nguyĂŞn táť­ kim loấi trung tâm Pt liĂŞn káşżt váť›i pháť‘i táť­ 2-aminopiriÄ‘in qua nguyĂŞn táť­ N cᝧa vòng piriÄ‘in mĂ khĂ´ng liĂŞn káşżt qua nguyĂŞn táť­ N cᝧa nhĂłm NH2 vĂŹ cạp electron chĆ°a liĂŞn káşżt trĂŞn nguyĂŞn táť­ N cᝧa nhĂłm NH2 Ä‘ĂŁ báť‹ vòng piriÄ‘in hĂşt nĂŞn máş­t Ä‘áť™ electron giảm, khĂł cĂł khả năng tấo phᝊc. Còn cạp electron chĆ°a liĂŞn káşżt trĂŞn nguyĂŞn táť­ N cᝧa vòng piriÄ‘in (náşąm trĂŞn obitan lai hĂła sp2), máş­t Ä‘áť™ electron láť›n hĆĄn nĂŞn dáť… tấo liĂŞn káşżt pháť‘i trĂ­ váť›i nguyĂŞn táť­ kim loấi trung tâm.

Câu 6: (2 Ä‘iáťƒm) Quan hᝇ giᝯa hiᝇu ᝊng cẼu trĂşc vĂ tĂ­nh chẼt a. Sắp xáşżp cĂĄc chẼt sau Ä‘ây theo chiáť u tăng dần láťąc axit, giải thĂ­ch: O

O

O

O

O H

O

O OEt

O Cl

O NO 2

O CF3


b. Hãy so sánh tính bazơ của các nguyên tử N trong mỗi phân tử các hợp chất sau: O N H

N

O N

N

N N

N N CH3

O NH

N(CH3)2

COOH

Hướng dẫn giải a HD: Giá trị pKa của các chất:

O

O Cl 16.5

O

<

O OEt

10.7

O

<

O <

9

H 5.9

O

O

O

NO2

< 5.1

O

< 4.7

CF3

Tất cả các axit cacbon này cho anion mà đt (-) chủ yếu trên oxi. 3 chất đầu đều chứa nhóm xeton bên trái bên phải là các nhóm xeton, and, este tương ứng. Điện tích (-) đều được giải tỏa trên cả 2 nhóm cacbonyl. And có lực axit mạnh nhất trong nhóm này vì là dạng cacbonyl đơn giản, xeton có nhóm CH3 tạo hiệu ứng siêu liên hợp đẩy vào làm giảm hiệu quả giải tỏa đt (-), este giải tỏa kém nhất do sự cạnh tranh của nguyên tử O trong nhóm este.

3 chất cuối chỉ có 1 nhóm cacbonyl. Với chất chứa Cl chỉ có hiệu ứng cảm ứng âm của Cl không có hiệu ứng liên hợp ở nhóm này nên lực axit yếu nhất. 2 chất cuối có 2 nhóm hút e mạnh, sự giải tỏa đt (-) mạnh nhất nên lực axit mạnh nhất. Trong đó chất chứa CF3 có 3F hút e mạnh lực axit mạnh nhất.

b. So sánh tính bazơ của các nguyên tử N trong mỗi hợp chất dưới đây:


Câu 7: (2 điểm)Hidrocacbon Chất A có công thức phân tử C7H12. Khi thực hiện phản ứng ozon phân A tạo HCHO và xiclohexanon.Thực hiện quá trình chuyển hoá A liên tiếp sau: 1, HBr ; 2, Mg/ ete; 3, CO2 sau đó với H3O+ ta thu được hợp chất B (C8H14O2) .Nếu cho A tác dụng với HBr/ peoxit tiếp theo cho sản phẩm tác dụng với KCN sau đó xử lý bằng dung dịch axit loãng ta thu được chất C (C8H14O2). C cũng có thể tạo ra bằng cách cho sản phẩm của A với HBr/ peoxit tác dụng với Mg/ete, tiếp theo CO2 và xử lý bằng dung dịch axit. a. Xác định cấu trúc các chất trung gian và viết tên A, B, C b. So sánh tính axit của các B,C với các chất D (axit Benzoic) và E (axit Phenyletanoic) ; F(axit 3phenylpropanoic)

Hướng dẫn giải a.

CH3

CH2

Br

HBr

CH3

MgBr H3O+

CH2Br 1. KCN 2. H3O+

COOH

1. CO2

Mg/ ete

HBr/peoxit

CH3

CH2COOH

(hoÆc 1. Mg/ ete ) 2. CO2 3. H3O+

A. Metylen xiclohexen B. Axit 1- Metyl xiclocacboxylic C. Axit 2- xiclophenyl etanoic

b. Tính axit B< C < F< E < D

H3C COOH

+I2

CH2COOH

+I1

< <

+I1

< +I2

-I1CH2CH2COOH

CH2COOH

- I2

<

< -I1

<

-I2

<

Câu 8: (2 điểm) xác định cấu trúc, đồng phân lập thể, danh pháp

-I3

COOH

-I3


Tinh dầu tràm được dùng trong sản xuất dược phẩm và mỹ phẩm. Hai hợp phần đóng vai trò chính cho ứng dụng đó là eucaliptol (kí hiệu là A) và một đồng phân cấu tạo của nó kí hiệu là B. Hợp chất A không làm mất màu dung dịch Br2/CCl4. Khi đun nóng A với dung dịch axit sunfuric loãng người ta thu được hợp chất không quang hoạt C kết tinh trong nước thành D công thức C10H20O2.H2O. Đun nóng D với axit photphoric người ta thu được sản phẩm chính là E không quang hoạt và có bộ khung cacbon giống như D (biết rằng có thể dùng phương pháp thích hợp để tách lấy từng đối quang từ E). Sau khi ozon phân chế hóa khử thì từ E và từ B đều thu được 3-(1-hydroxy-1-metyletyl)-6-oxoheptanal.

a. Hãy xác định công thức cấu tạo của A, B, C và E. b. Giải thích vì sao B quang hoạt còn E thì không. So sánh có giải thích nhiệt độ nóng chảy, nhiệt độ sôi của A, B, C và E.

c. Hãy vẽ công thức phối cảnh của A, C và giải thích vì sao A dễ dàng bị chuyển hóa bởi axit sunfuric loãng thành C, chất này lại dễ kết tinh trong nước thành D.

d. Hãy đề nghị một sơ đồ tổng hợp E từ những hợp chất chứa không quá 5C. Có thể dùng sơ đồ đó để tổng hợp B được không, vì sao?

e. Hãy đề nghị 2 phương pháp tổng hợp A từ những hợp chất chứa không quá 6C. a. Hãy xác định công thức cấu tạo của A, B, C và E.

E có 10 C như D. Sản phẩm ozon phân E và B là 3-(1-hydroxy-1-metyletyl)-6-oxoheptanal cũng có 10 C suy ra chúng có cấu tạo là:

Công thức phân tử của A (đồng phân của B) là C10H18O, của C là C10H20O2 chứng tỏ A đã cộng với 1 phân tử H2O để thành C. Độ không no của A bằng 2, nó không làm mất màu dung dịch Br2/CCl4 suy ra không có liên kết bội cacbon-cacbon. Các chuyển hóa từ A thành C, D rồi E cho thấy A có bộ khung cacbon như B. Vậy A là một ete 2 vòng no, trong đó 1 vòng chứa liên kết ete (C-O-C). Nhờ xúc tác H+ vòng ete cộng nước mở ra tạo thành diol không quang hoạt C. Hydrat D mất 2 phân tử H2O tạo ra E, tức là C mất 1 phân tử H2O tạo ra E. Sự phân tích trên cho phép suy ra cấu tạo của các chất A, C, E và sự chuyển hóa giữa chúng là như sau:

b. Giải thích vì sao B quang hoạt còn E thì không. So sánh có giải thích nhiệt độ nóng chảy, nhiệt độ sôi của

A, B, C và E.


B và E có cùng công thức cấu tạo với 1 nguyên tử cacbon bất đối. B được tạo ra trong lá cây tràm nên cacbon bất đối chỉ có thể ở một cấu hình (hoặc R, hoặc S), E được hình thành trong bình phản ứng qua ion cacboni C2, khả năng mất H+ từ 2 nhóm CH2 là như nhau, tạo ra cacbon cấu hình R và cacbon cấu hình S với sác xuất như nhau nên E là biến thể raxemic. Nhiệt độ sôi và nóng chảy của từng đối quang thường thấp hơn biến thể raxemic (vì ở biến thể raxemic hai phân tử đối quang kiên kết chặt chẽ với nhau tạo ta hợp chất mới chứ không phải là hỗn hợp thông thường). Hợp chất A là ete không tạo được liên kết hydro liên phân tử, diol C tạo được nhiều liên kết hydro liên phân tử hơn so với monoalcol B và E. Do đó nhiệt độ sôi và nhiệt độ nóng chảy biến đổi theo trật tự sau:

A<B<E<C c. Hãy vẽ công thức phối cảnh của A, C và giải thích vì sao A dễ dàng bị chuyển hóa bởi axit sunfuric loãng thành C, chất này lại dễ kết tinh trong nước thành D.

Ba vòng 6 cạnh ở A và A1 đều ở dạng thuyền kém bền nên ion oxoni A1 dễ mở vòng thành ion

cacboni bậc ba, sau đó cộng với nước thành ancol C có cấu dạng ghế bền hơn. Các monoancol tecpen như

B, mentol, … không kết tinh ngậm nước. Hai nhóm OH ở diancol C dễ ở cùng một phía của vòng 6 cạnh dễ hình thành được 2 liên kết hydro với 1 phân tử nước (H2O vừa cho H vừa nhận H) tạo ra cấu trúc bền thắng

được tính kị nước của gốc hydrocacbon C10H18. d. Hãy đề nghị một sơ đồ tổng hợp E từ những hợp chất chứa không quá 5C. Có thể dùng sơ đồ đó để tổng hợp B được không, vì sao?

Trong sơ đồ trên, ở giai đọan phản ứng Diels-Alder, nhóm metyl của dien và nhóm axetyl của dienophin phân bố ở xa nhau để tránh hiệu ứng không gian nên sản phẩm chính là xeton có cấu tạo như trên sơ đồ. Tuy nhiên sác xuất hình thành cacbon bất đối R và S là như nhau đối với xeton đó nên sau phản ứng vơi MeMgBr thì tạo ra biến thể raxemic E mà không tạo ra B quang hoạt được. e. Hãy đề nghị 2 phương pháp tổng hợp A từ những hợp chất chứa không quá 6C.


Phương pháp 1 [K. Alder, 1949]: H 2O/H +

1) MeMgBr/ete

to

OH P2O5

O

2) H3O+

OH

O

O

OH

(A)

Phương pháp 2 [W. Perkin, 1907]: O

O H2O/H+

KCN

O

O

OH 2) H3O

Br

CN

điểm) Cơ chế phản ứng Giải thích các phản ứng sau bằng cơ chế phản ứng? a. H/H2O

2 OH

b.

c. O

OH OH

-

OH (A)

d.

O (B)

O

+

OH

COOMe

COOH

P2O 5

1) 3 MeMgI

MeOH/H+

(A )

Câu 9: (2


CO2H O CO2Me

O DBU, MeOH

MeO2C

DBU: 1,8-Diazabicyclo[5.4.0]undec-7-ene

N N

Hướng dẫn giải a.

H 2O

OH

b.

c.

d. Giai đoạn đầu tiên của quá trình chuyển hóa là phản ứng cộng Michael:


Câu 10: (2 điểm) Tổng hợp các chất hữu cơ (Dạng sơ đồ phản ứng) (+)-Artemisinin (Quinhaosu) là một hoạt chất tự nhiên có tác dụng chữa bệnh sốt rét mà không gây tác dụng phụ được tách ra từ cây Thanh hao hoa vàng (Artemisia annua L., họ Asteriaceae). Đây là một lacton dạng secquitecpen. Hoạt tính kháng sốt rét của hợp chất này là do nó có cấu trúc cầu endoperoxit. Do nguồn thu tự nhiên hạn chế, trong khi yêu cầu chữa bệnh sốt rét vẫn luôn cấp thiết nên các nhà hóa dược đặt ra vấn đề phải tổng hợp hoặc bán tổng hợp hoạt chất này. Năm 2003, một nhóm nghiên cứu tại viện Hóa kỹ thuật Ấn

Độ đã đề xuất một qui trình tổng hợp (+)-Artemisinin từ (+)-Isolimonen theo sơ đồ dưới đây:

Trong sơ đồ trên, PCC là piriđinium clorocromat; KHMDS là kali hexametilen đisilan (hay kali


bis(trimetylsilyl) amiđua. Đây là một bazơ rất mạnh, tương tự NaNH2, nhưng cồng kềnh nên không thể hiện tính nucleophin). Hãy cho biết cấu trúc các hợp chất từ A đến I trong sơ đồ tổng hợp (+)-Artemisinin trên.

Hướng dẫn giải Hoàn thành sơ đồ phản ứng tổng hợp (+)-Artemisinin:

Cho: H = 1; C = 12; N = 14; O = 16; Na = 23; Mg = 24,3; Al = 27; P = 31; S = 32; Cl = 35,5; K = 39,1; Ca = 40,1; Ti = 47,9; Cr = 52; Mn = 54,9; Fe = 55,8; Co = 58,9; Ni = 58,7; Cu = 63,5; Zn = 65,4; Ag = 107,9; Ba = 137,3. ---------------------------------------------------

Thí sinh không được sử dụng tài liệu. Cán bộ coi thi không giải thích gì thêm. Giáo viên: Quách Phạm Thùy Trang 0975855880


SỞ GIÁO DỤC – ĐÀO TẠO THÁI BÌNH Trường THPT Chuyên Thái Bình

ĐỀ ĐÓNG GÓP CHO KỲ THI C10 Môn Hoá học lớp 11 Năm học : 2016-2017 Thời gian làm bài: 180 phút (Đề gồm 03 trang)

Bài 1: (2 điểm) Tốc độ phản ứng- Cân bằng hóa học k1 k2 Có hai phản ứng bậc nhất nối tiếp nhau A  → B  → C nồng độ của B có giá trị cực đại ở thời điểm

τ tính theo phương trình τ = ln [( k2/ k1) / (k2 − k1)] a) Viết phương trình động học vi phân cho các chất A, B, C. _ . b) Tỷ số k2 / k1 phải như thế nào để τ bằng nửa - chu kỳ chuyển hoá chất A?

Bài 2: (2 điểm) Cân bằng trong dung dịch điện li 1 Cho 0,01 mol NH3, 0,1 mol CH3NH2 và 0,11 mol HCl vào nước được 1 lít dung dịch. Tính pH của dung dịch thu được? Cho: pK NH+ = 9,24; pK CH NH+ = 10,6; pK H2 O = 14. 4

3

3

2 Tính độ tan của AgSCN trong dung dịch NH3 0,003 M. Cho T AgSCN = 1,1.10-12 và hằng số phân li của phức [Ag(NH3)2]+ bằng 6.10-8.

Bài 3: (2 điểm) Điện hoá học Lắp 1 pin bằng cách nối điện cực hydro chuẩn với một nửa pin bởi 1 dây đồng nhúng vào 40ml ddCuSO4 0,01M có thêm 10ml ddNH3 0,5M. Chấp nhận rằng chỉ tạo phức Cu(NH 3 ) 4

2+

+

với nồng độ NH 4 là không

đáng kể so với nồng độ NH3. 2+ a. Xác định E Cu /Cu .

o 2+ b. Tính E Cu(NH3 )4 /Cu . o

2+

Biết E Cu /Cu = 0,34v;

Cu(NH3 )4 2+ /Cu

lgβ Cu(NH3 )42+ =13,2 và ECu 2+ /Cu=ECu(NH3 )4 2+ /Cu Bài 4: (2 điểm) Nhóm N-P, nhóm C-Si A là một hợp chất của nitơ và hidro với tổng điện tích hạt nhân bằng 10. B là một oxit của nitơ, chứa 36,36% oxi về khối lượng. a. Xác định các chất A, B, X, D, E, G và hoàn thành các phương trình phản ứng: A + NaClO → X + NaCl + H2O X + HNO2 → D + H2O D + NaOH → E + H2O


A + Na → G + H G + B → E + H2O b. Viết công thức cấu tạo của D. Nhận xét về tính oxi hóa - khử của nó. c. D có thể hòa tan Cu tương tự HNO3. Hỗn hợp D và HCl hòa tan được vàng tương tự cường thủy. Viết phương trình của các phản ứng tương ứng.

Bài 5: (2 điểm) Phức chất Hòa tan 2,00 gam muối CrCl3.6H20 vào nước, sau đó thêm lượng dư dung dịch AgNO3 và lọc nhanh kết tủa AgCl cân được 2,1525 gam. Cho biết muối crom nói trên tồn tại dưới dạng phức chất.

1.

Hãy xác định công thức của phức chất đó.

2.

Hãy xác định cấu trúc (trạng thái lai hóa, dạng hình học) và nêu từ tính

của phức chất trên.

Bài 6: (2 điểm) Quan hệ cấu trúc - tính chất a) Sắp xếp các chất trong dãy sau đây theo trình tự tăng dần tính bazơ : 2,4,6- trinitro- N,N – đimetylanilin, p- nitroanilin, N,N – đimetylanilin và anilin,? Giải thích? b) Hãy giải thích vì sao chất béo thực vật thường có nhiệt độ đông đặc thấp hơn chất béo động vật?

Bài 7: (2 điểm) Hidrocacbon. Ankin A có công thức phân tử C6H10, có đồng phân quang học. Hidro hóa hoàn toàn A thu được A1. a. Viết công thức cấu tạo của A và A1. Cho biết A1 có đồng phân quang học hay không? b. Ankin B cũng có công thức phân tử C6H10. B tác dụng với H2 (xúc tác Ni, t0) thu được 2-metylpentan. B không tác dụng với dung dịch AgNO3/NH3. B tác dụng với H2O (xúc tác HgSO4, t0) tạo chất C6H12O (B1). Xác định công thức cấu tạo của B và B1. c. Hidro hóa B (xúc tác Pd/PbCO3, t0) thu được chất C. Chất C tác dụng với H2SO4 rồi thủy phân tạo chất D. Viết công thức cấu tạo của C và D. Biết C và D là sản phẩm chính. Cho biết C là đồng phân cis hay trans? d. Tách nước chất D với xúc tác H2SO4 đặc và đun nóng. Viết phương trình hóa học và nêu sản phẩm chính. Cho biết tên cơ chế phản ứng. Bài 8: (2 điểm) Xác định cấu trúc Thủy phân hợp chất A (C13H18O2) trong môi trường axit HCl loãng cho hợp chất B (C11H14O). Khi B phản ứng với brom trong NaOH, sau đó axit hóa thì thu được axit C. Nếu đun nóng B với hỗn hợp hiđrazin và KOH trong glicol thì cho hiđrocacbon D. Mặt khác, B tác dụng với benzanđehit trong dung dịch NaOH loãng (có đun nóng) thì tạo thành E (C18H18O). Khi A, B, C, D bị oxi hóa mạnh thì đều cho axit phtalic. Hãy viết công thức cấu tạo của các hợp chất từ A đến E. Bài 9: (2 điểm) Cơ chế Ancol tert-butylic cũng như iso-butilen khi đun nóng với metanol có H2SO4 xúc tác đều cho sản phẩm chính là A (C5H12O). Ngoài ra, tuỳ thuộc chất đầu là ancol tert-butylic hay iso-butilen mà còn tạo ra các sản phẩm phụ khác như B (C8H18O), C (C9H20O), D (C8H16)... Khi cho A, B, C, D tác dụng với CH3MgI đều không thấy khí thoát ra. a) Hãy xác định công thức cấu tạo của A và giải thích vì sao A lại là sản phẩm chính trong cả hai trường hợp đã cho.


b) Dùng công thức cấu tạo để viết sơ đồ phản ứng giải thích sự hình thành sản phẩm phụ trong trường hợp dùng ancol tert-butylic và trường hợp dùng iso-butilen. Bài 10: (2 điểm) Tổng hợp hữu cơ Thành phần chính của dầu thông là α-pinen (2,6,6-trimetylbixiclo[3.1.1]hept-2-en). Cho α-pinen tác dụng với axit HCl được hợp chất A, sau đó cho A tác dụng với KOH/ancol thu được hợp chất camphen (B). Viết cơ chế phản ứng chuyển hóa α-pinen thành A và A thành B. Trong môi trường axit, B quang hoạt chuyển hóa thành B raxemic. Giải thích hiện tượng này.

Cl

OAc A

B

C

Viết sơ đồ các phản ứng tổng hợp B từ xiclopentađien và acrolein cùng các hóa chất cần thiết khác, biết rằng một trong số các sản phẩm trung gian của quá trình tổng hợp là một enol axetat C.

------------------------------HẾT-----------------------------SỞ GIÁO DỤC – ĐÀO TẠO THÁI BÌNH Trường THPT Chuyên Thái Bình *****

ĐÁP ÁN ĐỀ ĐÓNG GÓP CHO KỲ THI C10 Môn Hoá học lớp 11 Năm học : 2016-2017 Thời gian làm bài : 180 phút (Đề gồm 02 trang)

Bài 1: (2 điểm) Tốc độ phản ứng- Cân bằng hóa học k1 k2 Có hai phản ứng bậc nhất nối tiếp nhau A  → B  → C nồng độ của B có giá trị cực đại ở thời điểm τ tính theo phương trình τ = ln [( k2/ k1) / (k2 − k1)] a) Viết phương trình động học vi phân cho các chất A, B, C. _ . b) Tỷ số k2 / k1 phải như thế nào để τ bằng nửa - chu kỳ chuyển hoá chất A? Câu Câu 1

Hướng dẫn a) Ph-¬ng tr×nh vi ph©n ®éng häc:

dC A = − k1.CA ; dt

dCB dCC = k1.CB – k2.CB ; = k2.CC . dt dt k ( 2) a ln 2 ln 2 k k1  → ln b) τ = t 1/2 = = ®Æt 2 = a ta cã ln k1 k2 − k1 k1 k1 a −1

Điểm 1,0

1,0

=ln 2 → a = 2

Bài 2: (2 điểm) Cân bằng trong dung dịch điện li 1 Cho 0,01 mol NH3, 0,1 mol CH3NH2 và 0,11 mol HCl vào nước được 1 lít dung dịch. Tính pH của dung dịch thu được? Cho: pK NH+ = 9,24; pK CH NH+ = 10,6; pK H2 O = 14. 4

3

3

2 Tính độ tan của AgSCN trong dung dịch NH3 0,003 M.


Cho T AgSCN = 1,1.10-12 và hằng số phân li của phức [Ag(NH3)2]+ bằng 6.10-8.

Câu 2a.

Hướng dẫn +

NH3 + H  → NH

1.

0,01

0,01

Điểm 1,0đ

+ 4

0,01 mol

CH3NH2 + H

+

 → CH3NH 3+

0,1 0,1 0,1 mol + Tổng số mol H phản ứng: n = 0,01 + 0,1 = 0,11 mol. Vậy H+ vừa hết. Dung dịch thu được gồm: 0,01 mol NH +4 và 0,1 mol CH3NH 3+ Vì K H O << K CH NH+ < K NH+ nên có thể bỏ qua cân bằng điện ly của nước 3 3 4 2 trong dung dịch. Ban đầu Điện ly Cân bằng

⇀ CH3 NH2 + H+ CH3NH 3+ ↽ 0,1 x x x 0,1 - x x x+y

⇀ NH3 + H+ NH +4 ↽ Ban đầu 0,01 Điện ly y y y Cân bằng 0,01 – y y x+y Áp dụng định luật tác dụng khối lượng cho các cân bằng điện li ta có : K CH NH+ = 3

3

[CH3 NH 2 ].[H + ] [CH3 NH3+ ]

=

x(x + y) = 10-10,6 0,1- x

⇒ x (x + y) = 10-11,6 (1) +

K NH+ 4

=

[NH3 ][H ] [NH +4 ]

=

(giả sử coi x << 0,1)

y(x + y) = 10-9,24 0, 01- y

⇒ y (x + y) = 10-9,24 . 10-2 = 10-11.24 (2) (giả sử coi y << 0,01) Từ (1) và (2) ta có x (x + y ) + y (x + y) = 10-11,6 + 10-11,24 = 8,27.10-12 8,27 .10 −12

⇔x+y=

⇒ [ H+ ] = x + y = 2,88 . 10-6 mol.lit-1. ⇒ pH = -lg [ H+ ] = - lg 2,88 . 10-6 = 5,54

2b.

Các cân bằng xảy ra trong dung dịch: +

⇀ Ag + SCN AgSCN ↽

1,0đ -

⇀ [Ag(NH3)2]+ Ag+ + 2NH3 ↽

-2

TAgSCN = 1,1.10 1 1 = Kb = K pl 6.10 −8


(2)

⇀ [Ag(NH3)2]+ + SCNAgSCN + 2NH3 ↽ 0,003 – 2s

s

Kcb = TAgSCN.

1 K pl

s

Áp dụng định luật tác dụng khối lượng cho cân bằng (3) ta có: T s2 = AgSCN 2 (0,003 − 2s) K pl Thay số vào ta có s = 1,27.10-5 mol.lit-1.đ

Bài 3: (2 điểm) Điện hoá học Lắp 1 pin bằng cách nối điện cực hydro chuẩn với một nửa pin bởi 1 dây đồng nhúng vào 40ml ddCuSO4 2+ + 0,01M có thêm 10ml ddNH3 0,5M. Chấp nhận rằng chỉ tạo phức Cu(NH3 ) 4 với nồng độ NH 4 là không

đáng kể so với nồng độ NH3. a. Xác định E Cu 2+ /Cu . o

2+

b. Tính E Cu(NH3 ) 4 /Cu . Biết E oCu 2+ /Cu = 0,34v;

Cu(NH3 )42+ /Cu

lgβ Cu(NH3 )4 2+ =13,2 và ECu 2+ /Cu=ECu(NH 3 )4 2+ /Cu Câu 3a.

Hướng dẫn 2+ 

Cu   = 0,8.10

Điểm 1đ

−2

[ NH3 ] = 0,1 Cu 2+ +2e → Cu

E o =0,34(v)

o E Cu 2+ /Cu =E Cu + 2+ /Cu

Cu 2+

(1)

+4NH3 ⇌ Cu(NH3 ) 4 2+

0,8.10−2

pöù

0,8.10

−2

cb

0

[ ⇒ Cu 2+  =

3b.

0,059  2+  lg Cu  2

0,1

β=1013,2

0 −2

0,8.10−2

6,8.10−2

0,8.10−2

3, 2.10

Cu(NH3 ) 4 ]

2+

[ NH3 ]4 .β

0,8.10−2 = =2,4.10−11 4 13,2 (0,068) .10

(1) ⇒ E Cu 2+ /Cu =0,34+

0,059 lg2,4.10−11 =0,026(v) 2

b/ E Cu 2+ /Cu =E Cu(NH

/Cu

3 )4

2+

Cu(NH 3 ) 4 2+ +2e → Cu+4NH3

(2) 1đ


0,059 o E Cu(NH ) 2+ /Cu =E Cu(NH + lg 2+ 3 4 3 ) 4 /Cu 2

(1)(2)(3)

⇒ E oCu(NH

3 )4

2+

/Cu

[Cu(NH3 )4 ]2+ [ NH3 ]4

(3)

= -0,05(V)

Bài 4: (2 điểm) Nhóm N-P, nhóm C-Si A là một hợp chất của nitơ và hidro với tổng điện tích hạt nhân bằng 10. B là một oxit của nitơ, chứa 36,36% oxi về khối lượng. a. Xác định các chất A, B, X, D, E, G và hoàn thành các phương trình phản ứng: A + NaClO → X + NaCl + H2O X + HNO2 → D + H2O D + NaOH → E + H2O G + B → D + H 2O b. Viết công thức cấu tạo của D. Nhận xét về tính oxi hóa - khử của nó. c. D có thể hòa tan Cu tương tự HNO3. Hỗn hợp D và HCl hòa tan được vàng tương tự cường thủy. Viết phương trình của các phản ứng tương ứng. Câu 4a.

4b.

4c.

Hướng dẫn * Do N có Z=7 và H có Z=1; mà chất A có tổng ĐTHN là 10. A là NH3. * Đặt oxit nito là NxOy => N2O * Các phản ứng: NH3 + NaClO → N2H4 + NaCl + H2O N2H4 + HNO2 → HN3 + 2H2O HN3 + NaOH → NaN3 + H2O 2NH3 + 2Na → 2NaNH2 + H2 NaNH2 + N2O → NaN3 + H2O A là NH3; B là N2O; X là N2H4; D là HN3; E là NaN3; G là NaNH2. D: Axit hidrazoic H-N(-3)=N(+5) ≡N(-3) Trong phân tử HN3 vừa có N(+5), vừa có N(-3) nên nó vừa có tính oxi hóa, vừa có tính khử. Về tính oxi hóa nó giống HNO3 nên hòa tan Cu: Cu + 3HN3 → Cu(N3)2 + N2 + NH3 Khi trộn với HCl: 2Au + 3HN3 +8HCl → 2H[AuCl4] + 3N2 + 3NH3

Đ iể m

0,25 đ 0,25 đ 0,25 đ 0,25 đ 0,25 đ 0,25đ

0,25đ 0,25đ

Bài 5: (2 điểm) Phức chất Hòa tan 2,00 gam muối CrCl3.6H20 vào nước, sau đó thêm lượng dư dung dịch AgNO3 và lọc nhanh kết tủa AgCl cân được 2,1525 gam. Cho biết muối crom nói trên tồn tại dưới dạng phức chất. 1. Hãy xác định công thức của phức chất đó. 2. Hãy xác định cấu trúc (trạng thái lai hóa, dạng hình học) và nêu từ tính của phức chất trên.


Câu 1.

Hướng dẫn n(AgCl) = (2,1525:143,5) = 0,015; n(CrCl3 . 6H2O) = (2:266,5) = 7,5.10-3 n(Cl- tạo phức) = 3(7,5.10-3) - 0,015 = 7,5.10-3 Trong phân tử phức chất tỷ lệ mol Cl − : Cr3+ = (7,5.10-3) : (7,5.10-3) = 1:1 Công thức của phức: [Cr(H2O)5Cl]2+

2.

24

Cr3+ (1s2 2s2 2p6 3s2 3p6 3d3) →

24

Cr3+ : [Ar] 3d3 Cl

3d

3

4s

4p

H2 O

A

Ar H 2O

Phøc thuËn tõ

900

H2 O

Đi ể m 1đ

Cr lai hãa sp3d2

(0,25đ)

900

H 2O

H 2O B¸t diÖn ®Òu

(0,25đ)

Bài 6: (2 điểm) Quan hệ cấu trúc - tính chất a) Sắp xếp các chất trong dãy sau đây theo trình tự tăng dần tính bazơ : 2,4,6- trinitro- N,N – đimetylanilin, p- nitroanilin, N,N – đimetylanilin và anilin,? Giải thích? b) Hãy giải thích vì sao chất béo thực vật thường có nhiệt độ đông đặc thấp hơn chất béo động vật?

Câ u 6a.

6b.

Hướng dẫn / a) Trình tự tăng dần tính bazơ : p- nitroanilin < anilin < N,N – đimetylanilin < pKa: 1,02 4,58 5,06 do nhóm - NO2 có 2 nhóm –CH3 có –I hiệu ứng +I -C hút e mạnh => tính bazơ tăng =>tính bazơ giảm

Đi ể m 1,5đ

2,4,6- trinitro- N,N – đimetylanilin 9,3 nhóm –NO2 có hiệu ứng –I, nhưng do hi không gian loại II: 2 nhóm –NO2 ở các v trong nhân benzen và 2 nhóm –CH3 ở nguyên t đã gây cản trở không gian làm cho các e p c không liên hợp được với các e của π của nhân th (trục của e p không song song với trục c phạm nguyên tắc của hệ liên hợp) nên tính baz Do thành phần của dầu thực vật chủ yếu este chứa gốc axit không no, còn mỡ động vật chủ yếu 0,5 chứa este chứa gốc axit no. đ Do thành phần của dầu thực vật chủ yếu este chứa gốc axit không no, còn mỡ động vật chủ yếu chứa este chứa gốc axit no.

Bài 7: (2 điểm) Hidrocacbon. Ankin A có công thức phân tử C6H10, có đồng phân quang học. Hidro hóa hoàn toàn A thu được A1. a. Viết công thức cấu tạo của A và A1. Cho biết A1 có đồng phân quang học hay không? b. Ankin B cũng có công thức phân tử C6H10. B tác dụng với H2 (xúc tác Ni, t0) thu được 2-metylpentan. B không tác dụng với dung dịch AgNO3/NH3. B tác dụng với H2O (xúc tác HgSO4, t0) tạo chất C6H12O (B1). Xác định công thức cấu tạo của B và B1.


c. Hidro hóa B (xúc tác Pd/PbCO3, t0) thu được chất C. Chất C tác dụng với H2SO4 rồi thủy phân tạo chất D. Viết công thức cấu tạo của C và D. Biết C và D là sản phẩm chính. Cho biết C là đồng phân cis hay trans? d. Tách nước chất D với xúc tác H2SO4 đặc và đun nóng. Viết phương trình hóa học và nêu sản phẩm chính. Cho biết tên cơ chế phản ứng.

Câu 7a.

7b. 7c.

7d.

Hướng dẫn

Điểm 0,25 đ

A: CH3-CH2-CH(CH3)-C≡CH A1: CH3-CH2-CH(CH3)-CH2-CH3 A1 không có đồng phân quang học. B: (CH3)2CH-C≡C-CH3 B1: (CH3)2CH-CO-CH2-CH3 C: (CH3)2CH-CH=CH-CH3 C có cấu hình cis D: (CH3)2CH-CH2-CH(OH)-CH3 * Viết ptpư tạo: (CH3)2CH-CH=CH-CH3 (SPC) và: (CH3)2CH-CH2-CH=CH2 * Cơ chế tách E1.

0,25 đ 0,25 đ 0,25 đ 0,25 đ 0,25 đ 0,25 đ 0,25 đ

Bài 8: (2 điểm) Xác định cấu trúc Thủy phân hợp chất A (C13H18O2) trong môi trường axit HCl loãng cho hợp chất B (C11H14O). Khi B phản ứng với brom trong NaOH, sau đó axit hóa thì thu được axit C. Nếu đun nóng B với hỗn hợp hiđrazin và KOH trong glicol thì cho hiđrocacbon D. Mặt khác, B tác dụng với benzanđehit trong dung dịch NaOH loãng (có đun nóng) thì tạo thành E (C18H18O). Khi A, B, C, D bị oxi hóa mạnh thì đều cho axit phtalic. Hãy viết công thức cấu tạo của các hợp chất từ A đến E. Câu 8.

Đ iể m Hướng dẫn Sự tạo thành axit phtalic cho thấy các hợp chất là dẫn xuất của benzen bị thế hai 1đ lần ở vị trí ortho. B là một xeton có nhóm CH3CO-. HO

B + C6H5CHO

-

E (C18H18O)

Cho thấy B chỉ ngưng tụ với một phân tử benzanđehit, vậy nhóm CH3CO- sẽ đính trực tiếp vào nhân benzen và xeton B phải là o-C3H7C6H4COCH3. 1đ C3H7 CH3 O

A

O

B

C3H7

C3H7

C3H7

C3H7

COCH3

COOH

C2H5

COCH=CHPh

C

D

E

Bài 9: (2 điểm) Cơ chế Ancol tert-butylic còng nh- iso-butilen khi ®un nãng víi metanol cã H2SO4 xóc t¸c ®Òu cho s¶n phÈm chÝnh lµ A (C5H12O). Ngoµi ra, tuú thuéc chÊt ®Çu lµ ancol tert-butylic hay iso-butilen mµ cßn t¹o ra c¸c s¶n phÈm phô kh¸c nh- B (C8H18O), C (C9H20O), D (C8H16)... Khi cho A, B, C, D t¸c dông víi CH3MgI ®Òu kh«ng thÊy khÝ tho¸t ra.


a) H·y x¸c ®Þnh c«ng thøc cÊu t¹o cña A vµ gi¶i thÝch v× sao A l¹i lµ s¶n phÈm chÝnh trong c¶ hai tr-êng hîp ®· cho. b) Dïng c«ng thøc cÊu t¹o ®Ó viÕt s¬ ®å ph¶n øng gi¶i thÝch sù h×nh thµnh s¶n phÈm phô trong tr-êng hîp dïng ancol tert-butylic vµ tr-êng hîp dïng iso-butilen. Câ Hướng dẫn Đ iể u m 9a,b

(CH3)3C - OH

H

+

- H 2O

+

CH3 C

CH3

H

+

CH2 C

CH3

CH3

CH3 CH3OH

CH3 O

CH3 C

CH3OH

(A)

CH3

H

+

- H 2O

CH3

(B) CH3+

(CH3)3C+ bÒn h¬n +CH3 ; (CH3)3C – OH cång kÒnh h¬n CH3 – OH V× vËy, ph¶n øng chÝnh lµ: H+

+

(CH3)3C + CH3OH

CH3 O

CH3 C

CH3

(CH3)3C + CH3 C

OH

H+

(CH3)3C + CH2 C

O

CH3

C

+

CH2 C

(CH3)3C

CH3 (B)

CH3

CH3

CH3 +

CH3

CH3 CH3 C

(A)

CH3

CH3 +

CH3

H+

(CH3)3C

CH3

CH3 H

+

CH3

(D)

C

C

CH3

CH3OH

CH3

CH3 CH3 C

CH

CH2

C

O-CH3 (C)

CH3

Bài 10: (2 điểm) Tổng hợp hữu cơ Thành phần chính của dầu thông là α-pinen (2,6,6-trimetylbixiclo[3.1.1]hept-2-en). Cho α-pinen tác dụng với axit HCl được hợp chất A, sau đó cho A tác dụng với KOH/ancol thu được hợp chất camphen (B). Viết cơ chế phản ứng chuyển hóa α-pinen thành A và A thành B. Trong môi trường axit, B quang hoạt chuyển hóa thành B raxemic. Giải thích hiện tượng này. Cl

OAc A

B

C

Viết sơ đồ các phản ứng tổng hợp B từ xiclopentađien và acrolein cùng các hóa chất cần thiết khác, biết rằng một trong số các sản phẩm trung gian của quá trình tổng hợp là một enol axetat C.


Hướng dẫn: Sơ đồ phản ứng tổng hợp camphen (B) từ α-pinen: Cl HCl

chuyÓn vÞ - HCl

α - Pinen

A

B

Cơ chế phản ứng chuyển hóa α-pinen thành A: Cl

H+

chuyÓn vÞ

Cl

-

α− Pinen

A

Cơ chế tạo thành B từ A: 3

7

Cl

1

2

3

chuyÓn vÞ

-

4

1

2

-Cl

3

+

-H

4 4

A

B

Giải thích hiện tượng raxemic hóa hợp chất B: H+

-H+

chuyÓn vÞ

H+

-H+

B quang ho¹t

B ®èi quang

Tổng hợp camphen (B) từ xiclopentađien và acrolein: CHO

CHO CHO

H2

+ O Na, CH3I

OAc Ac2O

O

O3

HO CH3MgI -HOH

Camphen

------------------------------HẾT-------------------------------


TRƯỜNG THPT CHUYÊN VĨNH PHÚC

KÌ THI CHỌN HỌC SINH GIỎI KHU VỰC ĐỒNG BẰNG DHBB NĂM 2017 MÔN HÓA HỌC – KHỐI 11 Thời gian làm bài: 180 phút (Đề gồm 05 trang )

ĐỀ THI ĐỀ XUẤT

Câu 1 : (2,0 điểm) Động học (Có cơ chế) – Cân bằng hóa học 1. Sự có mặt của clo ở tầng bình lưu của khí quyển Trái đất dẫn đến sự bào mòn lớp ozon. Quá trình này

được mô tả một cách đơn giản hóa như sau: k1 Cl2  → 2Cl

Cl ClO

k2 O3  → ClO

+ +

k3 O3  → Cl

+

O2

+ 2O2

k4 2Cl  → Cl2

a. Có thể áp dụng gần đúng trạng thái ổn định cho những tiểu phân nào? Tại sao? b. Tìm biểu thức mô tả tốc độ mất đi của ozon. Bậc riêng phần của Cl2 trong biểu thức bằng bao nhiêu? 2. Một hệ gồm 1 mol CO được đốt cháy với một lượng không khí vừa đủ (thành phần mol: 20% O2; 80% N2) theo phản ứng:  → 2CO2 2CO + O2 ← 


Nhiệt độ bắt đầu đốt là 250C, dưới áp suất chung không đổi P = 1 atm. Xác định nhiệt độ ngọn lửa (nhiệt độ cực đại mà hệ có thể đạt được)? Cho các đại lượng nhiệt động ở 298K; Áp suất tiêu chuẩn P0 = 1 atm. Cấu tử

O2(k)

N2(k)

∆G 0 tạo thành (Kj/mol)

CO(k)

CO2(k)

–137,3

–394,5

S0 (J/mol.K)

205

192

198

214

C p0 (J/mol.K)

29,4

29,1

29,1

37,1

(Chấp nhận C p0 của các chất; ∆S0 của phản ứng không phụ thuộc vào nhiệt độ)

Câu 2 : (2,0 điểm) Cân bằng trong dung dịch điện li

1. Để xác định hàm lượng axit salixylic (axit o-hidroxibenzoic) trong một mẫu phân tích với tạp chất trơ, người ta tiến hành như sau: + Hoà tan 1,7614 gam mẫu phân tích vào nước tạo 100 ml dung dịch. + Hoà tan 0,595 gam KBr và 0,167 gam KBrO3 vào nước tạo 50 ml dung dịch. + Trộn 10 ml dung dịch mẫu phân tích với dung dịch H2SO4 loãng dư và 10 ml dung dịch (KBr + KBrO3). Lắc kĩ, đậy nắp kín, để yên vài phút. + Chuẩn độ hỗn hợp thu được hết 12,5 ml dung dịch NaAsO2 0,016M a. Tính thành phần phần trăm khối lượng của axit salixylic trong mẫu phân tích? b. Chuẩn độ 10 ml dung dịch axit ở trên bằng dung dịch NaOH 0,01M thì nên chọn chất chỉ thị nào sau đây cho phù hợp? (Metyl đỏ-4,2 ; Brom thymolxanh-7,6; Trung tính -8). c. Tính sai số chuẩn độ nếu dùng metyl đỏ? E0 BrO3-/Br2 = 1,52V ; E0 Br2/Br- =1,085V; Pka =2,975; Bỏ qua quá trình phân li của nước. 2. Một dung dịch A gồm HAc 0,010 M và NH4Cl 0,200 M.

a. Tính pH của dung dịch A. b. Chuẩn độ 25,0 ml dung dịch A bằng dung dịch NaOH 0,020 M đến màu vàng rõ của Metyl đỏ (pT=6,2). Tính sai số chuẩn độ. Biết : K NH+ = 10-9,24; KHAc = 10-4,76. 4

Câu 3: (2,0 điểm) Điện hóa học Dung dịch X gồm Fe2(SO4)3 0,1500M ; FeSO4 0,0150M và KCl 2M. 1. Cần đặt điện thế tối thiểu là bao nhiêu để có quá trình oxi hóa và quá trình khử xảy ra đầu tiên ở mỗi điện cực khi điện phân dung dịch X ở pH=0. 2. Điện phân 100ml dung dịch X với cường độ dòng điện một chiều không đổi có I = 9,650A và trong thời gian 150 giây, thu được dung dịch Y. a) Tính khối lượng dung dịch giảm trong quá trình điện phân. b) Tính pH của dung dịch Y. c) Lắp một pin điện gồm một điện cực hiđro tiêu chẩn với một điện cực Pt nhúng vào dung dịch Y. Tính sức điện động của pin khi pin bắt đầu phóng điện và viết sơ đồ của pin.


(Giả thiết rằng H2O bay hơi không đáng kể và thể tích của dung dịch không thay đổi trong quá trình điện phân) Cho: E0(Fe3+/Fe2+) = 0,771V; E0(2H+/H2) = 0,00V; β*[Fe(OH)]2+= 10-2,17;

β*[Fe(OH)]+= 10-5,92; E0(Cl2/2Cl-)= 1,36V. Câu 4: (2,0 điểm) N – P, C – Si và hợp chất 1. Cho dãy chuyển hóa dưới đây: N2O Na HCl E A → B  → C  → D  →Z → E + Y + A H2O2 p,t NaOCl X + Y → A  → E ← Ure Fe O NaOH 3

4

Biết A và E là những bazơ yếu; X và Y là các đơn chất đều tồn tại ở thể khí, khi phân hủy 1 mol Z thu được 35,5 lít khí Y (đktc). a. Xác định các chất A, B, C, D, E, X, Y, Z. b. Viết đầy đủ các phương trình phản ứng xảy ra trong dãy chuyển hóa đã cho. 2. Một nguyên tố X có nhiều dạng thù hình, đa hóa trị, là nguyên tố thiết yếu cho cơ thể sống, không bao giờ tồn tại ở trạng thái đơn chất trong tự nhiên. Cho 1,55 gam X màu trắng tan hết trong axit HNO3 đặc nóng dư thu được 5,6 lít khí NO2 (đktc) là sản phẩm khử duy nhất và dung dịch Y chỉ chứa axit. Cho sơ đồ phản ứng sau đây: 0

+ ddBa ( OH )2 + H 2 SO4 + ddCuSO 4 +2 NaOH 600 C (a) X  → A   → B  → D  → E  →F 0

0

0

+ H 2O + ddAgNO3 + Ca ,t C 200 C 260 C (b) X   → G  → L  → M  → D  →Q

Biết A, B, D, E, F, G, L, M, Q đều là hợp chất của X có phân tử khối thỏa mãn: MA

+

MG = 449;

MF

+

MQ = 444; MD

MB +

+

ML = 100;

MM = 180

1. Xác định nguyên tố X và các chất A, B, D , E, F, G, L, M, Q. 2. Viết các phương trình phản ứng theo sơ đồ phản ứng ở trên.

Câu 5: (2,0 điểm) Phức chất, trắc quang 1. a. Thêm dần dung dịch NaCN vào dung dịch NiCl2 lúc đầu thu được kết tủa xanh X, sau đó kết tủa này tan ra tạo thành dung dịch màu vàng của chất Y. Nếu cho thêm tiếp NaCN đặc thì thu được dung dịch màu đỏ của chất Z. Hãy viết các phương trình phản ứng xảy ra trong thí nghiệm này. b. Cho biết Y và Z đều nghịch từ, dựa theo thuyết liên kết hóa trị (VB), hãy dự đoán cấu trúc phân tử của chúng. 2. Phương pháp phổ đo quang là phương pháp tiêu chuẩn để nghiên cứu các cân bằng hóa học có sự biến đổi màu sắc. Kỹ thuật này dựa vào định luật Beer phát biểu rằng độ hấp thụ tỉ lệ tuyến tính với đường đi của ánh sáng l (quãng đường mà ánh sáng phải đi qua chất) và nồng độ mol của tiểu phân hấp thụ. Xét phản ứng: 2NO2(k) ⇌ N2O4(k). (Lưu ý rằng khí NO2 có màu nâu đỏ còn N2O4 không có màu). Có hai tế bào chứa tỉ lệ NO2/N2O4 và có đường đi của ánh sáng là l1, l2 khác nhau, giả thiết rằng có thể đặt một áp suất p1 và p2 lên các tế bào để cho hai hỗn hợp đều có độ hấp thụ ngang nhau. Như vậy ta có thể xác định được hằng số cân bằng của phản ứng này. a. Xây dựng biểu thức tính hằng số cân bằng Kp phụ thuộc vào p1, p2 và tỉ lệ r = l1/l2


b. Trong một thí nghiệm thì l1 = 250mm và l2 = 50mm. sử dụng các giá trị thí nghiệm cho dưới đây hãy tính hằng số cân bằng Kp ở nhiệt độ thí nghiệm. Độ hấp thụ AI AII

Thí nghiệm I II

P1, mmHg 2,00 4,00

P2, mmHg 11,00 23,5

c. Tính tỉ lệ độ hấp thụ AI/AII Câu 6 (2,0 điểm): Quan hệ giữa hiệu ứng cấu trúc và tính chất 1. Cho cấu tạo của hợp chất hữu cơ E

Hãy chỉ rõ trạng thái lai hóa của từng nguyên tử N ở cấu tạo E và ghi giá trị Pka (ở 25 oC): 1,8; 6,0; 9,2 vào từng trung tâm axit trong công thức tương ứng với E. Giải thích. 2. Cho dãy hợp chất sau:

a. So sánh khả năng phản ứng thế electrophin của A với benzen và cho biết vị trí phản ứng ưu tiên ở A. Giải thích. b. So sánh nhiệt độ nóng chảy, nhiệt độ sôi của dãy hợp chất trên. Giải thích. Câu 7 (2,0 điểm) : Hidrocacbon 1. Một hidrocacbon mạch hở A có công thức phân tử C10H18 (khung cacbon gồm hai đơn vị isopren nối với nhau theo quy tắc đầu – đuôi). Oxi hóa A bằng dung dịch KMnO4 trong H2SO4, thu được hỗn hợp các chất A1, A2 và A3. Chất A1 (C3H6O) tác dụng với H2 (xúc tác Ni) tạo ancol bậc 2. Chất A2 (C2H4O2) phản ứng được với Na2CO3. Chất A3 (C5H8O3) chứa nhóm cacbonyl (C=O), phản ứng được với Na2CO3. a. Viết công thức cấu tạo của A1, A2, A3 và A. b. Viết công thức các đồng phân hình học của A. 2.a. Hãy hoàn thành sơ đồ phản ứng sau, giải thích sự hình thành X5 và X6: Br2 to

NaI

X1 (C8H 6Br 4)

Mg

X2 (C8H 6Br 2)

X3 (C8H 6)

X4 (C16H12) t o X7 (C16H12)

X5

HCl

X6 (C16H13Cl) ,, (khong mat' mau nuoc brom) -

o-Xilen

b. Hãy tổng hợp nona-2,7-đien từ những hiđrocacbon chứa không quá 5C chỉ nhờ một phản ứng. Câu 8 (2,0 điểm): Xác định cấu trúc, đồng phân lập thể, danh pháp 1. Viết công thức cấu tạo và gọi tên các đồng phân bixiclooctan. 2. Vẽ công thức cấu trúc của các dẫn xuất 1,4-đioxan là sản phẩm đime hóa hợp chất (R)-1,2-epoxi-2-metylpentan. 3. Một peptit X tự nhiên được chiết tách dưới dạng tinh thể màu trắng, có phân tử khối là 485. Thủy phân X và các phương pháp phân tích phù hợp đã xác định được thứ tự sắp xếp các α-amino axit trong X: phenylalanin,


alanin, glyxin, prolin, isoleuxin. Biết rằng X phản ứng với axit nitrơ không giải phóng khí nitơ. Hãy xác định công thức cấu trúc của peptit X; công thức cấu tạo của các α-amino axit như sau: Ph

COOH

COOH

NH2

H2N

COOH NH

NH2

Alanin

Phelylalanin

COOH

COOH

NH2

Prolin

Glyxin

Isoleuxin

Câu 9 (2,0 điểm): Cơ chế phản ứng Hãy trình bày cơ chế của các chuyển hóa sau:

O

O O

a)

MeONa C2H5OH

O

O O

OH OH Ph

O

Ph

CH2O

b)

N

CSA

NH CHPh2

Ph

Ph

Câu 10 (2,0 điểm): Tổng hợp các hợp chất hữu cơ ( Dạng sơ đồ phản ứng).

Loline là một thành viên của họ 1- aminopyrrolizidines ( thường được gọi là lolines ), là một ankaloid . Các lolines là những hợp chất diệt côn trùng, ngăn chặn sự sinh sản của cỏ nấm cộng sinh trong cỏ endophytic thuộc chi Epichloë ( loài anamorphic : Neotyphodium ). Loline được tổng hợp theo sơ đồ sau: O MeOH N

H+

oxh

A

B

OMe OMe HOH2C

HgO

COOMe C

N O

MsCl

D

H2/Pd

E

1. LiAlH4 2. Ac2O

F

H

+

G

H 2/Pt

H

SOCl2 Py

I


HẾT.

Mạc Thị Thanh Hà, SDDT0904769299- GV THPT Chuyên Vĩnh Phúc TRƯỜNG THPT CHUYÊN VĨNH PHÚC

HƯỚNG DẪN CHẤM MÔN HÓA HỌC – KHỐI 11 (Hướng dẫn chấm gồm 11 trang )

ĐỀ THI ĐỀ XUẤT Câu 1

ý 1 a

b

Nội dung Có thể áp dụng nguyên lý trạng thái tĩnh cho Cl và ClO vì chúng là các gốc tự do rất hoạt động do đó có khả năng tạo phức cao dẫn đến tốc độ sinh ra chậm hơn nhiều so với tốc độ tạo thành nên nồng độ của chúng ít bị thay đổi. Áp dụng nguyên lí nồng độ dừng ta có:

d [Cl ] = k1[Cl2 ] − k2 [Cl][O3 ] + k3 [ClO][O3 ]-k4 [Cl ]2 = 0 dt d [ClO] = k2 [Cl][O3 ]-k3 [ClO][O3 ] = 0 dt k ⇒ [ClO ] = 2 [Cl ] k3

k1[Cl2 ] − k2 [Cl][O3 ] + k3 . ⇒ [Cl ] =

0,25

k2 [Cl ][O3 ]-k4 [Cl ]2 = 0 k3

k1 [Cl2 ] k4

d [O3 ] = k2 [Cl][O3 ]+k3 [ClO][O3 ] = 0 dt d [O3 ] k k k1 ⇒− = k2 1 [Cl2 ][O3 ]+k3 . 2 [Cl2 ][O3 ] = 0 dt k4 k3 k 4 −

⇒v=−

Điểm 1,0 0,25

0,25

1 1 d [O3 ] k k k = 2k2 1 [Cl2 ][O3 ] = 2k2 1 [Cl2 ] 2 [O3 ] ⇒ v=2k2 1 [Cl2 ] 2 [O3 ] dt k4 k4 k4

Vậy: * v=2k2

1 k1 [Cl2 ]2 [O3 ] ; k4

0,25

* Bậc của Cl2 là 1/2. 2

1,0


Phản ᝊng

CO

+

Ban Ä‘ầu 1 Cân báşąng (1 – x)

 → â†? 

½ O2

0,5 0,5(1 – x)

→ K P (T ) =

x.(7 − x ) (1 − x )

x

CO2

(N2)

0

2 (mol) (mol)

2

0,25

1 2

3 2

0 0 0 ∆G298 (1) = −257, 2 KJ/mol; ∆S 298 (1) = −86,5 J/mol.K → ∆H 298 (1) ≈ −283, 0 KJ/mol;

0,25

0 ∆ST0 = −86,5 J/mol.K; ∆H T0 = ∆H 298 + ∆C P (T − 298) = −281003, 4 − 6, 7T J/mol;

Ta cĂł:

0,25

∆G = ∆H − T .∆S = − RT ln K P (T ) 0 T

0 T

0 T

281003, 4

→T =

(1)

1

8,314.ln

x.(7 − x ) 2 (1 − x)

3 2

+ 79,8

Mạt khĂĄc, ta cĂł: áťž 298K, khi tấo thĂ nh x mol CO2, nhiᝇt phản ᝊng 283.x (KJ) lĂ m nĂłng 0,25 hᝇ còn lấi tᝍ 298 Ä‘áşżn T

→ T = 298 +

283000 x 102 − 6, 7 x

(2)

Tᝍ (1) vĂ (2): => T ≈ 2653K 2

1 a

b

-

-

+

5Br + BrO3 + 6H → 3Br2 + 3H2O (o) C6H4(OH)COOH + 3Br2 → C6H2(OH)Br3 + 3H+ + 3Br- + CO2 Br2 + AsO2- + 2H2O → 2Br- + H2AsO4- + 2H+ nBr2 sinh ra = 0,6. 10-3 mol nBr2 pᝊ AsO2- = 0,2. 10-3 mol => nBr2 pᝊ axit = 0,4. 10-3 mol n axit/ 10ml = 0,4. 10-3 /3; n axit/ mẍu = 4. 10-3 /3 % Mc7H6O3 = (4. 10-3 /3.138)/1,7614 = 10,45% C6H5OCOOH + NaOH →C6H5OCOONa + H2O Nc7H6O3 = 0,4. 10-3 /3 => Vdd NaOH =40/3(ml) C0C7H5O3Na =

!, " ! "

= 5,7.10-3 (M) (Ä‘ạt C7H5O3Na = NaA )

 → HA + OHA– + H2O â†?  Kb =

#

, . %" #

Kb= 10-11,025

= 10-11,025 => x= 2,32.10-7 (M)

Ph= 7,37 => cháť?n brom thimol xanh Ph=7,6

1,0 0,25

0,25


c

Nếu dùng metyl đỏ thì Ph= 4,2 => [H+] =10-4,2 0,25 Đặt Vdd NaOH đã dùng chuẩn độ = V(ml) Nồng độ ban đầu: 1 .10 HA (axit salixylic): 75 M V + 10 0, 01.V NaOH: : M V + 10 Dừng chuẩn độ trước điểm tương đương nên thành phần của hệ sau chuẩn độ: HA; A– Điều kiện proton: [H+] = [A-] – C0NaOH 0,25 Ka 0 – C0NaOH [H+] = C HA + [H ] + K a

=> 10 ', =

) . *+

% ,-*+

, % , % ,-*+

=> V ≈12,4422 ml

, ., ,

Vtương đương = 40/3 (ml); Sai số q = – 6,684% 2 a

1,0 0,25

Tính pH của dung dịch A NH4Cl →

NH4+ +

Cl -

Trong dung dịch có các cân bằng sau: HAc

 → Ac← 

 → NH3 NH4+ ← 

H2O

 → H+ ← 

+

H+

K1 = 10 - 4,76

+

H+

K1 = 10 - 9,24

+

OH-

Kw=10 -14

K1C1 >> K2C2, KW Bỏ qua sự phân li của nước và NH4+, tính theo: HAc

 → Ac← 

C

0,01

[]

0,01 - x

+

x

Theo đltdkl ta có:

K1 = 10 - 4,76

x..x = K1 = 10−4,76 (0, 01 + x) pH = 3,39.

Giả sử chuẩn độ hết HAc, chưa chuẩn độ NH4Cl vì KNH4 rất nhỏ vì pT = 6,2 << pK : HAc

+

0,25

x

x= [H+] = 4,083.10-4 ⇒ b

H+

NaOH →

NaAc +

H2O

0,25


VTD = 25.0,01/0,02 = 12,50 ml. Thành phần tại điểm tương đương: H2O, NH4+, Ac0,25

-Tính sai số chuẩn độ Ta có : q = P-1 =

CV C01V 0

−1 =

CV − C01V 0 C01V 0

=−

C'HAc C HAc

Theo đk proton mức không C/HAc, ,H2O, NH4+, Ac[H+]= [OH- ] + [NH3] – ([HAc] - C'HAc) => C'HAc = [H+ ] - [OH-] - [NH3] + [HAc]

K  C + C01 C02 K NH +4 h  ⇒ q = − h − W  + 01 − = -0,0169 01 h  CC C K NH + + h K HAc + h  4

3

q =− 1,69 %.

1 Bán phản ứng đầu xảy ra ở mỗi điện cực là + Điện cực A (+) : 2Cl- ⇌ Cl2 + 2e + Điện cực K (-) : Fe3+ + 1e ⇌ Fe2+

2 a

b

0,5 0,25

Trong dung dịch X có C(Fe3+) = 0,3M; C(Fe2+) = 0,03M; C(H+) = 1M; C(Cl-) = 2M; Na+; 0,25 SO42-. Thế khử của mỗi cặp ở mỗi điện cực là: Ea = E(Cl2/2Cl-) = 1,36 + (0,0592/2)lg(1/22) = 1,3422V Ở Ph = 0; không có quá trình proton hóa của ion kim loại, vì vậy ta có: Ec = E(Fe3+/Fe2+) = 0,771 + 0,0592lg(0,3/0,015) = 0,848V => Cần đặt điện thế tối thiểu để xảy ra quá trình oxi hóa Cl- và quá trình khử ion Fe3+ là: V= 1,342 – 0,848 = 0,494 V 1,5 Số mol electron trao đổi trong quá trình điện phân là: 0,25 ne = It/F = 9,65.150/96500 = 0,015 (mol) Các phản ứng xảy ra tại các điện cực: Cực (+): 2Cl- → Cl2 + 2e (1) n 7,5.10-3 ← 0,015 Cực (-) : Fe3+ + 1e → Fe2+ (2) n 0,015 0,015 Theo (1) và (2) và giả thiết cho thấy ion Cl- và Fe3+ đều dư. 0,25 Vậy khối lượng dung dịch giảm là: m = m(Cl2) = 71.0,015/2 = 0,5325 gam. Dung dịch Y có: C(Fe3+)=(0,03-0,015)/0,1= 0,15M; C(Fe2+)=(0,0015+ 0,015)/0,1= 0,165M; C(Cl-)= (0,2 – 0,015)/0,1=1,85M; Na+; SO42-. Có cân bằng: Fe3+ + H2O ⇌ Fe(OH)2+ + H+ β*[Fe(OH)]2+= 10-2,17 (3) Fe2+ + H2O ⇌ Fe(OH)+ + H+ β*[Fe(OH)]+= 10-5,92 (4) + -14 H2O ⇌ H + OH Kw = 10 (5)

0,25


Xét [Fe(OH)2+].[ H+] ≃ 0,15. 10-2,17 >> [Fe(OH)+].[ H+] ≃ 0,165. 10-5,92 >> Kw Vì vậy Ph là do cân bằng (3) quyết định. Xét cân bằng: Fe3+ + H2O ⇌ Fe(OH)2+ + H+ β*[Fe(OH)]2+= 10-2,17 0,15 C0 [] 0,15 – x x x 2 -2,17 x /(0,15 – x) = 10 => x = 0,0286M => pH = 1,544.

4

0,25

c

Theo kết quả tính ở phần (b) và cho thấy ion Fe2+ tạo phức hidroxo không đáng kể, nên ta 0,25 có: E(Fe3+/Fe2+) = 0,771 + 0,0592.lg(0,15- 0,0286)/0,165 = 0,763 (V) Vậy E(pin) = E(cao) - E(thấp) = 0,763 – 0,00 = 0,763 (V) Do E(Fe3+/Fe2+) > E0(2H+/H2) => sơ đồ pin là: 0,25 A(-) Pt,H2 (1 atm)/H+ (1M)// Fe3+(0,077M); Fe2+(0,11M) / Pt (+) K.

1 a

1,0 0,25

Xác định các chất A -> Z: A: NH3

B: NaNH2

D: HN3

E: N2H4

X: H2

Z: [N2H5]+[N3]-

Y: N2 b

C: NaN3

Viết phương trình phản ứng + 2Na → 2NaNH2

2NH3

+

N2H4 → [N2H5] [N3] +

12[N2H5] [N3] 2N2 + 3H2 2NH3

+ NaOH

+ HCl → NaCl

NaN3 +

+ H2

N2O → NaN3

+

2NaNH2 HN3

0,25

-

+ HN3

(2) (3)

(4) +

16NH3

+ 19N2

2NH3

+ H2O2 → N2H4

(NH2)2CO + NaOCl

+ NH3

-

→ 3N2H4 Fe3O4 ⇀ ↽ P ,t

(1)

(6)

+ 2H2O

+ 2NaOH → N2H4

0,25

(5)

(7)

+ H2O + NaCl+ Na2CO3 (8)

2

0,25 1,0

* Theo giả thiết, X phải là phi kim. 0,25

Áp dụng bảo toàn mol electron X

X+n

a

+

N+5

ne a.n

+

1e 0,25

N+4 0,25 (mol)

=> a.n = 0,25 (1) => MX. a = 1,55 (2) (1), (2) => M X =

31 .n 5

=> n = 5 ; MX = 31 => X là P (photpho)


* Xác định các chất trong sơ đồ (a) và (b) : - X tác dụng Ca → G là Ca3P2. - MA

MG = 449 → MG = 449 – 182 = 267, mặt khác X tác dụng với dung

+

dịch Ba(OH)2 tạo A phải là muối → A là Ba(H2PO2)2. 0,25

- G tác dụng với H2O → L là PH3. - MB

ML = 100 → MB = 100 – 34 = 66, mặt khác A tác dụng với H2SO4

+

tạo B → B là H3PO2 - B tác dụng với CuSO4 có tính oxi hóa tạo D tác dụng được với NaOH → D là H3PO4 → E là Na2HPO4 - MD

MM = 180 → MM = 180 – 98 = 82, mặt khác L tác dụng với AgNO3

+

có tính oxi hóa tạo M, M mất nước tạo D → M là H3PO3. - Nhiệt phân D mất nước tạo Q → Q là H4P2O7 MQ = 444 → MF

- MF +

= 444 - 178 = 266, mặt khác nhiệt phân E tạo F

→ F là Na4P2O7 Vậy các chất trong sơ đồ phản ứng đã cho là: A – Ba(H2PO2)2 ; F- Na4P2O7 ;

b

;

B – H3PO2 ;

G - Ca3P2; L – PH3 ;

E – Na2HPO4;

M – H3PO3 ;

Q – H4P2O7

Phương trình phản ứng: 8P

6H2O → 3Ba(H2PO2)2

3Ba(OH)2 +

+

Ba(H2PO2)2

+

→ 2H3PO2

H2SO4

H3PO2 + 2CuSO4 + 2H2O → 2Cu H3PO4

+

2NaOH →

+

0

2P

Ca3P2 PH3 +

0

t C 3Ca →

+

+

3H2O 0

200 C  →

2H3PO4

260 C  →

0

H3PO4

+

+ 2PH3

(2)

+ 2H2SO4

(3)

+

H2O

H2O

(5) (6)

6Ag + 6 HNO3

H4P2O7

0,05x 10

(4)

3Ca(OH)2 + 2PH3

PH3

(1)

BaSO41

Ca3P2

6AgNO3 + 3H2O → 4H3PO3

+

Na2HPO4

600 C → Na4P2O7 2Na2HPO4 

5

D- H3PO4 ;

+ +

(7) +

H3PO3

(8)

3H3PO4

(9)

H2 O

(10)

1 a

b

1,0 –

NiCl2 + 2CN + 2H2O Ni(OH)2↓ (X, xanh) + 2HCN + 2Cl 0,25 – Ni(OH)2 + 4CN [Ni(CN)4]2– (Y, màu vàng) + 2OH– [Ni(CN)4]2– + CN– [Ni(CN)5]3– (Z, màu đỏ) Ni2+ cấu hình d8, ion phức chất [Ni(CN)4]2– nghich từ do vây sẽ lai hóa trong, hai e độc 0,25 thân sẽ ghép đôi. Vói phối trí 4 sẽ phù hợp với dạng dsp2, cấu trúc hình học vuông phẳng. ( có thể suy luận do CN- là phối tử trường mạnh )


Ion phức chất [Ni(CN)5]3– nghịch từ do vậy sẽ lai hóa trong dạng dsp3 lưỡng chóp tam giác. Số phối trí 5 trong [Ni(CN)5]3– (có thể suy luận từ sự lai hóa vì ion d8 chỉ còn tối đa 0,25 5 AO trống trong trường hợp lai hóa trong).

Cấu trúc hình học

0,25

2 a

1,0 0,25

Dựa vào phương trình trạng thái khí lý tưởng ta có: n P C= = V RT Theo định luật Beer có: A = ε cl = kPl , với A là độ hấp thụ, ɛ và k là những hằng s ố. Hằng số cân bằng của phản ứng 2NO2(k) ⇌ N2O4(k) là: pN O 1 xN O 1 (1 − xN2O4 ) K P = 22 4 = . 22 4 = . 2 pNO2 P xNO2 p xNO 2 Với p là áp suất chung của hệ; x là phần mol của NO2 Do độ hấp thụ cuat hai cuvet bằng nhau ⇒ p1(NO2).I1 = p2(NO2).I2 (vì chỉ có NO2 hấp thụ ánh sáng). Biểu thức này có thể viết: p1.x1.(NO2).I1 = p2.x2(NO2).I2 (1)

Tỉ số phản ứng ở các cuvet phải ngang nhau do các hỗn hợp đều cân bằng. 1 (1 − x1 ( NO2 )) ⇒ (2) KP = . p1 x12 ( NO2 ) KP =

1 (1 − x2 ( NO2 )) . p2 x22 ( NO2 )

Từ (1), (2), (3) và r = l1/l2 ta có: r (r − 1).( p2 − rp1 ) KP = ( r 2 p2 − p2 ) 2

(3)

0,25


b

Thay các giá trị thực nghiệm vào các biểu thức ta thu được Thí nghiệm 1: KP = 0,0131 Thí nghiệm 2: KP = 0,0120 ⇒ KP = (0,0131 + 0,0120)/2 = 0,01255

0,25

c

Từ định luật Beer ta có với mỗi tế bào (i = 1, 2)

0,25

p ( NO2 ) p x ( NO2 ) AI = = AII Pi ( NO2 ) p x ( NO2 ) I i II

I I i i II II i i

Với pIi và pIIi là các áp suất chung, xiI và xiII là phần mol của NO2. (1 − x) Áp dụng biểu thức: K P = ⇒ x = [-1 + (1 + 4KP.p)1/2]/2KP.p 2 px Đối với cuvet thứ nhất ta có:

AI p I x I ( NO2 ) [-1+(1+4K P . p1I )1/2 ] = II1 1II = = 0,511 AII p1 x1 ( NO2 ) [ − 1 + (1 + 4 K P . p1II )1/ 2 ] Tương tự đối với cuvet thứ hai có:

AI p I x I ( NO2 ) [-1+(1+4K P . p2I )1/2 ] = II2 2II = = 0,516 AII p2 x2 ( NO2 ) [ − 1 + (1 + 4 K P . p2II )1/2 ] ⇒ AI/AII = (0,511 + 0,516)/ 2 = 0,5235 6

1 sp 2

COOH

N

sp 3 N sp 2

*

H

NH 2

(E)

- Nguyên tử N nhóm NH ở trạng thái lai hóa sp2, cặp e chưa chia ở obitan p xen phủ với 5 obitan p khác tạo thành hệ thơm được lợi về mặt năng lượng nhưng “mất” tính bazơ. - Nguyên tử N thứ hai ở trạng thái lai hóa sp2, cặp e chưa chia ở obitan sp2 không tham gia vào hệ thơm nên còn tính bazơ. - Nguyên tử N nhóm NH2 ở trạng thái lai hóa sp3. * 6,0

1,0 0,25

0,25

0,25

COOH 1,8

H N N

NH 3 9,2

H

- Nhóm NH3+ là axit liên hợp của nhóm H2Nsp3 , nhóm NH+ là axit liên hợp của nhóm Nsp2. - Bazơ càng mạnh thì axit liên hợp càng yếu, vì thế giá trị 9,2 là thuộc nhóm NH3+ còn giá trị 6,0 thì thuộc nhóm NH+. 2 a

0,25

1,0 - Mật độ e π ở mỗi vị trí của A (6e/5 vị trí) lớn hơn ở mỗi vị trí trong vòng benzen (6e/6 0,25 vị trí) nên A dễ tham gia phản ứng thế electrophin hơn benzen. - Sự tạo thành phức σ ở vị trí 2 (ở giai đoạn quyết định tốc độ phản ứng) bền hơn ở vị trí


3 do điện tích dương 0,25

b

* Nhiệt độ nóng chảy: G > E > D > A > B > 0,25 * Nhiệt độ sôi: G > E > D > B > C > A 0,25 * Giải thích: - G, E và D có phân tử khối lớn hơn và có nhiều nhóm phân cực hơn so với A, B, C; - G tạo liên kết hiđro liên phân tử mạnh hơn E nên tonc và tos đều biến đổi theo thứ tự: > E > D > A, B, C. - Ở trạng thái rắn, lực Van deVan (Fv~ p.p’/rn với n ≥ 4) phụ thuộc chủ yếu vào khoảng cách giữa các phân tử (r). Vì r < r < r nên tonc theo giảm theo thứ tự A

B

C

A > B > C. - Ở trạng thái sôi, lực Van deVan phụ thuộc chủ yếu vào điện tích p và p’ của lưỡng cực (vì khi đó khoảng cách giữa các phân tử quá lớn). Vì µ B> µ C> µ A nên tos giảm theo thứ tự B > C > A.

7

1 a

Công thức cấu tạo: A1: CH3 – CO – CH3

A2: CH3 – COOH CH3 - C = CH - CH2 - CH2 - C = CH- CH3

HOOC - CH2 - CH2 - C - CH3

b

A3 : 2 đồng phân hình học H3C

H2C C

H3C

2 a

CH2

H

CH 3

CH3

C

C

Br2

o CH3 t

H3C

C

H3C

CHBr 2

CH3

A:

O

H2C C

H

CH3

CH2

H3C

H

H3C

C CH3

CHBr NaI

(X7)

Mg

CHBr 2

CHBr (X2)

(X1)

0,25 cho mỗi công thức

H

C

C

(X3) (X5)

(X6)

Cl-

H+

to

Cl

b H 3CHC CHCH 3

8

LnM=C<

1,0 0,125 điểm cho mỗi chất

(X4)

1,0 0,05 cho mỗi công thức 0,5

H 3CHC CHCH 2 CH 2CH 2CH=CHCH3

1

0,5 Bixiclooctan có các đồng phân sau:

bixiclo[5.1.0]octan

bixiclo[4.2.0]octan

bixiclo[3.3.0]octan

0,25


bixiclo[4.1.1]octan bixiclo[3.2.1]octan

bixiclo[2.2.2]octan 0,25

2

0,5 Công thức cấu trúc của các dẫn xuất 1,4-đioxan thế khi đime hóa hợp chất (R)-1,2-epoxi-2metylpentan: CH3

CH 3

O

n-C 3 H 7

CH3 C 3 H 7 -n

O

n-C 3 H 7

3

O O

CH 3

C 3 H 7 -n CH3

O

n-C 3 H 7

C 3 H 7 -n CH3

O

O

CH 3

CH 3

O C 3 H 7 -n

n-C 3 H 7

O n-H 7 C 3 H 3C

H 3C

O

O

O

C 3 H 7 -n

O CH3 C 3 H 7 -n

C 3 H 7 -n CH3

n-H 7 C 3 H 3C

CH3

C 3 H 7 -n CH3

O

CH3 C 3 H 7 -n O

0,25

O C 3 H 7 -n

0,25 1,0 * Theo giả thiết, X được tạo bởi các α-amino axit là: Phe: 165; Ala: 89; Gly: 75; Pro: 0,25 115; Ile: 131. Mỗi liên kết peptit tạo thành từ hai α-amino axit sẽ loại đi 1 phân tử nước. Dãy Phe-Ala-Gly-Pro-Ile có 4 liên kết peptit, số khối mất đi là 4x18 = 72, số khối còn lại là 575 - 72 = 503. * Trong khi đó, phân tử khối của peptit X là 485, sự chênh lệch về số khối là 503-485 = 0,25 18, đúng bằng phân tử khối của 1 phân tử nước. Mặt khác, X phản ứng với axit nitrơ không giải phóng khí nitơ, chứng tỏ X không còn nhóm NH2 tự do, tức là X có cấu trúc vòng khép kín. * X là một peptit tự nhiên nên các α-amino axit cấu tạo nên X phải có cấu hình L (L- 0,25 Phenylalanin, L-Alanin, L-Prolin và L-Isoleuxin, Glyxin không có cacbon bất đối nên không có đồng phân quang học). Vậy X có công thức cấu trúc như sau (có thể sử dụng công thức chiếu Fisơ để viết công thức của X): Phe

0,25

Ala

O

NH

O

HN

NH

Gly

O HN

N

Ile O

9

O Pro

2,0


a

b

c

d

10

0,25 x2 (viết đúng được 3 chất cho 0,25đ ) 0,25x 2

0,25x 2 (viết đúng 3 chất cho 0,25) 0,25x 2

2,0 0,25x 3

0,25x 3

0,25x 2


HẾT. HỘI CÁC TRƯỜNG CHUYÊN VÙNG DUYÊN HẢI VÀ ĐỒNG BẰNG BẮC BỘ

ĐỀ THI MÔN HÓA HỌC KHỐI 11 NĂM 2017

TRƯỜNG PT VÙNG CAO VIỆT BẮC

ĐỀ THI ĐÈ XUẤT

Thời gian làm bài 180 phút (Đề này có 4 trang, gồm 10 câu)

Câu 1: (2 điểm) Tốc độ phản ứng- cân bằng hóa học

Xét sự thủy phân của este metyl axetat ở 250C trong các môi trường sau: Trường hợp 1: Trong môi trường kiềm, khi tăng nồng độ kiềm lên gấp đôi thì tốc độ phản ứng tăng lên hai lần. Nhận xét này cũng được thấy khi tăng nồng độ của este lên hai lần. Trường hợp 2: Phản ứng thuỷ phân được thực hiện trong môi trường đệm Trường hợp 3: Người ta tiến hành phản ứng thuỷ phân trong môi trường axit HCl 0,05M dư. Động học của phản ứng được nghiên cứu bằng cách chuẩn độ 25ml hỗn hợp phản ứng bằng dung dịch NaOH ở từng thời điểm t với kết quả như sau : t [phút]

0

21

75

119

VNaOH [cm3]

24,4

25,8

29,3

31,7

47,2

a. Hãy viết phương trình động học của phản ứng. Cho biết bậc của phản ứng trong từng trường hợp. b. Trong TH1: Nếu cho 0,01 mol xút và 0,01 mol este vào 1lit nước (xem như thể tích thay đổi không đáng kể). Sau 200 phút thì 3/5 lượng este chưa bị phân huỷ. Tính hằng số tốc độ phản ứng k1 c. Trong TH 3: Hãy tính hằng số tốc độ phản ứng k3 và thời gian để este phân huỷ hết 50%. Từ đó hãy so sánh giá trị k1 và k3 Câu 2: (2 điểm) Cân bằng trong dung dịch điện li 1. Tính pH của dung dịch H3PO4 0,1M 2. Cần cho vào 100ml dung dịch H3PO4 0,1M bao nhiêu gam NaOH để thu được dung dịch có pH= 4,72. 3. Trộn 20,00 ml dung dịch H3PO4 0,50 M với 37,50 ml dung dịch Na3PO4 0,40 M, rồi pha loãng bằng nước

cất thành 100,00 ml dung dịch A. Cần phải thêm bao nhiêu ml dung dịch HCl 0,050 M vào 20,00 ml dung dịch A để thu được dung dịch có pH =5,00 (metyl đỏ đổi màu). Cho: H2SO4 : pKa2 = 2 ; H3PO4 : pKa1 = 2,23 , pKa2 = 7,21 , pKa3 = 12,32 Câu 3: (2 điểm) Điện hoá học


0 Xét khả năng phản ứng của Cl-, Br- với KMnO4.Biết E Br

2

= 1,085V ; ECl0

/ 2 Br −

2

/ 2 Cl −

= 1,359V ;

0 E MnO = 1,51V . − / Mn 2 + 4

a. Ở pH=0 b. Trong dung dịch axit axetic 1,00 M. Biết CH3COOH có Ka=10-4,76. Câu 4: (2 điểm) Bài tập tính toán vô cơ tổng hợp

Kim lo¹i A ph¶n øng víi phi kim B t¹o hîp chÊt C mµu vµng cam. Cho 0,1 mol hîp chÊt C ph¶n øng víi CO2 (d-) t¹o thµnh hîp chÊt D vµ 2,4 gam B. Hßa tan hoµn toµn D vµo n-íc, dung dÞch D ph¶n øng hÕt 100 ml dung dÞch HCl 1 M gi¶i phãng 1,12 l khÝ CO2 (®ktc). H·y x¸c ®Þnh A, B, C, D vµ viÕt c¸c ph-¬ng tr×nh ph¶n øng x¶y ra. BiÕt hîp chÊt C chøa 45,07 % B theo khèi l-îng; hîp chÊt D kh«ng bÞ ph©n tÝch khi nãng ch¶y. Câu 5: (2,0điểm) Phức chất.

Cấu hình electron của nguyên tố M ở trạng thái cơ bản chỉ ra rằng: M có 4 lớp electron, số electron

độc thân của M là 3. a. Dựa vào các dữ liệu trên cho biết M có thể là các nguyên tố nào. b. M tạo được ion phức có công thức [M(NH3)6]3+, phép đo momen từ chỉ ra rằng ion này là nghịch từ.

- Cho biết tên gọi của [M(NH3)6]Cl3 - Cho biết trạng thái lai hoá của M trong ion phức trên và chỉ ra dạng hình học của ion phức này. Câu 6: ( 2 điểm) Quan hệ giữa cấu trúc và tính chất

1. Sắp xếp (có giải thích) theo trình tự tăng dần tính axit của các chất trong từng dãy sau: a. Axit: benzoic, phenyletanoic, 3-phenylpropanoic, xiclohexyletanoic,

1-metylxiclohexan-cacboxylic. COOH

b.

N

;

COOH

;

; N

(A)

CH2COOH

COOH

(D)

(C)

(B)

2. Sắp xếp (có giải thích) theo trình tự tăng dần nhiệt độ nóng chảy của các chất sau: COOH

S

(A)

COOH

COOH ;

; N

(B)

(C)

Câu 7: ( 2 điểm) Hidrocacbon

Trộn hiđrocacbon khí A với oxi theo tỉ lệ thể tích A:O2 = 1:9 rồi cho vào bình kín thấy áp suất trong bình là 1 atm ở 00C. Bật tia lửa điện để A cháy hết, hh sau pư có áp suất là 1,575 atm ở 136,50C. a.Tìm công thức phân tử của A?


b. Viết công thức cấu tạo có thể có của A biết tất cả các nguyên tử cacbon trong A đều có cùng một dạng lai hóa? c. Chọn công thức cấu tạo của A ở trên để viết sơ đồ tổng hợp :

+ Chất B(Anthracen) có CTCT như sau: + bixiclo[ 4.1.0] heptan + Đietyl phtalat( DEF) Câu 8: (2,0 điểm) Xác định cấu trúc, đồng phân lập thể, danh pháp

Hợp chất A đóng vai trò là chất gốc để tạo thành các hợp chất E1 và E2

(Gợi ý: Các hợp chất E1 và E2 là trans-diol có tỉ lệ 1: 1). 1. Xác định cấu trúc của A, D, E1 và E2 trong sơ đồ phản ứng trên. 2. Viết tên các hợp chất A, B, C, D, E1 và E2. 3. Thêm các chất phản ứng cần thiết bổ sung ở những nơi có dấu "?". 4. Đánh dấu tất cả các trung tâm bất đối của các hợp chất A, B, C, D, E1 và E2. Câu 9: (2,0 điểm) Cơ chế phản ứng

sau:

2,7-đimetylnaphtalen có thể được tổng hợp bằng phản ứng giữa tác nhân Grignard A và axetal B theo sơ đồ

i MgBr

A CHO O

O

ii O

+A/H+

O

B

a. Đề nghị điều kiện tạo thành A và B. Viết phương trình phản ứng b. Trình bày cơ chế phản ứng tạo thành 2,7-đimetylnaphtalen


Câu 10. (2 điểm): Tổng hợp các chất hữu cơ

Xác định các chất A, B, C, D,E trong sơ đồ phản ứng sau: COOH OH

AlCl3

(CH3CO)2O A

N H

Br2 (CHCl3) B

C

D LiAlH4 H2/Pd-C

Salbutamol

E


ĐÁP ÁN VÀ THANG ĐIỂM Câu

Nội dung

Đ iể m 2,0

Câu 1

a.

Nhận xét: TH1: v = k1[este]a.[OH-]b → khi tăng nồng độ của este hoặc bazơ lên gấp hai lần thì v cũng tăng lên gấp đôi → a = b = 1 → v = k1[este][OH-] Vậy trong TH1 bậc của phản ứng là bậc 2 TH2: Ta có v = k[este][OH-] . Nhưng trong môi trường đệm nên [OH-] = const → v =

0,5

k2[este] với k2 = k[OH-] TH3: v = k[este][axit] Do axit lấy dư nên k[axit] = const =k3 → v = k3[este] Vậy trong TH2, 3 bậc phản ứng là bậc 1

b.

Vì nồng este và axit bằng nhau và [A0] = 0,01M nên ta có 1 1 − = kt với [A0] là nồng độ của este; [A0] – x là nồng độ của este tại [ A0 ] − x [ A0 ]

thời điểm t Lượng este chưa bị thuỷ phân là 2/5[A0] →

1 2 [ A0 ] 5

1 = kt [ A0 ]

0,75

→ k= 0,75 mol-1.L.phút-1 = k1

c.

Ta có k =

[ A0 ] 2,303 lg t [ A0 ] − x

Nếu V∞ ở thời điểm t = ∞ là thể tích ứng với sự kết thúc thuỷ phân este trong môi trường axit, V0 là thể tích ứng với thời điểm t= 0 thì hiệu V∞ - V0 sẽ tỉ lệ với nồng độ

đầu của este. Còn hiệu V∞ - Vt sẽ tỉ lệ với nồng độ este tại thời điểm t ( Vt là thể tích ứng với thời điểm t ). Do đó: t [phút]

21

75

119

VNaOH [cm3]

25,8

29,3

31,7

k3 phút-1

0,003016

0,003224

0,003244

k 3 = 0,003161 phut −1 Vậy k1 gấp k 3 khoảng 237,27 lần.

0,75


t k 3 = ln2 → t = 219 phút Câu 2 1

2,0 H+ + H2PO4- (1) K1 = 10-2,23

H3PO4

H2PO4-

H+ + HPO42- (2) K2 = 10-7,21

HPO42-

H+ + PO43-

(3) K3 = 10-12,32

H2 O

H+ + OH-

(4) Kw

K3 << K2 << K1 ⇒ chủ yếu xảy ra cân bằng (1) H+ + H2PO4- K1 = 10-2,23

H3PO4 C(M)

0,5

0,1

[ ](M) 0,1 – x

x

x

x2 -2,23 ⇒ x2 + 10-2,23 x – 10-3,23 = 0 (0,1 - x) = 10 ⇒

NaOH + H3PO4

x = 0,0215 (M)

⇒ pH = 1,66

= NaH2PO4 + H2O

NaOH + NaH2PO4 = Na2HPO4 + H2O NaOH + Na2HPO4 = Na3PO4 + H2O Trung hòa nấc 1:

2

pH1 =

pK1 + pK2 2.23 + 7.21 = = 4,72 2 2

0,5

⇒ trong dung dịch thu được có pH = 4,72 chỉ chứa NaH2PO4:

nH3PO4 = 0,1 x 0,1 = 0,01 (mol) ⇒ nNaOH = 0,01 (mol) mNaOH = 0,01 x 40 = 0,4(g) CH3PO4 =

0,50.20 0,40.37,5 = 0,10 (M); C Na 3PO4 = = 0,15 (M). 100 100

C Na3PO4 = 1,5. CH3PO4 → phản ứng xảy ra như sau: 3

H3PO4 + PO34 0,1

0,15

0

0,05

H 2 PO-4 + PO34

= HPO 2+ 4 0,1 =

2 HPO 24

0,1

0,05

0,1

0,05

0

0,2

H 2 PO-4

10,17 K1 = Ka1. K -1 a3 = 10

0,1 5,11 K2 = Ka2. K -1 a3 = 10

Dung dịch A thu được là hệ đệm gồm: H 2 PO-4 0,05 M và HPO 24 0,2 M

1,0


pH = 5,00 ≈ pH (NaH 2PO4 ) ≈

pK a1 + pK a2 = 4,68 → có thể coi lượng HCl thêm vào 20,00 2

ml dung dịch A sẽ phản ứng vừa đủ với HPO 24 tạo thành H 2 PO 4 : + HPO 2→ H 2 PO-4 4 + H

→ VHCl =

0,2.20 = 80 (ml) 0,05 2,0

Câu 3

a

Các cặp oxi hoá- khử: 2Br-

Br2 + 2e

0 E Br

2

/ 2 Br −

2Cl-

Cl2 + 2e MnO4- + 8H+ + 5e

= 1,085V

ECl0

Mn2+ + 4H2O

2

/ 2 Cl −

(1)

= 1,359V

0 E MnO = 1,51V − / Mn 2 + 4

(2) (3)

Từ (1) và (2) ta thấy thế của các cặp không phụ thuộc vào pH( trong môi trường axit), tuy vậy thế của cặp MnO4-/Mn2+ lại phụ thuộc pH:

[ [

][ ] ]

MnO4− 8 0,0592 0,0592 E=E + . lg . H + → E = E0 + . lg H + 2+ 5 5 Mn 0

[ ]

E = E 0 − 0,095 pH + 0,0118 lg

[MnO ] [Mn ]

8

[ [

] ]

MnO4− 0,0592 + . lg 5 Mn 2+

0,5

− 4 2+

(4)

Ở pH = 0 Ở điều kiện tiêu chuẩn E = E 0 = 1,51V > E Cl0

2

/ 2 Cl −

0 > E Br

2

/ 2 Br −

. Vì vậy trước hết MnO4-

oxi hoá Br- thành Br2 và sau đó Cl- thành Cl2.

0,5

b

Trong dung dịch CH3COOH 1,00M H+ + CH3COO-

CH3COOH C

1,00

[]

1,00-x

x

x

Ka = 10-4,76


[H+] = x =

1,0

K a = 10-2,38 → pH = 2,38

Từ (4) rút ra: E = E0- 0,095pH = 1,51- 0,095.2,38 = 1,28V 0 Bởi vì E Br

2

/ 2 Br −

< E < ECl0

2

/ 2 Cl −

, nên trong dung dịch CH3COOH 1M, MnO4- chỉ oxi hoá

được Br- thành Br2 mà không oxi hoá được Cl- thành Cl2. 2,0

Câu 4

nHCl = 0,1 mol

; nCO2 = 0,05 mol

Dung dÞch D ph¶n øng hÕt 0,1 mol HCl gi¶i phãng khÝ CO2 →

nH + nCO2

=

0, 1 2 = 0, 05 1

suy ra h¬p chÊt D lµ muèi cacbonat kim lo¹i. h¬p chÊt D kh«ng bÞ ph©n tÝch khi nãng ch¶y, vËy D lµ cacbonat kim lo¹i kiÒm.

2 H+ + CO32- = H2O + CO2

C + CO2 = D + B → C lµ peroxit hay superoxit, B lµ oxi.

§Æt c«ng thøc ho¸ häc cña C lµ AxOy . L-îng oxi trong 0,1 mol C (AxOy ) lµ 16 x 0,05 + 2,4 = 3,2 (g); mC =

3, 2.100 = 7,1 45, 07

gam Mc = 7,1 : 0,1 = 71 (g/mol). x:y=

mA trong C = 7,1 - 3,2 = 3,9 (g).

3, 9 3, 2 → MA = 39 (g). VËy A lµ K ; B lµ O2 ; C lµ KO2 ; D lµ K2CO3 : M A 16

K + O2 → KO2

C¸c ph-¬ng tr×nh ph¶n øng:

4 KO2 + 2 CO2 → 2 K2CO3 + 3O2 ↑ K2CO3 + 2 HCl → 2 KCl + H2O + CO2 ↑

Câu 5 1

2,0 Vì có 4 lớp điện tử do vậy phân lớp cuối cùng trong phân bố điện tử chỉ có thể là 4S, 3d, 4P. Vì có 3 điện tử độc thân do vậy, phân lớp cuối cùng chỉ có thể là 3

2

2

6

2

6

3

2

3d → Cấu hình hoàn chỉnh 1S 2S 2P 3S 3P 3d 4S Nguyên tố 23V 3d7 → Cấu hình hoàn chỉnh 1S2 2S2 2P6 3S2 3P6 3d7 4S2 Nguyên tố 27Co 3P3 → Cấu hình hoàn chỉnh 1S2 2S2 2P6 3S2 3P6 3d10 4S2 4P3 Nguyên tố 33As

1,0


2

Tạo phức với NH3 có công thức [M(NH3)6]3+ do vậy không thể là As. Vì phức nghịch từ do vậy không có điện tử độc thân → M chỉ có thể là Coban [CO] Tên gọi [CO(NH3)6]Cl3

:

Hexa amin coban (III) Clorua:

1,0

CO3+: 1S2 2S2 2P6 3S2 3P6 3d6 Vì NH3 là phối tử trường mạnh do vậy khi tạo phức có sự dồn 2 electron vào vậy:

NH3 NH3 NH3 NH3NH3NH3

Vậy Co lai hoá d2sp3 Hình dạng phân tử bát diện: NH3 NH3

H3N Co NH3

H3N NH3

Câu 6

2,0 a

H3C COOH

+I2

CH2COOH

+I1

< <

+I1

< +I2

-I1CH2CH2COOH

CH2COOH

-I2

<

< -I1

<

-I2

<

COOH

-I3

0,75

-I3

Các gốc hiđrocacbon có hiệu ứng +I lớn thì Ka giảm và -I lớn thì Ka tăng

b

0,75


-I1

CH2COOH

COOH -I2

<

<

(D)

C O H O (A)

-C3 N -I3

(C)

COOH -I4 < N -C4 (B)

- I1 < - I2 nên (C) có tính axit lớn hơn (D).

Vì:

(A) và (B) có N nên tính axit lớn hơn (D) và (C) (A) có liên kết hiđro nội phân tử làm giảm tính axit so với (B).

2

Tăng dần nhiệt độ nóng chảy của các chất:

COOH

COOH <

Vì:

<

S (A)

(C)

0,5

COOH

N (B)

M C < MA. (B) có thêm liên kết hiđro liên phân tử với N của phân tử khác.

Câu 7

2,0 Gọi CxHy là công thức của A ta có:

Mol:

CxHy +(x+

y y ) O2 → xCO2 + H2O 4 2

a

y ) 4

a(x+

ax

a

y 2

+ Số mol khí trước pư = a + 9a = 10a mol 1

+ Số mol khí sau pư = ax + a

V bình kín =

y y y + 9a – a(x+ ) = 9a + a 2 4 4

0,5

10a.273 (9a + 0, 25ay ).409,5 n1RT1 n 2 RT2 hay: = y = 6. = P1 P2 1 1,575

+ Vì A là chất khí nên A có thể là: C2H6; C3H6 và C4H6 0,5 đ

Vì tất cả các nguyên tử C trong A đều có cùng một dạng lai hóa nên CTCT của A: CH3-CH3(sp3); (CH2)3(xiclopropan = sp3); CH2=CH-CH=CH2(sp2) và 2

CH

CH2

0,5

(sp3)

CH2 CH

3

Ta chọn A là buta-1,3-đien để hoàn thành sơ đồ: buta-1,3-đien → butan → etilen.


1,0 CH2

HC HC

+

+ C H2 C H2

CH2

Cho sp cuối cùng trong sơ đồ pư với H2/Ni, t0 thì thu được B

CH2N2 Cu,t0

CH2=CHCH=CH2

0,5 đ

Br + CH2=CHCH=CH2

NBS Br -HBr

+ C2H5OH

C6H4(CO)2O

SP

-H2

+O2

Câu 8 OH

*

* *

A

D

*

*

OH

*

S

R R

O

1,4

OH

OH

S

*

OH

E1

*

OH

E2

OH

1,0

C

A: xiclohexanol; B: xiclohexen; C: cis-xiclohexan-1,2-diol; D: xiclohexanepoxit E1: (1R, 2R)-xiclohexan-1,2-diol; E2: (1S, 2S)-xiclohexan-1,2-diol 2,3

B B

KMnO4 lo·ng, l¹nh mCPBA

D

1,0

C +

H3O

E1 + E2

axit meta-Chloroperoxybenzoic (mCPBA) là một axit peroxycarboxylic sử dụng rộng rãi như một chất oxy hóa trong tổng hợp hữu cơ.


Câu 9

2,0

Mg/THF

NBS

MgBr M Br

A

HO O

a. CHO

H+

O

1,0

O

HO O O

O

B

O O

O

A +

HC

H+/H2O B

OH

OMgBr

OH

OH

b.

HC+

SE

H+

OH

O OH

1,0 -2H2O

Câu 10

2,0 A: HO

B:

O O

HO

O


C:

HO

D:

O

HO

OH

OH

N

Br

O

O HO

OH E:

N OH

O


TRƯỜNG THPT CHUYÊN LÊ QUÝ ĐÔN ĐIỆN BIÊN

ĐỀ THI MÔN HOÁ HỌC LỚP 11 Thời gian làm bài : 180 phút;

ĐỀ THI ĐỀ XUẤT ĐỀ BÀI Câu 1 (2 điểm): Động học

Để phân hủy hiđro peoxit (H2O2) với chất xúc tác là ion iođua trong dung dịch có môi trường trung tính, người ta trộn dung dịch H2O2 3% (chấp nhận tương đương với 30 gam H2O2 trong 1 lít dung dịch) và dung dịch KI 0,1 M với nước theo tỉ lệ khác nhau về thể tích để tiến hành thí nghiệm xác định thể tích oxi ( VO2 ) thoát ra. Thí nghiệm

VH2O2 (ml)

VH 2O (ml)

VKI (ml)

υ O2 (ml/phút) ở 298 K và 1 atm 1 25 50 75 4,4 2 50 50 50 8,5 3 100 50 0 17,5 4 50 25 75 4,25 5 50 100 0 16,5 1. Xác định bậc phản ứng phân huỷ đối với H2O2 và đối với chất xúc tác I-. 2. Viết phương trình hoá học và biểu thức tính tốc độ phản ứng. 3. Tính nồng độ mol của H2O2 khi bắt đầu thí nghiệm 4 và sau 4 phút. 4. Cơ chế phản ứng được xem là một chuỗi hai phản ứng sau: H2O2 + I-

k

1 → H O + IO 2

k2

(1)

-

IO + H2O2 → O2 + I + H2O (2) Hãy cho biết hai phản ứng này xảy ra với tốc độ như nhau hay khác nhau? Phản ứng nào quyết định tốc độ phản ứng giải phóng oxi? Giải thích. Câu 2 (2 điểm): Cân bằng trong dung dịch điện li

Trộn 10,00 mL dung dịch CH3COOH 0,20 M với 10,00 mL dung dịch H3PO4, thu được dung dịch A có pH = 1,50. 1. Tính CH3PO 4 trong dung dịch H3PO4 trước khi trộn. 2. Tính độ điện li của CH3COOH trong dung dịch A. 3. Thêm từ từ Na2CO3 rắn vào dung dịch A cho đến pH = 4,0, thu được dung dịch B. Tính số gam Na2CO3 đã dùng. Cho biết: H3PO4: pKa1 = 2,15; pKa2 = 7,21; pKa3 = 12,32; CH3COOH: pKa = 4,76; CO2 + H2O có pKa1 = 6,35; pKa2 = 10,33;

Câu 3 (2 điểm): Điện hóa học

Cho sức điện động của pin: AgCl Ag AgNO3 0,001M Na2S2O3 0,10M HCl 0,05M 1. Viết phương trình phản ứng khi pin hoạt động . 2. Tính

0 E Ag ( S O ) 3 − / Ag 2

3 2

Ag Là 0,341V.


3. Tính KsAgCl . 4. Thêm 0,01 mol KCN vào 1 lít dung dịch ở anot .Tính Epin 0 Cho: E Ag =0,80V , Ag+ + 2S2O32Ag(S2O3)23- lgβ1 =13,46 + / Ag

Ag+ + 2CN-

Ag(CN)2- lgβ2 = 21

Câu 4 (2 điểm): N –P, C- Si và hợp chất 1. Cho 3 gam kim loại M có hoá trị 2 tác dụng với dung dịch HNO3 đặc, nóng, dư. Khí tạo thành

cho hấp thụ hoàn toàn vào dung dịch NaOH vừa đủ , thu được dung dịch X. Cô cạn dung dịch X rồi nhiệt phân hoàn toàn chất rắn thu được trong môi trường trơ thì được chất Y. Lượng chất Y này làm mất màu vừa hết 0,5 lit dung dịch KMnO4 0,2M trong môi trường H2SO4. a) Xác định kim loại M. b) Viết các phương trình phản ứng. 2. Trong điều kiện thường, NF3 là chất khí không màu, hóa lỏng ở -129 độ C, hóa rắn ở -2090C. Còn NH3

cũng là chất khí nhưng hóa lỏng ở -33,350C, hóa rắn ở -77,750C. Amoniac dễ tham gia phản ứng cộng, có tính khử, còn NF3 thì không có những tính chất này. a) So sánh góc HNH và góc FNF, giải thích ? b) Dựa vào cấu trúc phân tử, hãy giải thích sự khác nhau về các hiện tượng thực nghiệm trên. Câu 5 (2 điểm): Phức chất

Hòa tan 1,066 gam tinh thể hiđrat A của một kim loại M tan trong nước được dung dịch màu xanh. Cho dung dịch này tác dụng với dung dịch AgNO3 dư, thu được ngay 1,148 gam kết tủa trắng X và dung dịch B. Kết tủa X là chất dễ bị phân hủy. Đun nóng B với H2O2 trong môi trường kiềm thu được dung dịch C. Cho C tác dụng với dung dịch BaCl2 dư thu được 1,013 gam kết tủa Y màu vàng; Y đồng hình với BaSO4. Dung

dịch của A nếu pha loãng hoặc đun nóng đều có màu sắc thay đổi. Khi theo dõi quá trình phân hủy nhiệt của A trong môi trường HCl thấy trước khi A bị phân hủy ở khoảng nhiệt độ <1800C thì khối lượng giảm 6,75%. Nếu ở 3000C thì khối lượng giảm 40,52%. Hãy xác định các công thức của A, B, C, X, Y và viết các phương trình hóa học xảy ra trong các thí nghiệm trên. Câu 6 (2,0 điểm): Quan hệ giữa hiệu ứng cấu trúc và tính chất 1. Vẽ cấu trúc của 1,3-diazole (imidazole, C3H4N2), anion imidazol-1-yl, cation imidazolyl, 1,3oxazole (oxazole, C3H3NO) and 1,3-thiazole (thiazole, C3H3NS). Cấu trúc nào thơm? 2. Sắp xếp imidazole, 1,3-oxazole and 1,3-thiazole theo chiều giảm dần nhiệt độ sôi, nhiệt độ nóng chảy, giải thích? 3. Sử dụng cấu trúc viết quá trình ion hóa imidazole, oxazole, và thiazole trong nước. Sắp xếp theo chiều giảm dần tính bazơ và giải thích? Câu 7 (2,0 điểm): Hidrocacbon


1. Hợp chất 2,2,4-trimetylpentan (A) được sản xuất với quy mô lớn bằng phương pháp tổng hợp xúc

tác từ C4H8 (X) với C4H10 (Y). A cũng có thể được điều chế từ X theo hai bước: thứ nhất, khi có xúc tác axit vô cơ, X tạo thành Z và Q; thứ hai, hiđro hoá Q và Z. a) Viết các phương trình phản ứng để minh họa và tên các hợp chất X, Y, Z, Q theo danh pháp IUPAC. b) Ozon phân Z và Q sẽ tạo thành 4 hợp chất, trong đó có axeton và fomanđehit, viết cơ chế phản ứng. 2. Hợp chất A có công thức phân tử C9H16. Khi cho A tác dụng với hiđro dư, xúc tác Ni, thu được

hỗn hợp D gồm ba chất đồng phân của nhau là D1, D2, D3 có công thức cấu tạo lần lượt là:

(D1)

(D2)

(D3)

a) Hãy xác định công thức cấu tạo của A. b) Viết công thức cấu dạng bền có thể có của D1 và D2. Câu 8 (2,0 điểm): Xác định cấu trúc, đồng phân lập thể, danh pháp

Khi xử lí chất A (C13H18O2) bằng dung dịch HCl loãng, thu được chất B (C11H14O) không quang hoạt. Khi B phản ứng với Br2/NaOH, sau đó axit hóa sản phẩm phản ứng, thu được chất C. Khi đun nóng B với hiđrazin/KOH trong etylen glicol, thu được chất D. Đun B với benzanđehit trong môi trường kiềm, thu

được chất hữu cơ E (C18H18O) duy nhất. Khi bị oxi hóa mạnh, các chất B, C, D và E đều cho axit phtalic (axit benzen-1,2-đicacboxylic). Xác định công thức cấu tạo của các chất A, B, C, D và E. Câu 9 (2,0 điểm): Cơ chế phản ứng

Cho các quá trình phản ứng sau: a)

COPh

Et

O Et

(1)

Ph

Ph

(2)

AlCl3 /xilen

COEt

AlCl3 /xilen

o

o

t , 48h H = 35%

t , 4h H = 90%

b) O

Br H

O

o

KOH/EtOH, t

Br

OH

O O

Dùng cơ chế để giải thích sự tạo thành sản phẩm trong các quá trình phản ứng trên. Câu 10 (2,0 điểm): Tổng hợp các hợp chất hữu cơ ( Dạng sơ đồ phản ứng). Cấu tạo của hợp chất K (tách từ quả hồi) đã được xác định theo sơ đồ phản ứng sau: K (C 7H10O 5)

O3

Me 2S

L (C 7H 10 O7 )

CH3OH H+

M

HIO4

N

H 3O+

OHCCHO + OHCCH(OH)CH2COCOOH

1. Hãy vẽ công thức cấu tạo của L, M, N và K, biết rằng K không chứa nhóm chức ancol bậc ba. 2. Hãy viết sơ đồ phản ứng tổng hợp ra K từ những hợp chất chứa không quá 4C.


-------------------Hết--------------------------

HƯỚNG DẪN CHẤM ĐỀ ĐỀ XUẤT LỚP 11 Câu

Nội dung 1. Từ phương trình phản ứng: 2 H2O2 → 2 H2O + O2 ta có: thể tích oxi thoát ra trong 1 đơn vị thời gian tỉ lệ thuận với tốc độ phản ứng Theo các thí nghiệm 1, 2, 3 khi tăng gấp đôi thể tích dung dịch H2O2 và giữ nguyên thể tích của dung dịch KI thì tốc độ phản ứng tăng gấp đôi, điều đó có nghĩa là tốc độ phản ứng tỉ lệ thuận với nồng độ của H2O2 → phản ứng là bậc 1 đối với H2O2. Tương tự, từ các thí nghiệm 2, 4, 5 ta thấy tốc độ phản ứng tỉ lệ thuận với nồng độ của I- → phản ứng là bậc 1 đối với I-.

Đ iể m

0,25 0,25

2. Phương trình phản ứng: 2 H2O2 → 2 H2O + O2 Biểu thức của định luật tốc độ phản ứng: V = k . CH 2O2 .CICâu 1

0,25

3. Khi pha loãng 3 lần thì nồng độ của H2O2 (C0) ở thí nghiệm 4 giảm 3 lần: 10 = 0,294 M. → C0 = 10 gam H2O2/1 lit. Hay C0 = 34 Vì phản ứng xảy ra chậm nên có thể coi như tốc độ phản ứng (thể tích oxi thoát ra) không thay đổi trong khoảng thời gian ngắn (4 phút). 0,25 Sau 4 phút sẽ thoát ra: 4,25 . 4 = 17 (ml) oxi, khi đó:

n O2 =

P.V 1 . 17.10-3 = = 0,695.10-3 (mol) R.T 0,082 . 298

Lúc đầu có: n H 2O2 = 0,294 . 0,15 = 44,1.10-3 (mol). Sau 4 phút, số mol H2O2 chỉ còn: 44,1.10-3 – 2 . 0,695.10-3 = 42,71.10-3 (mol). 0, 04271 Vậy sau 4 phút: CH 2 O2 = = 0,285 (M). 0,15

0,25

0,25 4. Phản ứng: -

I 2 H 2 O2  → 2 H 2O + O2

v=−

(*)

1 d [ H 2 O2 ] 2 dt

Cơ chế: k1 H 2 O 2 + I-  → H 2 O + IO-

k2 IO- + H 2 O 2  → H 2 O + I- + O2

Xét 3 trường hợp:

(1) (2)


1/ Nếu phản ứng (1) chậm và quyết định tốc độ thì tốc độ của phản ứng tổng hợp (*) bằng tốc độ của phản ứng (1): 1 d [ H 2O2 ] v=− = k1[H 2 O2 ][I- ] 2 dt 0,125 Cơ chế phù hợp với định luật tốc độ. 2/ Nếu phản ứng (2) chậm thì: 1 d [ H 2 O2 ] v=− = k2 [H 2 O 2 ][IO- ] (a) 2 dt Chấp nhận nồng độ của IO- là ổn định ta có:

k d [IO- ] = k1[H 2 O2 ][I- ] − k2 [IO- ][H 2 O2 ] = 0 → [IO- ] = 1 [I- ] dt k2

(b)

Thay (b) vào (a) ta được: 1 d [ H 2O2 ] v=− = k1[H 2 O2 ][I- ] 0,125 2 dt Cơ chế phù hợp với định luật tốc độ. 3/ Nếu hai phản ứng có tốc độ xấp xỉ nhau thì: 1 d [H 2 O 2 ] 1 v=− = ( k1[H 2 O 2 ][I- ] + k2 [H 2 O 2 ][IO - ]) 2 dt 2 Chấp nhận nồng độ của IO là ổn định, rồi tính [IO-] như ở trường hợp 2 và thay vào biểu thức trên ta được: 1 d [ H 2 O2 ] = k1[H 2 O2 ][I- ] v=− 0,125 2 dt Cơ chế phù hợp với định luật tốc độ. Trong 3 trường hợp, trường hợp đầu hợp lí hơn cả vì ở đây không cần chấp nhận điều kiện gì; mặt khác ở trường hợp 2, nếu đã giả thiết phản ứng (2) là chậm thì việc chấp 0,125 nhận nồng độ của IO- ổn định là không hợp lí. 1. pHA = 1,50 → không cần tính đến sự phân li của nước Các quá trình xảy ra trong dung dịch A:

⇌ H+ + H 2 PO-4

CH3COOH ⇌ H + CH3COO

Ka1 = 10-2,15

HPO 24

Ka3 = 10-12,32

H3PO4

+

H 2 PO -4

⇌H + +

-

HPO 24

⇌ H+ + PO34 ⇌

Ka = 10

-4,76

Ka2 = 10

-7,21

(1) (2) (3) (4)

0,25

Vì Ka1 >> Ka >> Ka2 >> Ka3 nên pHA được tính theo (1):

Câu 2

H3PO4 [] 2. []

0,5C – 10−1,5

H+ + H 2 PO-4 10−1,5

Ka1 = 10-2,15

10−1,5 → C H 3 PO 4 = C = 0,346 M

CH3COOH ⇌ H+ + CH3COO0,1-x 10−1,5 x

→ x = 5,49.10−5 M → αCH3COOH =

0,25 Ka = 10−4,76 0,25

5, 49.10−5 .100 = 0,055% 0,1

0,25


[H2 PO −4 ] K a1 10 −2,15 [H 2 PO −4 ] = = → = 0,986 [H3PO 4 ] [H + ] 10 − 4 [H 2 PO −4 ] + [H3PO 4 ]

3. Tại pH = 4,00 ta có:

K 10 −7, 21 [HPO24 − ] − = a2 = = 10−3,21 → [ HPO 24 ] << [H 2 PO 4 ] − − 4,0 + [H2 PO 4 ] [H ] 10

0,25

[CH3COO − ] [CH3COO− ] K a 10 −4,76 = + = − 4, 0 → = 0,148 [CH3COOH] [H ] 10 [CH 3COO − ] + [CH3COOH] Tương tự:

[CO32 − ] 10−10,33 = << 1 → [ CO32− ] << [ HCO3− ]; [HCO3− ] 10− 4 [HCO3− ] 10−6,35 = << 1 → [ HCO3− ] << [CO2]. 10− 4 [CO2 ]

0,25

Như vậy khi trung hòa đến pH = 4,00 thì chỉ có 14,8% CH3COOH và 98,6% nấc 1 của H3PO4 bị trung hòa, còn bản thân Na2CO3 phản ứng với H+ của 2 axit tạo thành CO2: 2 H3PO4 + CO32− → 2 H 2 PO −4 + CO2 + H2O 2 CH3COOH + CO32− → 2 CH3COO− + CO2 + H2O

n CO2- = 0,5.(14,8%. n CH3COOH + 98,6%. n H3PO4 )= 0,5.20.10−3(14,8%.0,1+ 98,6%.0,173)

0,25

3

→ n CO2- = 1,85.10−3 (mol) → m Na 2CO3 = 0,1961 (gam) 3

0,25 +

2S2O32-

3-

1+2. Ở điện cực trái : Ag + Ag(S2O3)2 0,001 0,10 0,098 0,001 23Do S2O3 rất dư nên [Ag(S2O3)2 ] ≈ 0,001(M) Xét cặp Ag(S2O3)23-/Ag: Ag(S2O3)23Ag+ + 2S2O32- - lgβ1

Ag+ + e Câu 3

Ag(S2O3)23- + e

Ag

0,25

lgK1

Ag + 2S2O32-

lgK2=lgK1.( - lgβ1)

0,25

E20 E10 = − lg β1 ⇒ E20 = E10 − 0,0592 lg β1 = 3,168.10− 3 (V ) 0,0592 0,0592 Vậy

0 -3 E Ag ( S O ) 3 − / Ag =3,168.10 (V) 2

0,25

3 2

Khi pin hoạt động:

AgCl + e Ag + 2S2O32AgCl +

3. E A = E 20 + 0,0592 lg

2S2O32-

Ag + ClAg(S2O3)23- + e

0,25 -

Ag(S2O3)2 + Cl

3− 2 3 2 2− 2 3

[ Ag ( S O ) ] 0,001 = 3,168.10 −3 + 0,0592 lg ≈ −0,055(V ) [S O ] 0,098 2

EC = Epin + EA = 0,341 +(- 0,055) = 0,286(V) Xét điện cực phải: Ta có : AgCl + e

3-

Ag + Cl-

0,25


0 0 E AgCl / Ag = E Ag + / Ag + 0,0592 lg Ks AgCl

0 − 0 − ⇒ EC = E AgCl / Ag − 0,0592 lg[Cl ] = E Ag + / Ag + 0,0592 lg Ks AgCl − 0,0592 lg[Cl ]

⇒ lg Ks AgCl =

0 + 0,0592 lg[Cl − ] EC − E Ag + / Ag

0,0592

≈ −10

0,25

Vậy KsAgCl = 10-10 4. Ag(S2O3)23- + 2CNAg(CN)2- + 2S2O32- K = 107,54 0,001 0,01 0,098 -3 8.10 0,001 0,1 + Ag(CN)2 Ag + 2CN Kkb= 10-21 C: 0,001 8.10-3 []: 0,001-x x 2x+8.10-3

⇒ K kb =

x(2 x + 8.10 −3 ) = 10 − 21 0,001 − x

Coi x << 0,001 ⇒ x ≈

10 −21.0,001 ≈ 1,5625.10 − 20 ( M ) << 0,001 −3 2 (8.10 )

EA = E0(Ag+/Ag) + 0,0592lg[Ag+] =-0,373(V) Epin = 0,286 – (- 0,373) = 0,659 (V) M + 4HNO3  → M(NO3)2 + 2H2O + 2NO2 (1) x 2x 2NO2 + 2NaOH NaNO3 + NaNO2 + H2O (2) 2x x x o tC NaNO3  → NaNO2 + 1/2O2

0,25

0,25

1.

Câu 4

5NaNO2+2KMnO4 +3H2SO4  → 5NaNO3 + K2SO4 +2MnSO4+ 3H2O (4) Từ phương trình 4 : n NaNO2= 5/2. n KMnO4 =0,25 mol Đặt nM=x từ các phương trình ta thấy nNO2 =2x nNaNO3=nNaNO2(2) =x Vậy 2x= 0,25 nên x=0,125 MM= 3/0,125 = 24(g/mol) vậy M là Mg 2. (sp3) (sp3) ɺɺ ɺɺ N N H H F F H F Đều là tháp tam giác. Góc liên kết giảm theo chiều HNH > FNF vì độ âm điện của F lớn hơn của H làm điện tích lệch về phía F nhiều hơn. b. - NH3 có liên kết H liên phân tử còn NF3 thì không có, do đó amoniac có nhiệt độ sôi và nhiệt độ nóng chảy cao hơn NF3. - Số oxi hóa của Nitơ trong NH3: -3 Số oxi hóa của Nitơ trong NF3: +3 Như vậy, nguyên tử Nitơ trong amoniac với số oxi hóa thấp nhất dễ bị oxi hóa lên

0,5

0,25 0,25

0,25

0,25

0,25


mức oxi hóa cao hơn. - NH3 ngoài tính khử còn có thể tham gia phản ứng cộng do khả năng tạo liên kết cho nhận của cặp e tự do trong nguyên tử Nitơ. Tuy nhiên, nguyên tử F có độ âm điện cao nhất nên có khả năng hút electron mạnh, làm giảm mật độ của cặp e tự do của nguyên tử Nitơ trong NF3. Do đó khả năng tạo liên kết 0,25 cho nhận trong phản ứng cộng bị giảm mạnh. Khi cho dung dịch tác dụng với AgNO3 dư thu được kết tủa trắng, dễ bị phân hủy → Kết tủa trắng là AgCl. nAgCl = 8.10-3 mol → Số mol Cl- tạo được kết tủa là 8.10-3

Đun dung dịch B với H2O2 trong môi trường kiềm thu được dung dịch C, cho C tác dụng với BaCl2 dư thu được kết tủa Y màu vàng, là đồng hình với BaSO4 → Y là BaCrO4

A là tinh thể hiđrat của kim loại crom

0,25

-3

-3

nBaCrO4 = 4.10 mol → Số mol kim loại crom = 4.10 mol nCr : nCl − = 1: 2

0,25 0

Đun nóng tinh thể ở 300 C, trong môi trường HCl thì thấy khối lượng giảm 40,52% → Trước khi bị phân hủy khối lượng giảm 40,52% → đó là toàn bộ nước trong A Số mol H2O trong A = 0,024 mol → Số phân tử H2O = 6

0,25

- Theo đề bài m = 1,066 gam → Số mol A= 0,04 → M = 266,5 Công thức CrCl2.nH2O → n = 8 → vô lí - Ta thấy khối lượng H2O giảm ở 2 giai đoạn → có 2 loại H2O khác nhau (H2O trong cầu Câu 5

nội, H2O trong cầu ngoại) Khi làm khô tinh thể A bằng H2SO4 đặc thấy khối lượng giảm 6,75 % → , đó là khối lượng

0,25

nước trong cầu ngoại. Số mol H2O = 4.10-3 mol Số mol H2O trong cầu nội = 0,02 Thành phần A có Cr, Cl, H2O Tổng số mol Cl trong A = (1,066-4.10-3.52-0,024.18):35,5 = 0,012 mol Tuy nhiên chỉ có 8.10-3 mol Cl- tạo kết tủa với AgNO3 → có clo trong cầu nội của A Số mol Cl trong cầu nội = 0,012-8.10-3 = 4.10-3 mol Công thức của A [CrxCly(H2O)z]Cla.bH2O

0,25

x : y : z : a : b = 0,004 : 0,004: 0,02: 0,008 : 0,004 = 1: 1: 5: 2: 1 Công thức của A là: [CrCl(H2O)5]Cl2.H2O [CrCl(H2O)5]Cl2.H2O → [CrCl(H2O)5]2+ + 2Cl- + H2O Cl- + Ag+

→ AgCl (X)

0,25

[CrCl(H2O)5]Cl2 + 2 AgNO3 → [CrCl(H2O)5](NO3)2 + 2 AgCl B 2[CrCl(H2O)5]2+ + 3H2O2 + 10OH- → 2CrO42- + 2Cl- + 18H2O

0,25


C CrO42- + Ba2+ → BaCrO4 ↓ (vàng) (Y) 0,25 1.(0,75 điểm)

0 Cấu trúc

thơm

Imidazole (C3H4N2) Imidazol-1-yl (C3H3N2)

Thơm hay không thơm

anion

thơm

Imidazolyl cation (C3H5N2)

thơm

Oxazole (C3H3NO)

thơm

Thiazole (C3H3NS)

thơm

2. (0,75 điểm)

Tonc Câu 6

Giải thích Tos Giải thích

Imidazole > Thiazole > Oxazole Imidazole lớn nhất do liên kết H liên phân tử. Thiazole lớn hơn oxazole do khối lượng phân tử và độ phân cực lớn hơn. Imidazole > Thiazole > Oxazole Imidazole lớn nhất do liên kết H liên phân tử. Thiazole lớn hơn oxazole do khối lượng phân tử và độ phân cực lớn hơn.

3. (0,5 điểm)

Phản ứng ion hóa

Kb Lý do

Imidazole > Thiazole > Oxazole Axit liên hợp của imidazole có sự giải tỏa đối xứng, tạo liên kết H mạnh hơn với nước, do đó có lực bazo mạnh hơn oxadiazole và thiazole. O có độ âm điện lớn hơn N và S, nên làm giảm mật độ e trên N ở oxazole, làm giảm độ bền của axit liên hợp của oxazole nên lực bazo yếu nhất.


1. a (0,75 ®Óm).

CH3 C

H3C

CH3 CH2

H

+

C

to , p

CH3

H3C

CH3 2-Metylpropan (Y)

2-Metylpropen (X)

CH3 H

CH3

C

C

C

H

H (A)

CH3

CH3

Bước thứ nhất gồm tương tác giữa hai phân tử trong môi trường axit:

H 3C

CH 3 C

2 H 3C

H+

CH 2

CH 3 H

CH 3

C

C

C

CH 2

CH 3 H

H 3C

2,4,4-trimetyl-1-penten

CH 3 H

CH 3

C

C

C

CH 3 2,4,4-trimetyl-2-penten

CH 3

H3 C

CH3 H

CH3

C

C

C

CH2

+

H2

CH3 H

H 3C

o

Ni , t

CH3 H

CH3

C

C

C

CH3

+

CH3 H

CH3

C

C

C

CH3 H

H

H 3C

H2

CH3

CH3

Câu 7 .

b (0,75 ®Óm). R1

C

O CH2

O3

R1

O

O

O

C CH2

R1

CH2 O

C

Zn/H3O

+

R2

C

C

O

O O3

R2

CH3

H

H

O

O

O C

C

CH3

R2

CH3

O + O

CH3

Z CH3

C

R1

CH3

CH3

CH3

O

C

C O

CH3 CH3

H Zn/H3O

Q R2HC

O + CH3COCH3

2. (0,5 điểm) a. Công thức cấu tạo của A là: CH3 H3C

b. D1 và D2 có thế ở dạng cis hoặc trans, nên D1 có thể có cấu dạng bền: hay

H3CH2C

CH3

H3CH2C

CH3

cis-

D2 có thể có cấu dạng bền:

trans-

+

CH2


CH2CH3 H3C

CH2CH3

hay H3C

trans-

cis-

(0,75 điểm) - Khi bị oxi hóa mạnh, các hợp chất B, C, D và E đều cho axit phtalic chứng tỏ các hợp chất này là dẫn xuất của benzen bị thế hai lần ở vị trí 1,2. B có độ không no k = 5 và có phản ứng bromofom nên B có thể là: CH2CH2CH3

B1

C2H5

CH(CH3)2

COCH3

COCH3

B2

B3

CH3

CH2COCH3

B4

CH(CH3)COCH3

(0,75 điểm) - Do B không quang hoạt nên loại B4. Đun B với benzanđehit trong môi trường kiềm, thu được chất hữu cơ E (C18H18O) duy nhất nên loại B3 do B3 ngưng tụ với benzanđehit tạo ra 2 sản phẩm hữu cơ khác nhau. C 3H7 COCH3

B

C 2H5 D

C3H7

Câu 8

C 3H7

N2H4/KOH

PhCHO/OH

C 3 H7

-

COCH3

COCH=CHPh

B

E

(0,5 điểm) - Khi xử lí hợp chất A (C13H18O2) bằng dung dịch HCl loãng thu được hợp chất B chính là phản ứng thủy phân xetal. C3 H 7

C3H7 H3O+

CH3 A

O

OH

+ HO

O

B

COCH3

Vậy A là một trong hai công thức dưới đây: CH2CH2CH3

CH(CH3)2

CH3 O

CH3

O

O

O

a) (1,0 điểm) Cl3Al

Câu 9

R

1

O

Cl3Al R2

R1

O

2

R

R1

R2

O AlCl3

R

chậm -AlCl3

X1

X2

1

R2

O

( ) (


b) (1,0 điểm) Br H

..O

H

O

Br

O

O

O

Br

Br OH

-

O

H

O

O

OH Br

Br O

OH

OH

O

OH

O

O

1(1,0điểm) COOH HO

COOH O3

O CHO

Me2S

HO

HO OH

Câu 10

COOH

COOH

COOH

HO O O HO

OH

K

O O

CH3OH H+ OH HO

HIO4

OMe OH

OH L

M

O O CHO OHC N

OMe

2. (1,0điểm) Viết sơ đồ phản ứng tổng hợp ra K từ những hợp chất chứa không quá 4C.

--------------------Hết ------------------

HỘI CÁC TRƯỜNG THPT CHUYÊN

KÌ THI HỌC SINH GIỎI NĂM HỌC 2016-2017

VÙNG DUYÊN HẢI VÀ ĐỒNG BẰNG BẮC BỘ

MÔN THI: HÓA HỌC. LỚP 11

TRƯỜNG THPT CHUYÊN TUYÊN QUANG

Ngày thi: 15 tháng 4 năm 2017

---------------------

Thời gian làm bài: 180 phút

ĐỀ THI ĐỀ XUẤT

(Đề thi gồm 03 trang)

Câu 1: (2,0 điểm) Động học (Có cơ chế) – Cân bằng hóa học 1. Phản ứng phân hủy nhiệt metan xảy ra như sau: k1 CH 4  → .CH3 + H. k2 CH 4 + .CH 3  → C 2 H 6 + H. k3 CH 4 + H.  → .

CH 3 + H

.

.

CH 3 + H 2

→ CH 4 k4

Áp dụng nguyên lí nồng độ dừng đối với H. và .CH3, hãy chứng minh rằng:


k .k .k d[C2 H 6 ] = k[CH 4 ]3/ 2 với k = 1 2 3 dt k4 2. a) Tính nhiệt tạo thành chuẩn của khí CO từ những dữ kiện thực nghiệm sau:

C(graphit) + O2 (k) → CO2 (k)

∆H1o = - 94,05 kcal

2CO(k) + O2 (k) → 2CO2 (k)

∆H o2 = - 135,28 kcal

b) Kết quả này có phù hợp với công thức cấu tạo của CO nếu giả thiết là C=O không? Vì sao? Cho biết nhiệt thăng hoa của than chì là 170 kcal/mol, năng lượng liên kết trong phân tử O2 là 118 kcal/mol và năng lượng liên kết C=O trong CO2 là 168 kcal/mol. Câu 2: (2,0 điểm) Cân bằng trong dung dịch điện li a) Ion Ni2+ tạo phức với etylenđiamin (NH2-CH2-CH2-NH2, viết tắt là en) theo các cân bằng sau:

Ni2+ + en

Ni(en)2+

lgβ1 = 7,52

+ 2 en

Ni(en)22+

Ni2+ + 3 en

Ni(en)32+

2+

Ni

-3

lgβ2 = 13,84 lgβ3 = 18,33 2+

Trộn 1 mol en với 10 mol Ni và pha loãng thành 1 lít dung dịch bằng bazơ loãng. Tính nồng độ mol của các phức chất trong dung dịch ở trạng thái cân bằng. b) Hãy tính nồng độ tối thiểu của SCN- để có thể phát hiện ra Fe3+ (nồng độ 1 mg/lit) có trong nước

máy? Biết βFe(SCN)2+ = 1,4 ⋅102, mắt quan sát được màu khi [Fe(SCN)2+] ≥ 6,4 ⋅10-6M. Câu 3: (2,0 điểm) Điện hóa học a) Hãy tính thế điện cực chuẩn của cặp [Fe(CN)6]3– / [Fe(CN)6]4– từ các số liệu sau:

E°(Fe3+(aq) | Fe2+(aq)) = + 0,770 V Fe3+(aq) + 6 CN–(aq) Fe2+(aq) + 6 CN–(aq)

[Fe(CN)6]3–(aq) logβ6 = 43,9 [Fe(CN)6]4–(aq)

logβ'6 = 36,9

Có các thế điện cực chuẩn sau: In+ (aq) + e 3+

In (aq) + 3e +

Tl (aq) + e 3+

Tl (aq) + 3e

In(r)

E° = –0,13 V

In(r)

E° = –0,34 V

Tl (r)

E° = –0,34 V

Tl(r)

E° = +0,72 V

b) Hãy tính hằng số cân bằng của các phản ứng dị ly:

3 M+ (aq)

M3+ (aq) + 2M (r)

trong đó M là In và Tl. Nhận xét kết quả thu được. Câu 4: (2,0 điểm) N - P, C – Si và hợp chất

Hợp chất XY3 và KYO3 được sử dụng rộng rãi trong các túi khí bảo hiểm được lắp đặt trong ô tô. Tổng số hạt proton, nơtron và electron trong XY3 là 97, trong đó số hạt mang điện nhiều hơn số hạt không mang điện là 31. Phân tử khối của XY3 nhỏ hơn phân tử khối của KYO3 là 36. Tổng số hạt mang điện trong nguyên tử X bằng tổng số electron trong anion Y3– . Cho số khối của K bằng 39, của O bằng 16. a) Xác định tên nguyên tố và số khối của X, Y. b) Các phản ứng trong túi khí bảo hiểm xảy ra theo sơ đồ sau:


(1) XY3 → X + Y2 (2) X + KYO3 → X2O + K2O + Y2 (3) X2O + K2O + SiO2 → “thủy tinh” Hãy tính khối lượng XY3 tối thiểu cần để tạo ra khí Y2 nạp đầy túi khí an toàn có thể tích 17 lít ở 25oC, áp suất 1,25 atm. Câu 5: (2,0 điểm) Phức chất, trắc quang

Phổ hấp thụ phân tử vùng UV –Vis có độ hấp thụ quang (mật độ quang A) phụ thuộc tuyến tính vào nồng độ. Ngoài ra độ hấp thụ quang còn có tính cộng tính trong dung dịch chứa các cấu tử màu không tương tác hóa học với nhay. Hãy thiết lập công thức tính hằng số phân ly của thuốc thử HR và tính hằng số phân ly của HR biết rằng khi đo mật độ quang ở cùng một bước sóng đối với các dung dịch có tổng nồng độ thuốc thử như nhau (Co mol/L) và có pH khác nhau. Kết quả thực nghiệm cho dưới đây: – pH < 2 thì được A1 = 0,016. – pH = 7,33 được A2 = 0,442. – pH > 11

được A3 = 0,655.

Câu 6 (2,0 điểm): Quan hệ giữa hiệu ứng cấu trúc và tính chất 1. Cantharidin (A) có công thức như hình bên:

a) Hãy xác dịnh số nguyên tử cacbon bất đối và vẽ công thức lập thể của (A). b) Đọc tên (A) theo danh pháp hệ thống IUPAC c) (A) có tính quang hoạt hay không? Giải thích 2.

a) Hãy cho biết giá trị pKa = 5,06 và 9,3 tương ứng với chất nào trong 2 chất sau đây? CH3 CH3

N

CH3

N

CH3

O2N

NO2

NO2

N,N-đimetylanilin (A)

2,4,6-trinitro-N,N-đimetylanilin (B)

b) Tính axit của nhóm –OH nào sau đây thể hiện mạnh nhất? O

HO

O

(1)

OH (2)

OH (3)

Câu 7 (2,0 điểm): Hidrocacbon

Từ dầu mỏ người ta tách được các hiđrocacbon A(C10H16) ; B (C10H18) và C(C10H18). Cả ba đều không làm mất màu dung dịch brom và chỉ chứa C bậc hai và bậc ba. Tỉ lệ giữa số nguyên tử CIII : số nguyên tử CII ở A là 2 : 3; còn ở B và C là 1 : 4. Cả A, B,C đều chỉ chứa vòng 6 cạnh ở dạng ghế. a) Hãy xác định công thức cấu tạo và viết công thức lập thể của A, B, C. b) So sánh nhiệt độ nóng chảy của A, B, C và giải thích. Câu 8 (2,0 điểm): Xác định cấu trúc, đồng phân lập thể, danh pháp


Cho hợp chất B tác dụng với axit malonic/piridin tạo thành D. Đun D trong dung dịch axit tạo thành E và F có cùng công thức phân tử: C5H8O2 . Khi oxi hoá E cho axit oxalic còn khi khử hoá F bằng LiAlH4 thu được diol G. 1) Xác định cấu trúc B, D ,E ,F ,G. Giải thích 2) Viết sơ đồ chuyển F thành 5–etylheptan–2,5–diol. Câu 9 (2,0 điểm): Đề nghị cơ chế các phản ứng sau: O

OEt Br2/EtOH

a)

Br

O

OEt O

Cl

b)

Etylen

O

MeO

AlCl3

MeO

Câu 10 (2,0 điểm): Tổng hợp các hợp chất hữu cơ ( Dạng sơ đồ phản ứng).

Hoàn thành chuỗi phản ứng: Hoàn thành sơ đồ tổng hợp Efaviren: O

Cl

Cl

A

OCH3

1. BuLi O 2. BF3

F

Ce(NO3)2 NH4NO3

C

OEt

Efaviren

Biết F là: F3C

C

C

O N

C

D

C

NH2

COCl2

B

HCl

Cl

O OCH3

HẾT Người ra đề: Phan Khánh Phong- 0983.713.890

CLi

E


HỘI CÁC TRƯỜNG CHUYÊN

ĐỀ THI MÔN HÓA HỌC -KHỐI 11

VÙNG DUYÊN HẢI VÀ ĐỒNG BẰNG BẮC BỘ

NĂM 2017

TRƯỜNG THPT CHUYÊN TUYÊN QUANG

Thời gian làm bài 180 phút

HƯỚNG DẪN CHẤM CÂU 1

HƯỚNG DẪN GIẢI

ĐIỂM

1.

d[C2 H 6 ] = k 2 [CH 4 ].[H 3C. ] dt .

(1) .

.

. .

d[H ]/dt = k1[CH4] + k2[CH4][ CH3] - k3[CH4][H ] - k4[H ][ CH3] = 0 .

.

.

.

0,25

(2)

.

d[ CH3]/dt = k1[CH4] - k2[CH4][ CH3] + k3[CH4][H ] - k4[H ][ CH3] = 0

(3)

Cộng (2) và (3)

0,25 . .

=> k1[CH4] = k4[H ][ CH3] → [H.] = k1[CH4]/k4[.CH3] .

.

(4) .

.

Lấy (2) trừ đi (3), ta có: k2[CH4][ CH3] = k3[CH4][H ] → [ CH3] = k3[H ]/k2

(5)

k1k 3[CH 4 ] k 2k 4

(6)

Thay (4) vào (5) và biến đổi đơn giản, ta thu được: [.CH 3 ] = Thay (6) vào (1), ta có:

k1.k 2 .k 3 d[C2 H 6 ] = k[CH 4 ]3/ 2 với k = k4 dt

0,25

0,25

2.

a) C(graphit) + O2 (k) → CO2 (k)

∆H1 = - 94,05 kcal

(1)

2CO(k) + O2 (k) → 2CO2 (k)

∆H2 = - 135,28 kcal

(2)

C(graphit) + ½ O2 (k) → CO (k)

∆Hx

(3)

Ta có: (1) – ½ (2) → (3) nên: ∆Hx = ∆H1 - ½∆H2 = - 26,41(kcal/mol)

0,5

b) Giả sử CO có công thức cấu tạo là C=O thì nhiệt tạo thành chuẩn của CO được tính như sau: C(graphit, r) → C(k)

∆H3 = 170 kcal

½ O2(k) → O(k)

∆H4 = 118/2 kcal

C(k) + O(k) → C=O

∆H5 = -168 kcal

C(graphit, r) + ½ O2(k) → C=O

∆H6

∆H6 = ∆H3 + ∆H4 + ∆H5 = (170kcal) + (118/2kcal) + (-168kcal) = 61 (kcal) Kết quả này không phù hợp với thực tế (có sự khác nhau quá lớn). Vậy công thức

0,25


cấu tạo của CO là C=O là không đúng.

0,25

2

a) Phương trình bảo toàn nồng độ đối với Ni2+: [Ni2+] + [Ni(en)2+] + [Ni(en)22+] + [Ni(en)32+] = 10-3 M

(1)

0,25

(2)

0,25

Phương trình bảo toàn nồng độ đối với en: [en] + [Ni(en)2+] + 2[Ni(en)22+] + 3[Ni(en)32+] = 1 M 2+

2+

Giả sử: [Ni ], [Ni(en) ], Ni2+)

[Ni(en)22+]

2+

<< [Ni(en)3 ] (Vì nồng độ en >> nồng độ

(1) => [Ni(en)32+] = 10-3 M (2) => [en]= 1- 3⋅10-3= 0,997 M 2+ Có: β 3 = [ Ni2(+en)3 ]3

[ Ni ][en]

2+ => [ Ni 2+ ] = [ Ni(en) 33 ] =

β 3 [en]

18, 33

10

10 −3 = 4,72 ⋅ 10 −22 M × (0,997) 3

2+ Có: β 2 = [ Ni2(+en) 2 ]2

0,25

[ Ni ][en]

=> [ Ni (en) 22+ ] = β 2 [ Ni 2+ ][en]2 = 1013,84 × 4,72 ⋅ 10 −22 × (0,997) 2 = 3,25 ⋅ 10 −8 M Có: [ Ni(en) 2+ ] = β1[ Ni 2+ ][en] = 107,52 × 4,72 ⋅ 10−22 × 0,997 = 1,56 ⋅ 10−14 M

0,25

KTGT: thỏa mãn

0,25

b) Có: C Fe

3+

1 ⋅ 10 −3 g 1mg 55,85 g / mol = = = 1,79 ⋅ 10 −5 M lit 1 lit

Fe3+

+

SCN-

1,79⋅10-5

b đ: pư:

6,4⋅10

cb:

C0

-6

1,15⋅10

6,4⋅10

-5

[ Fe ][ SCN ]

0

mol/l -6

C0 -6,4⋅10

2+ ] Có: β = [ FeSCN = 3+ −

0,25

FeSCN2+ -6

6,4⋅10 mol/l -6

6,4⋅10-6

mol/l

6,4 ⋅ 10−6 = 1,4 ⋅ 102 1,15 ⋅ 10−5 × (C0 − 6,4 ⋅ 10− 6 )

0,25

-3

=> C = 3,98 ⋅10 M 0,25 3

a) 3−

Fe3+ + 6 CN-

[Fe(CN)6]3-

β6 =

[ Fe(CN ) 6 ] = 1043,9 [ Fe3+ ][CN − ]6

Fe2+ + 6 CN-

[Fe(CN)6]4-

β 6' =

[ Fe(CN )6 ] = 1036,9 [ Fe 2 + ][CN − ]6

Fe3+ + e

Fe2+

4−

0,5


0 E Fe 3+ / Fe 2+ = EFe 3+ / Fe 2+

3+

[ Fe ] 0 + 0,0592 log = EFe 3+ / Fe 2+ [ Fe 2 + ]

[ Fe(CN )36− ] β [CN − ]6 + 0,0592 log 6 [ Fe(CN ) 64 − ] β 6' [CN − ]6

0,25

Điều kiện chuẩn: [Fe(CN)63-] = [Fe(CN)64-] = 1M => EFe0 ( CN )

3− 6

/ Fe ( CN ) 64−

b) Có: 3EIn0 => EIn0

3+

+

/ In

3+

/ In

=

=> ETl0

3+

3+

/ Tl

β 6' = 0,356V β6

3EIn0 3+ / In − EIn0 + / In 2

0,25

= −0,445V

In3+ (aq) + 2 In (r) K = 10 / Tl

+

0,25

= 2 EIn0 3+ / In + + EIn0 + / In

3 In+ (aq) Có: 3ETl0

0 = EFe + 0,0592 log 3+ / Fe 2+

2 × ( −0,13+ 0, 445) 0 , 0592

= 4,4 ⋅ 1010

= 2 ETl0 3+ / Tl + + ETl0 + / Tl

=

3ETl0 3+ / Tl − ETl0 + / Tl 2

3 Tl+ (aq)

0,25

= +1,25V

Tl3+ (aq) + 2 Tl (r)

K = 10

2 × ( −0, 34 −1, 25) 0, 0592

= 1,9 ⋅ 10 − 54

0,25

Nhận xét: In+ bị phân hủy dị ly hoàn toàn còn Tl+ thì không GT: tăng độ trơ của lớp vỏ ns

0,25

4 a) Gọi số p và n của X lần lượt là a và b; của Y lần lượt là c và d. Có hệ phương trình:

2a + b + 6c + 3d = 97

(1)

2a – b + 6c – 3d = 31

(2)

(39 + c + d + 16.3) – (a + b + 3c + 3d) = 36

(3)

2a = 3c +1 ⇒ 2a – 3c = 1

(4)

giải hệ ta được a = 11; b = 12; c = 7; d = 7

0,5 0,25 0,25

⇒ X là Natri, Y là Nitơ; ANa = 23, AN = 14 b)

nN2 =

PV 1,25 . 17 = = 0,8696 mol RT 0,082 . 298

0,25

(1) 2NaN3 → 2Na + 3N2 x

x

0,25

1,5x

(2) 10Na + 2KNO3 → 5Na2O + K2O + N2 x

0,1x

0,25

1,6x = 0,8696 mol ⇒ n NaN3 = x = 0,5435 mol ⇒

m NaN3 = 0,5435 . 65 = 35,3275 gam

0,25


5  → H+ + R – Từ cân bằng: HR ← 

Ka

 H +  .  R −   R −  K= hay pK = pH − lg [ HR ] [ HR ]

0,25

Ta đi tìm [H+] và [R –] – Khi pH < 2: Thì giả sử toàn bộ thuốc thử ở dạng axit HR và nồng độ của nó tỷ lệ với mật độ quang A1 = ε HR .[HR].l = ε HR .C HR .l = ε HR .Co .l ⇒ ε HR =

A1 Co .l

(1)

0,5

– Khi pH > 11: Thì giả sử toàn bộ thuốc thử ở dạng bazơ R – và nồng độ của nó tỷ lệ với mật độ quang: A 3 = ε R - [R - ].l = ε R - .CR - .l = ε R - .Co .l ⇒ ε R - =

A3 Co .l

(2)

0,5

Đo ở pH=7,33 là đo ở pH trung gian trong dung dịch tồn tại ở cả hai dạng cho nên:

A 2 = ε HR [HR].l + εR - [R - ].l =

A1 [HR].l + Co .l

A3 [R - ].l (3) Co .l

⇒ A 2 Co = A 2 ([HR] + [R - ]) = A1.[HR] + A 3 .[R - ] ⇒

[R - ] A 2 − A1 = [HR] A 3 − A 2

 R −  A − A1 0, 442 − 0, 016 pK = pH − lg = pH − lg 2 = 7, 33 − lg = 7, 03 A3 − A 2 0, 655 − 0, 442 [ HR ]

0,5

K = 9, 35.10−8 025 6 1. a) Có 4 C*. Cấu trúc có hai dạng endo và exo.

0,5

b) Anhidrit 2,3-dimetyl-7-oxabixiclo[2.2.1]heptan-2,3-dicacboxilic

0,25

c) Do có mặt phẳng đối xứng đi qua nguyên tử oxi và qua giữa liên kết C5-C6 và liên kết C2-C3 nên phân tử không có tính quang hoạt.

0,25

2.

a) - Tính bazơ (B) > (A) => pKa (A) = 5,06 ; pKa (B) = 9,3

0,25

- B có hiệu ứng không gian loại II: Hai nóm NO2 ở vị trí 2, 6 trong nhân benzen và hai nhóm –CH3 ở nguyên tử N gây cản trở không gian làm cho cặp electron tự do của N không liên hiợp được với vòng thơm (vi phạm nguyên tắc của hệ liên hợp) => Tính bazơ của B mạnh hơn A.

0,5

b) Tính axit của nhóm –OH (2) mạnh nhất do hiệu ứng liên hợp.


0,25 7 a)

- A, B, C không làm mất màu dung dịch brom => không chứa liên kết bội mà chứa vòng no.

0,25

- A có 4CIII và 6CII ; B, C có 2CIII và 8CII nên ta có: + CTCT của A, B, C 0,5 A

B, C

+ Công thức lập thể

0,75 A

C

B

b)

- A có cấu trúc kim cương đặc khít nên là chất rắn có nhiệt độ nóng chảy cao nhất. B có cấu hình trans gọn gàng hơn C cấu hình cis => nhiệt độ nóng chảy B > C. Nhiệt độ nóng chảy: A > B > C

0,5

a) Xác định cấu trúc B, D ,E ,F ,G. Mỗi chất đúng được 0,25 đ

1,25

8

– E và F: C5H8O2 có độ không no bằng 2. O] – E [→ HOOC–COOH

=> E là HOOC–CH=CH–CH2–CH3. – D là sản phẩm tổng hợp malonic, mặt khác D đun trong H+ tạo thành E. Vậy D là: CH3CH2CH(OH)CH(COOH)2, vì: COOH (R,S)

COOH OH

+

H -HOH

COOH

t

COOH

CO2

(cis, trans)

COOH (E)

(D)

– B phản ứng với axit malonic tạo thành D. Vậy B là CH3CH2CHO. COOH H O

+

COOH CH

COOH COOH

(B)

O

COOH +HOH - OH

COOH OH

(D)


– Sản phẩm F tạo thành từ D khi đun với H+: COOH -HOH CO2

COOH OH

O H

(D)

O (R, S)

+

O

(F)

O

– F khử bằng LiAlH4 cho diol G: O

O

LiAlH4

(R,S) HO

(G)

(F)

2. O

O

C2H5MgBr

(F)

OH

BrMgO Et

O

C2H5MgBr

Et

Et

OMgBr

0.75

BrMgO

OH

HOH +

H

HO

Et

5-etylheptan-2,5-diol

9 a) 1,0

b)

1,0

10


O Cl

Cl

CF3

NH

O

1,0

(C) CF3

CF3 Cl

NH OCH3

O

OH

NH (D)

O NH2

O

(B)

CF3 Cl

Cl

NH O

(A)

CF3

(E)

HỘI CÁC TRƯỜNG CHUYÊN VÙNG DUYÊN HẢI VÀ ĐỒNG BẰNG BẮC BỘ TRƯỜNG THPT CHUYÊN HÙNG VƯƠNG ĐỀ THI ĐỀ XUẤT

N OCH3

H (Efaviren)

O

1,0

ĐỀ THI MÔN HÓA HỌC KHỐI 11 NĂM 2017 Thời gian làm bài 180 phút (Đề này có 03 trang, gồm 10 câu)

Câu 1: (2,0 điểm) Động học (Có cơ chế) – Cân bằng hóa học

Người ta thực hiện phản ứng 2 NO2 (k) + F2 (k) 2 NO2F (k) trong một bình kín có thể tích V (có thể thay đổi thể tích của bình bằng một pittông). ¸p suất ban đâu của NO2 bằng 0,5 atm, còn của F2 bằng 1,5 atm. Trong các điều kiện đó tốc độ đầu vo = 3,2. 10−3 mol.L−1.s−1. 1. Nếu thực hiện phản ứng trên ở cùng nhiệt độ với cùng những lượng ban đầu của chất phản ứng nhưng thêm một khí trơ vào bình đó để thể tích thành 2V, còn áp suất tổng quát vẫn bằng 2 atm, thì tốc độ đầu bằng 8.10−4 mol.L−1.s−1. Kết qủa này có cho phép thiết lập phương trình động học (biểu thức tóc độ) của phản ứng hay không? 2. Người ta lại thực hiện phản ứng trên ở cùng điều kiện nhiệt độ và cùng những lượng NO2, F2 và khí trơ như ở (1) nhưng giảm thể tích xuống bằng 0,5V. Tính gía trị của tốc độ đầu vo . 3. Nếu thay cho việc thêm khí trơ, người ta thêm NO2 vào đó cho áp suất tổng quát bằng 4 atm và thể tích bằng V thì tốc độ đầu vo = 1,6.10−2 mol.L−1.s−1. Kết qủa này cho phép kết luận như thế nào về phương trình động học của phản ứng? 4. Dự đoán cơ chế của phản ứng. Câu 2: (2,0 điểm) Cân bằng trong dung dịch điện li


1. Trộn 15,00 ml dung dịch CH3COONa 0,030 M với 30,00 ml dung dịch HCOONa 0,15 M. Tính pH của dung dịch thu được. Biết pKa (CH3COOH) = 4,76 và pKa (HCOOH) = 3,75. 2. Tính nồng độ ban đầu của HSO4- (Ka = 10-2), biết giá trị sức điện động của pin: PtI- 0,1 M; I3- 0,02 MMnO4- 0,05 M, Mn2+ 0,01 M, HSO4- C MPt

ở 25oC đo được bằng 0,824 V. Cho E oMnO − / Mn 2+ = 1,51V và E oI − / I − = 0,5355V . 4

3

3. Tính nồng độ ion H+ đủ để làm giảm nồng độ Ag(NH3)2+ 0,10 M xuống còn 1,0.10-8 M. Biết pKb (NH3) = 4,76 và hằng số bền β [Ag(NH3)2+] = 7,24. Câu 3: (2,0 điểm) Điện hóa học Cho biến thiên thế đẳng áp, đẳng nhiệt hình thành chuẩn của Al(OH)3(r) là -1157kJ/mol, của H2O(l)là 237,4 kJ/mol, tích số tan của Al(OH)3 là 10-32 và tích số ion của nước là W H 2 O = 1,0.10 −14 . 0 1) Xác định ∆G 298 của phản ứng sau trong dung dịch nước: Al(r) + 3H+.aq = 3/2 H2(k)+ Al3+.aq 0 2) Từ giá trị của ∆G 298 của phản ứng trên, tính thế điện cực chuẩn E0 ( Al3+/Al).

3) Có thể xác định thế điện cực chuẩn E0 (Al3+/Al) bằng thực nghiệm như thế nào?. 4) Ở pH = 7 , thực tế nhôm có tác dụng với nước không ? tại sao ? Câu 4: (2,0 điểm) N - P, C – Si và hợp chất Dẫn 1 luồng hơi nước qua than nóng đỏ thì thu được V lít (ở đktc) hỗn hợp khí X gồm CO2, CO, H2, tỉ khối hơi của X so với H2 là 7,8. Toàn bộ V lít hợp khí X trên khử vừa đủ 24 gam hỗn hợp CuO, Fe2O3 nung nóng thu được rắn Y chỉ có 2 kim loại. Ngâm toàn bộ Y vào dung dịch HCl dư thấy có 4,48 lít H2 bay ra (ở đktc). Tính Giá trị V. Câu 5: (2,0 điểm) Phức chất, trắc quang Hiện nay, để xác định pH thì người ta thường dùng điện cực thuỷ tinh. Tuy nhiên, trong một số trường hợp, do có nhiều yếu tố ảnh hưởng nên việc xác định pH bằng điện cực thuỷ tinh cho kết quả không chính xác. Trong trường hợp này, ta có thể xác định pH bằng phương pháp đo quang sử dụng chất chỉ thị axit bazơ. Ví dụ như khi xác định pH trong mẫu nước biển. Vì hàm lượng muối tan trong nước biển là rất cao dẫn đến sai số hệ thống rất lớn khi đo pH bằng điện cực thủy tinh. Thymol xanh là chỉ thị có màu thuộc dạng axit hai lần axit. Nồng độ của dạng không phân li, H2In, ở pH của nước biển là rất nhỏ và có thể bỏ qua. Ở 25oC, hằng số phân li nấc thứ 2 của thymol xanh, đã được hiệu chỉnh, là Ka2 = 10–8,09. Hệ số hấp thụ phân tử (ελ) của HIn– và In2− tại hai bước sóng (λ) như trong bảng sau: Ion ε436 (L.mol−1.cm−1) ε596 (L.mol−1.cm−1) HIn− 13900 44,2 2− In 1930 33800 Tiến hành đo mẫu nước biển trong cuvet có bề dày 10,00 cm, kết quả như sau: A436 A596 Mẫu nước biển 0,052 0,023 Mẫu có thêm chỉ 0,651 0,882 thị thymol xanh Tính pH của mẫu và nồng độ mol/l của thymol xanh trong mẫu. Câu 6 (2,0 điểm): Quan hệ giữa hiệu ứng cấu trúc và tính chất Hiđro hóa hoàn toàn naphtalen người ta thu được đecalin (C10H18). Oxi hóa đecalin thì thu được hỗn hợp các đecalon (C10H16O). 1).Trong hỗn hợp đecalon nói trên có tối đa bao nhiêu đecalon đồng phân ? Giải thích. 2). Hãy vẽ công thức các đồng phân lập thể của 1-đecalon, biết rằng hai vòng 6 cạnh trong phân tử đecalon đều ở dạng ghế.


3). Hòa tan cis-1-đecalon vào dung dịch bazơ thì nó bị đồng phân hóa thành trans-1-đecalon tới 95%. Hãy giải thích sự đồng phân hóa của cis-1-đecalon và cho biết cis-2-đecalon có bị đồng phân hóa thành trans-2-đecalon trong điều kiện đã cho hay không, vì sao? 4). Trong dung dịch bazơ, 1-đecalon phản ứng với benzanđehit cho hợp chất T, phản ứng với metyl vinyl xeton cho hợp chất U. T làm mất màu nước brom còn U thì không. Hãy viét công thức cấu tạo của T và U. Câu 7 (2,0 điểm): Hidrocacbon 1). Viết phương trình phản ứng cộng brom vào propen và đề nghị cơ chế phản ứng 2). Viết phương trình phản ứng cộng các halogen dạng Br – Cl; I – Cl; I – Br vào propen và cho biết thứ tự tăng tốc độ phản ứng theo dãy I2 < ? ... < Br2 < ? ... < Cl2 .

3) .Viết phương trình phản ứng của Br – Cl với: But – 1 – en ; pent – 2 – en; axit propenoic; brom eten. Xếp các chất trên theo thứ tự tăng tốc độ phản ứng. Câu 8 (2,0 điểm): Xác định cấu trúc, đồng phân lập thể, danh pháp 1). Streptimidon là một loại kháng sinh có công thức sau:

O O

NH

H OH

H H

H3C H CH3

H

O

(a) Mô tả các trung tâm lập thể của streptimidon bằng kí hiệu E, Z và R, S. (b) Streptimidon có bao nhiêu đồng phân lập thể, trong đó có bao nhiêu đồng phân enan và bao nhiêu

đồng phân dia? 2). So sánh và giải thích:

(a) Nhiệt độ sôi của các chất xyclopentan, tetrahidrofuran và pirolidin. (b) Độ mạnh tính bazơ của dimetylamin, piperidin và piridin. Câu 9 (2,0 điểm): Cơ chế phản ứng Cho n – propylbenzen tác dụng với clo 1). Trong trường hợp chiếu sáng và đun nóng người ta thu được hỗn hợp ba dẫn xuất monoclo A1,A2,A3 với tỉ lệ % lần lượt bằng 68:22:10. 2). Trong trường hợp đun nóng với bột sắt người ta thu được hỗn hợp ba dẫn xuất monoclo B1,B2,B3 với tỉ lệ % lần lượt bằng 60:38:2. Hãy nêu cơ chế phản ứng và xác định cấu trúc của các dẫn xuất monoclo, giải thích. Câu 10 (2,0 điểm): Tổng hợp các hợp chất hữu cơ ( Dạng sơ đồ phản ứng). Viết công thức cấu tạo của A, B, C và giải thích quá trình chuyển hóa tạo thành C trong sơ đồ phản ứng đươi đây:


HỘI CÁC TRƯỜNG CHUYÊN

ĐÁP ÁN ĐỀ THI MÔN HÓA HỌC

VÙNG DUYÊN HẢI VÀ ĐỒNG BẰNG BẮC BỘ

KHỐI 11

TRƯỜNG THPT CHUYÊN HÙNG VƯƠNG

NĂM 2017

ĐỀ THI ĐỀ XUẤT

Thời gian làm bài 180 phút

Câu 1: (2,0 điểm) Động học (Có cơ chế) – Cân bằng hóa học Người ta thực hiện phản ứng 2 NO2 (k) + F2 (k) 2 NO2F (k) trong một bình kín có thể tích V (có thể thay đổi thể tích của bình bằng một pittông). ¸p suất ban đâu của NO2 bằng 0,5 atm, còn của F2 bằng 1,5 atm. Trong các điều kiện đó tốc độ đầu vo = 3,2. 10−3 mol.L−1.s−1. 1. Nếu thực hiện phản ứng trên ở cùng nhiệt độ với cùng những lượng ban đầu của chất phản ứng nhưng thêm một khí trơ vào bình đó để thể tích thành 2V, còn áp suất tổng quát vẫn bằng 2 atm, thì tốc độ đầu bằng 8.10−4 mol.L−1.s−1. Kết qủa này có cho phép thiết lập phương trình động học (biểu thức tóc độ) của phản ứng hay không? 2. Người ta lại thực hiện phản ứng trên ở cùng điều kiện nhiệt độ và cùng những lượng NO2, F2 và khí trơ như ở (1) nhưng giảm thể tích xuống bằng 0,5V. Tính gía trị của tốc độ đầu vo .


3. Nếu thay cho việc thêm khí trơ, người ta thêm NO2 vào đó cho áp suất tổng quát bằng 4 atm và thể tích bằng V thì tốc độ đầu vo = 1,6.10−2 mol.L−1.s−1. Kết qủa này cho phép kết luận như thế nào về phương trình động học của phản ứng? 4. Dự đoán cơ chế của phản ứng.

ĐÁP ÁN 1. Ở thí nghiệm 2, sau khi thêm khí trơ để cho thể tích tăng gấp đôi thì P NO2 và PF đều giảm 2 lần so với thí nghiệm 1, nghĩa là nồng độ của chúng cũng giảm đi 2 2 lần (vì PA = CA.RT), còn tốc độ đầu của phản ứng giảm 4 lần. Từ đây, chỉ có thể kết luận bậc của phản ứng là 2.

ĐIỂM

0,50

Phương trình động học có thể có các dạng sau đây: v = k [NO2] [F2] (a) , v = k [NO2]2 (b) , v = k [F2]2 (c) ë thí nghiệm 3, P(NO2) và P(F2) đều tăng gấp đôi so với thí nghiệm 1. Cũng lập luận như trên, ta thấy tốc độ đầu của phản ứng ở thí nghiệm 3 phải bằng 4 lần tốc độ đầu của phản ứng ở thí nghiệm 1. vo = 3,2 × 10−3 mol.L−1.s−1 × 4 = 1,28 × 10−2 mol.L−1.s−1.

0,50

3. ë thí nghiệm 4, P không đổi, P = 4 atm − 1,5 atm = 2,5 atm. P tăng 5 lần so với NO2 NO2 F2 thí nghiệm 1, còn tốc độ đầu của phản ứng tăng 5 lần. Vậy phươnng trình động học của phản ứng là: v = k [NO2] [F2]

0,50

4. Căn cứ vào phương trình động học của phản ứng, cơ chế phản ứng có thể là: NO2 +

F2

F

NO2F

+ NO2

+ F (chậm)

0,50

NO2F (nhanh).

Câu 2: (2,0 điểm) Cân bằng trong dung dịch điện li 4. Trộn 15,00 ml dung dịch CH3COONa 0,030 M với 30,00 ml dung dịch HCOONa 0,15 M. Tính pH của dung dịch thu được. Biết pKa (CH3COOH) = 4,76 và pKa (HCOOH) = 3,75. 5. Tính nồng độ ban đầu của HSO4- (Ka = 10-2), biết giá trị sức điện động của pin: PtI- 0,1 M; I3- 0,02 MMnO4- 0,05 M, Mn2+ 0,01 M, HSO4- C MPt

ở 25oC đo được bằng 0,824 V. Cho E oMnO − / Mn 2+ = 1,51V và E oI − / I − = 0,5355V . 4

3

6. Tính nồng độ ion H+ đủ để làm giảm nồng độ Ag(NH3)2+ 0,10 M xuống còn 1,0.10-8 M. Biết pKb (NH3) = 4,76 và hằng số bền β [Ag(NH3)2+] = 7,24.

ĐÁP ÁN 0,03 × 15 0,15 × 30 = 0,010M ; C HCOO - = = 0,100M 45 45 Các cân bằng: H2O ⇌ H+ + OHKw = 10-14 (1) -9,24 CH3COO + H2O ⇌ CH3COOH + OH Kb = 10 (2) ’ -10,25 HCOO + H2O ⇌ HCOOH + OH Kb = 10 (3)

1. C CH

3COO

-

ĐIỂM

=

0,50


Do K b × C CH

3COO

-

= 10 −11, 24 ≈ K 'b × C HCOO − = 10 −11, 25 cho nên không thể tính gần

đúng theo một cân bằng. Điều kiện proton: h = [H+] = [OH-] - [CH3COOH] - [HCOOH]

h=

1+

Kw −1 K a [CH 3 COO - ] + (K 'a ) −1 [HCOO - ]

(4)

Chấp nhận [CH3COO-]o = 0,010; [HCOO-]o = 0,10 và thay vào (4) để tính h1:

10 −14

h1 =

−2

4 , 76

−1

3, 75

= 2,96.10 −9

1 + 10 .10 + 10 .10 Từ giá trị h1 tính lại [CH3COO-]1 và [HCOO-]1 theo các biểu thức:

[CH 3 COO - ]1 = 0,010 [HCOO - ]1 = 0,10

10 −4,76 ≈ 0,010 = [CH 3 COO - ]o 10 −4,76 + 2,96.10 −9

10 −3,75 ≈ 0,10 = [HCOO - ]o −3, 75 −9 10 + 2,96.10

Kết quả lập lại. Vậy h = 2,96.10-9 = 10-8,53 ⇒ pH = 8,5 2. Ở điện cực phải: MnO4- + 8H+ + 5e ⇌ Mn2+ + 4H2O Ở điện cực trái: 3I- ⇌ I3- + 2e

E p = E oMnO− / Mn 2 + + 4

E t = E oI − / 3I − + 3

0,059 [MnO 4− ][H + ]8 0,059 0,05[H + ]8 = + lg 1 , 51 lg 5 5 0,01 [Mn 2+ ]

0,75

0,059 [I 3− ] 0,059 0,02 lg − 3 = 0,5355 + lg = 0,574V 2 2 [I ] (0,1) 3

∆E = Ephải - Etrái ⇔ 0,824 = 1,51 +

0,059 lg(5[H+]8) – 0,574 5

Suy ra h = [H+] = 0,053 M Mặt khác từ cân bằng: H2SO4- ⇌ H+ + SO42[] C – h h h Suy ra

Ka = 10-2

h2 h2 = Ka ⇒ +h=C C−h Ka

Thay giá trị h = 0,053 và Ka = 1,0.10-2, tính được C HSO − = 0,334M 4

+

-8

3. Do [Ag(NH3)2 ] = 1,0.10 M << 0,10 M, suy ra phức bị phân hủy hoàn toàn. Hơn nữa môi trường axit, nên có thể bỏ qua quá trình tạo phức hidroxo của Ag+. Phản ứng phân hủy phức: Ag(NH3)2+ + 2H+ ⇌ Ag+ + 2NH4+ K = 10-7,24.(109,24)2 = 1011,24 o C 0,1 C Co C – 0,2 0,1 0,2

0,75


Vì môi trường axit và K NH + = 10 −9, 24 nhỏ nên sự phân li của NH4+ có thể bỏ qua. 4

Xét cân bằng: Ag+ + 2NH4+ ⇌ Ag(NH3)2+ + 2H+ C 0,1 0,2 C – 0,2 -8 -8 -8 [] 0,1-10 0,2 - 2.10 10 C-0,2 + 2.10-8

(C − 0,2) 2 .10 −8 0,1.(0,2)

2

K-1 = 10-11,24

= 10 −11, 24 ⇒ C = 0,2015M

Câu 3: (2,0 điểm) Điện hóa học Cho biến thiên thế đẳng áp, đẳng nhiệt hình thành chuẩn của Al(OH)3(r) là -1157kJ/mol, của H2O(l)là 237,4 kJ/mol, tích số tan của Al(OH)3 là 10-32 và tích số ion của nước là W H 2O = 1,0.10 −14 . 0 1) Xác định ∆G 298 của phản ứng sau trong dung dịch nước: Al(r) + 3H+.aq = 3/2 H2(k)+ Al3+.aq 0 2) Từ giá trị của ∆G 298 của phản ứng trên, tính thế điện cực chuẩn E0 ( Al3+/Al).

3) Có thể xác định thế điện cực chuẩn E0 (Al3+/Al) bằng thực nghiệm như thế nào?. 4) Ở pH = 7 , thực tế nhôm có tác dụng với nước không ? tại sao ? ĐÁP ÁN 1)

Al(r) + 3/2 O2(k) +

ĐIỂM

3/2 H2(k) → Al(OH)3 ∆G S0, Al ( OH )3

1 3 H 2 O (l ) → H 2 ( k ) + O2 − 3∆G S0, H 2O ( l ) 2 3+ Al(OH)3(r) Al .aq+ 3OH-.aq ∆G 0 = − RT ln T Al ( OH )3 − ⇔ H 2 O(l ) ∆G 0 = 3(− RT ln 3 H + (aq) + OH aq

Al(r) +

3H+.aq

=

3+

Al

.aq+

3/2 H2(k)

1 W

)

0,50

H 2O

∆G tổng cộng 0

∆G 0 tổng cộng = ∆G S0, Al ( OH ) 3 − 3∆G S0, H 2O ( l ) − RT ln T Al ( OH )3 + 3(− RT ln

1 W

)

H 2O

∆G 0 tổng cộng = 501,9 kJ

2) ∆G 0 = − nFE P0

⇒ 501,9 = - 3. 96487. E0pu⇒ E0pu = 1,73V

Mặt khác E0pu = E0(2H+/H2) - E0(Al3+/Al) ⇒ E0(Al3+/Al)

= - 1,73V

3) Muốn xác định thế điện cực chuẩnE0(Al3+/Al), ta ghép điện cực nhôm chuẩn với điện cực hidro chuẩn thành một pin điện ...

0,50

0,50

4) Ở pH = 7 , thực tế nhôm không tác dụng với nước. Tính : nồng độ Al3+.aqkhi pH = 7 dựa vào Tt Al(OH)3 = 1,0.10-32 ⇒ Giá trị quá nhỏ ⇒ 0,50 coi như thực tế phản ứng không xảy ra Câu 4: (2,0 điểm) N - P, C – Si và hợp chất Dẫn 1 luồng hơi nước qua than nóng đỏ thì thu được V lít (ở đktc) hỗn hợp khí X gồm CO2, CO, H2, tỉ khối hơi của X so với H2 là 7,8. Toàn bộ V lít hợp khí X trên khử vừa đủ 24 gam hỗn hợp CuO,


Fe2O3 nung nóng thu được rắn Y chỉ có 2 kim loại. Ngâm toàn bộ Y vào dung dịch HCl dư thấy có 4,48 lít H2 bay ra (ở đktc). Tính Giá trị V.

ĐÁP ÁN

ĐIỂM

Theo bảo toàn nguyên tố Fe, bảo toàn electron và bảo toàn khối lượng, ta có : 2n Fe O = n Fe = n H = 0,2 n Fe O = 0,1 2 3 2   2 3  m (CuO, Fe O ) − m Fe O ⇒  24 − 0,1.160 2 3 2 3 = 0,1  nCuO = nCuO = 80  80 

1.00

Theo giả thiết, theo bảo toàn electron trong phản ứng của C với H2O và phản ứng của CO, H2 với CuO, Fe2O3, ta có :  28nCO + 44nCO + 2nH 2 2  = 7,8.2 = 15,6  + + n n n  CO CO2 H2 nCO = 0,1   ⇒ nCO = 0,1 ⇒ n(CO, CO , H ) = 11,2 lít 2nCO + 4nCO2 = 2nH2 2 2   2 2n + 2n = 2n + 6n n = 0,3  CO  H2 H2 CuO Fe2O3

 0,1 0,1 

1,00

Câu 5: (2,0 điểm) Phức chất, trắc quang Hiện nay, để xác định pH thì người ta thường dùng điện cực thuỷ tinh. Tuy nhiên, trong một số trường hợp, do có nhiều yếu tố ảnh hưởng nên việc xác định pH bằng điện cực thuỷ tinh cho kết quả không chính xác. Trong trường hợp này, ta có thể xác định pH bằng phương pháp đo quang sử dụng chất chỉ thị axit bazơ. Ví dụ như khi xác định pH trong mẫu nước biển. Vì hàm lượng muối tan trong nước biển là rất cao dẫn đến sai số hệ thống rất lớn khi đo pH bằng điện cực thủy tinh. Thymol xanh là chỉ thị có màu thuộc dạng axit hai lần axit. Nồng độ của dạng không phân li, H2In, ở pH của nước biển là rất nhỏ và có thể bỏ qua. Ở 25oC, hằng số phân li nấc thứ 2 của thymol xanh, đã được hiệu chỉnh, là Ka2 = 10–8,09. Hệ số hấp thụ phân tử (ελ) của HIn– và In2− tại hai bước sóng (λ) như trong bảng sau: Ion ε436 (L.mol−1.cm−1) ε596 (L.mol−1.cm−1) HIn− 13900 44,2 2− In 1930 33800 Tiến hành đo mẫu nước biển trong cuvet có bề dày 10,00 cm, kết quả như sau: A436 A596 Mẫu nước biển 0,052 0,023 Mẫu có thêm chỉ 0,651 0,882 thị thymol xanh Tính pH của mẫu và nồng độ mol/l của thymol xanh trong mẫu.


ĐÁP ÁN

ĐIỂM

Với A = ε.l.c ta có Tại λ = 436 13.900.10. [HIn-] + 1930.10[In2-]=0,651 - 0,052 Tại λ = 596 44,2 . 10. [HIn-] _+33800.10[In2-] =0,882 - 0,023 Từ 2 phương trình trên có: [HIn-] = 3,96.10-6 M; [In2-] =2,54.10-6 M HIn ⇌ In + H -

-2

1,00 +

K=10

-8,09

[In 2− ].[H + ] −8,09 10 = [HIn − ]

1,00

2,54.10−6.[H + ] −8,09 10 ⇔ = 3,96.10−6 ⇒ [H + ] = 1, 27.10−8 (m) ⇒ p H = 7,9

Câu 6 (2,0 điểm): Quan hệ giữa hiệu ứng cấu trúc và tính chất Hiđro hóa hoàn toàn naphtalen người ta thu được đecalin (C10H18). Oxi hóa đecalin thì thu được hỗn hợp các đecalon (C10H16O). 1).Trong hỗn hợp đecalon nói trên có tối đa bao nhiêu đecalon đồng phân ? Giải thích. 2). Hãy vẽ công thức các đồng phân lập thể của 1-đecalon, biết rằng hai vòng 6 cạnh trong phân tử đecalon đều ở dạng ghế. 3). Hòa tan cis-1-đecalon vào dung dịch bazơ thì nó bị đồng phân hóa thành trans-1-đecalon tới 95%. Hãy giải thích sự đồng phân hóa của cis-1-đecalon và cho biết cis-2-đecalon có bị đồng phân hóa thành

trans-2-đecalon trong điều kiện đã cho hay không, vì sao? 4). Trong dung dịch bazơ, 1-đecalon phản ứng với benzanđehit cho hợp chất T, phản ứng với metyl vinyl xeton cho hợp chất U. T làm mất màu nước brom còn U thì không. Hãy viét công thức cấu tạo của T và U.

ĐÁP ÁN

ĐIỂM

1) Trong hỗn hợp có 12 decalon đồng phân gồm: trans-1-decalon: 2 đối quang; cis-1-

0,50


decalon: 2 cặp đối quang (xem hình ở câu 2); trans-2-decalon: 2 đối quang; cis-2decalon: 2 cặp đối quang. 2) 6 đồng phân lập thể của 1-decalon:

O

O

O

O

0,50 O

O -

+

3). Nguyên tử Cα ở cầu nối trở thành C và bị nghịch đảo khi kết hợp với H . Cân bằng chuyển dịch về phía đồng phân trans bền hơn:

H

H

H

- H+

H

+ H+

+ H+

- H+ O

O

O

0,50

H

Cũng có thể giải thích bằng cách dựa vào dạng enol có cấu tạo phẳng .

cis-2-decalon không có sự đồng phân hóa như vậy vì Cα ở nó không phải là cầu nối, nơi quyết định cấu hình cis hoặc trans. 0,50

4). T là sản phẩm ngưng tụ, U là sản phẩm cộng Michael rồi ngưng tụ tiếp theo: O O T:

U:

CHC6H5

Câu 7 (2,0 điểm): Hidrocacbon 1) Viết phương trình phản ứng cộng brom vào propen và đề nghị cơ chế phản ứng 2) Viết phương trình phản ứng cộng các halogen dạng Br – Cl; I – Cl; I – Br vào propen và cho biết thứ tự tăng tốc độ phản ứng theo dãy I2 < ? ... < Br2 < ? ... < Cl2 . 3) Viết phương trình phản ứng của Br – Cl với: But – 1 – en ; pent – 2 – en; axit propenoic; brom eten. Xếp các chất trên theo thứ tự tăng tốc độ phản ứng.

ĐÁP ÁN

ĐI Ể M

1) Cơ chế AE: theo tiến trình lập thể là cộng kiểu transH3C

H C

H

C H

+ + Br

H3C C

Br H

Br +

CH2Br

H + Br

C H

Br *

H

CH3 CÆp ®èi quang

0,75


2) Tốc độ phản ứng công tăng theo thứ tự I2 ; IBr ; Br2 ; ICl ; BrCl ; Cl2. 3) (1) CH3CH2CHCl − CH2Br (3) Cl − CH2 − CHBr − COOH

0,50

( 2) CH3CH2CHCl − CHBr − CH3. (4) BrCHCl − CH2Br

Thứ tự tăng tốc độ phản ứng: (3) < (4) < (1) < (2) Giải thích: Theo cơ chế cộng AE : giai đoạn chậm của phản ứng là giai đoạn các tiểu

0,75

phân tích điện dương tấn công; nên nếu mật độ e ở nối đôi càng cao thì phản ứng càng dễ xảy ra.

Câu 8 (2,0 điểm): Xác định cấu trúc, đồng phân lập thể, danh pháp

1. Streptimidon là một loại kháng sinh có công thức sau: O

O NH

H OH

H H

H3C H CH3

O

H

(c) Mô tả các trung tâm lập thể của streptimidon bằng kí hiệu E, Z và R, S. (d) Streptimidon có bao nhiêu đồng phân lập thể, trong đó có bao nhiêu đồng phân enan và bao nhiêu

đồng phân dia?

2. So sánh và giải thích: (c) Nhiệt độ sôi của các chất xyclopentan, tetrahidrofuran và pirolidin. (d) Độ mạnh tính bazơ của dimetylamin, piperidin và piridin.

ĐÁP ÁN

ĐI Ể M

1. (a) Streptimidon có ba trung tâm lập thể: S O

E H

O *

H

*

H3C H CH3

NH

H OH

0,50

O

H

R (b) Streptimidon có 2 = 8 đồng phân lập thể, trong đó streptimidon và đối quang của nó tạo cặp đồng phân enan, còn sáu đồng phân lập thể còn lại là đồng phân dia của streptimidon. 2. (a) Trật tự tăng dần nhiệt độ sôi: 3

O

N H

0,50

0,50


không phân cực

có phân cực

tạo được kiên kết H liên phân tử

(b) Trật tự tăng dần độ mạnh tính bazơ:

N

N H

N lai hóa sp2

N lai hóa sp3

N H N lai hóa sp3 và cấu trúc vòng ít gây án ngữ không gian với N

0,50

Câu 9 (2,0 điểm): Cơ chế phản ứng Cho n – propylbenzen tác dụng với clo 1) Trong trường hợp chiếu sáng và đun nóng người ta thu được hỗn hợp ba dẫn xuất monoclo A1,A2,A3 với tỉ lệ % lần lượt bằng 68:22:10. 2) Trong trường hợp đun nóng với bột sắt người ta thu được hỗn hợp ba dẫn xuất monoclo B1,B2,B3 với tỉ lệ % lần lượt bằng 60:38:2. Hãy nêu cơ chế phản ứng và xác định cấu trúc của các dẫn xuất monoclo, giải thích.

ĐÁP ÁN

ĐIỂM

1) Phản ứng xảy ra theo cơ chế gốc tự do (SR) 3 giai đoạn. Tỷ lệ % lần lượt là của sản phẩm thế Clo

CHCl CH2 CH3

CH2 CHCl

CH3

CH2 CH2 CH2Cl

0,75

Do ảnh hưởng hút e của vòng benzen. 2) Phản ứng xảy ra theo cơ chế thế electrofin vào vòng ben zen (SEAr). Tỷ lệ % lần lượt là của sản phẩm para - ; ortho - ; meta. Do hướng thế vào vòng benzen với nhóm thế có sẵn là nhóm thế loại (I) định hướng nhóm thế tiếp theo thuận lợi vào vị trí ortho- và para-. Mặt khác do nhóm thế có sẵn có kích thước cồng kềnh gây hiệu ứng không gian

0,50

0,75

ở vị trí ortho- nên sản phẩm thế ở vị trí para- có % cao nhất.

Câu 10 (2,0 điểm): Tổng hợp các hợp chất hữu cơ ( Dạng sơ đồ phản ứng). Viết công thức cấu tạo của A, B, C và giải thích quá trình chuyển hóa tạo thành C trong sơ đồ phản ứng đươi đây:


ĐÁP ÁN

ĐIỂM

Sự tách nước của A xảy ra qua giai đoạn tạo thành cacbocation kèm theo sự giải tỏa điện tích dương nên tạo ra B. Khi B bị đun nóng có thể xảy ra phản ứng Diels-Alder nội phân tử tạo thành C nhờ có phần đienophin -C6=C7- và hợp phần đien -C1=C2-C3=C4-.

0,75

Hiệu suất của phản ứng Diels-Alder nội phân tử kể trên không cao do hợp phần đienophin (C6=C7) và hợp phần đien (C1=C2-C3=C4) chỉ cách nhau mỗi một nguyên tử C5

nên khó sắp xếp phù hợp với yêu cầu lập thể của phản ứng Diels-Alder và vì phải tạo ra 0,50 vòng 3 cạnh có sức căng lớn. Trong trường hợp đun nóng B, phản ứng Diels-Alder bình thường (liên phân tử) tạo ra D1 và D2 có lẽ sẽ diễn ra thuận lợi hơn và tiếp theo có thể tạo ra E1 và E2:

0,75


SỞ GIÁO DỤC-ĐÀO TẠO HƯNG YÊN TRƯỜNG THPT CHUYÊN

ĐÁP ÁN ĐỀ THI CHỌN HỌC SINH GIỎI DH & ĐBBB NĂM HỌC 2016- 2017

ĐỀ ĐỀ NGHỊ

MÔN THI: HÓA HỌC LỚP 11.

(Thời gian làm bài 180 phút không kể thời gian giao đề)

Đề thi gồm 03 trang Câu 1: (2,0 điểm) Động học (Có cơ chế) – Cân bằng hóa học 1.1 Động học (Có cơ chế) Cho phản ứng thực hiện ở pha khí: A2(g) + 2 B(g) → 2 AB(g) Phản ứng xảy ra nhanh là do chất xúc tác C. Hằng số tốc độ tổng quát được tìm thấy có sự tăng tuyến tính với nồng độ chất xúc tác. Sau khi thực hiện đo tại 400K với [C]=0.050 mol.L-1

Thí nghiệm

Tốc độ ( mol·L-1·s-

[A2] -1 0.010

[B]

2

0.010

0.20

1.600×10-10 3.200×10-10

3

0.100

0.20

1.012×10-9

1

0.10

-1

a) Hãy tìm bậc riêng phần của phản ứng. b) Tính hằng số k tại 400k c) Cho cơ chế đã được đề xuất như sau: A2(g)

2A

(nhanh)

A(g) + B(g) +C(g) –– > ABC(g) (chậm) ABC(g) ––> AB(g) + C(g) Hãy kiểm tra lại cơ chế đã được đề nghị bằng cách giải thích sự tổ hợp của phản ứng. d) . Chứng minh rằng cơ chế được đề xuất ở trên phù hợp với định luật tốc độ được xác định bằng thực nghiệm. e) . Tính sự biến thiên enthalpy của liên kết trong phân tử A2 bằng cách sử dụng các dữ kiện cho sau đây: e-1) Tại 400K, [A2]= 1.10-1 mol.L-1, [A]= 4.10-3mol.L-1. e-2) Khi thí nghiệm đầu tiên được lặp lại ở 425K, phản ứng tăng gấp 3 lần giá trị ban đầu. e-3) Năng lượng hoạt hóa của giai đoạn chậm nhất là 45 kJ


1.2. Cân bằng hóa học Ở 1396K và áp suất 1,0133.105 N.m-2, độ phân li của hơi nước thành hiđro và oxi là 0,567.10-4; độ phân li của cacbon đioxit thành cacbon monooxit và oxi là 1,551.10-4. Từ hai thể tích như nhau của cacbon monooxit và hơi nước ở điều kiện trên xác định thành phần hỗn hợp khí ở trạng thái cân bằng được tạo thành theo phản ứng:

H2 O +CO

Câu 1.1

CO2 + H2

Đáp án

Điểm

a)

Từ thí nghiệm 1 và 2 ta có: y=1 Từ thí nghiệm 2 và 3 ta có : x=0.5 Do đó v=k[A2]1/2.[B2].[C]=kob[A2].[B2]

0.25

b)

kob=k[C] V=1.6.10-10= kob.0.011/2.0.1 Kob=1.6.10-8 L1/2.mol-1/2.s-1

0.125

c)

Nhân phản ứng thứ hai với thứ 3 với hệ số 2 và cộng với phản ứng thứ nhất ta được phương trình phản ứng: A2(g) + 2 B(g) → 2 AB(g)

0.125

d)

Ta có v[B]=

∆ [ B] ∆t

= k2 [ A][ B ][C ]

0,25

Theo cân bằng xảy ra nhanh ta có: k [ A] = 1 × [ A2 ] k−1 Thay vào ta có: 1 k B = k2 ( 1 ) [ A2 ] 2 [ B ][C ] k−1

∆ [ A2 ] 1 ∆ [ B ] 1 ∆ [ AB ] =− = ∆t 2 ∆t 2 ∆t 1 1 1 k Nên v= = k2 ( 1 ) [ A2 ] 2 [ B ][C ] = kob [ A2 ] 2 [ B2 ] 2 k−1 Ta có vpư= −

1 k k2 ( 1 ) [C ] 2 k−1 Kết luận: cơ chế được đề xuất phù hợp với công thức rút ra từ thực nghiệm

Với kob =

e)

1 k k2 ( 1 ) [C ] 2 k−1 Để tìm lượng biến thiên enthalpy của A2 (liên kết) ta cần tìm k2 tại 400K và 425K. k1 (4.0 ×10−3 ) 2 = = 1.6 ×10−4 k−1 0.10 Theo ý d , ta có: kob =

0,25


1 k 2 1.6 × 10 −4 × 0.050 = 1.6 × 10 −8 2 k2 (400) = 5.1× 10−5 k2 45000 25 K2 tại 425K ln k2 (425) = 1.1×10−4 = ×( ) −5 5.06 × 10 8.314 400 × 425 Tại 425K tỉ lệ của thí ghiệm thứ nhất được tăng 3 lần . Do vậy: 3i1.6 × 10−10 kob = = 4.8 × 10−8 1 0.012 i0.1 1 k kob = × 1.12 ×10−4 × 1 × 0.05 = 4.8 ×10−8 2 k−1 kob =

k1 = 1.71× 10−2 k−1

K (425) =

k1 2.93 × 10 −4 k −1

2.93 × 10−4 △H 25 = .( ) H=34kJ −4 1.6 × 10 8.314 400.425 Kết luận: vậy bến thiên enthalpy bằng 34 kJ ln

1.2

Theo phương trình phản ứng: CO + H2O cân bằng: PCO2 PH2 Kp = PCO PH2O

H2 + CO2, ta có hằng số

0, 25

(a)

Giá trị hằng số cân bằng của phản ứng này có thể tính từ hằng số phân li của hơi nước và hằng số phân li của cacbon đioxit: 2H2O 2CO2

2H2 + O2

K p,H 2O =

2CO + O2

K p,CO2 =

PO2 PH2 2

(b)

PH2 2O 2 PO2 PCO

(c)

2 PCO 2

Chia vế (b) cho (c), dễ dàng suy ra Kp =

K p,H 2 O K p,CO2

(d)

* Xác định các hằng số cân bằng: Gọi độ phân li của hơi nước là α1 = 0,567.10-4; 2H2O === 2H2 + Ban đầu 2 0 Phân li 2 α1 2α1 Cân bằng 2(1 - α1) 2α1 Tổng số mol hỗn hợp ở trạng thái cân bằng: 2 + α1 2(1-α1 ) 2α1 α → PH 2O =P , PH2 =P , PO2 =P 1 2+α1 2+α1 2+α1 Thay (e) vào (a) với lưu ý α1 ≪ 1 , ta có:

0,25 O2 0 (mol) α1 α1 (e)


K p,H2O =

Pα31 2

=

1, 0133.105 (0,567.10 −4 )3 =0,923.10-8. 2

Với cách tính hoàn toàn tương tự nhận được: 3 2

5

0,25

−4 3

Pα 1, 0133.10 (1,551.10 ) = =18,90.10-8. 2 2 Thay các giá trị của K p,H2O và K p,CO2 vào (d), có Kp = 0,221. K p,CO2 =

* Xác định thành phần hỗn hợp khí theo Kp Vì phản ứng tạo hỗn hợp khí xảy ra ở điều kiện thể tích không đổi nên nồng độ của các chất phản ứng có thể biểu diễn bằng bất kì đơn vị nào. Trong trường hợp này, thuận tiện nhất là biểu diễn nồng độ bằng phần trăm thể tích. Phần trăm thể tích ban đầu của CO và của H2O đều bằng 50%. Gọi x % là phần trăm thể tích của H2 và CO2 sinh ra ở trạng thái cân bằng, theo phản ứng: CO + H2O === H2 + CO2

0,25

x2 = 0, 221→ x = 15,99%. ta có: (50 − x)2

Câu 2: (2,0 điểm) Cân bằng trong dung dịch điện li

Histidine là một amino axit thiết yếu trong cơ thể người, động vật và thực vật. Cấu trúc phân tử của histidin khi đã bị proton hoá có dạng như sau:

Ở dạng này, histidin được coi như một axit 3 lần axit (được kí hiệu là H3A2+) có các hằng số phân li axit tương ứng: pKai = 1,82; 6,00 và 9,17. 1) Hòa tan hoàn toàn 4´10−4 mol H3ACl2 (có thể viết dạng HA.2HCl) trong nước, thu được 40,0 mL dung dịch A. Tính pH và nồng độ cân bằng của các ion trong dung dịch A. 2) Nếu dùng dung dịch NaOH 0,01 M để chuẩn độ dung dịch A thì pH của dung dịch thu được là bao nhiêu sau khi đã cho hết 40,0 mL dung dịch NaOH?

3) Điện tích trung bình trên toàn phân tử amino axit trong dung dịch nước có thể được tính theo điện tích của từng dạng tồn tại của các cấu tử trong dung dịch. Hãy tính điện tích trung bình của Histidin trong dung dịch khi pH của dung dịch là 6.0.


4) Ion Cu2+ và Histidin có thể tạo ra hai phức bền với tỉ lệ 1:1 và 1:2 với các hằng số của các quá trình tương ứng như sau: Cu2+ + A− === CuA+ k1 = 2,0´108 CuA+ + A− ==== CuA2

k2 = 4,0´1010

Trộn 50 mL dung dịch Cu(NO3)2 0,02 M với 50 mL dung dịch Histidine 0,20 M rồi điều chỉnh pH của dung dịch thu được bằng 6,0 thu được 100,0 mL dung dịch B. a) Tính nồng độ cân bằng của các ion Cu2+, CuA+, CuA2, A− trong dung dịch B. b) Nồng độ cân bằng của ion Cu2+ trong dung dịch B sẽ thay đổi như thế nào nếu điều chỉnh cho pH của dung dịch B tăng? Điều chỉnh cho pH của dung dịch B giảm? Giải thích ngắn gọn. (Giả sử thể tích của dung dịch B không đổi sau khi điều chỉnh pH).

Cho biết: Ks(Cu(OH)2) = 10−19,8;

Câu 2 1

Cu2+ + H2O ⇌ Cu(OH)+ + H+ * b = 10−8,0.

Đáp án

Điểm 0,125

Nồng độ H3A2+ trong dung dịch A là 0,01 M. Các cân bằng:

H3A2+ −1,82 10 (1)

⇌ H 2A + −6,0 = 10 (2) HA = 10

−9,17

KW = 10

+ + H2A + H Ka1 =

HA + H

Ka2

− + A + H

(3) H2O ⇌

−14

+

Ka3

+ H +

OH

(4)

So sánh (1) – (4) ta thấy: Ka1 >> Ka2 >> Ka3 và Ka1.C >> KW

Do đó pH của dung dịch tính theo cân bằng (1) H3A2+

2

C

0,01

[ ]

0,01 – x x x = 6,876.10−3 ;

⇌ H2A+ + H+

0,125 0,25

+

+

Ka1 = 10

−1,82

x

pH = 2,16.

Nồng độ cân bằng trong dung dịch A: [H+]= 6,867.10-3 M [OH-]= 1,454.10-12M [H3A2+]= 3,124.10-3M [H2A+]= 6,876.10-3M [HA]= 1.10-6M [A-]= 9,83.10-14M Tại thời điểm cho hết 40,00 mL dung dịch NaOH 0,01 M vào 40,0 mL dung

0,25

0,25


dịch A. Số mol NaOH bằng số mol H3A2+ Phản ứng chuẩn độ xảy ra như sau: H3A2+ + OH- → H2A+ + H2O Dung dịch thu được có H2A+: 5.10−3 M. Áp dụng biểu thức điều kiện proton.

 H 2 A+  .K a 2  H +  = 1 +  H 2 A+  .K a−11

3

→ pH= 4,21 Nếu pH của dung dịch bằng 6,0.

-4 = 0,50; a = 0,50; a = 3,79 .10 . H 2A + HA A−

-5 = 3,30 .10 ; a

a H3A2+ Vì a

4

rất nhỏ, do đó có thể bỏ qua nồng độ của H3A2+ và

và a

H3A2+

0,25

A-

A−. Hai dạng chính là H2A+ và HA có nồng độ bằng nhau. Trong đó H2A+ mang điện tích + 1 còn HA không mang điện tích. Do vậy điện tích trung bình của trên toàn phân tử của histidin trong dung dịch có pH = 6,0 là + 0,5. Nồng độ sau khi trộn: Cu2+: 0,01 M; Histidin: 0,10 M.

0,25

Phản ứng:

Cu2+ + A− ⇌ CuA+

b1 = 2,0´108

Cu2+ + 2A− ⇌ CuA2

b2 = 8,0´1018

Giả sử chỉ tạo ra phức CuA2. TPGH: CuA2: 0,01 M; Histidin: 0,08 M. Tại pH = 6,0 CuA2 [ ]’

0,01 – x 2+

2

Cu   A  β 2−1 =  =

[CuA2 ]

x.

x 1 −1

x= 1,373.10-12 Do đó ta có nồng độ các chất là [Cu2+]= 1,30.10-12M [CuA2]= 0,01M

2A−

b2−1 = 1,25.10

0,08+2x

.(0, 08 + 2 x) 2 .α A2 −

1 + *β  H  0, 01 − x +

Cu2+ +

= 1, 25 × 10−19

0,25


[A-]= 3,032.10-5M [CuA+]= 8,247.10-9M Nồng độ của phức CuA+ rất nhỏ so với nồng độ của phức CuA2, do vây giả thiết chỉ tạo ra phức CuA2 là hợp lí : - Nếu điều chỉnh pH của dung dịch tăng thì Cu2+ tạo phức hidroxo nhiều hơn và dạng tồn tại của A- cũng tăng , do vậy sự tạo phức hiệu quả hơn nên làm giảm nồng độ của ion Cu2+tự do Ngược lại, nếu điều chỉnh pH của dung dịch thì [Cu2+] tăng

0,25

Câu 3: (2,0 điểm) Điện hóa học Cho các thế của urani trong dung dịch nước: U3+ + 3e → U Eo = -1,798V Eo = -0,607V U4+ + e → U3+ UO22+ + e → UO2+ Eo = +0,062V 2+ + 4+ UO2 + 4H + 2e → U + 2H2O Eo = +0,327V UO22+ + 4H+ + 6e → U + 2H2O Eo = -1,444V Eo = +0,620V UO2+ + 4H+ + e → U4+ + 2H2O 1) Xác định các mức oxy hóa của những tiểu phân có chứa urani xuất hiện trong các bán phản ứng trên. 2) Bằng cách phân tích các bán phản ứng trên, hãy xác định diễn biến hóa học tối ưu của một mẫu nhỏ urani rắn để tiếp xúc với dung dịch 1M của một axit mạnh đơn chức HX, có mặt hydro dưới áp suất 1atm, tại 25oC. Viết các phương trình phản ứng, cân bằng và thế điện cực của tất cả các phản ứng (Giả thiết rằng bazơ liên hợp X- không phản ứng đáng kể với urani hoặc hợp chất của urani). 3) Hãy cho biết tiểu phân bền nhất của urani tại pH = 6 là gì? (và các điều kiện khác đều coi như chuẩn). 4) Xác định khoảng pH của dung dịch hoặc axit trung hoà mà dung dịch 1M của UO2+ là bền: a) Với các điều kiện khác đều chuẩn (như P(H2) = 1), nồng độ của các tiểu phân có chứa urani = 1. b) Với P(H2) = 1,0.10-6 atm và các điều kiện khác đều coi như chuẩn.Điều kiện nào là thích hợp hơn cho sự hình thành urani trong các luồng nước thiên nhiên?

Câu 3

Đáp án

Điểm

1

Urani kim loại có số oxi hóa bằng không theo định nghĩa. Số oxi hóa các tiểu phân khác của U: U(III) [U3+]; U(IV) [U4+]; U(V) [UO2+]; U(VI) [UO22+]. Các điều kiện mô tả là điều kiện chuẩn, nên có thể dùng thế khử chuẩn để xác định các phản ứng tự xảy ra theo chiều nào. Cũng cần phải xét các bước khử: 2H+ + 2e → H2 Eo = 0,000V Có hai bán phản ứng tạo ra U kim loại: U3+ + 3e → U UO22+ + 4H+ + 6e → U + 2H2O Từ đó Urani bị oxi hóa kế tiếp: 2U + 6H+ → 2U3+ + 3H2 Eopin = +1,798V U + 2H+ + 2H2O → UO22+ + 3H2 Eopin = +1,444V Bất kể qúa trình nào trên đây chiếm ưu thế, không có ‘sản phẩm sơ cấp’ nào 3+ U hoặc UO22+ là sản phẩm chính. U(III) tự oxy hóa chuyển thành U(IV):

0,25

2

0,25

0,25


2U3+ + 2H+ → 2U4+ + H2 Eopin = +0,607V Trong khi U(VI) lại tự khử để tạo lại U(IV) hoặc U(V): UO22+ + 2H+ + H2 → U4+ + 2H2O Eo = +0,327V UO22+ + H2 → 2UO2+ + 2H+ Eo = +0,062V và U(V) cũng tự khử thành U(IV): Eo = +0,620V 2UO2+ + 6H+ + H2 → 2U4+ + 4H2O Vì U4+ là tiểu phân duy nhất không thể tự phản ứng với H+ hoặc H2, nên sẽ có nhiều nhất trong dung dịch nước ở điều kiện này. (Bổ sung: khi vẫn còn U kim loại, qúa trình: U4+ + 3U → 4U3+ Eopin = +1,191V là thuận lợi, nhưng chỉ có thể xảy ra đến khi hết U, sau đó U(III) sẽ bị oxi hóa thành U(IV) như đã nói ở trên).

3

4-a

a) Do các điều kiện là tiêu chuẩn trừ [H+] = 1,0.10-6M, có thể dùng dạng đơn giản của phương trình Nernst như sau cho các phương trình có liên quan: 2U + 6H+ → 2U3+ + 3H2 Eopin = +1,798V o + -6 Epin = E pin – (RT/6F)ln([H ] ) = +1,444V U + 2H+ + 2H2O → UO22+ + 3H2 Eopin = +1,444V Epin = Eopin – (RT/6F)ln([H+]-2) = +1,326V 3+ 2U + 2H+ → 2U4+ + H2 Eopin = +0,607V Epin = Eopin – (RT/2F)ln([H+]-2) = +0,253V UO22+ + 2H+ + H2 → U4+ + 2H2O Eopin = +0,327V Epin = Eopin – (RT/2F)ln([H+]-2) = -0,027V 2+ UO2 + H2 → 2UO2+ + 2H+ Eopin = +0,062V Epin = Eopin – (RT/2F)ln([H+]2) = +0,293V 2UO2+ + 6H+ + H2 → 2U4+ + 4H2O Eopin = +0,620V Epin = Eopin – (RT/2F)ln([H+]-6) = -0,444V Toàn bộ các qúa trình trên có trị số Eopin dương khi xét theo chiều thuận, nên tất cả đều xảy ra được theo chiều này tại pH = 0. Trái lại, U(VI) → U(IV) và U(V) → U(IV) tự xảy ra theo chiều nghịch tại pH = 6 (như cho thấy qua trị số âm của Epin); nay U(IV) lại bị oxy hóa thành U(V) hoặc U(VI). Do U(VI) là mức oxy hóa duy nhất không tự phản ứng được với H+ hay H2 trong điều kiện này nên tiểu phân chiếm số lượng lớn sẽ là UO22+. Ta có các phản ứng xác định thể của UO2+. Eo = +0,062V UO22+ + H2 → 2UO2+ + 2H+ o + 2 -2 Epin = E pin – (RT/2F)ln([H ] .P(H2) ) 2UO2+ + 6H+ + H2 → 2U4+ + 4H2O Eo = +0,620V Epin = Eopin – (RT/2F)ln([H+]-6.P(H2)-2) Tại các điều kiện lúc đầu, Epin = 0, tương ứng với các trường hợp sau:

[H ] = {P [H ] = {P +

2 H2

.e

( 2 FE opin / RT )

+

2 H2

.e

( −2 FE opin / RT )

0,25

0,25

0,25

}1 / 2 cho qúa trình U(VI) ⇌ U(V) và

}1 / 6 cho qúa trình U(IV) ⇌ U(V)

Các biểu thức trên cho giá trị ban đầu của:

4-b

[H+] < 11,2M hay pH > -1 để U(V) bền hơn U(IV) và [H+] < 3,19.10-4M hay pH > 3,50 để U(V) bền hơn U(IV).

0,25


4-c

[H+] < 1,12.10-5M hay pH > 4,95 để U(V) bền hơn U(IV) và [H+] < 3,19.10-2M hay pH > 1,50 để U(V) bền hơn U(IV). Như vậy UO2+ bền hơn các mức oxy hóa khác trên khoảng pH 3,5 – 7 (ta chỉ đang xét các dung dịch axit và trung hòa) dưới khí quyển hydro tiêu chuẩn, nhưng chỉ bền và không bị oxy hóa thành U(VI) tại pH có gía trị lớn hơn 4,95. Áp suất riêng phần của H2 trong khí quyển rất thấp, nên điều kiện của câu sau (câu ii) gần hơn với điều kiện thực của môi trường qủa đất. UO2+ không thực sự bền như kết qủa của pin cho thấy.

0,25

Câu 4: (2,0 điểm) N - P, C – Si và hợp chất Một chất rắn màu vàng X1 được hòa tan trong dung dịch axit nitric cô đặc xúc tác nhiệt độ cao, khí thoát ra có khối lượng phân tử gấp 1,586 lần không khí. Khi thêm một lượng Bari clorua dư vào dung dịch, ta thu được kết tủa trắng X2 tách ra. Lọc tách kết tủa. Dung dịch sau phản ứng trên cho tác dụng với một lượng dư dung dịch bạc sunfat tạo thành hai kết tủa rắn X2 và X3 tách ra khỏi dung dịch. Để lọc mới dung dịch, nhỏ từng giọt dung dịch natri hydroxit đến khi dung dịch thu được có môt trường gần trung tính pH̴ 7. Khi đó bột màu vàng X4 (77,31% khối lượng Ag) được kết tinh từ dung dịch. Khối lượng của X4 gấp 2,9 lần khối lượng của X2. 1) Xác định các chất X1-X4. 2) Tìm cong thức phân tử của khí trên và viết các phương trình hóa học xảy ra. 3) Vẽ công thức cấu tạo cua X1. 4) Viết các phản ứng của X1 với: a) O2. b) H2SO4, t(C) c) KClO3

Câu 4

Đáp án

Điểm

1

+) Chất kết tủa X2 được hình thành bằng cách thêm dung dịch bari clorua trong dung môi axit nên X2 là BaSO4 +) Chất kết tủa X3 được hình thành bằng cách thêm dunh dịch bạc sunfat nên X3 là AgCl +) Chất kết tủa màu vàng X4 được hình thành bằng cách thêm vào kiềm có thể là thủy ngân oxit HgO hoặc bạc phosphate Ag3PO4 . Tỷ lệ các khối mol X4: X2 là 0.931 đối với HgO: BaSO4 không hợp lệ và 1.798 đối với Ag3PO4:BaSO4 cho 2.4 nhân với 4/3. Vì vậy, tỷ lệ phân tử là 4Ag3PO4:3BaSO4 tương ứng với P: S = 4: 3, nên công thức của X1 : P4S3 X4 là Ag3PO4 dkhí/kk= 1,586 suy ra khí có CTPT là NO2 Các phương trình hóa học : 6 x 0,125 = 0,75

0,125

2

P4S3 + 38HNO3 → 4H3PO4 + 3H2SO4 + 38NO2+ 10H2O H2SO4 + BaCl2 →BaSO4 + 2HCl Ag2SO4 + 2HCl → 2AgCl + H2SO4 BaCl2 + Ag2SO4 →BaSO4 + 2AgCl H2SO4 + 2NaOH → Na2SO4 + 2H2O 2H3PO4 + 6NaOH +3Ag2SO4 → 2Ag3PO4 +3Na2SO4 + 6H2O

0,125 0,25

0,75


3

4

0,25

a) P4S3 + 8O2 → 2P2O5 + 3SO2 b) P4S3 + 16H2SO4 → 4H3PO4 + 19SO2 + 10H2O c) 3P4S3+ 16KClO3 → 16KCl + 6P2O5 + 9SO2

0,5

Câu 5: (2,0 điểm) Phức chất, trắc quang 5.1. Chiếu một chùm tia đơn sắc(có bước sóng lamda xác định) qua dung dịch mẫu chát nghiên cứu thì cường độ của ttia sáng tới I0 giảm đi chỉ còn I. Tỉ số được gọi là độ truyền qua. T phụ thuộc vào nồng độ mol C(mol.L-1)

Của chất hấp thụ ánh sáng tronh dung dịch, chiều dày lớp dung dịch l (cm) và hệ số hấp thụ mol (L.mol1 .cm-1) đặc trưng cho bản chất của chất hấp thụ ( định luật Lambert-Beer): Để xác định giá trị Ka của một axit hữu cơ yếu HA, người ta đo độ truyền qua của một chùm tai đơn sắc( tại bước songslamda xác định) với dunh dịch axit HA 0,05M đựng trong thiết bị đo với chiều dày lớp dung dịch l= 1cm. Kết quả cho thấy 70% tia sáng tới bị hấp thụ. Giả thiết, chỉ có anion A- hấp thụ tia đơn sắc tại bước sóng này và hệ số hấp thụ mol cảu A- là 600 L.mol-1.cm-1. Tính giá trị Ka của HA trong điều kiện thí nghiệm. 5.2 Phức [Fe(CN)6]4- có năng lượng tách là 394,2 kJ/mol, phức [Fe(H2O)6]2+ có năng lượng tách là 124,2 kJ/mol và năng lượng ghép electron là 210,3 kJ/mol. a) Hãy vẽ giản đồ năng lượng của hai phức trên và cho biết phức nào là phức spin cao, phức nào là phức spin thấp? b) Hỏi với sự kích thích electron từ t2g đến eg thì phức [Fe(CN)6]4- hấp thụ ánh sáng có bước sóng λ bằng bao nhiêu. Câu 5

Đáp án

Điểm

5.1

Gọi cường độ ánh sáng ban đầu là I0, cường độ ánh sáng sau khi đi qua dung dịch là I. Theo đầu bài, cường độ ánh sáng sau khi đi qua dung dịch có giá trị: I=I0-70%I0=30%Io Từ định luật Lambert-Beer ta có:

0,25

D= 600.l.CA- =

= 0,5229

Từ đó, nồng độ của A- tại cân bằng là:8,715.10-4 (M);

0,25

Xét cân bằng

0,5


H+

HA 0,05-8,715.10

-4

8,715.10

Từ đó: Ka= 5.2

A-

+ -4

Ka

8,715.10

=1,55.10

-4

-4

Vậy hằng số phân ly của HA là Ka=1,55.10-4 Các phức [Fe(CN)6]4- và [Fe(H2O)6]2+ đều là phức bát diện. Trong phức [Fe(CN)6]4có năng lượng tách (∆) > năng lượng ghép electron nên phức này có giản đồ năng lượng như sau:

0,25

0,125

eg

∆ ↑↓ ↑↓ ↑↓

t2g

↑↓

Trong giản đồ trên tổng spin S = 0 và là phức spin thấp. Trong phức [Fe(H2O6)]2+ có năng lượng tách thấp hơn năng lượng ghép electron nên phức này có giản đồ năng lượng như sau: ∆

↑↓

↑↓

0,125

eg

t2g

Trong giản đồ trên tổng spin S = 4 x 1/2 = 2 và là phức spin cao λ=

hc 6,625.10 −34. 3.10 8 0 = = 3,034.10-7 m hay 3034 A 3 E 394,2.10

0,5

6,02.10 23

Câu 6 (2điểm). Quan hệ cấu trúc – tính chất

Phân tử hợp chất hữu cơ A công thức C12H4Cl4O2 có tâm đối xứng và có 3 mặt phẳng đối xứng. A bền với nhiệt, không làm mất màu dung dịch brom và dung dịch kali pemanganat. (a) Hãy lập luận để xác định các công thức cấu trúc có thể của A. (b) Hãy dự đoán trạng thái của A ở nhiệt độ thường và tính tan của nó. (c) Hãy dựa vào cấu tạo để suy ra độ bền của A đối với ánh sáng, kiềm và axit. Câu 6

Đáp án

Điểm

a.

A là hợp chất thơm vì không làm mất màu dung dịch brom và dung dịch kali pemanganat. Độ không no của A bằng 9, là hợp chất thơm, bền nhiệt, nên nó chứa 2 vòng benzen nối với nhau bằng 2 nguyên tử O ete ở 2 vị trí ortho (0,5). A có tâm đối xứng và 3 mặt phẳng đối xứng nên có công thức là:

0.75

b)

A có phân tử khối lớn, có nhiều liên kết phân cực nên là chất rắn. Ở A phần ưa nước (2 nguyên tử O) rất nhỏ so với phần kị nước (phần còn lại trừ 2O) nên nó

0,5


tan trong dung môi hữu cơ, không tan trong nước. - A tương đối bền với ánh sáng vì là hợp chất thơm không có liên kết nào dễ bị phân cắt bởi ánh sáng. - A bền với kiềm vì các nguyên tử Cl đính với vòng benzen nên A thuộc loại dẫn 0,75 xuất halogen khả năng phản ứng thấp; - A bền với axit vì mật độ electron ở 2 nguyên tử O di chuyển vào nhân benzen nên khó tác dụng với axit (0,5).

c)

Câu 7 (2 điểm) Hiđrocacbon: 1. Một số hidrocacbon C10H16 tham gia vào các chuyển hóa sau:

Xác định cấu trúc của C10H16 và các chất từ A-D biết C và D là đồng phân của hidrocacbon ban đầu. Ozon phân C, rồi xử lý hỗn hợp sản phẩm với dung dịch kiềm H2O2 tạo một sản phẩm, nếu tiến hành tương tự với

D tạo 2 sản phẩm. 2. Hoàn thành sơ đồ sau: 1. O3 2. CH3SCH3

A

CrO3, H2SO4 H2O

EtOH

B

H2SO4

C

1. EtONa, EtOH +

D

2. H3O

1. LiAlH4

F

2. H2O

C13H24O2

Câu 7

Đáp án

Điểm

1

Xử lý A với bazo tạo xeton 2 vòng không no (C10H14O) có cùng số C so với

0,5

hidrocacbon ban đầu C10H16. Do đó, hidrocacbon ban đầu chứa liên kết endoxiclic không no; A chứa 2 nhóm cacbonyl và CTPT là C10H16O2; xeton 2 vòng không no là sản phẩm của sự ngưng tụ andol nội phân tử. Do đó, A là xiclodecan-1,6-đion, hidrocacbon ban đầu là: octahidronaphthalen:

Sự ozon phân octahidronaphthalen tiếp theo khử ozonit với NaBH4 tạo xiclodecan-1,6-điol, sau đó tách nước tạo 2 xiclodecadien C và D. Ozon phân C tạo một sản phẩm duy nhất cho thấy C là sản phẩm đối xứng, đó là xiclodeca-1,6dien, do đó D là xiclodeca-1,5-dien.


0,5

2

Công thức các chất:

1,0

Câu 8 (2 điểm) Xác định cấu trúc, đồng phân lập thể : 1. Chất A có CTPT là C8H16O, cho phản ứng Iođofom nhưng không cộng được H2. Khi đun A với H2SO4 đặc ở 170oC ta thu được chất B và C (cả hai đều có CTPT là C8H14). Nếu ôxi hóa B rồi đề cacboxyl sản phẩm sẽ thu được metylxiclopentan. Chât B không có đồng phân hình học. Xác định CTCT của A,B,C và giải thích sự tạo ra chất C. 2. Metylisopropylxeton phản ứng với đietyl cacbonat trong môi trường kiềm-rượu, tạo thành hợp chất A. Cho axeton tác dụng với fomanđehit và đietylamin, thu được chất B. Metyl hoá B bằng metyl iođua, sau đó tiến hành tách loại Hopman, thu được C. Khi C phản ứng với A trong môi trường kiềm- rượu thì được D. Cho D phản ứng với NaOH, sau với axit HCl và cuối cùng đun nóng thì được E. Hãy xác định công thức cấu tạo của các hợp chất từ A đến E và hoàn thành các phương trình phản ứng. Câu 8

Đáp án

Điểm


v =1, cho phản ứng Iođofom nhưng. Chất A có không cộng được H2 A có

0,75

CH3-CHOH nhóm Và A có 1 vòng. Nếu ôxi hóa B rồi đề cacboxyl sản phẩm sẽ thu được metylxiclopentan nên A cũng phải có bộ khung giống metylxiclopentan. Vậy A là 1 trong 3 chất sau đây:

OH

HO

OH

(1) (2)

(3)

Chất 3 phù hợp do tách nước sinh ra B không có đồng phân hình học. B là : C là

Sự tạo thành chất C:

0,25 - H+

H+ OH

2

O

(+)

- H2O

O

CO(OEt)2

A + CH2O + Et2NH

Manich

1,0

O

EtONa

O

(+)

OEt

O

1. CH3I

NEt2 B

O

2. T¸ch Hopman

O A, EtONa

1. NaOH + 2. H o

3. t

C

E: 3-isopropylxiclohex-2-enon Câu 9 (2,0 điểm): Cơ chế phản ứng 1. Hãy cho biết cơ chế hình thành sản phẩm của các phản ứng sau:

COOEt D


O

O O

a)

MeONa C2H5OH

O

O O

OH OH Ph

O

Ph

CH2O

b)

N

CSA

NH CHPh2

Ph

Ph

2. Giải thích tại sao R-C6H13CHBrCH3 trong rượu - nước cho 17% sản phẩm cấu hình R, 83% cấu hình S. Còn R-C6H5CHClCH3 cùng điều kiện trên cho 48% sản phẩm cấu hình R, 52% cấu hình S.

Câu 9

Đáp án

1-a

O

O

O

O

O

O

-

O

OMe

OMe

OMe -

O

-

O H

OH

OH

HO

OMe

OMe O

MeONa

O

0,5

O

O

O a)

Điểm

O

O

-

O

O O O


1-b

OH Ph

0,5

OH Ph CH2O

b) NH

-H2O

N

CHPh2

CHPh2 O

Ph

HO

Ph

[3.3]

N

+

-H

N

Ph

Ph

CHPh2

2

0,25

R A

(1) RC6H13CHBrCH3 R A H2O:

Br H

CH3

Br H

CH3 S

SN 2

+

CH3 OH H 2

A

-H

CH3 H OH (SPC)

A

0,25

R (2) R B

B

RC6H4CHClCH3

Cl

CH3

SN1

Cl H

CH3 R

S B

CH3

H2O -H+

B

CH3 H OH

B

HO H

C

hon hop raxemic

Phản ứng (1) xảy ra theo cơ chế SN2 là chủ yếu vì cacbocation tạo thành trong

0,25

phản ứng SN1 là cacbocation bậc hai kém bền. Nhóm thế gắn với cacbon bậc 2 phản ứng theo SN2 thuận lợi hơn. -Phản ứng (2) xảy ra theo cơ chế SN1 là chủ yếu vì cacbocation tạo thành: điện

0,25

tích dương được giải tỏa bởi hiệu ứng +C của vong thơm.Nên sản phẩm thu được gồm hai đồng phân S, R tương đương nhau. Đồng phân S lớn hơn một chút do có phản ứng SN2 xảy ra nhưng không nhiều. Cl

H

+

SN 1 R R

-Cl

+

+

R

H

HO H H2O -H

R

S 52%

R

R 48%


Bài 10. (2 điểm): Tổng hợp hữu cơ 1.Viết công thức cấu tạo của các chất từ C đến C5 và hoàn thành sơ đồ chuyển hóa sau:

(Biết C3 không làm mất màu KMnO4 loãng) 2. Viết công thức cấu tạo của các chất từ D đến D4 và hoàn thành sơ đồ chuyển hóa sau:

Câu 10

Đáp án

Điểm


1

1,0

2

1,0


HỘI CÁC TRƯỜNG CHUYÊN VÙNG

ĐỀ THI MÔN HÓA HỌC – KHỐI 11

DUYÊN HẢI VÀ ĐỒNG BẰNG BẮC BỘ

NĂM 2017

TRƯỜNG THPT CHUYÊN LAM SƠN

Thời gian làm bài: 180 phút

TỈNH THANH HÓA

(Đề này có 05 trang, gồm 10 câu)

ĐỀ THI ĐỀ XUẤT

Câu 1: (2,0 điểm) Tốc độ phản ứng 1. Cho phản ứng: k

1  →B A ←

(các hằng số tốc độ phản ứng k1 = 300 s–1; k2 = 100 s–1).

k2

(xe là nồng độ chất lúc cân bằng; x là nồng độ chất đã phản ứng).

(1) k1 + k2 =

xe 1 ln t xe - x

(2) Ở thời điểm t = 0, chỉ có chất A mà không có chất B. Trong thời gian bao lâu thì một nửa lượng chất A chuyển thành chất B? 2. Cho phản ứng pha khí: 2NO (k) + O2 (k) → 2NO2 (k) (3) 2 Phản ứng (3) tuân theo định luật tốc độ thực nghiệm v = k[NO] [O2]. Giả định rằng phản ứng không diễn ra theo một giai đoạn sơ cấp. Hãy đề nghị một cơ chế có khả năng cho phản ứng (3) và chứng tỏ rằng cơ chế ấy phù hợp với thực nghiệm động học. Câu 2:(2,0 điểm) Cân bằng trong dung dịch chất điện li

1.Tính pH bắt đầu kết tủa và kết tủa hoàn toàn Cr(OH)3 từ dung dịch CrCl3 0,010M. 2.Tính độ tan của Cr(OH)3? (coi như khi kết tủa hoàn toàn nồng độ cation còn lại là 10-6 M) pKs của Cr(OH)3 = 29,8;

Lg *βCrOH2+ = - 3,8

Cr(OH)3↓ = H+ + CrO2- + H2O

K = 10-14

Câu 3: (2 điểm) Điện hóa học 1. Thiết lập khu vực pH sao cho K2Cr2O7 có thể oxi hóa được hơn 80% Br- và ít hơn 2% Cl- từ hỗn hợp KBr 0,010M và KCl 1,0M. 0

Cho: E Cl

2 / 2 Cl

= 1,36 V; E 0Br

2 ( l ) / 2 Br

= 1,065 V; E 0Cr

2

O7 2 − / 2 Cr 3 +

= 1,33 V;

[Cr2O72-] = [Cr3+] = 1M. Độ tan của Br2 trong nước là 0,22 M.

2. Đánh giá thành phần cân bằng trong hỗn hợp KClO3 0,10M và FeBr2 0,060M ở pH= 2,0. Cho:

E 0ClO− /Cl− = 1,45 V; E 0Br 3

*

2 (H 2 O) / 2 Br

= 1,085 V; E 0Fe3+ / Fe2+− = 0,771 V;

βIII = 10 −2,17 (Fe3+ ) ; * βII = 10 −5,92 (Fe2+ )

Câu 4: (2 điểm) N - P, C - Si và hợp chất


Cho m gam hợp chất X ( được tạo thành từ hai nguyên tố) phản ứng hoàn toàn với H2SO4 đặc, nóng chỉ thu được 20,16 lít (đktc) hỗn hợp khí A gồm hai khí và H2O. A làm mất màu vừa đủ 1,6 lít dung dịch Br2 0,5M

và A không có phản ứng với dung dịch CuCl2. Cho A vào dung dịch Ca(OH)2 dư, thu được 106 gam kết tủa trắng. Xác định công thức của X, và tính m.

Câu 5: (2 điểm) Phức chất, trắc quang Chất A được tạo từ cation K+ và anion Xn– . Chất B được tạo từ cation K+ và anion Xm–. Hai anion này đều là anion phức bát diện nhưng khác nhau về momen từ: µ X n − = 0; µ X m− = 1,72D. Trong phối tử của hai anion trên chỉ chứa hai nguyên tố thuộc chu kỳ 2. Khi cho 20mL dung dịch 0,1M của A tác dụng với 1,3240 gam Pb(NO3)2 thì tạo thành 1,2520 gam kết tủa trắng và trong dung dịch chỉ còn lại muối kali. Khi cho 1,2700 gam FeCl2 vào một lượng dư dung dịch của A thì tạo thành 1,6200 gam kết tủa trắng C (chứa 51,85% khối lượng là sắt). Khi để ra ngoài không khí C trở thành xanh lơ và chuyển thành D. Dung dịch của B tác dụng với FeCl2 cũng tạo thành D. Biết momen từ : µ = n (n + 2) ; trong đó n là số electron độc thân của ion trung tâm. a) Các chất A, B, C là những chất gì? b) Viết các phương trình phản ứng xảy ra.

Câu 6: (2 điểm) Quan hệ giữa hiệu ứng cấu trúc và tính chất 1.Cho các chất: anilin, glyxerol, axit photphoric. Viết sơ đồ các phương trình phản ứng để điều chế N

(quinolin).

2. Giải thích sự khác nhau về nhiệt độ sôi trong dãy chất sau: N

N N

S

N 1150C

1170C

H 2560C

N

N

H 1870C

Câu 7: (2 điểm) Hiđrocacbon 1. Chất K có công thức:

Từ chất A là 1,2,3-trimetylbenzen người ta tổng hợp ra chất K theo sơ đồ: −

A

+ 0

NBS CN du H3O ,t  → B  → C   → 1:2

ThO2 1.(CH ) CHMgCl  → E  →F 2.H O

CH3Cl H2SO4 ,t 2 ,Pd  → G  →K → H → I H AlCl3 400 C 0

F

D

CH2 N2 as

3 2

2


a) Hãy cho biết cấu tạo của các chất từ B đến I. Trong phản ứng I

2,Pd H  → K có thể tạo ra sản phẩm nào 400C

khác không ? b) So sánh nhiệt độ sôi của các chất: D, E và G. Giải thích ngắn gọn? Hợp chất F có 2 đồng phân cấu hình, hãy cho biết nhiệt độ sôi của chúng giống nhau hay khác nhau? Tại sao? 2. Từ metyl xiclopropyl xeton và hợp chất cơ magie tuỳ ý chọn, viết sơ đồ phản ứng điều chế 2,6-đimetyl-9bromnona-2,6-đien. Câu 8: (2 điểm) Xác định cấu trúc, đồng phân lập thể, danh pháp 1. Khi cùng một lượng buta-1,3-dien và brom phản ứng với nhau ở nhiệt độ cao sẽ tạo ra hai hợp chất

G (sản phẩm chính) và H (sản phẩm phụ) với công thức C4H6Br2. Hợp chất G phản ứng với lượng dư Br2 để tạo hợp chất I (C4H6Br4) là hợp chất meso. Hợp chất H phản ứng với lượng dư Br2 tạo ra chất I và đồng phân dia J. Vẽ công thức cấu trúc của G và H. Vẽ công thức chiếu Fischer của I và J. Chỉ ra cấu hình tuyệt đối ở các trung tâm bất đối. 2. a) Tiến hành phản ứng Diels-Alder giữa 2,5-dimetylfuran và anhydrit maleic cho hợp chất K tồn tại ở hai dạng đồng phân lập thể. Vẽ cấu trúc của hai đồng phân này.

b) Dehydrat hóa K xúc tác axit thu được chất L (C10H8O3). Vẽ cấu trúc của L. Câu 9: (2 điểm) Cơ chế phản ứng 1.Trình bày cơ chế các phản ứng sau:

a. O

Br Br

N

Zn/THF

b. HO H+ t0

2.Cho sơ đồ phản ứng sau:

O N

O

HO

O

O


a.Cho biết công thức cấu tạo các chất A, B, C, D, F. b.Cho biết F có thể có tất cả bao nhiêu đồng phân cấu hình? c.Trong số các đồng phân cấu hình của F, hãy viết công thức cấu hình một đồng phân và biểu diễn cấu

dạng của đồng phân đó.

Câu 10: (2 điểm) Tổng hợp các chất hữu cơ ( Dạng sơ đồ phản ứng) 1. Hoàn thành sơ đồ phản ứng sau bằng cách xác định cấu trúc các chất từ (A) đến (L)

2. Chỉ thị axit- bazơ phenolphthalein được điều chế bằng phản ứng giữa anhidritphtalit và phenol xúc tác H2SO4 có phản ứng sau:

Viết cơ chế cho phản ứng tổng hợp phenolphthalein và cơ chế cho quá trình chuyển hóa phenolphtalein thành đianion màu đỏ trong môi trường bazơ.


Người ra đề

Lê Văn Đậu (Sđt: 0912137307) HỘI CÁC TRƯỜNG CHUYÊN VÙNG DUYÊN

ĐỀ THI MÔN HÓA HỌC – KHỐI 11

HẢI VÀ ĐỒNG BẰNG BẮC BỘ

NĂM 2017

TRƯỜNG THPT CHUYÊN LAM SƠN

Thời gian làm bài: 180 phút

TỈNH THANH HÓA

ĐÁP ÁN ĐỀ THI ĐỀ NGHỊ Câu 1(2 điểm) Động học - Cân bằng hóa học k

1  → B A ←

1) (1 điểm)

k2

Nồng độ đầu: a 0 Nồng độ cân bằng: a - xe xe Ta có xe được xác định qua hằng số cân bằng (K): xe [B] aK K= = → xe = [A] a - x e 1+K Tại thời điểm một nửa lượng chất A đã tham gia phản ứng: x = a/2; t = t1/2

a aK a 2aK - a - aK a(K - 1) = - = = 2 1+K 2 2(1 + K) 2(1 + K) x aK/(1+K) 2K → e = = xe - x a(K-1)/[2(1+K)] K - 1

→ xe – x = x e -

Thay

(0,5 điểm)

xe 2K 2,303 2K 2,303 2K vào (2), ta có: k1 + k2 = → t1/2 = = lg lg xe - x K-1 t1/2 K-1 k1 + k 2 K - 1

Vì K =

k1 , nên: k2

2k1 2,303 2,303 2 . 300 lg = lg = 2,75.10-3 (s). k1 + k 2 k1 - k 2 300 + 100 300 - 100 Vậy sau 2,75.10-3 giây thì một nửa lượng chất A đã chuyển thành chất B.

t1/2 =

2) Phản ứng có thể xảy ra theo cơ chế hai giai đoạn: 2NO → N2O2 (a) (nhanh) k2 N2O2 + O2  → 2NO2

(b)

(chậm)

(0,5 điểm)


Cộng (a) với (b) sẽ thu được phản ứng tổng cộng (3). Giai đoạn (b)chậm, quyết định tốc độ chung của phản ứng, nên: v = k2[N2O2][ O2] (*) (0,5 điểm) Do giai đoạn (b) chậm và (a) nhanh nên có thể coi cân bằng (a) được thiết lập, khi đó có: [N2O2]/[NO]2 = k1/k-1 → [N2O2] = (k1/k-1)[NO]2 (2*) Thay (2*) vào (*) thu được: v = (k1/k-1)k2[NO]2[ O2] = k[NO]2[ O2] với k = (k1/k-1)k2. Như vậy từ cơ chế giả định có thể rút ra được định luật tốc độ thực nghiệm. Cơ chế là có khả năng. (0,5 điểm) Chú ý: Thí sinh có thể đưa ra cơ chế khác. Nếu chứng minh chặt chẽ rằng cơ chế đó phù hợp với thực nghiệm thì cho đủ điểm.

Câu 2(2,0 điểm): Cân bằng trong dung dịch chất điện li ' 3 ’ 1. - Khi bắt đầu kết tủa thì: C'Cr3+ .(COH − ) ≥ Ks (C là nồng độ ion trước khi tạ o kết tủa)

Trong đó: C'OH− = [OH-];

C'Cr3+ = CCr3+ - CCrOH2+

Cr 3+ +

Xét cân bằng:

H2 O

CrOH 2+ + H+

*

β = 10-3,8

0,01 0,01-x

x

Giải ra được: x = 1,182.10-3 → →

C'OH− = [OH-] =

3

x

C'Cr3+ = 8,818.10-3M Ks = 5,643.10-10 M ' CCr3+

(0,5điểm)

→ pH = 4,75.

Khi kết tủa hoàn toàn thì nồng độ còn lại của Cr3+ là 10-6M tức là: [Cr3+] → [Cr3+] =

+ [CrOH2+] = 10-6 M

Ks K s .h 3 10−6 = = 1 + β.h −1 [OH]3 (K w )3

→ Ks.h3 + *β.h2.Ks – (Kw)3.10-6 = 0

→ h = 6,038.10-8 M → pH = 7,2

2. Tổ hợp các cân bằng:

Cr(OH)3 ↓ Cr3+ + H2O ⇌ H+ + OH- ⇌ Ta được cân bằng:

Cr(OH)3 ↓

Cr3+

+

CrOH2+ + H+

3 OH*β = 10-3,8

Kw-1 = 1014

H2 O

Ks = 10-29,8

CrOH2+ + 2 OH- (1) có K1 = 10-19,6

So sánh các cân bằng, nhận thấy tính độ tan theo các cân bằng sau:

Cr(OH)3 ↓

CrOH2+ + 2 OH-

H2 O

H+ + OH-

K1 = 10-19,6 Kw = 10-14

(0,5điểm)


H+ + CrO2- + H2O

Cr(OH)3↓ ⇌

K3 = 10-14

Áp dụng định luật bảo toàn nồng độ, ta có: [OH-] = 2.[CrOH2+] + [H+] -  CrO−2  =   → [OH − ] . ( 1 +

2K1 [OH− ]2

2K1 Kw K3 ) = + − 2 Kw [OH ] [OH− ]

+

Kw K 3 .[OH− ] [OH− ] Kw [OH− ] = 2,986.10-7 M (0,75điểm)

Ta có: Độ tan S =  CrO−2  + [CrOH2+ ] + [Cr3+] (trong đó [Cr3+] rất nhỏ, bỏ qua)

K s .[OH− ] K1 = + = 5,8.10-7 M − 2 Kw [OH ]

→S

(0,25điểm)

Câu 3: (2 điểm) Điện hóa học Để oxi hóa hơn 80% Br- và ít hơn 2% Cl- thì:

1)

E Br /2Br− < E Cr O 2 − /2 Cr3+ < E Cl 2

2

7

2

/2Cl −

⇒ 1,1733 < 1,33 – 0,138pH < 1,3013

(0,5điểm)

⇒ 0,21 < pH < 1,14

E 0Br

Ở pH = 2,0

E 'Br

− 2 (H 2 O) / 2 Br

2 (H 2 O) / 2 Br

= E 0Br

2 (H 2 O) / 2 Br

= 1,085 V không phụ thuộc vào pH vì vậy thế điều kiện

= 1,085 V;

Đối với cặp ClO3Nhận xét: Vì lượng Br2 sinh ra tối đa chỉ có thể bằng 5.10-3M, bé hơn độ tan của 0

Br2 trong nước vì vậy phải tính E Br

2 ( H 2 O ) / 2 Br

: 0

Br2 (l) + 2e 2Br-

K1 = 102E1 / 0,0592

Br2 (H2O)

K2 =

Br2 (l)

0

K = 102E3 /0,0592 = K1.K2

Br2 (H2O) + 2e 2Br-

⇒ E 30 = E10 E 0Br

− 2 ( H 2 O ) /2Br

0

Ta có: E Cl

2 / 2 Cl

[Br2 (l)] = S−Br12 Br2 (H 2 O)

0, 0592 lg S Br2 2

= E 30 = 1,065 -

= 1,36 V > E 0Br

0,059 lg 0,22 = 1,0845 V 2

2 ( H2 O ) /2Br

(0,5điểm)

= 1,0845 V nên Cr2O72- oxi hóa Br- trước, Cl- sau.


- Để oxi hóa 80% Br- thì [Br-] còn = 0,2.0,01 = 2.10-3M [Br2] = 1/2.(0,01 – 2.10-3) = 4.10-3M 0 E Br /2Br − = E Br + /2Br − 2

2

0, 0592 [Br2 ] lg − 2 = 1,1733V 2 [Br ]

Oxi hóa 2% Cl- thì [Cl-] = 0,98M; [Cl2] = 0,01M

E Cl

2

/2Cl

0 = E Cl

2 /2Cl

+

0,0592 [Cl2 ] lg − 2 = 1,3013V 2 [Cl ]

Cr2O72- + 14H+ + 6e 2Cr3+ + 7H2O

0,0592 [Cr2 O7 2− ][. H + ]14 lg 2 7 6 [Cr 3+ ]2 14.0,0592 = 1,33 pH 6

0 E Cr O 2 − /2 Cr3+ = E Cr + O 2 − /2Cr 3 + 2

7

-

/Cl-:

ClO3- + 6H+ + 6e Cl- + 3H2O

0, 0592 [ClO3− ] 0, 0592 E=E + lg + lg[H + ]6 − 6 [Cl ] 6 0

Tính thế điều kiện E’:

0, 0592 [ClO3− ]' E=E + lg - 0,0592pH 6 [Cl − ]' '

Vì [ClO3-]’ = [ClO3-]; [Cl-]’ = [Cl-] nên E’ = E0 – 0,0592pH Ở pH = 2 thì E’ = 1,45 – 0,0592.2 = 1,33V Đối với cặp Fe3+/Fe2+: *

Fe3+ + H2O FeOH2+ + H+

*

Fe2+ + H2O FeOH+ + H+

E=E

' Fe3+ /Fe2 +

βIII = 10 −2,17 βII = 10 −5,92

[Fe3+ ]' [Fe3+ ] 0 + 0,0592 lg 2+ ' = E Fe3+ /Fe2+ + 0,0592 lg [Fe ] [Fe 2+ ]

Vì [Fe3+]’ = [Fe3+] + [FeOH2+] = [Fe3+].(1 + βIII .h-1) *

[Fe2+]’ = [Fe2+] + [FeOH+] = [Fe2+].(1 + β II .h-1) *

Do đó tổ hợp lại ta có:

h + * βIII 10 −2 + 10 −2,17 = 0,771 - 0,0592 lg −2 = 0,758V E = E - 0,0592 lg h + * βII 10 + 10 −5,92 '

0

Phản ứng đầu tiên xảy ra: ClO3- + 6Fe2+ Cl- + 6Fe3+

K = 106(1,33-0,758)/0,0592 = 1057,97

(0,5điểm)


C0

0,10

0,06

C

0,09

-

0,01

0,06

Phản ứng tiếp theo: ClO3- + 6Br- Cl- + C0

0,09

0,12

C

0,07

-

3Br2

K = 106(1,33-1,085)/0,0592 = 1024,83

0,01 0,03

0,06

Xét cân bằng ngược: 3Br2 []

+ Cl-

0,06 -3x

ClO3- + 6Br-

0,03-x

0,07+x

10-24,83

6x

(6x)6 .(0, 07 + x) = 10 −24,83 3 (0, 06 - 3x) .(0, 03 -x) ⇒ 6x = [Br-] = 1,55.10-5M

[Br2] = 0,060M; [Cl-] = 0,03M; [ClO3-] = 0,07M Từ (1): []

6Fe3+

+ Cl- 6Fe2+ + ClO3-

0,06-6x

0,03-x

6x

10-57,97

0,07+x

Giả thiết x << 0,01 ⇒ [Fe2+] = [Fe2+]’ = 6x = 10-10,94M [Fe3+] = [Fe3+]’ .

1 0, 06 = 0,0358M = −1 1 + βIII .h 1 + 10 −2,17 .10 2 *

[FeOH2+] = 0,0242M

(0,5điểm)

Câu 4: (2 điểm) N - P, C - Si và hợp chất

nA =

20,16 = 0,9(mol ) 22, 4 Trong A có SO2 và một khí Y , Y không phản ứng với dung dịch Br2 nBr2 = 0,5.1,6 = 0,8 (mol) => nSO2 = 0,8 (mol) (0,5điểm)

=> nY = 0,1 (mol) Kết tủa gồm 0,8 mol CaSO3 và kết tủa do Y tạo ra. m (CaSO3) = 0,8. 120 = 96 (gam) => kết tủa do Y tạo ra = 106 – 96 = 10 (gam) Mà nY = 0,1 (mol) => Y là CO2 và kết tủa là CaCO3 => A gồm 0,1 mol CO2 và 0,8 mol SO2 => X chứa hai nguyên tố là C và S Giả sử công thức của X là CSx => CSx  → C + 4 + xS+ 4 + (4 + 4x)e S+ 6 + 2e  → S+ 4 n(CO2) : n(SO2) = 1 :8 => x + 2 + 2x = 8 => x = 2 Công thức của X là CS2 và m = 0,1.76 = 7,6 gam

Câu 5: (2 điểm) Phức chất, trắc quang

(0,5điểm)

(0,5điểm)

(0,5điểm)


a)

n Pb(NO nA

3 )2

=

2Pb2+

1,3240 : 331 2 ⇒ Anion trong A là X4= 0,1 .0,02 1

X4-

+

4.10-3

2.10-3

M Pb X = 2

2Fe2+

 →

Pb2X↓

2.10-3 (mol)

1,252 = 626 (g / mol) ⇒ M X4− = 626 – 207.2 = 212 (g/mol) 2.10 −3

+

X4-

 →

0,01

Fe2X↓ (C) 0,005 (mol)

n FeCl = 2

1,62 1,27 = 324 (g / mol) = 0,01 (mol) ; MC = M Fe X = 2 127 0,005

Số nguyên tử Fe trong chất C =

324.51,85 =3 100.56

⇒ ion X4- có 1 nguyên tử Fe (0,5điểm)

Vì Xn- là phức bát diện nên số phối tử là 6 Mà M X4− = 212 (g/mol) ⇒ Mphối tử =

212 − 56 = 26 6

⇒ phối tử là CN-

µ X 4− = 0 ⇒ ion X4- chứa Fe2+ ⇒ X4- là [Fe(CN)6]4-.

Vậy : A là K4[Fe(CN)6]

(0,5điểm)

Anion Xm- : [Fe(CN)6]m-

µ X m− = 1,72 = [n(n+2)]1/2 ⇒ n ≈ 1 ⇒ Xm- chứa Fe3+. Vậy B là K3[Fe(CN)6] Suy ra C: Fe2[Fe(CN)6];

(0,5điểm)

b) K4[Fe(CN)6] + 2Pb(NO3)2 → Pb2[Fe(CN)6]↓ + 4KNO3 K4[Fe(CN)6] + 2FeCl2 → Fe2[Fe(CN)6]↓ + 4KCl


2Fe2[Fe(CN)6] + 2K4[Fe(CN)6] + O2 + H2O → 4KFe[Fe(CN)6] + 4KOH K3[Fe(CN)6] + FeCl2 → KFe[Fe(CN)6]↓ + 2KCl

(0,5điểm)

Câu 6: (2 điểm) Quan hệ giữa hiệu ứng cấu trúc và tính chất 1. (1.0 điểm) Điều chế quinolin.

HOCH2-CHOH-CH2OH

H3PO4

CH2=CH-CHO

H

CHO

+

+

H

+ H2C=CH-CHO NH2

NH

OH

NH

-2H

NH

H - H2O

N

2. (1.0 điểm) Nhiệt độ sôi:

N

< N

S

M = 79

M = 85

Vì hai chất này không có liên kết hidro nên có nhiệt độ sôi thấp nhất và nhiệt độ sôi phụ thuuộc vào phân tử khối. Và hai chất N N

N

H

H

Có nhiệt độ sôi cao hơn vì chúng đều tạo liên kết hidro. Nhưng N N H

Hình thành liên kết hidro liên phân tử

N N H. . . N N H

Còn

N


N

N

H Hình thành liên kết nội phân tử tạo thành dạng dime N H.

N . .. . .N

H N

Vậy: Nhiệt độ sôi N

< S

N 1150C

1170C

<

N

N

N<

H 1870C

N H 2560C

Câu 7: (2 điểm) Hiđrocacbon 1. a) B

D

C Br

CN

COOH

Br

CN

COOH

G

F

E

OH

OH

O

H

I

Sản phẩm khác:

(1 điểm)


b. - Nhiệt độ sôi giảm dần theo trình tự: D > G > E - D, G có liên kết hyđro và liên kết hyđro giữa các phân tử D bền hơn giữa các phân tử G. + 2 đồng phân cấu hình của F có nhiệt độ sôi khác nhau vì chúng vừa là đồng phân quang học vừa là đồng phân hình học. (0,5điểm) O C

CH3

. . 3 3. CH3MgBr 1 Mg 2 H O+

. .

1 CH3MgBr 2 H2O

OH C CH 3 CH3

OH C

CH2CH2CH=C CH3

+ HBr - H2O

Br CH2CH2CH=C

CH3 HBr CH3

CH3 CH3

Br CH2CH2CH=C CH2CH2CH=C

CH3 CH3

CH3

(0,5điểm) Câu 8: (2 điểm) Xác định cấu trúc, đồng phân lập thể, danh pháp

(1 điểm)

a.

b.

(0,5 điểm)

(0,5 điểm)

Câu 9: (2 điểm) Cơ chế phản ứng


Br Br OZnBr

O

O

OZnBr

-ZnBr2

O N

N

N

O

(0,5điểm)

(0,5đ) Sơ đồ phản ứng: O

O 1.O3

CrO3, H2SO4

2.(CH3)2S

CHO

A

b.Số đồng phân cấu hình có thể có của F là 25 = 32 đồng phân.

Câu 10: (2 điểm) Tổng hợp các chất hữu cơ ( Dạng sơ đồ phản ứng) 1) ( 1 điểm)

C2H5OH COOH H2SO4

H2O

B

(1 điểm)


2- Viết cơ chế cho phản ứng tổng hợp phenolphtalein:


-

Cơ chế cho quá trình chuyển hóa phenolphtalein thành đianion màu đỏ trong môi trường bazơ.

( 1 điểm)


TRƯỜNG THPT CHUYÊN LÊ HỒNG PHONG-NAM ĐỊNH

ĐỀ THI CHỌN HỌC SINH GIỎI KHU VỰC MỞ RỘNG NĂM HỌC 2016- 2017 MÔN: HOÁ HỌC LỚP 11 Thời gian làm bài 180 phút

Câu 1: Tốc độ phản ứng – cân bằng hóa học. 1. Heroin và morphin là các chất gây nghiện. Khi heroin được vào cơ thể người chuyển hóa thành morphin (1 phân tử heroin biến đổi thành 1 phân tử morphin), sau đó sẽ bị đào thải ra khỏi cơ thể qua nước tiểu. Giả thiết các quá trình trên là các phản ứng bậc 1 đơn giản và được thể hiện bằng sơ đồ sau:

Heroin

k1 (1)

Morphin

k2 (2)

Saûn phaåm baøitieátra khoûicô theå

Phản ứng (1) và (2) có chu kì bán hủy lần lượt là 2,4 giờ và 20,0 giờ. a. Một đối tượng đã tiêm 18,45 mg heroin vào máu. Một ngày sau (24,0 giờ), đối tượng này bị kiểm tra ma túy bằng cách xét nghiệm máu. Hãy cho biết mẫu máu của đối tượng đó khi xét nghiệm có hàm lượng morphin và heroin là bao nhiêu (tính theo đơn vị nmol/l)? Giả thiết cơ thể đối tượng có 5 lít máu. Heroin có công thức phân tử là C21H23NO5. b. Sau bao lâu xét nghiệm kiểm tra morphin của đối tượng này cho kết quả âm tính? (kể từ thời điểm đối tượng sử dụng heroin). Biết rằng khi hàm lượng morphin dưới 30,0 nmol/l, xét nghiệm không phát hiện ra và cho kết quả âm tính. Chú ý: đơn vị nmol đọc là nanomol, 1 nmol= 10-9 mol. Hàm lượng morphin 30,0 nmol/l nghĩa là trong 1 lít máu có chứa 30,0 nmol morphin. 2. Thực hiện phản ứng sau: CaCO3(r) CaO(r) + CO2(k). Giả thiết ∆H và ∆S không thay đổi theo nhiệt độ. CaCO3(r) CaO(r) CO2(k) o ∆H (kJ/mol) -1207 -634 -394 ∆So (J/mol.K)

89

40

214

a. Xác định nhiệt độ tối thiểu để phản ứng phân hủy CaCO3 xảy ra. b. Thực hiện phản ứng ở 1000oC. Cho 5,0 gam CaCO3 vào bình phản ứng có thể tích không đổi 2,00 lít. Tính áp suất trong bình phản ứng khi phản ứng kết thúc. Hướng dẫn chấm: 1. Động hóa học: Ý 1 được 1 điểm. Tính đúng k1, k2 được 0,25 điểm. Tính đúng hàm lượng heroin, morphin sau 24h và ý b, mỗi ý được 0,25 điểm. Chú ý: Ý b nếu lập phương trình nhưng không giải được vẫn cho điểm tối đa. a. Tính các giá trị k1 và k2:

k1 =

ln2 = 0,289 2,4

k2 =

Hàm lượng heroin đối tượng đã tiêm vào máu:

ln2 = 0,035 20,0


C0Heroin =

18,45.10−3 109 . = 104 (nmol/l) 369 5

Sau thời gian 24 giờ hàm lượng heroin là

CHeroin = C0Heroin e-k1t = 104 . e-0,289.24 = 9,766 (nmol/l) Và hàm lượng morphin là

k1 C0Heroin (e-k1t - e-k2t ) k 2 -k1 0,289 = 104 . (e-0,289.24 - e-0,035.24 ) = 4,935.103 (nmol/l) 0,035-0,289

CMorphin =

b. Thời gian để phép xét nghiệm cho kết quả âm tính:

k1 C0Heroin (e-k1t - e-k2t ) k 2 -k1 0,289 = 104 . (e-0,289.t - e-0,035.t ) = 30,0 (nmol/l) 0,035 − 0,289

CMorphin =

⇒ t = 169,7 giờ. 2. Cân bằng hóa học: Ý 2 được 1 điểm. Tính đúng ý a được 0,5 điểm. Tính đúng ý b được 0,5 điểm. a. ∆Ho = -394 – 634 + 1207 = 179 kJ/mol. ∆So = 214 + 40 – 89 = 165 J/mol.K Nhiệt độ tối thiểu để phản ứng xảy ra: ∆Go = ∆Ho – T∆So < 0 ⇒ ∆Go = 179.103 – T.165 < 0 ⇒ T > 1085 K = 812oC. b. Tại 1000oC: ∆Go = ∆Ho – T∆So = 179.103 – 1273.165 = -31045 J/mol ∆Go = -RTlnKP ⇒ KP = 18,79 Nếu hệ đạt trạng thái cân bằng thì KP = PCO2 = 18,79 ⇒

nCO2 = PV = 2,0.18,79 = 0,36 mol > nCO2(tối đa) = 0,05 mol RT

0,082.1273

⇒ hệ chưa đạt được đến trạng thái cân bằng, toàn bộ CaCO3 bị phân hủy hết. Áp suất trong bình khi phản ứng kết thúc: nCO2 = 0,05 mol ⇒

PCO2 =

n CO2 .R.T 0,05.0,082.1273 = = 2,61 atm. VCO2 2,00

Vậy áp suất trong bình PCO2 = 2,61 atm. Câu 2: Cân bằng trong dung dịch điện ly. 1. Tính độ tan của CaCO3 trong dung dịch có pH = 7. Cho biết pKs của CaCO3 là 7,77; pKa1 và pKa2 của H2CO3 lần lượt là 6,35 và 10,33. 2. Cho 0,10 mol CaCO3 vào 1,00 lít nước. Cần đưa pH của nước đến giá trị nào để hòa tan hết lượng muối trên? Hướng dẫn chấm: 1. Có các cân bằng: Làm đúng ý 1 được 1 điểm. CaCO3(r)

Ca2+(dd) + CO32-(dd)

KS = [Ca 2+ ] . [CO32- ]


Ka1 =

H+(dd) + HCO3-(dd)

H2CO3(dd) HCO3-(dd)

H

+

(dd)

+

[H+ ] . [HCO3- ] [H2CO3 ]

[H + ] . [CO23 ] K a2 = [HCO3 ]

CO32-(dd)

Áp dụng bảo toàn nồng độ ta có: 2+

[Ca ] = [H 2CO3 ] +

[HCO3- ]

+

[CO32- ]

=

[HCO3- ]

+

[H [CO32- ]+

+

] . [HCO-3 ] Ka1

+ 2-     + + - ]  [H ] +1 = [CO2- ]+ [H ] . [CO3 ]  [H ] +1 = [CO2]+[HCO 3 3  K 3   K  Ka2 a1 a1 

 + + 2  = [CO32- ] 1+ [H ] + [H ]  Ka2 Ka1Ka2   

=

KS  [H + ] [H + ]2  +  1+  K a2 K a1K a2  [Ca 2+ ] 

 [H + ] [H + ]2  2+ ⇒ [Ca ] =KS 1+ +  ⇒ [Ca ] =  Ka2 K a1Ka2   Thay vào ta có: tại pH = 7: 2+ 2

[Ca 2+ ] = 10-7,77 1+

KS 1+ 

[H + ] [H + ]2  +  Ka2 Ka1Ka2 

10-7 (10-7 )2  + = 6,67.10-3 M -10,33 -6,35 -10,33   10 10 10 

 Độ tan của CaCO3 tại pH = 7 là 6,67.10-3 mol/lít. 2. Làm đúng ý 2 được 1 điểm. Khi 0,10 mol CaCO3 tan hết trong 1,00 lít thì nồng độ của Ca2+ là 0,1M

[Ca 2+ ] = KS 1+ 

+

 [H + ] [H + ]2  [H + ] [H + ]2 -7,77  + 0,10 = 10 1+ + ⇒   -10,33 -6,35 -10,33  Ka2 Ka1Ka2  10 10  10 

-6

[H ] = 3,29.10 M ⇒ pH = 5,48. Câu 3: Điện hóa học. Người ta chế tạo một pin điện hoạt động ở 25oC như sau: Điện cực đồng gồm một thanh Cu nhúng vào dung dịch CuSO4 0,15M; Điện cực kẽm gồm một thanh Zn nhúng vào dung dịch ZnSO4 2,00M. Ghép 2 điện cực trên thành một tế bào Ganvani. 1. Viết sơ đồ của tế bào Ganvani trên. Viết phương trình phản ứng xảy ra tại các điện cực và phản ứng tổng quát khi tế bào hoạt động. 2. Tính sức điện động của pin tại nhiệt độ 25oC. 3. Khi pin phóng hết điện, nồng độ các chất trong mỗi dung dịch là bao nhiêu? 4. Đặt một hiệu điện thế ngoài vào tế bào để tiến hành điện phân trong vòng 1 giờ với cường độ dòng điện 1,0 A, sau đó ngắt ra khỏi nguồn điện ngoài. Hãy cho biết sức điện động của pin ngay sau khi điện phân là bao nhiêu? Biết thể tích dung dịch ở mỗi điện cực là 1 lít. Bỏ qua ảnh hưởng của quá thế. Cho EoCu2+ /Cu = 0,337 V và Eo 2+ = -0,760 V Zn /Zn

Hướng dẫn chấm: Làm đúng ý 1 được 0,5 điểm: Viết sơ đồ pin 0,25 điểm + viết phương trình 0,25 điểm.


Làm đúng ý 2 được 0,5 điểm: Tính các E điện cực được 0,25 điểm + tính Epin 0,25 điểm. Làm đúng ý 3 được 0,5 điểm: Tính KC được 0,25 + Tính nồng độ 0,25 điểm. Làm đúng ý 4 được 0,5 điểm: Tính được nồng độ các chất sau khi điện phân 0,25 điểm + Epin 0,25 điểm. 1. Sơ đồ tế bào Galvani: Zn | ZnSO4 (2,00M) || CuSO4 (0,15M) | Cu. Tại anot: Zn → Zn2+ + 2e. Tại catot: Cu2+ + 2e → Cu. Phản ứng chung: Zn + Cu2+ → Zn2+ + Cu. 2. Sức điện động của tế bào tại 25oC:

RT 8,314.298 ln[Zn 2+ ] = -0,760 + ln2,00 = -0,751 (V) 2F 2.96500 RT 8,314.298 o ECu2+ /Cu = ECu ln[Cu 2+ ] = 0,337 + ln 0,15 = 0,313 (V) 2+ /Cu + 2F 2.96500 E Zn2+ /Zn = EoZn2+ /Zn +

Epin = 1,064 (V). 3. Pin phóng hết điện khi hệ đạt trạng thái cân bằng: o o Eopin = ECu 2+ /Cu - E Zn2+ /Zn = 1,097V

Eopin =

RT 8,314.298 lnK ⇒ 1,097 = lnK ⇒ K = 1037,11. nF 2.96500

K phản ứng rất lớn nên coi như phản ứng xảy ra hoàn toàn: Zn + Cu2+ → Zn2+ Ban đầu: 0,15 2,00 Sau phản ứng 0 2,15 Nồng độ ZnSO4 trong dung dịch là 2,15M. Tính nồng độ CuSO4 trong dung dịch: Cu + Zn2+ Cu2+ + Zn o C: 2,15 0,00 Cpu x x Csau (2,15 – x) x ⇒ K=

+

Cu

[Cu 2+ ] x = = 10 - 37,11 2+ [Zn ] 2,15 - x

⇒ x = 1,67.10-37 (M) Nồng độ CuSO4 trong dung dịch là 1,67.10-37 M. 4. Khi tiến hành điện phân sẽ diễn ra các quá trình: Tại catot: Zn2+ + 2e → Zn Tại catot: Cu → Cu2+ + 2e. Phản ứng chung: Zn2+ + Cu → Zn + Cu2+.

Số mol e trao đổi: ne = ⇒ n Zn2+pu = n Cu2+sinhra

It 1.1.60.60 = = 0,037 mol F 96500 = 0,019 mol

⇒ CZn2+saudp = 1,981 M và CCu2+saudp = 0,169 M

E Zn2+ /Zn = E oZn2+ /Zn +

RT 8,314.298 ln[Zn 2+ ] = -0,760 + ln1,981 = -0,751 (V) 2F 2.96500


o ECu2+ /Cu = ECu 2+ /Cu +

RT 8,314.298 ln[Cu 2+ ] = 0,337 + ln 0,169 = 0,314 (V) 2F 2.96500

⇒ Epin = 1,065 (V) Câu 4: Nhóm N – P, nhóm C – Si. Cho sơ đồ chuyển hóa sau:

X7

O2 + N 0o C -8

X3

Tia

+O

H2

X2

+ cao löû 2 ,p o C añi X e eä 1 500 xt F n

NaOH

t o,

+O

2

xt P

t

HClO

CO

X8 1. N a /H 2. g A gN O

3

X9

Cl + H to

X6

X3

20 0o C

X2

X4

X5

Các chất từ X2 đến X9 là các hợp chất của nguyên tố hóa học A. X1 là đơn chất của nguyên tố A. Một số tính chất của X1, X2, X3, X7 được thống kê ở bảng sau: Chất Màu sắc Tính tan (dung Môi trường dung dịch Nhiệt độ Nhiệt độ môi nước) chất (dung môi nước) nóng chảy sôi o Không màu Ít tan Trung tính -210 C -196oC X1 o Không màu Tan tốt Bazơ -78 C -33oC X2 Không màu Ít tan Trung tính -164oC -152oC X3 Màu xanh Tan tốt Axit -102oC 4,5oC X7 Xác định các chất từ X1 đến X9. Viết các phương trình hóa học xảy ra. Hướng dẫn chấm: Làm đúng câu 4 được 2 điểm: 1 điểm xác định công thức chất + 1 điểm phương trình Xác định đúng từ 4 đến 7 công thức được 0,5 điểm, từ 8 công thức trở đi được 1 điểm. Viết đúng từ 4 đến 8 phương trình được 0,5 điểm, từ 9 trở lên được 1 điểm. X1: N2; X2: NH3; X3: NO; X4: HNO3; X5: NH4NO3; X6: N2O; X7: N2O3; X8: NaNO2; X9: Ag2N2O2. Phương trình hóa học: 1. N2 + 3H2 → 2NH3. 2. 4NH3 + 5O2 → 4NO + 6H2O. 3. 2NO + 3HClO + H2O → 2HNO3 + 3HCl 4. HNO3 + NH3 → NH4NO3. 5. NH4NO3 → N2O + 2H2O. 6. N2O + CO → CO2 + N2. 7. N2 + O2 → 2NO. 8. NO + NO2 → N2O3. 9. N2O3 + 2NaOH → 2NaNO2 + H2O. 10. 2NaNO2 + 2AgNO3 + 4Na/Hg + 2H2O → Ag2N2O2 + 2NaNO3 + 4NaOH + 4Hg 11. Ag2N2O2 + 2HCl → N2O + 2AgCl + H2O. Câu 5: Phức chất, trắc quang. 1. Khi tiến hành oxi hóa muối CoCO3 bằng oxi không khí và dung dịch muối NH4Cl thu được dung dịch A màu hồng của một muối clorua (muối X) của phức Co (có công thức CoN4H12CO3Cl). Thêm dung dịch HCl dư vào dung dịch A và đun nóng thấy có khí không màu thoát ra, dung dịch chuyển dần sang màu tím (dung dịch B). Làm bay hơi nước của dung dịch B thu được phức Y (có công thức CoN4H12Cl3). Phức Y khi đun nóng với dung dịch HCl đặc thu được phức Z (có công thức CoN4H13Cl4) có màu xanh. Viết các phương trình phản ứng xảy ra trong thí nghiệm trên. Hãy biểu diễn cấu trúc của các phức X, Y, Z.


2. Độ hấp thụ riêng của phân tử anilin ở λmax = 279,1 nm là ε = 1,48.103 (L/mol.cm), trong khi đó, dạng proton hoá của anilin lại không hấp thụ ánh sáng có bước sóng trên. Độ truyền qua của một dung dịch chứa anilin với nồng độ 2.10-4 M trong cuvet dày 20 mm ở 279,1 nm bằng 0,92. Tính giá trị pH của dung dịch này. Cho hằng số bazơ của anilin là pKb bằng 9,2. Hướng dẫn chấm: 1. Làm đúng ý 1 được 1 điểm: 0,5 điểm xác định công thức chất + 0,5 điểm phương trình Xác định đúng từ 4 đến 7 công thức được 0,25 điểm, từ 8 công thức trở đi được 0,5 điểm. Muối clorua X có công thức CoN4H12CO3Cl, khi đun nóng với dung dịch HCl có thoát ra khí CO2 chứng tỏ trong muối X có gốc CO32-. Vì X là muối clorua nên cation phức có công thức: [CoN4H12CO3]+. Vậy công thức của muối X là

Phức Y có công thức CoN4H12Cl3 thu được khi cho X phản ứng với HCl, có giải phóng CO2. Y có công thức:

Phức Z thu được khi cho Y tác dụng với dung dịch HCl đặc. Z có công thức CoN4H13Cl4. Phản ứng có sự chuyển màu phức từ tím sang xanh. Phản ứng đồng phân hóa Y thành Z:

Cl H3N H3N Phương trình phản ứng:

Co Cl

NH3 NH3

Cl- . HCl


2. Ta có: Làm đúng ý 2 được 1 điểm. AnH+ + OH- (trong đó An là PhNH2) An + H2O

K [An] K b [An] [AnH + ][OH - ] ⇒ [OH - ] = b = [An] [AnH + ] C-[An] lg0,92 ⇒ [An]= = 1,22.10-5 A= -lgT = εl[An] 1,48.103 .2 6,31.10-10 .1,22.10-5 [OH - ]= = 4,09.10-11 ⇒ pH = 14 + lg(4,09.10-11 ) = 3,61 1,88.10-4 Kb =

Câu 6: Quan hệ giữa hiệu ứng cấu trúc với tính chất. 1. So sánh và giải thích ngắn gọn các tính chất sau đây: a. So sánh tính axit của H trong các phân tử sau:

b. So sánh nhiệt độ sôi của các chất sau:

S

N F 2. Cho các chất:

a. Giải thích tại sao mômen lưỡng cực của D (0,7 D), nhỏ hơn của C (1,7D)

O

O

b. So sánh nhiệt độ sôi của A, B, C, D, giải thích ngắn gọn.


c. Bằng phương pháp hóa học, hãy nhận biết 2 chất lỏng trong suốt riêng biệt: xiclopentan và THF (chất C). Hướng dẫn chấm: 1. Làm đúng ý 1 được 1 điểm: Mỗi ý so sánh và giải thích được 0,25 điểm. a.Tính axit của A > C > B. A tạo ra anion là hệ thơm, B tạo ra anion là hệ phản thơm, C tạo ra anion hệ không thơm. b. Nhiệt đội sôi D > F > E. D tạo liên kết H liên phân tử, F có phân tử khối lớn hơn E. 2. Làm đúng ý 2 được 1,5 điểm: Mỗi ý nhỏ được 0,5 điểm. a. Giải thích: C có hiệu +I của 2 nhóm CH2, do vậy vectơ momen lưỡng cực được biểu diễn như sau:

D có hiệu ứng liên hợp của O vào hệ electron π nên electron dồn ngược lại vào hệ liên kết C-C:

Hiệu ứng –I của O trong phân tử D không lớn bằng C do nguyên tử Csp2 có độ âm điện lớn hơn Csp3. b. So sánh nhiệt độ sôi: A > C > B > D. Phân tử A có M lớn hơn, tương tác khuếch tán mạnh hơn do có 2 nguyên tử O. B mạch hở, nên 2 nhóm thế có thể quay tự do, phân tử khó liên kết, hơn nữa làm momen lưỡng cực nhỏ hơn so với C. Phân tử D có mômen lưỡng cực nhỏ hơn B. c. Nhận biết: THF có nguyên tử oxi còn 2 cặp e không tham gia liên kết, do vậy THF có tính bazơ. Thuốc thử nhận biết là dung dịch H2SO4 đặc. THF tan trong dung dịch H2SO4 đặc tạo thành dung dịch đồng nhất. Xiclopentan khi cho vào dung dịch H2SO4 đặc không phản ứng và phân lớp. Câu 7: Hiđrocacbon. 1. Từ hiđrocacbon 5,5 – đimetylxiclopenta-1,3-đien (A), người ta tổng hợp được 2 xicloankan C và D theo sơ đồ sau:

Xác định công thức cấu tạo của B, C, D. 2. Một hiđrocacbon X quang hoạt có chứa 89,55% khối lượng là cacbon. Hiđro hóa hoàn toàn X bằng H2 dư (xúc tác Ni, đun nóng) thu được butylxiclohexan. Mặt khác, khi khử X bằng H2 với hệ xúc tác Lindlar, đun nóng, thu được hiđrocacbon Y. Cho Y phản ứng với O3 rồi xử lý sản phẩm thu được với dung dịch H2O2 trong môi trường kiềm thu được axit tricacboxylic Z quang hoạt (C8H12O6). Đun nóng Z với xúc tác P2O5 thu được hợp chất T (C8H10O5). Xác định công thức cấu tạo của các chất X, Y, Z, T.


Hướng dẫn chấm: 1. Làm đúng ý 1 được 0,75 điểm: Xác định đúng mỗi chất được 0,25 điểm. Công thức của các chất:

2. Làm đúng ý 2 được 1,25 điểm: Lập luận được công thức phân tử X được 0,25 điểm. Xác định đúng cấu tạo mỗi chất được 0,25 điểm. Hiđro hóa hoàn toàn X bằng H2 dư (xúc tác Ni, đun nóng) thu được butylxiclohexan. ⇒ X có 10 nguyên tử C Mà 89,55% khối lượng là cacbon ⇒ Công thức phân tử X là C10H14. (phân tử có độ không no k = 4) Mặt khác, khi khử X bằng H2 với hệ xúc tác Lindlar, đun nóng, thu được hiđrocacbon Y. ⇒ Y có liên kết C≡C. Ozon phân oxi hóa Y thu được axit tricacboxylic Z quang hoạt (C8H12O6). ⇒ Phản ứng tách ra CH3COOH. Đun nóng Z với xúc tác P2O5 thu được hợp chất T (C8H10O5) → phản ứng tách nước ra anhiđrit. Vậy công thức của các chất như sau:

Câu 8: Xác định cấu trúc. 1. Một hợp chất hữu cơ X có công thức phân tử C18H22O7. Để nghiên cứu cấu tạo của X, người ta tiến hành các thí nghiệm sau: Cho X phản ứng với thuốc thử Tollens, không thấy xuất hiện bạc kết tủa. Khi cho X phản ứng với PCC thấy dung dịch thuốc thử chuyển sang màu xanh, thu được hợp chất hữu cơ M. Cho X vào dung dịch KMnO4 loãng ở nhiệt độ thường, thấy dung dịch mất màu, thu được hợp chất N. Đun nóng X với I2/NaOH thấy xuất hiện kết tủa vàng và thu được hợp chất P. Cho cấu tạo các chất M, N, P như sau:


Xác định cấu tạo của X. 2. Một hợp chất hữu cơ A quang hoạt có công thức phân tử C5H10O5. Cho A phản ứng thuốc thử Tollens thấy cho kết tủa Ag. A có khả năng làm mất màu dung dịch nước brom cho sản phẩm hữu cơ B. Tiến hành khử B bằng HI, đun nóng thu được hợp chất C. Hợp chất C không quang hoạt và có khả năng làm quỳ tím ẩm chuyển sang màu đỏ. Cho B vào dung dịch Ca(OH)2 rồi lấy sản phẩm thu được xử lý bằng dung dịch H2O2 có mặt Fe(AcO)3 thu được hợp chất D. Đun nóng nhẹ D thu được sản phẩm E có công thức C4H8O4. Hợp chất E không quang hoạt. Xác định cấu tạo của các chất A, B, C, D, E. Hướng dẫn chấm: 1. Làm đúng ý 1 được 0,75 điểm: Lập luận và xác định đúng cấu tạo của X được 0,75 điểm. Cho X không phản ứng với thuốc thử Tollens ⇒ X không chứa chức anđehit. Khi cho X phản ứng với PCC thấy dung dịch thuốc thử chuyển sang màu xanh (màu của Cr2O3). ⇒ X chứa chức ancol, oxi hóa thành xeton, anđehit. Cho X phản ứng với dung dịch KMnO4 loãng ở nhiệt độ thường ⇒ X có liên kết π kém bền. Đun nóng X với I2/NaOH thấy xuất hiện kết tủa vàng ⇒ Có nhóm CH3CONgoài ra, I2 còn phản ứng với dung dịch ancol bậc I oxi hóa thành anđehit rồi thành axit. Công thức cấu tạo của X:

2. Làm đúng ý 2 được 1,25 điểm: Lập luận và xác định đúng cấu tạo mỗi chất được 0,25 điểm. Thoái phân B thu được D, đun nóng làm mất CO2 thu được E (C4H8O4). ⇒ Phân tử E có chức anđehit. E không quang hoạt suy ra công thức của E:

Công thức cấu tạo của các chất:


Câu 9: Cơ chế phản ứng. 1. Trình bày cơ chế các phản ứng sau:

a.

b. 2. Người ta thực hiện chuyển hóa đecan-2-on thành 2 – metyl đecanal theo sơ đồ sau:

Để nghiên cứu cơ chế của quá trình chuyển hóa, người ta đã đánh dấu nguyên tử cacbon ở nhóm metylen trong tác nhân metyl monocloaxetat bằng đồng vị 13C như sơ đồ trên. Hãy đề xuất cơ chế cho phản ứng (1) và (3). Hướng dẫn chấm: 1. Làm đúng ý 1 được 1 điểm: Viết đúng mỗi cơ chế được 0,5 điểm. Cơ chế phản ứng: a.

b.


2. Làm đúng ý 1 được 1 điểm: Xác định đúng công thức mỗi chất được 0,25 điểm và viết đúng mỗi cơ chế được 0,25 điểm. Công thức cấu tạo các chất trong sơ đồ:

Cơ chế phản ứng (1):

O

O CH3(CH2)7

+ ClCHCH3OOMe

COOMe

CH3(CH2)7

Cơ chế phản ứng (3):

A

Cl

Câu 10: Sơ đồ chuyển hóa, tổng hợp chất hữu cơ. 1. Flutriazol fungicide (kí hiệu là X) là một loại thuốc trừ nấm cho cây trồng có công thức cấu tạo như sau:

Để tổng hợp flutriazol fungicide người ta thực hiện sơ đồ sau:

Xác định công thức cấu tạo các chất A, B, C trong sơ đồ.

2. Xác định công thức cấu tạo các chất trong sơ đồ chuyển hóa sau:


Biết rằng chất N có công thức cấu tạo như sau:

Hướng dẫn chấm: 1. Làm đúng ý 1 được 0,75 điểm: Xác định đúng công thức mỗi chất được 0,25 điểm. Xác định công thức cấu tạo các chất trong sơ đồ:

2. Làm đúng ý 2 được 1,25 điểm: Viết đúng mỗi công thức được 0,1 điểm, sau đó tính tổng lại rồi làm tròn. Xác định chất sai từ đâu thì các chất sau đó không được tính điểm. Xác định công thức cấu tạo các chất trong sơ đồ:


HO 1. NaNO2 + HCl

CH3

2. H3O+, to

1. H2/Ni dö +

H

D

HCHO

OH 2. PCC E

O Me2NH2Cl F

Giáo viên soạn đề: Phạm Trọng Thịnh 0943666387

TRƯỜNG THPT CHUYÊN

ĐÁP ÁN ĐỀ ĐỀ NGHỊ

LÊ QUÝ ĐÔN ĐÀ NẴNG

MÔN: HÓA HỌC KHỐI 11

-------------------

NĂM HỌC 2016 – 2017 Thời gian làm bài 180 phút

Câu 1: (2,0 điểm) Động học (Có cơ chế) – Cân bằng hóa học


1.1. Năng lượng Gibbs chuẩn của phản ứng (1) trong pha khí có giá trị dương ∆G°(1) = 66 kJ.mol–1 tại Т =

k1  → HBr + H 600 К: Br + H 2 ←  k−1

(1)

(a) Xác định tỉ số của tốc độ phản ứng thuận và nghịch,

r1 , tại nhiệt độ này, áp suất của H2 và HBr là r−1

áp suất chuẩn còn áp suất của H và Br là bằng nhau. Nếu học sinh không làm được câu này, trong các tính toán tiếp theo, sử dụng giá trị tham khảo r1/r–1 = 3,14⋅10–7.

Phản ứng (1) xảy ra theo chiều thuận bởi sự có mặt đồng thời của phản ứng (2) trong hệ

k1  → HBr + H Br + H 2 ←  k −1

(1)

k2 H + Br2 → HBr + Br

(2)

k1, k–1 lần lượt là hằng số tốc độ phản ứng thuận và nghịch của phản ứng (1), k2 là hằng số tốc độ phản ứng thuận (2).

Giả thiết áp suất của các phân tử trung hòa là áp suất chuẩn, nghĩa là p(H2) = p(Br2) = p(HBr) = 1 bar, áp suất p(H), p(Br) đạt tới giá trị ở trạng thái dừng, hằng số tốc độ k2 lớn gấp 10 lần k–1. (b) Tính ∆G(1) và

r1 trong các điều kiện đã cho. r−1

1.2. Cho bảng số liệu sau: CH4 (k) ⇌ C(gr) + 2H2 (k) ∆H 0298,15 = 74,85 kJ.mol −1

(3)

CH4 (k)

C(gr)

H2 (k)

S0298,15 (J.K -1.mol−1 )

186,19

5,69

130,59

C0p,298,15 (J.K -1.mol −1 )

35,71

8,64

28,84

(a) Tính Kp của phản ứng (3) ở 250C. (b) Xác định ∆H 0T và Kp ở 7270C, coi C0p không phụ thuộc vào nhiệt độ. Cho T(K) = t0(C) + 273,15.

Câu

Nội dung

1.1. (a)

Điểm 1,0

Năng lượng Gibbs chuẩn của phản ứng (1) tại 600К is 66 kJ/mol. Hằng số cân bằng là: 0,5

K = e−66000/8.314/600 = 1,8.10−6 = k1 / k−1 . Xét phản ứng trong điều kiện áp suất chuẩn cho cả tác nhân và sản phẩm. Tỉ lệ tốc độ phản ứng thuận và nghịch:

r1 k [Br][H 2 ] k = 1 = 1 = 1,8.10−6 r−1 k−1[HBr][H] k−1


r1 = 1,8.10–6 r−1 (b)

à p d᝼ng nguyên lý trấng thåi dᝍng, ví d᝼ lẼy H:

d [H] = k1[Br][H 2 ] − k−1[HBr][H] − k2 [H][Br2 ] = 0 dt [H] k1[H 2 ] = [Br] k−1[HBr] + k2 [Br2 ] VĂŹ náť“ng Ä‘áť™ cᝧa cĂĄc phân táť­ trung hòa nhĆ° nhau (Ä‘áť u tĆ°ĆĄng ᝊng ĂĄp suẼt lĂ 1 bar) nĂŞn:

[H] k1 k /k 1,8.10−6 = = 1 −1 = = 1, 6.10−7 [Br] k−1 + k1 1 + k2 / k−1 1 + 10

0,25

Khi Ä‘Ăł năng lưᝣng Gibbs cᝧa phản ᝊng (1) lĂ :

∆G = ∆G + RT ln

[H][HBr] = 66 + 8,314.10−3.600.ln (1, 6.10−7 ) = −12 kJ.mol−1 [Br][H2 ]

Tᝉ lᝇ táť‘c Ä‘áť™ thuáş­n vĂ ngháť‹ch cᝧa phản ᝊng (1):

r1 k [Br][H 2 ] k [Br] k1 1 + k2 / k−1 k = 1 = 1 = = 1 + 2 = 11 r−1 k−1[HBr][H] k−1 [H] k−1 k1 / k−1 k−1

0,25

r1 = 11 r−1

∆G(1) = –12 kJ.mol–1;

1,0

1.2. (a)

∆S0298,15 = 5,69 + 130,59.2 - 186,19 = 80,68 (J.K-1.mol-1);

0,5

→ ∆G 0298,15 = ∆H 0298,15 - T ∆S0298,15

= 74,85.103 – 298,15.80,68 = 50,8.103 (J.mol-1) → ∆G 0298,15 = - RTlnKp → Kp = exp(

(b)

−50,8.103 ) = 1,26.10-9 8,314.298,15

∆C0298 = 8,64 + 28,84.2 - 35,71 = 30,61 (J.K-1.mol-1)

VĂŹ C0p,298,15 cĂĄc chẼt khĂ´ng thay Ä‘áť•i theo nhiᝇt Ä‘áť™ nĂŞn T

∆H 0T = ∆H 0298,15 +

âˆŤ

∆C0p dT

298,15

0,25

3

3

= 74,85.10 + 30,61(T-298,15) = 65,723.10 + 30,61.T 0 = 65,723.103 + 30,61.1000,15 = 96,34.103 (J.mol-1) → ∆H1000,15

(

∂ ln K ∆H 0T ) P0 = ∂T RT 2

→ KP(1000) = 13,116.

T

→

âˆŤ 298

T

d ln K =

1 65,723.103 + 30,61.T dT âˆŤ R 298 T2

0,25


Câu 2: (2,0 điểm) Cân bằng trong dung dịch điện li 2.1. Thêm từ từ dung dịch Pb(NO3)2 vào 20,0 mL dung dịch hỗn hợp gồm Na2SO4 0,02M; Na2C2O4 5,0.103

M; KI 9,7.10-3M; KCl 0,05M và KIO3 0,001M. Khi kết tủa màu vàng sáng của PbI2 bắt đầu xuất hiện

thì đã tiêu tốn hết 21,6 mL dung dịch Pb(NO3)2. (a) Xác định thứ tự các kết tủa trong cả quá trình. (b) Tính nồng độ dung dịch Pb(NO3)2 ban đầu. Cho: pK

s(PbSO4 )

= 7,66; pK

s(Pb(IO3 )2 )

= 12,61; pK

s(PbI2 )

= 7,86; pK

s(PbC2O4 )

= 10,05;

pK s(PbCl2 ) = 4, 77 . Các quá trình khác của ion xem như không đáng kể.

2.2. Nêu các hiện tượng, viết các phương trình hoá học và giải thích bằng tính toán cho các quá trình sau: (a) Cho 1 mL dung dịch KI 2.10-2M vào 1 mL dung dịch hỗn hợp gồm FeCl3 0,2M và HCl 0,2M; thêm vài giọt hồ tinh bột, thu được dung dịch hỗn hợp (1). (b) Thêm 1 mL dung dịch KF 1,5M vào dung dịch (1), thu được dung dịch (2). Cho: E 0 3+ 2 + = 0, 770V; E 0 Fe /Fe I

2 /2I

= +0,536V; β[FeF ]3− = 1016,1 . 6

Câu

Nội dung

2.1 (a)

Điểm 1,0

Điều kiện để có kết tủa:

0,5

10-7,66 PbSO4 : C 2+ ≥ = 1,09 × 10-6 (M) Pb (1) 0,02

≥ PbC2O4 : C 2+ Pb (2) PbI2 : C

Pb2+ (3)

Pb(IO3)2: C

PbCl2: C

Pb2+ (4)

Pb2+ (5)

10-10,05 5,0 × 10

−3

10-7,86 −3 2

(9,7 × 10 ) ≥

10-12,61 (0,001) 10-4,8

(0,05)

2

2

= 1,78 × 10-8 (M) = 1,47 × 10-4 (M) = 2,45 × 10-7 (M)

= 6,34 × 10-3 (M)

C Pb2+ (2) < C Pb2+ (4) < C Pb2+ (1) < C Pb2+ (3) < C Pb2+ (5) ⇒ Thứ tự kết tủa: PbC2O4, Pb(IO3)2, PbSO4, PbI2 and PbCl2.

(b)

Khi PbI2 bắt đầu kết tủa (giả sử I- tham gia phản ứng chưa đáng kể)

0,5


[SO 24 ]=

K s(PbSO4 ) CPb2+ (3)

=

10-7.66 1,47 × 10

-4

= 1,49 × 10-4 (M)

= K s(PbSO4 ) = 1,48 × 10-4 (M) = SPbSO4 (S là độ tan của PbSO4 trong dung dịch bão hòa). Do đó: PbC2O4, Pb(IO3)2 và PbSO4 đã kết tủa hoàn toàn.

→ 21,60 × C Pb(NO3 )2 = 20,00 × ( CC O2- + 0,5× CIO- + CSO2- ) 2

4

3

4

−3

= 20,00(5,0 × 10 + 0,5 × 0,0010 + 0,020)

→ C Pb(NO3 )2 = 0,0236 (M) 2.2. (a)

1,0 Khi thêm 1 mL dung dịch KI 2.10-2M vào 1 mL dung dịch FeCl3 0,2M và HCl 0,2M thì

0,5

C(Fe3+) = 0,1M; C(I-) = 10-2M; C(H+) = 0,1M. Trong hệ có phản ứng: 2Fe3+ + 2I − ⇌ 2Fe2 + + I2

(1)

Fe3+ + e ⇌ Fe 2+

E 0Fe3+ /Fe2+ = 0, 770V

2I− ⇌ I2 + 2e

E 0I

2 /2I

= 0, 536V

∆E0(1) = 0, 77 − 0, 0536 = 0, 234(V) Hằng số cân bằng phản ứng (1): K1 = 10

2. ∆E 0 (1) 0,0592

2.0,234

= 10 0,0592 = 107,91

K1 lớn nên có thể coi phản ứng xảy ra hoàn toàn. 2Fe3+ + 2I- → 2Fe2+ + Bđ []

0,1 9.10

I2

(1)

0,01 -2

0

0,01

5.10-3

Dung dịch hỗn hợp (1) có màu xanh (hoặc xanh đen) do tạo hỗn hợp I2 với hồ tinh bột.

(b)

Thêm 1 mL dung dịch KF 1,5M vào dung dịch (1) được 3 mL dung dịch (2). Thành phần chính của hệ: C(Fe3+) = 9.10-2.

2 2 = 6.10-2 (M); C(Fe2+) = .10-2 (M) 3 3

1 2 10−2 C(F-) = 1,5. =0,5 (M) ; C(I2) = 5.10-3. = (M) 3 3 3 Trong hệ xảy ra phản ứng (2) và (3) như sau: Fe3+

+ 6F-

[FeF6 ]3− (2) β = 1016,3 phản ứng hoàn toàn. [FeF ]3− 6

0,5


6.10-2

0,5

[]

0

0,14

6.10-2

Do tạo phức [FeF6 ]3− khá bền nên thế tiêu chuẩn điều kiện của hệ sắt giảm mạnh. 0 E[FeF

3− 2+ 6 ] /Fe

= E 0Fe3+ /Fe2 + − 0, 0592 log β[FeF ]3− = 0,77 - 0,0592.16,1= -0,18(V). 6

− 2 /2I

3− 2+ 6 ] /Fe

2+

2Fe

+ I2

nên I2 oxi hóa được Fe2+ theo phản ứng (3):

≪ E 0I

0 Như vậy, E[FeF

-

3-

+ 12F ⇌ 2[FeF6]

-

+ 2I (3) K3 = K 3 = 10

2[0,536 −( −0,18)] 0,0592

= 1024,19

K3 rất lớn nên phản ứng (3) xảy ra hoàn toàn. 2Fe2+ + I2 2/3.10-2

Bđ []

0

+ 12F- → 2[FeF6]3- + 2I-

10-2/3 0,14 0

(3)

6.10-2 (6+2/3).10-2 2/3.10-2

0,10

Do I2 bị khử hết thành I- nên dung dịch hỗn hợp (2) mất màu xanh. Câu 3: (2,0 điểm) Điện hóa học

Pin chì - axit thường được biết đến như loại pin chì được cấu tạo gồm 2 điện cực chì: cực dương phủ một lớp chì đioxit và cực âm được làm từ chì xốp. Các điện cực được nhúng trong dung dịch chất điện ly chứa nước và axit sunfuric. 3.1. Viết các quá trình hóa học xảy ra ở mỗi điện cực, phản ứng chung xảy ra khi pin phóng điện và sơ đồ

pin. Cho: E 0Pb 2+ /Pb = −0,126V; 2, 303

0 EPbO = 1, 455V ; / Pb2+

pK a(HSO− ) = 2, 00; 4

2

pK s(PbSO4 ) = 7, 66;

tại

25oC:

RT = 0, 0592V F

3.2. Tính: 0

0

(a) E PbSO /Pb ; E PbO /PbSO 4 2 4 (b) Suất điện động của pin khi CH SO ≈ 1,8 M. 2 4 Câu

Nội dung

3.1.

Catot:

PbO2 + 4 H

+

+ 2e

Pb HSO4-

Pb2+ + SO42-

2+

+ 2 H2O

Điểm 2(1,455) 10 0,0592

SO42− + H+

10-2

PbSO4

107,66

0,25

Quá trình khử tại catot: PbO2 + HSO4- + 3H+ + 2e

PbSO4 + 2 H2O K1

(*) 0,25


Anot:

−2( −0,126) 10 0,0592

Pb2+ + 2e

Pb HSO4-

SO42- + H+

Pb2+ + SO42Quá trình oxh tại anot: Pb + HSO4

10-2

PbSO4

107,66

PbSO4 + H+ + 2e

K2

(**)

0,25 0,25

Phản ứng chung khi pin phóng điện: PbO2 + Pb + 2 HSO4- + 2 H+

2 PbSO4 + 2 H2O +

(***)

-

(a) Pb│PbSO4, H , HSO4 │PbO2 (Pb) (c)

Sơ đồ pin: 3.2.

1,0

(a)

2E 0PbO2 /PbSO4

Theo (*): 10

0,0592

2(1,455) = K1 = 10 0,0592 .10−2 .107,66

0,25

→ E 0PbO2 /PbSO4 = 1,62 (V) Theo (**): −2E 0PbSO4 /Pb

10 (b)

0,0592

0,25

−2( −0,126) = K2 = 10 0,0592 .10-2 .107,66

→ E 0PbSO 4 /Pb = - 0,29 (V)

Theo (***): Epin = E(c) – E(a) = E 0PbO /PbSO - E 0PbSO /Pb + 2 4 4 -

0, 0592 2

- 2

0,5

+

Trong đó [HSO4 ], [H ] được tính từ cân bằng sau: -

[] 2-

1,8 – x

[ SO4 ] = x = 9,89×10-3 (M)

2-

H+ +

HSO4

Ka = 10−2

SO4

1,8 + x

+ 2

log[HSO 4 ] [H ]

x

→ [H+] = 1,81 (M); [ HSO-4 ] = 1,79 (M)

Epin = 1,62 + 0,29 +

0.0592 2

2

2

log(1, 79) (1,81) = 1,94 (V)

Câu 4: (2,0 điểm) N - P, C – Si và hợp chất

Hợp chất X1 màu vàng, gồm 2 nguyên tố, hòa tan được hoàn toàn trong axit nitric đặc khi đun nóng, giải phóng ra một chất khí có tỉ khối bằng 1,586 lần so với không khí. Khi thêm bari clorua dư vào dung dịch thu được ở trên, một chất rắn màu trắng X2 được tách ra. Lọc kết tủa. Phần nước lọc cho phản ứng với một lượng dư dung dịch bạc sunfat tạo thành một kết tủa gồm 2 chất rắn X2 và X3, chúng cũng được tách ra bằng cách lọc. Thêm từng giọt dung dịch natri hiđroxit vào phần nước lọc mới thu được

sau khi tách hai chất rắn trên đến khi dung dịch có môi trường gần như trung tính (pH khoảng bằng 7).


Tại thời điểm này một chất bột màu vàng X4 (trong đó Ag chiếm 77,31 % về khối lượng) được tách ra từ dung dịch. Khối lượng của X4 lớn hơn so với khối lượng của X2 trong phần kết tủa đầu tiên gần 2,4 lần. 4.1. Xác định công thức hóa học của các chất từ X1 đến X4. 4.2. Xác định công thức hóa học của chất khí thoát ra. Viết và cân bằng các phương trình ình hóa học ở dạng ion

hoặc dạng phân tử của tất cả các phản ứng xảy ra. 4.3. Trong một đơn vị cấu trúc của X1, cấu trúc tạo bởi các nguyên tử có tính đối xứng. Vẽ cấu trúc của X1. 4.4. Hãy dự đoán các sản phẩm của X1 khi tương tác với:

(a) lượng dư oxi; (b) lượng dư axit sunfuric đặc nóng; (c) KClO3 rắn khi nghiền. Viết và cân bằng các phương trình ình hóa học của các phản ứng xảy ra. Câu 4.1

Nội dung

Điểm

Kết tủa X2 là BaSO4.

0,75

Kết tủa X3 là AgCl. Kết tủa vàng X4 có thể là HgO hoặc Ag3PO4. Tỉ lệ khối lượng mol phân tử X4 : X2 là 0,931 nên HgO : BaSO4 không phù hợp và 1,798 cho Ag3PO4 : BaSO4 gấp 2,4 lần so với 4/3. Do đó, tỉ lệ mol là 4Ag3PO4: 3BaSO4 tương ứng với P : S = 4:3 ⇒ X1 là P4S3. X1 = P4S3 4.2.

X2 = BaSO4

X3 = AgCl

Khí thoát ra có M = 1,586 × 29 = 46 g/mol, là NO2.

X4 = Ag3PO4 0,5

Hòa tan X1: P4S3 + 38HNO3 = 4H3PO4 + 3H2SO4 + 38NO2+ 10H2O Tạo thành X2: H2SO4 + BaCl2 = BaSO4↓+ 2HCl Tạo thành X2 và X3: Ag2SO4 + 2HCl = 2AgCl↓ + H2SO4 BaCl2 + Ag2SO4 = BaSO4↓ + 2AgCl↓ Thêm NaOH và tạo thành X4: H2SO4 + 2NaOH = Na2SO4 + 2H2O 2H3PO4 + 6NaOH + 3Ag2SO4 = 2Ag3PO4↓ + 3Na2SO4 + 6H2O 4.3.

Photpho sunfua P4S3 có hình dạng lồng.

0,25


(Hs có thể vẽ bất kì công thức cộng hưởng nào phù hợp hóa trị đều được chấp nhận). 4.4.

a) P4S3 + 8O2 = 2P2O5 + 3SO2

0,5

b) P4S3 + 16H2SO4 = 4H3PO4 + 19SO2 + 10H2O (HS có thể viết sản phẩm khử S thay cho SO2 được chấp nhận) c) 3P4S3+ 16KClO3 = 16KCl + 6P2O5 + 9SO2 Câu 5: (2,0 điểm) Phức chất, trắc quang 5.1. Dựa vào thuyết VB hãy viết công thức cấu tạo của các phức chất sau: [Fe(CO)5]; [Fe(CO)6]Cl2, biết

rằng chúng đều nghịch từ. 5.2. Thuyết VB không thể giải thích được tại sao một bazơ Lewis yếu như CO lại có khả năng tạo phức chất

tốt và tạo nên những phức chất cacbonyl bền vững. Dựa vào cấu hình electron của phân tử CO theo thuyết MO, hãy giải thích sự tạo thành liên kết bền giữa kim loại và CO. 5.3. Cho phản ứng: [Fe(CO)5] + 2 NO → [Fe(CO)2(NO)2] + 3 CO

(a) Giải thích tại sao có thể thay thế 3 phối tử CO bằng 2 phối tử NO trong phản ứng trên. (b) Tìm một phức chất cacbonyl (chỉ chứa phối tử CO) đồng điện tử với [Fe(CO)2(NO)2]. Hãy dự đoán cấu trúc phân tử của [Fe(CO)2(NO)2]. Câu 5.1.

Nội dung

Điểm

- Cấu tạo [Fe(CO)5]: Fe0 cấu hình 3d8. Phối tử CO sẽ lai hóa trong, hai e độc thân của cấu hình d8 sẽ ghép đôi 0,5 tạo nên 1 obitan d trống. Dạng lai hóa dsp3, cấu trúc hình học lưỡng chóp tam giác, thỏa mãn tính nghịch từ.

- Cấu tạo của [Fe(CO)6]2+: Fe2+ cấu hình 3d6. Phối tử CO lai hóa trong, bốn e độc thân của cấu hình d6 sẽ ghép đôi tạo nên 2 obitan d trống. Dạng lai hóa d2sp3, cấu trúc hình học bát diện đều, thỏa mãn tính nghịch từ.

0,5


5.2.

0,5

4 2 0 - Cấu hình electron của CO: σs2 σ*2 s πxy σ z πxy

- Liên kết M-CO bền vì ngoài liên kết σ cho nhận kiểu CO→M (giữa đôi e trên MO σz liên kết của CO với obitan d trống của M) còn có liên kết π kiểu M→CO (giữa đối e trên obitan d của M với MO phản liên kết π* trống của CO). - Liên kết sau đóng vai trò quyết định độ bền bất thường của liên kết M-CO mà thuyết trường tinh thể, thuyết VB không giải thích được. - Như vậy CO dùng cả hai obitan σz và π* trống để hình thành liên kết phối trí. 5.3.

0,5

4 2 *1 Cấu hình electron của NO: σs2σ*2 s π xy σ z πxy

- Trên MO của phối tử NO có thêm một electron trên MO π*. Giống như CO, NO cũng dùng hai obitan σz và π* để hình thành liên kết phối trí. Như vậy, NO cho nguyên tử kim loại 3e (CO chỉ cho 2e). Do đó, 2 phối tử NO có thể thay thế được 3 phối tử CO. - Phức chất đồng điện tử với [Fe(CO)2(NO)2] là [Ni(CO)4] hoặc [Fe(CO)4]2+... Chúng đều là những phức chất tứ diện (giải thích bằng thuyết VB). Do vậy, cấu trúc của [Fe(CO)2(NO)2] cũng được dự đoán là tứ diện. Câu 6 (2,0 điểm): Quan hệ giữa hiệu ứng cấu trúc và tính chất 6.1. Hằng số tốc độ phản ứng xà phòng hóa bằng NaOH trong đioxin và nước các dẫn xuất thế m- và p-

metylbenzoat được liệt kê dưới đây. Dựa trên cấu trúc (gồm cả các trạng thái cộng hưởng có thể có) giải thích sự khác biệt vận tốc phản ứng của các phản ứng này.

k(M-1.phút-1)

k(M-1.phút-1)

Metyl p-nitrobenzoat

102

Metyl benzoat

1,70

Metyl m-nitrobenzoat

63

Metyl p-metylbenzoat

0,98

Metyl m-clobenzoat

9,1

Metyl p-metoxybenzoat

0,42

Metyl m-brombenzoat

8,6

Metyl p-aminobenzoat

0,06

6.2. Hãy giải thích vì sao µ của hợp chất sau có giá trị lớn (5,6 D):

6.3. Hãy giải thích vì sao (A) có hằng số axit Ka cao gấp khoảng 107 lần (B): O

O O

(A)

OEt

(B)

OEt

O O

O


Câu

Nội dung

6.1

Các nhóm thế hút electron hoạt nhóm cacbonyl, làm thuận lợi cho phản ứng thế SN2(CO).

Điểm

1,0

Nhóm thế m-Cl gây gây –I mạnh hơn m-Br (do độ âm điện Cl lớn hơn), nên vận tốc của dẫn xuất thế này lớn hơn. Nhóm thế O2N gây cả -I và –C nên vận tốc phản ứng nhanh hơn nhóm thế halogen. Tuy nhiên, nhóm thế p-O2N đặt điện tích dương lên C cạnh nhóm cacbonyl (cộng hưởng A) nên hoạt hóa nhóm này mạnh hơn nhóm thế m-O2N (xem cộng hưởng B).

Ngược lại, các nhóm đẩy electron phản hoạt hóa nhóm cacbonyl, làm giảm tốc độ phản ứng. Nhóm p-CH3 gây +H phản hoạt hóa ít hơn các nhóm p-CH3O và p-H2N (gây +C). Vì N lại dễ nhường cặp electron hơn O (độ âm điện nhỏ hơn), nên p-H2N phản hoạt hóa nhóm cacbonyl nhiều hơn.

6.2.

Hợp chất này có thể tồn tại ở dạng có hai vòng thơm: một vòng mang điện dương, vòng

0,5

còn lại mang điện âm:

6.3.

Đietyl malonat (B) tồn tại ở dạng enol có liên kết H nội phân tử. Nhóm OH enol của (B)

gây ra tính axit của hợp chất này. Tuy nhiên, liên kết H nội phân tử làm giảm mạnh tính axit của nhóm OH enol:

(A) không tạo được liên kết H nội phân tử. Nhóm metilen chịu ảnh hưởng của hai nhóm C=O este. Tính axit gây ra do H linh động của nhóm metilen này.

0,5


Câu 7 (2,0 điểm): Hidrocacbon 7.1. Khi chế hóa 2,3-đimetylbutan với brom thu được một hợp chất X (C6H10Br4) có cấu trúc đối xứng. X cũng có thể được tổng hợp trực tiếp khi brom hóa 3,3-đimetylbutan-2-ol trong môi trường axit. Biết rằng cả hai phản ứng tổng hợp X đều đi qua sự tạo thành một trung gian Y (C6H8Br2) có khả năng cộng Diels - Alder. Cho biết cấu trúc các chất và giải thích toàn quá trình bằng cơ chế phản ứng. 7.2. Một hợp chất A tác dụng với Br2 (hν) thu được hợp chất B duy nhất chứa 55,8% C; 6,98% H và 37,21% Br. Bằng các phương pháp vật lý người ta xác nhận hợp chất B là hỗn hợp 2 phân tử với thành phần tương đương nhưng phân tử khối hơn kém nhau 2 đơn vị. (a) Xác định công thức phân tử của A, B. (b) Biết A và B đều không làm mất màu dung dịch brom. Xác định công thức cấu tạo. (c) Viết công thức lập thể của A, B. (d) Dự đoán tính tan và so sánh nhiệt độ nóng chảy của chúng. Câu

Nội dung

7.1.

Điểm

1,0 Cơ chế như sau:

Giai đoạn chuyển Y thành X là sự cộng HBr trái Markovnikov. Ở đây do nguyên tử Br gây –I mạnh sẽ làm bất lợi cho sự hình thành cacbocation bậc cao. Mặt khác sản phẩm có tính đối xứng cao sẽ có tính bền nhiệt động cao hơn sản phẩm ít đối xứng hơn. 7.2. (a)

1,0 Sau khi tính toán thu được công thức đơn giản của B là C10H15Br. Vậy công thức thực

0,25

nghiệm của B là (C10H15Br)n. Do tỉ lệ đồng vị trong thiên nhiên nên B không thể chứa đồng vị của H, của C mà chỉ có thể chứa đồng vị Br79 và Br81 (có hàm lượng tự nhiên là 1

: 1). Nếu n = 2 thì trong B có 3 loại phân tử bao gồm 79 - 79, 79 - 81, 81 - 81 và không thể thỏa mãn đề bài. Do đó n = 1. Vậy A là C10H16. (b)

Công thức cấu tạo A, B lần lượt là

0,25


(c)

Công thức lập thể của A, B là

0,25

(d)

Do Br có tính chọn lọc cao nên sản phẩm bậc III có tỷ lệ rất lớn, hơn nữa gốc tự do trung

0,25

gian có cấu trúc tứ diện càng làm thuận lợi cho việc thế Br vào C bậc III. Coi như không có sản phẩm thế vào C bậc II trong trường hợp này vì tốc độ thế quá nhỏ. Tính tan: cả 2 chất đều không tan trong dung môi phân cực mà chỉ tan trong các dung môi không phân cực như CHCl3, CCl4… Nhiệt độ nóng chảy: yếu tố quan trọng nhất là đối xứng phân tử. Về mặt này A hoàn toàn lấn lướt so với B, do đó dĩ nhiên nhiệt độ nóng

chảy của A > B. Câu 8 (2,0 điểm): Xác định cấu trúc, đồng phân lập thể, danh pháp 8.1. Xử lý 1,3-dibrompropan với dietyl malonat trong sự có mặt của EtONa cho một dieste A. Thủy phân A trong môi trường base rồi đun nóng kế tiếp cho một axit B (C5H8O2). Chất này có thể bị biến đổi nhanh chóng thành chất C dưới tác dụng của thionyl clorua. Cho C tác dụng với diazometan rồi thêm vào đó dung dịch AgNO3/NH3 thu được chất D, chất này chuyển thành E dưới tác dụng của Br2/NaOH. Cuối cùng cho E tác dụng với axit nitrơ thu được một hỗn hợp gồm 4 sản phẩm bao gồm hai ancol đồng phân F và G cùng với hai hiđrocacbon H và I. Khử xiclopentanon bằng Wolff-Kishner nhận được H và điandehit glutaric là sản phẩm duy nhất thu được khi ozon phân I. Xác định cấu trúc A – I.

8.2. Một nhóm nhà khoa học Hàn Quốc đã tách và tổng hợp được hợp chất daumone, một loại pheromon gây hiện tượng ngủ đông cho loài giun Caenorhabditis elegans mỗi khi thức ăn trở nên khan hiếm. Dưới đây là mô hình của phân tử daumone.

(a) Cho biết cấu hình tuyệt đối của nguyên tử cacbon được đánh dấu sao. (b) Ngoài nguyên tử cacbon trên, trong daumone còn có thêm bốn nguyên tử cacbon bất đối nữa. Chỉ rõ các nguyên tử này và cho biết cấu hình tuyệt đối của chúng. (c) Vẽ công thức đồng phân đối quang của daumone.


(d) Đồng phân đối quang của daumone có thể gây hiện tượng ngủ đông cho loài giun Caenorhabditis elegans hay không? Tại sao?

Câu 8.1

Nội dung

Điểm

Cấu trúc các chất như sau:

1,0

8.2.

1,0 (a) Cấu hình tuyệt đối của nguyên tử cacbon được đánh dấu:

0,5

(b)

Cấu hình bốn nguyên tử cacbon bất đối khác:

0,5

Đồng phân đối quang (một cách thiết lập là nghịch đảo cấu hình của cả năm nguyên tử

(c)

cacbon bất đối): 0,5

(d)

Đồng phân đối quang của daumone không thể gây hiện tượng ngủ đông cho loài giun

Caenorhabditis elegans Enzim vì phản ứng của enzim có tính chọn lọc lập thể; enzim với

một cấu hình xác định sẽ tham gia một quá trình xác định. Câu 9 (2,0 điểm): Cơ chế phản ứng 9.1. Trình bày cơ chế của phản ứng sau:

0,5


(a)

(b)

O-tBu

(c) 9.2. Winkler (1999) và Boekmann (2002) đã tổng hợp thành công diterpen Saudin có khả năng ức chế sự hạ đường huyết trên chuột. Trong đó phản ứng cuối cùng tạo hệ vòng saudin của Winkler đã được đánh giá

rất cao theo sơ đồ như sau: O

O

O

O

O

H

H

1. LiOH/MeOH O O

O

2. H + , D

O

O

O

O

O

O

Sử dụng mũi tên cong, hãy thể hiện quá trình hình thành sản phẩm cuối. Biết rằng phản ứng đi qua một giai đoạn retro-andol dẫn đến sự tách loại một phân tử axeton. Câu

Nội dung

Điểm

9.1

1,5

(a)

0,5


(b)

0,5

(c)

0,5


9.2.

0,5

Câu 10 (2,0 điểm): Tổng hợp các hợp chất hữu cơ (Dạng sơ đồ phản ứng). 10.1. Xác định các chất từ A đến H để hoàn thành sơ đồ phản ứng tổng hợp chất (X) sau: CN

(CH 2CO2 Et)2

D

HCl

E (C 11H12O 3)

EtONa

1. EtOH/H+ 2. (CH2 OH)2 3, HCO2 Et/NaH

F (C 16H 19O5 Na) H+ G (C 12H 10O3 ) 1. LiAlH 4 2. H3 O+

Axeton A

Axetilen CuC2

1. H2 , ∆, P 2. H2 SO4 , ∆

B (C 8 H14 )

N2 CH2 CO2Et

O O

H C

DCC

Cu

O Ph

(X)

10.2. Từ 1,3-xiclohexađion, MeI, EtI và iPrMgBr cùng với các chất cần thiết khác hãy đề nghị sơ đồ tổng hợp chất sau đây: O

Et O O

Me iPr

Câu 10.1.

Nội dung Cấu trúc các chất:

Điểm

1,0


10.2

Sơ đồ tổng hợp đề nghị như sau: 1,0


HỘI CÁC TRƯỜNG CHUYÊN VÙNG DUYÊN HẢI VÀ ĐBBB TRƯỜNG THPT CHUYÊN NGUYỄN TẤT THÀNH TỈNH YÊN BÁI

KỲ THI CHỌN HỌC SINH GIỎI THPT CHUYÊN - DUYÊN HẢI BẮC BỘ NĂM 2017 MÔN: HÓA HỌC LỚP 11 Thời gian làm bài: 180 phút

ĐỀ THI ĐỀ XUẤT Câu 1. (2,0 điểm) Tốc độ phản ứng - Câng bằng hóa học Trong dung dịch nước có môi trường axit I- bị oxi hóa bởi BrO 3− theo phản ứng: 9I − + BrO3− + 6H +  → 3I3− + Br − + 3H 2 O

(1)

1. Thực nghiệm cho biết tốc độ của phản ứng có dạng:

d[BrO3− ] v=− = k[H + ]2 [BrO3− ][I− ] dt Với k là hằng số tốc độ của phản ứng, ở 298K. a) Hãy cho biết bậc của phản ứng (1). Bậc của phản ứng sẽ bằng bao nhiêu nếu phản ứng được thực hiện trong dung dịch đệm có pH = 3 ? b) Việc thực hiện phản ứng trong dung dịch đệm có pH = 3 có ảnh hưởng đến năng lượng hoạt hóa của phản ứng không? Tại sao ? 2. Cơ chế của phản ứng (1) được đề nghị như sau: k1  → H 2 BrO3+ BrO3− + 2H + ←

(1)

(nhanh, cân bằng)

k2 H 2 BrO3+ + I −  → IBrO 2 + H 2 O

(2)

(chậm)

k3 IBrO 2 + I −  → I 2 + BrO 2−

(3)

(nhanh)

k4 BrO −2 + 2I − + 2H +  → I 2 + BrO − + 3H 2 O

(4)

(nhanh)

k5 BrO − + 2I − + 2H +  → I 2 + Br − + H 2 O

(5)

(nhanh)

k6 → I 2 + I − ← I3−

(6)

(cân bằng)

k −1

k −6

a) Có thể áp dụng nguyên lí nồng độ dừng cho các tiểu phân trung gian: H 2 BrO 3+ , IBrO 2 được không? Tại sao ? b) Chứng minh rằng cơ chế này phù hợp với quy luật tốc độ xác định từ thực nghiệm từ đó tìm biểu thức của k. Câu Hướng dẫn trả lời Điểm 2 1 0,5 d[BrO3− ] = k[H + ]2 [BrO3− ][I− ] . a) Từ biểu thức: v = − dt Suy ra bậc của phản ứng: n = 2 + 1 + 1 = 4. Trong dung dịch đệm có pH = 3 ⇒ [H+] = 10-3 M Khi đó

v=−

d[BrO3− ] = k[H + ]2 [BrO3− ][I − ]=k[10−3 ]2 [BrO3− ][I − ]=10−6 k[BrO3− ][I− ]=k'[BrO3− ][I − ] dt


Suy ra phản ứng có bậc n’ = 1 + 1 = 2. −E b) Ta có: k 'T1 = 10 −6 k T1 = A.exp( a ) RT1 k 'T2 = 10 −6 k T2 = A.exp(

k 'T2 k

' T1

=

k T2 k T1

0,5

−E a ) RT2

 E 1 1  = exp  − a ( − )   R T2 T1 

⇒ Việc thực hiện ở pH = 3 không ảnh hưởng đến năng lượng hoạt hóa, Ea, của phản ứng.

2

a) H 2 BrO 3+ được tạo ra ở giai đoạn nhanh (1) và bị tiêu thụ ở giai đoạn chậm nên

0,5

không thể áp dụng nguyên lí nồng độ dừng cho tiểu phân này được. IBrO2 được tạo ra ở giai đoạn chậm (2) và bị tiêu thụ ở giai đoạn nhanh (3) nên có thể áp dụng nguyên lí nồng độ dừng đối với tiểu phân này. b) Vì (2) là giai đoạn chậm nên tốc độ của phản ứng: v = v 2 = k 2 [H 2 BrO3+ ][I − ]

(I) − 3

+ 2

0,5 + 3

Giai đoạn (1) là nhanh và cân bằng nên: k1[BrO ][H ] = k −1[H 2 BrO ] ⇒ [H 2 BrO3+ ] =

k1 [BrO3− ][H + ]2 k −1

Thay (II) vào (I) ⇒ v = k 2 Với k = k 2

(II)

k1 [H + ]2 [BrO 3− ][I − ]=k[H + ]2 [BrO 3− ][I − ] k −1

k1 k −1

Vậy cơ chế được đề nghị phù hợp với quy luật động học bậc 1.

Câu 2. (2,0 điểm) Dung dịch điện li Dung dịch A gồm HCOONa 0,1M và Na2SO3 có pHA = 10,4.

1. Tính nồng độ của SO 32− 2. Thêm 14,2 ml HCl 0,6M vào 20 ml dung dịch A tạo thành dung dịch B. Tính pHB. 3. Trộn 1 ml dung dịch A với 1 ml dung dịch Mg2+ 0,001M a. Hỏi có Mg(OH)2 tách ra không? Khi đó pH của hệ là bao nhiêu? b. Nếu có kết tủa Mg(OH)2 tách ra, hãy tính độ tan của Mg(OH)2 trong hỗn hợp thu được. Cho HCOOH có pK a = 3,75; H2SO3 có pK a1 = 1,76;pK a 2 = 7, 21 ; pTMg(OH)2 = 10,95

Câu 2

Hướng dẫn trả lời

Điểm


1

SO32− + H 2 O HSO3− + OH − K b1 = 10−6,79 (1) HCOO − + H 2O HCOOH + OH − K b = 10 −10,25 (2) HSO3− + H 2O H 2SO3 + OH − K b2 = 10−12,24 (3) H 2O H + + OH − K w = 10 −14

(4)

[HCOOH] [H + ] 10−10,4 pHA =10,4 nên [H ] <[OH ] . Khi đó = = −3,75 < 1 [HCOO − ] Ka 10 +

0,5

-

Kb2<<Kb1 nên cân bằng (1) là chính. Từ đó tính được CSO2 − = 0, 389 ( M ) 3

2

Thêm HCl thì: CH + = 0, 249(M);CSO2 − = 0, 228(M);C HCOO− = 0,0585(M) 3

Phản ứng: SO

2− 3

+

+ H → HSO

− 3

0,228 0,249 (M) 0,021 0,249(M)

HCOO − + H + → HCOOH 0,0585 0,0375

0,021(M) 0,021(M)

Vậy dung dịch B: HCOOH (0,021M); HCOO- 0,0375M); HSO 32− (0,249M)

HCOOH H + + HCOO − K a = 10−3,75 (1) HSO3− H + + SO32−

K a 2 =10−7,21 (2)

H 2O H + + OH −

K w = 10−14 (3)

HCOO − + H 2 O HCOOH + OH − K b = 10−10,25 (4) HSO3− + H 2O H 2SO3 + OH −

K b2 = 10 −12,24 (5)

Bỏ qua (2) và (3) so với (1), bỏ qua (5) nên tính pH dung dịch B theo (1) và (4)

pH B = 3,75 + lg

0,0375 =4 0,021

pH = 4 suy ra [OH-] =10-10 << 10-4 <<Ca, Cb

[ H 2SO3 ] = H +  =  HSO3− 

Ka1

10 −4 << 1 => [ H 2SO3 ] <<  HSO3−  −1,76 10

Vậy, bỏ qua (5) là hợp lý.

3

a. Trộn CSO2 − = 0,1945;C HCOO− = 0,05(M);C Mg2 + = 5.10 −4 (M) . 3

Điều kiện có kết tủa Mg(OH)2 ' 2 C'Mg2 + .(COH ≥ K s = 10 −10,5 − )

0,5


C’ là nồng độ của các ion trước khi tạo kết tủa

C'Mg2 + = C Mg 2+ = 5.10−4 (M) Tính C'OH− theo cân bằng :

SO32− + H 2O HSO3− + OH − K b = 10 −6,79 C C’

C

0,1945 0,1945-x ' OH −

x −4

= x = 1,78.10 ⇒ C

x ' Mg 2 +

' 2 .(COH = 5.10−4. (1,78.10−4 ) > 10−10,95 − )

bắt đầu có kết tủa Mg(OH)2 theo phản ứng:

Mg 2+ + 2SO32− + 2H 2O Mg(OH) 2 + 2HSO3− K = 10 −2,63 5.10-4 5.10-4-x

C C’

0,1945 0,1945-2x

2x

2x = CHSO−

0,75

3

2

(2x) = 10−2,63 Suy ra 2 (5.10 − x)(0,1945 − 2x) −4

x = 9, 47.10−5 ⇒ C HSO− = 1,89.10−4 (M) 3

Vậy thành phần giới hạn:

CSO2 − ≈ 0,1945(M);C HCOO− = 0,05(M);C HSO− = 1,89.10 −4 (M);C Mg2 + = 4,1.10−4 3

3

SO32− + H 2O HSO3− + OH − K b = 10−6,79 C C’

0,1945 0,1945-y

1,89.10-4 1,89.10-4 +y y

y(1,89.10 −4 + y) = 10 −6,79 ⇒ y = 1,068.10 −4 ⇒ pH = 10,03 0,1945 − y b.

0,25

Mg(OH) 2 Mg 2+ + 2OH − K s = 10−10,95

4,1.10-4+s 1,068.10-4 Cân bằng Ks=[Mg2+][OH-]2 =(4,1.10-4+s)(1,068.10-4)2 =10-10,95 s = 5,67.10-4

Câu 3. (2,0 điểm) Điện hóa học Cho giản đồ quá trình khử - thế khử: quá trình khử diễn ra theo chiều mũi tên, thế khử chuẩn được ghi trên các mũi tên và đo ở pH = 0. +0,293

Cr(VI)

(Cr2O27)

+0,55

Cr(V)

+1,34

0

Cr(IV)

Ex

3+

Cr

-0,408

2+

Cr

0

Ey

-0,744

1. Tính E 0x và E 0y . 2. Dựa vào tính toán, cho biết Cr(IV) có thể dị phân thành Cr3+ và Cr(VI) được không?

Cr


3. Viết quá trình xảy ra với hệ oxi hóa – khử Cr2 O 72- /Cr3+ và tính độ biến thiên thế của hệ ở nhiệt độ 298 K, khi pH tăng 1 đơn vị. 4. Phản ứng giữa K2Cr2O7 với H2O2 trong môi trường axit (loãng) được dùng để nhận biết crom vì sản phẩm tạo thành có màu xanh. Viết phương trình ion của phản ứng xảy ra và cho biết phản ứng này có thuộc loại phản ứng oxi hóa – khử hay không? Vì sao? Ghi số oxi hóa tương ứng trên mỗi nguyên tố. 0 Cho: E Cr O2- /Cr3+ = 1,33 V; Hằng số khí R = 8,3145 J.K–1.mol–1; Hằng số Farađay F = 96485 C.mol–1. 2 7

Câu 1 1

ĐÁP ÁN Hướng dẫn trả lời

Điểm 0,5

Từ giản đồ ta có: 3.(-0,744) = -0,408 + 2 E → E = -0,912 (V) 0 y

0 y

0,55 + 1,34 + E 0x – 3.0,744 = 6.0,293 → E 0x = +2,1

2

(V) Cr(IV) có thể dị phân thành Cr và Cr(VI) khi ∆G0 của quá trình < 0. 3+

2Cr(IV) + 2 e → 2Cr

3+

(1) E = E = 2,1 V → ∆G 0 1

0 x

0 1

0,5 0 1

= -n E F = -

2.2,1.F Cr(VI) + 2 e → Cr(IV) (2) E 02 =

0, 55 + 1,34 = 0,945 (V) → ∆G 02 = -n 2

E 02 F = - 2.0,945.F Từ (1) và (2) ta có: 3Cr(IV) → 2Cr3+ + Cr(VI)

∆G 30

∆G 30 = ∆G10 - ∆G 02 = - 2.(2,1 - 0,945).F < 0 → Vậy Cr(IV) có dị phân.

3

0,5

Cr2 O 72- + 14H+ + 6e ⇌ 2Cr3+ + 7H2O

E1 = 1,33 +

RT [Cr2 O72- ].(10-pH )14 ln 6.F [Cr 3+ ]2

E 2 = 1,33 +

RT [Cr2O 72- ].(10-(pH + 1) )14 ln 6.F [Cr 3+ ]2

Độ biến thiên của thế: E 2 - E1 =

+6 -2

Cr2 O

+1 -1 27

+1

8,3145 . 298 .14ln10-1 = -0,138 (V). 6 . 96485

+6,-2/-1

0,5

+1 -2

+

+ 4H2O2 + 2H → 2CrO5 + 5H2O

Đây không phải phản ứng oxi hóa khử vì số oxi hóa của các nguyên tử không thay đổi. Trong CrO5 số oxi hóa của crom vẫn là +6 do có cấu tạo như sau: -2

O O

-1

O Cr

O

+6

O

Câu 4. (2,0 điểm) Nhóm Nitơ và nhóm cacbon Nguyên tố X (có nhiều dạng thù hình) có một anion chứa oxy đóng vai trò quan trọng trong ô nhiễm nước. Độ âm điện của nó nhỏ hơn oxy. Nó chỉ tạo hợp chất phân tử với halogen. Ngoài hai oxit đơn phân


tử còn có những oxit cao phân tử. X có vai trò rất quan trọng trong sinh hóa. Các obitan p của nó chỉ có một electron. 1. Đó là nguyên tố nào? Viết cấu hình electron của nó. 2. X có thể tạo được với Hidro nhiều hợp chất cộng hóa trị có công thức chung là XaHb; dãy hợp chất này tương tự như dãy đồng đẳng của ankan. Viết công thức cấu tạo 4 chất đầu của dãy. 3. Nguyên tố X tạo được những axit có chứa oxy (oxiaxit) có công thức chung là H3XOn với n = 2, 3 và 4. Viết công thức cấu tạo của 3 axit này. Đánh dấu (dấu sao hoặc mũi tên) các nguyên tử H và ghi số oxy hóa của X trong các hợp chất này. 4. Một hợp chất dị vòng của X, với cấu trúc phẳng do J. Liebig và F.Wohler tổng hợp từ năm 1834, được tạo thành từ NH4Cl với một chất pentacloro của X; sản phẩm phụ của phản ứng này là một khí dễ tan trong nước và phản ứng như một axit mạnh a) Viết phương trình phản ứng. b) Viết công thức cấu tạo của hợp chất (NXCl2)3. 5. Hợp chất vô cơ vừa nêu ở trên có tính chất khác thường khi bị đun nóng: nó sôi ở 256oC khi bị đun nóng nhanh. Nếu đun nóng chậm nó bắt đầu nóng chảy ở 250oC; làm nguội nhanh chất lỏng này thì ta được một chất tương tự cao su. Giải thích tính chất đặc biệt này.

ĐÁP ÁN Hướng dẫn trả lời

Câu 1

Điểm

1) Photpho. Cấu hình [Ne]3s23p3

0,5

2) Công thức cấu tạo của 4 chất đầu tiên:

0,5 H

H P

H

H

P

H

H

P

H

1

H H

P

H

H P

P

H

H

H

H

3

P H

0,25 OH

+3

H

O

P

H

4

OH

+1

P

P

2

OH

P

P

3) Công thức cấu tạo của các chất:

O

H

H

+5

H

OH

4) a) 3NH4Cl + 3PCl5 = (NPCl2)3 + 12HCl b) Công thức cấu tạo:

O

P

OH

OH

0,5


Cl

N

Cl

P

P

N

N

Cl

Cl P Cl

Cl

5) Đun nóng nhanh → chất nóng chảy không bị gãy vòng Đun nóng chậm → vòng bị bẻ gãy tạo thành các phân tử polyme có hệ liên hợp pi: N

N

N

P Cl

0,25

P Cl Cl

P Cl

Câu 5. (2,0 điểm) Phức chất - trắc quang 1.Bài 5. Hãy gọi tên và vẽ các đồng phân của các phức sau: a. [Cr(NH3)4Cl2]Cl b. [Co(SCN)(H2O)5]Cl d. [PtCl(PMe3)3]Br, Me là CH3. e. [Co(en)2Cl2]Cl. 2. Màu sắc của dung dịch Cr(VI) phụ thuộc vào pH, theo cân bằng:

2CrO42− + 2 H + ⇌ 2Cr2O72 − + H 2O 2−

2− Độ hấp phụ quang (A) được đo ở bước sóng λ = 345 nm cho 2 ion CrO4 và Cr2O7 ở các giá trị pH

khác nhau. Trong một cuvet chiều dài l = 1 cm, nồng độ ban đầu của ion cromat và pH tương ứng cho các giá trị độ hấp phụ quang theo bảng sau:

pH

C (mol/l)

A

1

2.10-4

0,214

12

2.10-4

0,736

5,6

4.10-4

0,827

Hãy xác định hằng số cân bằng của phản ứng trên

Câu 5 1

Hướng dẫn trả lời a. [Cr(NH3)4Cl2]Cl: 2 đồng phân (cis, trans). Tetraammindiclorocrom(III)clorua b. [Co(SCN)(H2O)5]Cl: 2 đồng phân (liên kết). Pentaaquathioxianatocoban(II)clorua Pentaaquaisothioxianatocoban(II)clorua c. [PtCl(PMe3)3]Br: cấu hình vuông phẳng, 2 đồng phân (đồng phân ion hóa) [PtCl(PMe3)3]Br: Clorotris (trimetylphotphin)platin (II) bromua. [PtBr(PMe3)3]Cl: Bromotris (trimetylphotphin)platin (II) clorua. d. [Co(en)2Cl2]Cl: 3 đồng phân (±cis, trans).

Điểm 0,25

0,25

0,25


Dicloro bis(etylendiamin)coban(III) clorua

0,25 2− 7

Tại pH = 1: Cr2O là dạng chủ yếu 2−

2−

2[ Cr2O7 ] = 2 CCrO 2− =2.10-4 suy ra [ Cr2O7 ] = 10-4 (M) 4

0,25

A = ε Cr O2− .l.[Cr2O72− ] ⇔ 0,214 = ε Cr O 2− .1.10−4 ⇔ ε Cr O2− = 2,14.103 ( M −1.cm −1 ) 2 7

2 7

2 7

2−

Tại pH = 12: CrO4 là chủ yếu

0,25

A = ε CrO 2− .l.[CrO ] ⇔ 0, 736 = 2.10 .1.ε CrO2− ε Cr O2− = 3,68.10 ( M .cm ) 2− 4

4

−4

−1

3

4

−1

4

2− 4 và

Tại pH= 5,6: Cr(VI) tồn tại cả 2 dạng CrO

Cr2O72−

[CrO 24− ] + 2[Cr2O72− ] = 4.10−4 (1) A = ε CrO2− .l.[CrO42− ] + ε Cr O2− .l.[Cr2O72− ] = 0,827(2) 4

Từ

(1)

2 7

(2)

suy

[CrO24− ] = 1,5287.10−4 (M)

ra

[Cr2O72− ] = 1, 2356.10 −4 (M)

0,5

Hằng số cân bằng là:

K=

[Cr2O72− ] 1, 2356.10−4 = = 8,3798.1014 + 2 −4 2 −5,6 2 2− 2 [CrO 4 ] [H ] (1,5287.10 ) (10 )

Câu 6 (2,0 điểm): Quan hệ giữa hiệu ứng cấu trúc và tính chất 1. Sắp xếp (có giải thích ngắn gọn) theo thứ tự tăng dần tính axit của các hợp chất sau :

;

;

;

;

.

(1) (2) (3) (5) (6) 2. Sắp xếp (có giải thích ngắn gọn) theo thứ tự tăng dần nhiệt độ sôi của các hợp chất sau :

(1) Câu 6 1

(2)

(3)

(4) Hướng dẫn trả lời Tính axit (1)< (3) < (4) < (5) < (2). Giải thích : Tính axit phụ thuộc khả năng cho H+ ở liên kết N-H - Tính axit của (1) yếu nhất vì vòng no đẩy e làm tăng độ bền liên kết N-H. - (3) có hiệu ứng –C của nhóm C=O.

Điểm 0,5


- (4) có cặp e trên nguyên tử N tham gia vào hệ thơm, hệ này gây hiệu ứng –C mạnh hơn 1 nhóm C=O nhưng yếu hơn hai nhóm C=O. - (5) ngoài hiệu ứng –C của hệ thơm, còn hiệu ứng – I của nguyên tử N trong vòng thơm nên hút e mạnh hơn. Do đó tính axit của (3) < (4) < (5). - (2) có hiệu ứng –C của nhóm C=O làm giảm mạnh độ bền của liên kết N-H nên 0,5 có tính axit mạnh nhất. Nhiệt độ sôi của (3) < (2) < (1) < (4) 2 0,5 Nguyên tử khối của O < S < Se nên nhiệt độ sôi của (3) < (2) < (1). (4) có liên kết hidro liên phân tử nên có nhiệt độ sôi cao nhất. 0,5 Câu 7 (2,0 điểm): Hidrocacbon Một hợp chất A khi tác dụng với Br2(hv) thu được chất B duy nhất chứa 55,8%C; 6,98%H và 37,21%Br. Bằng phương pháp vật lí người ta xác nhận hợp chất B là hỗn hợp 2 phân tử với thành phần tương đương nhưng phân tử khối hơn kém nhau 2 đvC. a/ Xác định công thức phân tử của A, B. b/ Xác định công thức cấu tạo A, B. Biết rằng A, B đều không làm mất màu dung dịch Br2. c/ Vẽ công thức lập thể của A, B. d/ So sánh tính tan trong nước, nhiệt độ nóng chảy của A, B. Câu 7 Hướng dẫn trả lời Điểm Gọi công thức phân tử của B là CxHyBrx . 0,5 Tỉ lệ x : y : z = : 6,98 : = 10 : 15 : 1 CT của B là (C10H15Br)n, mà B là hỗn hợp 2 phân tử với thành phần tương đương nhưng phân tử khối hơn kém nhau 2 đơn vị, do đó trong mỗi phân tử B chứa 1 nguyên tử Brom và chúng là đồng vị Br79, Br81. Vậy CTPT của B là C10H15Br vầ CTPT của A là C10H16. A, B không làm mất màu nước brom, A tác dụng với Br2/

0,5

sinh ra B duy nhất.

0,5

Vậy cấu trúc của A là

0,5

cấu trúc của B là Tính tan của B > A, Nhiệt độ nóng chảy của B > A do B phân cực hơn A. Câu 8 (2,0 điểm): Xác định cấu trúc, đồng phân lập thể, danh pháp 1 .Axit retigeranic là một tecpenoit được phân lập từ một loại địa y. Cấu trúc của axit retigeranic được xác định dựa trên phương pháp phân tích tia – X. Công thức của axit retigeranic như sau:


Xác định cấu hình của axit retigeranic. Axit retigeranic có thể có bao nhiêu đồng phân cấu hình? 2. Một nhà hóa học điều chế axit tactric trong phòng thí nghiệm theo bốn thí nghiệm khác nhau được bốn sản phẩm là A, B, C, D. Chúng có tính chất vật lý như sau: tO nc pK1 pK2 [α]D25 A 170 –12 2,93 4,23 B 170 +12 2,93 4,23 C 140 0 3,11 4,8 D 206 0 2,96 4,24 a) Axit tactric có bao nhiêu đồng phân quang học. b) Viết công thức cấu trúc của A, B, C, D. Biết quan hệ cấu trúc và tính quang hoạt của axit tactric giống glyxerandehit. c) Điều chế C, D từ axit không no tương ứng. d) Đưa thêm một nhóm CHOH vào axit tactric. Gọi tên và xác định các đồng phân của axit tạo thành. Câu 8 Hướng dẫn trả lời Điểm Xác định cấu hình của axit retigeranic: 1 0,5 COOH R

H

E S

H

R

R

R

S

S

R

H

2

Axit retigeranic có thể có 28 đồng phân cấu hình. Chú ý đồng phân E không thể xuất hiện đồng phân Z. a) Các đồng phân: COOH

COOH

H

OH

HO

H

OH

H

COOH

H

H

OH HO COOH

0,5

COOH OH H COOH

b) A và B có các tính chất hóa lý giống nhau, chỉ ngược nhau về độ quay cực riêng → A và B là cặp đối quang. Vì quan hệ cấu trúc và tính quang hoạt của axit tactric giống glyxerandehit nên A là đồng phân cấu hình L, B là đồng phân cấu hình D. - C, D cùng có độ quay cực riêng bằng 0 → chúng là đồng phân meso hoặc hỗn hợp raxemic. - So sánh nhiệt độ nóng chảy có thể thấy D > A, B > C → C là hỗn hợp raxemic (vì hỗn hợp raxemic có nhiệt độ nóng chảy nhỏ hơn các chất tương ứng. c) Cần dựa thêm vào hóa lập thể cơ chế phản ứng để điều chế C, D. - C là hỗn hợp raxemic nên có thể điều chế bằng phản ứng thủy phân peoxit

0,5

0,25


- D là đồng phân meso nên có thể điều chế bằng phản ứng cộng oxi hóa (KMnO4 hoặc OsO4) d) Thêm một nhóm CHOH, sẽ được axit 2,3,4-trihidroxipentanđioic. Điều khác biệt là sẽ tạo ra một C* tương đối (khi thêm vào đồng phân meso). COOH

COOH

COOH

H

OH

H

H

OH

HO

H

OH

H

COOH

COOH

OH

H

OH

HO

H

H

H

OH

HO

H

HO

OH

H

H

COOH

COOH

0,25

OH COOH

Câu 9 (2,0 điểm): Cơ chế phản ứng 1. Phản ứng sau là một thí dụ của quá trình axyl hóa enamin: PhCO

2

N

COCl

Cl-

CHCl3

+N

N

Hãy viết cơ chế của phản ứng trên và so sánh với cơ chế phản ứng axyl hóa amoniac (sự giống nhau và khác nhau giữa hai cơ chế phản ứng). 2. Hãy trình bày cơ chế phản ứng của các chuyển hóa sau: b) a) 2

1. NaNH2

O

2. H3O

Câu 9 1

COOCH3

O

+

O

CH2COOCH3

1.CH3ONa

COOCH3

2. H3O

+

O

Hướng dẫn trả lời

Điểm

Cơ chế phản ứng axyl hóa enamin là cộng enamin vào nguyên tử C của nhóm cacbonyl và tách ion clo:

1,0

O

N Cl

N+

Cl C

Ph

O-

Cl-

N+

O Ph

Ph

Tiếp theo là chuyển proton đến phân tử enamin khác: N

N+

N

O

N+ COPh

Ph H

Cơ chế axyl hóa amoniac cũng tuân theo qui luật cộng – tách: O Cl

Ph NH3

O

OCl

C

Ph NH3+

H3N+ Cl-

Ph

Cl-


Chuyển proton đến phân tử NH3 thứ hai: O Cl-

O Ph

H2N+ H

2

H2N

NH3

Ph

+ NH4Cl

Cả hai phản ứng đều thực hiện theo cơ chế cộng tách. Sự khác nhau chủ yếu là liên kết CC được hình thành khi axyl hóa enamin, còn liên kết C-N được hình thành khi axyl hóa amoniac. Cơ chế phản ứng của các chuyển hóa: a) H

OH

O

O

O

H

CH3ONa

O

O

+

H COOCH3

C-OCH3

OCH3 O-

O

O-

O

-OCH 3

H

CHCOOCH3

O

O

H3O

O

b)

O NaNH2

NaNH2

1,0

COOCH3

COOCH3 O

COOCH3 H+

O-

O

Câu 10 (2,0 điểm): Tổng hợp các hợp chất hữu cơ Khảo sát một số giai đoạn trong tổng hợp prostaglandin của Woodward ơ sơ đồ dưới:

Xác định các chất chưa biết trong sơ đồ trên. Câu Hướng dẫn trả lời 10

Điểm


0,5

0,75

0,75

Trường THPT chuyên

KỲ THI CHỌN HỌC SINH GIỎI

Nguyễn Trãi

KHU VỰC DUYÊN HẢI VÀ ĐỒNG BẰNG BẮC BỘ

TỈNH HẢI DƯƠNG

Môn: Hóa học lớp 11 – Năm 2017

ĐỀ GIỚI THIỆU

Thời gian: 180 phút

Đề thi này gồm 6 trang, 10 câu Câu I: Tốc độ phản ứng – Cân bằng hóa học(2 điểm) Trong tầng bình lưu, nguyên tử Cl* có thể phân hủy tầng ozon theo phương trình sau: Cl* + O3

ka  → ClO* + O2 (a) với ka = 1,7.1010(M-1.s-1).exp(-260/T).


1.Hãy tính tốc độ phản ứng ở khoảng cách 20 km, khi đó nồng độ các chất : -17

[Cl]=

-9

5.10 M; [O3] = 8.10 M và T = 220K.

2.Ở độ cao 45 km, nồng độ thực tế các chất là [Cl]= 3.10-15M; [O3] = 8.10-11 M và

T = 270K, hãy tính

tốc độ phản ứng tại đó.

3.Tại 298K, hằng số tốc độ phản ứng (a) đo được là ka = 6,7.109M-1.s-1; thừa số Arrhenius, A = 6,7.1010M1 -1

.s . Hãy tính năng lượng hoạt hóa của phản ứng ở 298K.

4.Phản ứng lưỡng nguyên tử pha khí với các sản phẩm khác nhau của gốc clomonoxit: ClO* + ClO*

 → Cl2 + O2

k1 = 2,9.106

ClO* + ClO*

 → ClOO* + Cl*

k 2

ClO* + ClO*

 → OClO* + Cl*

k3 = 2,1.106.

= 4,8.106.

Hãy xác định hiệu suất cho sản phẩm không chứa gốc tự do.

5.Biết phản ứng xảy ra ở phần (4), chất đầu là (ClO)2 phân hủy nhanh thành ClO*rồi sau đó phân hủy theo 3 hướng sản phẩm trên. Biết thừa số Arrhenius và năng lượng hoạt hóa của các phản ứng tại 298K là: Phản ứng A (M-1.s-1) Ea (KJ/mol) 1 6,08.108 13,2 10 2 1,79.10 20,4. 3 2,11.108 11,4. Cho biết phản ứng nào có cho ∆H* lớn nhất và phản ứng nào cho ∆S* nhỏ nhất, tính các giá trị đó. Cho biểu thức của Eyring: k =

kB .T ∆G * exp() và ∆U* = Ea – RT; h RT

kB = 1,38.1023(J/K), h = 6,62.10-34(J.s).

Câu II: Cân bằng trong dung dịch điện li(2 điểm) Trộn 50 ml dung dịch NaOH C01(M) với 50 ml dung dịch H2S C02(M), thu được dung dịch A. Chuẩn độ 10 ml A với dung dịch HCl 0,025M, dùng chỉ thị Thymol (pKa = 9,4) hết 8,0 ml dung dịch HCl trên. Nếu dùng chỉ thị metyldacam (pKa= 4,0) thì hết 22,2 ml dung dịch HCl trên.

a.Cho biết phản ứng nào xảy ra khi chuẩn độ với chỉ thị metyldacam? b.Tính C01; C02 của 2 dung dịch trước khi trộn tạo ra A. c.Tính pH của dung dịch A? Cho: pKa(H2S) = 7,02; 12,9.

Câu III: Điện hóa học(2 điểm) Chuẩn độ Ce4+ người ta dùng As2O3, Hòa tan 0,198 gam As2O3 trong 100 ml dung dịch xút đặc, axit hóa bằng HClO4, chuẩn độ 10 ml dung dịch thu được ở pH = 1,0, chỉ thị thích hợp, hết 8,0 ml dung dịch Ce4+.

1.Viết các phương trình ion các phản ứng xảy ra. 2.Tính nồng độ M của Ce4+? 3.Tính thế của dung dịch tại thời điểm Vdd (Ce4+) = 6,0 ml.


4.Tính thế của dung dịch tại điểm tương đương? Cho: As = 75, O = 16, Thế khử chuẩn của H3AsO4/H3AsO3 = 0,56V; Ce4+/Ce3+ = 1,7V; pKa của H3AsO3= 9,23; của H3AsO4 = 2,13; 6,94; 11,5.

Câu IV: Nhóm N-P; C – Si(2 điểm) 1.Đơn chất X tác dụng mãnh liệt với dung dịch kiềm, nhưng chỉ tác dụng với nước ở nhiệt độ cao( khoảng 800oC). X bền trong các axit, ngay cả nước cường thủy, chỉ tan trong hỗn hợp hai axit HF và HNO3. Bột mịn X tác dụng được với hơi HF hay tác dụng được với Mg ở khoảng 800oC. X lại có thể được điều chế bằng cách dùng Mg tác dụng với oxit của nó khi đốt cháy hỗn hợp. Hãy cho biết X là đơn chất nào ? viết phương trình phản ứng xảy ra trong các trường hợp nêu trên ?

2.Hòa tan hoàn toàn 2,64 gam hỗn hợp FeS2 và FeO trong 62 gam dung dịch HNO3 vừa đủ, thu được 3,808 lít khí NO2(đktc) và dung dịch X ( biết trong X, sản phẩm oxi hóa là Fe3+ và SO42-). Dung dịch X phản ứng vừa đủ với 240 ml dung dịch NaOH 1M.

a)Xác định thành phần hỗn hợp 2 chất ban đầu ? b)Tính C% của dung dịch HNO3 đã dùng ? Câu V : Phức chất(2 điểm) Cho sơ đồ pin Ag|Ag2CrO4↓, CrO42- 1M | | AgNO3 0,1M| Ag. Thể tích dung dịch ở mỗi cực là 1 lít.

1.Tính sức điện động của pin. Viết phương trình phản ứng xảy ra trên từng điện cực và phản ứng xảy ra khi pin hoạt động.

2.Thêm 1 mol NH3 vào cực bên phải của pin. Tính Epin khi đó. Biết: Ag+ + 2NH3 Ag(NH ) + 3 2

K1 = 107,24.

3.Thêm 1 mol KCN vào dung dịch bên trái của pin, cho biết phản ứng xảy ra khi pin hoạt động, tính Epin khi đó. Biết: Ag++ 2CN- Ag(CN)2-

K2 = 1021.

0 Cho: Ks( Ag2CrO4) = 10-12, E Ag + / Ag = 0,8V .

Câu VI (2,0 điểm): Quan hệ giữa hiệu ứng cấu trúc và tính chất 1. Chất sau là axit, bazơ hay trung tính? Giải thích? NEt2 H3CO

NEt2 OMe

R CO CH2 R' A

B

NH=C(NH2)2 C

2. So sánh: a) So sánh nhiệt độ sôi của imidazol, axazol và thiazol. Giải thích.

D


N

N

Imidazol

N

O

NH

Oxazol

S

Thiazol

b) So sánh nhiệt độ nóng chảy của:

Câu VII (2,0 điểm): Hidrocacbon 1. Năm 1966, Masamune đã thực hiện một chuỗi phản ứng xuất phát từ phản ứng cộng giữa anhidrit maleic và xiclooctatetraen. Cấu trúc sản phẩm này đã được xác định chắc chắn nhưng Masamune vẫn chưa biết đặt tên nó là gì. Trong 1 lần đi uống cafe, ông nhìn thấy một giỏ trái cây trên bàn rất giống hình dạng của chất mình tổng hợp được, ông đặt tên hợp chất đó là Basketen ( xuất phát từ basket – cái giỏ). Sau đây là sơ đồ tổng hợp: O

O

O

t0

hv A

B

1. Na2CO3

C

Pb(OAc)4

2. HCl

Hãy xác định cấu trúc các chất A,B,C cho sơ đồ trên. 2. Đime hóa isopren với hỗn hợp xúc tác Fe(acac)3, Et3Al, và 2,2'-Dipyridin cho hỗn hợp 2 đồng phân 1,5dimethyl và 2,5-dimetylxiclooctadien. Sau đó cho 2 đồng phân xiclooctadien này phản ứng với lượng lớn dibromocarbene với xúc tác ancol terbutylic được sản phẩm A. Tiếp theo, thực nghiệm tiếp tục cho A tác dụng với methyl lithium trong ete thu được một bis- allen là vòng có 10 cạnh B. Đốt nóng bis-allene B tới 225oC, kết quả được dạng của 2 đồng phân tetraene. Ở 3500C, hai đồng phân có thể chuyển đổi lẫn nhau. Tiếp tục theo dõi phản ứng trong thiết bị dòng điện, các đồng phân tetraen này chuyển sang các hợp chất có vòng thơm. Xác định các cấu trúc trong các quá trình chuyển đổi trên. Câu VIII (2,0 điểm): Xác định cấu trúc, đồng phân lập thể, danh pháp Nepetalacton A là một chất tự nhiên có công thức phân tử là C10H14O2. Cấu trúc của hợp chất này được xác định qua chuỗi phản ứng sau:


Chất A được hidro hóa xúc tác cho ra axit B (C10H18O2). Qua khảo sát người ta nhận thấy phản ứng hidro hoá trên sử dụng đến 2 mol hidro cho một mol A, trong đó 1 mol dùng để khử nối đôi C=C còn 1 mol dùng để mở vòng lacton và khử ancol.

Chất B bị khử bởi LiAlH4 cho ra ancol C (C10H20O). Chất C tác dụng với anhydrit axetic cho ra một este D. Nhiệt phân chất D sẽ loại đi một phân tử axit axetic và tạo thành một anken E (C10H18). Tiến hành ozon phân hợp chất E thì thu được fomandehit và 2-metyl-5-isoproylxiclopentanon. 1.Hãy xác định cấu trúc các chất trung gian E, D, C, và B. Từ đó suy cấu trúc có thể có của chất A biết rằng vòng lacton ở chất A là vòng 6 và chứng tỏ cấu trúc của nó thoả quy tắc isopren. 2. Chất A hoà tan từ từ trong dung dịch kiềm loãng và sau khi axit hoá dung dịch này người ta thu được axit F. Trong dung dịch nước F tồn tại ở 3 dạng hổ biến F1, F2, F3. Hãy đề nghị các dạng hỗ biến F1, F2, F3 biết rằng F3 sẽ phải qua các quá trình hidrat hoá để tạo thành F2 và có một dạng dễ tạo oxim và thiosemicacbazon cũng như tham gia phản ứng tráng gương. Suy ra cấu trúc thực của nepetalacton. Câu 9 (2,0 điểm): Cơ chế phản ứng 1. Tropilidenes là một cycloheptatrienes có nhóm thế, khi bị nung nóng sẽ chuyển sẽ chuyển qua dạng bicycle[4,1,0]heptadiene (norcaradiene) rồi lại trở thành cycloheptatrienes nhưng nhóm thế đã ở vị trí khác so với ban đầu.

R1 R2

1

7

2

Trình bày cơ chế bằng mũi tên cong và chỉ rõ vị trí đã thay đổi bằng cách

6 3

đánh dấu số thứ tự các nguyên tử cacbon.

4

5

2. Trình bày cơ chế chuyển vị sau:

3. Từ các hợp chất đien có vòng 5 hoặc 6 cạnh, viết cơ chế chuyển dịch electron để mở rộng vòng tạo ra:

O

a)

b)

4. Dự đoán sản phẩm và dùng mũi tên cong để trình bày cơ chế của phản ứng: OH

CHCl3

CHCl3

a) NH

OH-

b)

OH-


Câu 10: (2 điểm) Tổng hợp các hợp chất hữu cơ 1. Cho sơ đồ tổng hợp sau:

Xác định các chất B,C,D phù hợp. 2. Phương pháp tổng hợp ngược (tổng hợp lùi) là một kĩ thuật giải quyết tổng hợp hữu cơ dựa vào sự biến đổi cấu trúc phân tử mục tiêu thông qua một chuỗi có hệ thống các cấu trúc đơn giản hơn.

Tổng hợp phân tử mục tiêu M được tiến hành cắt liên kết cho ra hai phân tử X và Y.

b

Sau đó thực hiện sơ đồ tổng hợp: X

1) (CH3)2O, H+ 2) CH3COOOH

A

Me2CuLi

B

HCl

1) TsCl, pyr C

D 2) NaI

+Y E C2H5ONa

Hoàn thiện sơ đồ theo hướng tổng hợp đề xuất? ---------------------------------Hết------------------------------

SnCl4

M


TỈNH HẢI DƯƠNG

KỲ THI CHỌN HỌC SINH GIỎI

ĐÁP ÁN

KHU VỰC DUYÊN HẢI VÀ ĐỒNG BẰNG BẮC BỘ

ĐỀ GIỚI THIỆU

Môn: Hóa học lớp 11 Thời gian: 180 phút

Đáp án này gồm 10 trang CÂU

ĐÁP ÁN

ĐI Ể M

I.1

va = ka.[Cl*][O3] = 2,08.10-15 (M.s-1)

0,25

I.2

Vb = kb.[Cl*][O3] = 1,56.10-15 (M.s-1)

0,25

I.3

K = A.exp(-Ea/RT) => Ea = 5,70585 kJ/mol.

0,25

I.4

d [Cl2 ] d [O2 ] = = k1[ClO*]2 ; dt dt

d [OClO*] = k3[ClO*]2 dt

;

d [ClOO*] = k2 [ClO*]2 ; dt

0, 5

d [Cl*] = ( k2 + k 3 )[ClO*]2 dt

[Cl2 ] [O2 ] k1 = = = 29, 6% ∑ sp ∑ sp 2(k1 + k2 + k3 ) I.5

Tính k = A. exp(

− Ea ) ta được : k1 = 106,47 ; k2 = 106,677 ; k3 = 106,32. RT

0,75

Theo phương trình Eyring :

k=

k B .T −∆G * exp( ) và từ ∆U* = Ea – RT ; ∆H* = ∆U* - ∆nRT với h RT

∆n = 1 ta suy ra :

∆H* = Ea -2RT.

Mà ∆G* = ∆H*-T∆S* = Ea-2RT- T∆S*

k= A=

−E −E k B .T ∆S * k B .T 2 ∆S * .exp( a ).e2 .exp = .e .exp .[exp( a )] h RT R h R RT

k B .T 2 ∆S * .e .exp h R

∆H* lớn nhất khi Ea lớn nhất đó là ở phản ứng (2): ∆H*= Ea – 2RT = 15,44 KJ. mol-1. ∆S* lớn nhất khi A lớn nhất; mà A ở (3) lớn nhất, khi đó:

A=

II

k B .T 2 ∆S * .e .exp thay số vào ta tính được ∆S* = -102,2(J/mol) h R

Khi trộn 2 dung dịch có thể có các phản ứng : H2S + OH- ⇌ HS-

HS- + OH- ⇌ S2- .

1,0


Dung dịch A có thể có OH-, HS-, H2S, S2-. Vì

K a1 10−7,02 = ≫ 104 ⇒ có thể chuẩn độ riêng nấc 1 được. K a 2 10−12,9

Kw = 10 −6,98 ≫ 10 −9 nên có thể chuẩn độ riêng nấc 2 được. K a1

*Với pH = 9,4 chuẩn độ nấc 1 hết V1 = 8 ml. * Với pH = 4 chuẩn độ nấc 2 hết V2 = 22,2 ml. V2 > 2 V1 => Dung dịch ban đầu ( dung dịch A) chứa HS- và S2-; OH- phản ứng hết. Ở pH = 4,0 =>

[HS − ] K a1 [S 2− ] K a 2 = = 10−3,02 ≪ 1 và = = 10−8,9 ≪ 1 − [H 2 S ] h [HS ] h

Nên dung dịch coi như chỉ chứa H2S => Xảy ra hoàn toàn : S2- + 2H+ ⇌ H2S

HS- + H+ ⇌ H2S.

b.Trong dung dịch A có : CS 2− =

CHS − + 2CS 2− =

8.0, 025 = 0, 02( M ) ; theo trung hòa nấc 1. 10

22, 2.0, 025 = 0, 0555( M ) ; theo trung hòa nấc 2. 10

. C HS − = 0, 0155( M ) .

0,5

Khi đó CO2 = 2( C HS − + CS 2− ) = 0,0355.2 = 0,071(M). CO1 = 2.

10( 2.CS 2− + CHS − ) 10

= 0,111( M )

c.Trong dung dịch A: CS 2− = 0,02 M; C HS − = 0,0155M. S2- + H2O ⇌ HS- + OH- (1) ; Kb1 = 10-1,1.

HS- + H2O ⇌ H2S + OH- (2) ; Kb2 = 10-6,92.. H2O ⇌ H+ + OH- (3) ; KW = 10-14.

Vì CS 2− .K b1 ≫ C HS − .K b 2 ≫ K w . => Tính [OH-] theo (1): S2- + H2O ⇌

Co

0,02

[]

0,02-x

K b1 =

HS- +

(1) ; Kb1 = 10-1,1.

0,0155 0,0155+x

0,5 x

(0,0155 + x).x = 10−1,1 => x = 0, 0328 . => [OH-] = 0,0328(M). 0, 02 − x

pOH = 1,484 => pH = 12,516.

III.1

OH-

As2O3 + 6OH- ⇌ 2 AsO33- + 3H2O .

AsO33- + 3H+ ⇌ H3AsO3 .


H3AsO3 + 2Ce4+ + H2O ⇌ H3AsO4 + 2 Ce3+ + 2H+.

III.2

nAs2O3 = 10−3 mol ⇒ 10 ml dung dịch có nAs2O3 = 10−4 mol nCe4+ = 410−4 mol ⇒ CCe4+ =

III.3

4.10−4 = 0, 05( M ) 8.10−3

0,5

Khi VCe4+ = 6ml ⇒ nCe4+ = 3.10−4 ( M ) ⇒ nH3AsO3 pu = 1,5.10−4 mol .

⇒ CH3AsO3 =

2.10−4 − 1,5.10−4 = 3,125.10−3 ( M ) −3 16.10

⇒ CH3AsO4 =

1,5.10−4 = 9,375.10−3 ( M ) 16.10−3

Edd = EH3AsO4 / H 3AsO3 = E o +

III.4

0,5

0,5

0, 0582 [H 3 AsO4 ] + 2 lg .[H ] = 0, 515(V ). 2 [H 3 AsO3 ]

Tại điểm tương đương:

H3AsO3 + 2 Ce4+ + H2O ⇌ H3AsO4 + 2Ce3+ + 2H+.

2[H3AsO4] = [Ce3+] và 2[H3AsO3] = [Ce4+] Mà Edd = EH3AsO4 / H 3AsO3 = ECe4+ / Ce3+ . 3

Edd = 2 EH3AsO4 / H3AsO3 + ECe4+ /Ce3+ = 2 E o H3AsO4 / H3AsO3 + E oCe4+ / Ce3+ +

0,0582 lg[H + ]2 2

0,5

= 2.0,56 + 1,7 + 0,0592 lg(0,1) ; => ETD = 0,9(V ). 2

IV.1

Nguyên tố với các tính chất trên là Si . Các phương trình phản ứng xảy ra được là : Si + 2KOH + H2O → K2SiO3 + 2 H2.

0,75

800 C Si + 2H2O  → SiO2 + 2H2. o

3Si 4HNO3 + 18HF → 3H2Sì6 + 4NO + 8H2O. Si + 4HFhơi → SiF4 + 2H2. 800 C → Mg2Si. Si + 2Mg  o

t cao SiO2 + 2Mg  → Si + 2MgO. o

IV.2

Gọi số mol FeS2 và FeO lần lượt là x và y.=> 120x + 72y = 2,64* Sơ đồ cho nhận e: FeS2 → Fe3+ + 2SO42- + 15 e FeO → Fe3+ + 1e . N+5 + 1e → NO2 Bảo toàn e : 15x + y = 0,17 ** Giải hệ (*) và (**) ta được x = 0,01 và y = 0,02.

0,5


%mFeS2 =

120.0, 01 = 45, 45% 2, 64

; %m FeO = 54,55%

Dung dịch X chứa Fe3+ ( 0,03 mol); H+( a mol); SO42-( 0,02 mol); NO3-( b mol). Theo 0,25 NaOH ta có : 0,03.3 + a = 0,24 => a= 0,15. Bảo toàn điện tích trong dung dịch ta có: 2.0,02 + b = 3.0,03 + 1.0,15 => b = 0,2.

0, 5

Theo bảo toàn N ta có : số mol HNO3 = n(NO3-) + n(NO2) = 0,37. C%(HNO3) =

V.1

0,37.63 = 37, 6% 62 Ag+ + e

(-) Ag

E1o = - 0,8 V => K1=10-13,5.

;

2Ag+ + CrO42-

Ag2CrO4

2Ag + CrO42-

Ks-1 =1012.

;

Ag2CrO4 + 2e ; K(-) = K12.Ks = 10-15

Eo(Ag2CrO4/Ag,CrO42-) = 0,444(v) = E(Ag2CrO4/Ag,CrO42-). (+) Ag+ + e

Ag ;

0,5

E(Ag+/Ag)= Eo(Ag+/Ag) + 0,0592lg[Ag+] = 0,741(V) S.đ.đ của pin Epin = E(+) – E(-) = 0,297 V.

V.2

0,75

Khi thêm 1 mol NH3 vào dung dịch bên phải của pin: Ag+ + 2NH3 Bđ:

0,1

[]

x

Ag(NH3)2+ ;

K2 = 107,24 coi hoàn toàn.

1 0,8

0,1

K2 = (0,1)/ (0,82.x) = 107,24.

X = 8,99.10-9 => E(Ag+/Ag)= 0,324V < 0,444V. Vậy cực sẽ đổi ngược lại : Ag+ + e

(-) Ag

Ag+ + 2NH3 (+)

Ag(NH3)2+. 2Ag+ + CrO42-

Ag2CrO4 + 2 e

Phản ứng xảy ra là : Ag2CrO4 + 4 NH3

V.3

.

2Ag(NH3)2+ + CrO42-.

Epin = 0,444 - 0,324 = 0,12 (V). Thêm 1 mol KCN vào bên trái pin : 2Ag+ + CrO42-.

Ag2CrO4 Ag+ + 2CNAg2CrO4 + 4CNBđ :

2Ag(CN)2- ; kb = 1021 . 2Ag(CN)2- + CrO42- ; K=1030 coi hoàn toàn.

1

[]:

1

x 2

KS = 10-12.

0,5 4

K = (0,5) .(1,25)/ x = 10

30

1,25

=> x = 2,364.10-8 (M).

[CrO42-] = 1,25M E(Ag2CrO4/Ag,CrO42-) = 0,441 V .

0,75


Epin = 0,741 – 0,441 = 0,3(V). VI.1

Chất A và B đều là axit vì dạng cacbanion bền 0,25

+H+ A

0,25

R CO CH R' +H+

R CO CH2 R' B

Chất C là bazơ vì khi tương tác với proton tạo muối bậc 4 với liên kết H nội phân tử bền 0,25

Chất D là một bazơ mạnh dù là amin không no vì dạng cộng hưởng bền khi proton hóa 0,25

VI.2

a) Oxazol < thiazol < imidazol

0,5

Imidazol: có nhiều liên kết hidro hơn nên đs cao nhất Thiazol có phân tử khối cao hơn oxazol. 0,5 C O N

C

N(C2H5)2

C NH2

<

O

N B

C

<

O N

A

A và B có liên kết hidro liên phân tử, nhưng A có nhiệt độ nóng chảy

O-H


cao hơn dotồn tại dạng ion C

O

O

lưỡng cực

N H

VII.1

1,0

O

O O

t

0

O

Diels - Alder O O

O

O

A

O

HOOC

hv

CO OOH

1. Na2CO3 Basketen

Pb(OAc)4

2. HCl B

VII.2

0,25

0,25 0,25 0,25


VIII.1

Từ dữ kiện đề bài suy ra: -Cấu trúc các chất trung gian

0,5

VIII.2

-Từ đó suy ra cấu trúc có thể có của A

0,5

Các cấu trúc trên của A có bộ khung thoả quy tắc isopren:

Thuỷ phân chất A trong môi trường kiềm sau đó axit hoá thu được F với 3 dạng hổ


biến thì chỉ có a1 thoả điều kiện.

0,25

Vậy A là:

IX.1

R2

R1 R2

1 2

4

IX.2

IX.3

R2

1 t0

7 6

3

R1

5

2

5

7

0

6

3 4

R1

R2

1 7

t

1

6

2

t 5

3

4

0,25 0,5

R1

0

6

7 5

2 3

4

0,5

a)

0,25

b)

0,25


IX.4

OH-

CHCl3

NH

0,25

: CCl2 + Cl- + H2O

: CCl2

OH-

Cl

Cl

OH

Cl

O

OH-

N

N

NH

Cl

Cl

O

O

O : CCl2

OH-

CCl2

0,25 CHO

X.1

1,0

X.2

1,0


KỲ THI HỌC SINH GIỎI CÁC TRƯỜNG THPT CHUYÊN KHU VỰC DUYÊN HẢI VÀ ĐỒNG BẰNG BẮC BỘ LẦN THỨ X, NĂM HỌC 2016 – 2017 TRƯỜNG THPT CHUYÊN THÁI NGUYÊN – TỈNH THÁI NGUYÊN ĐỀ THI ĐỀ XUẤT MÔN: HÓA HỌC 11 Thời gian: 180 phút (Không kể thời gian giao đề) Câu 1: (2,0 điểm) Động học (Có cơ chế) – Cân bằng hóa học 1. Cho phản ứng: 2N2O5(k) → 4NO2(k) + O2(k) Giá trị tốc độ đầu của N2O5 tại 250C được cho trong bảng dưới đây: [N2O5], M 0,150 0,350 0,650 -1 -1 -4 -4 Tốc độ, mol.l .phút 3,42.10 7,98.10 1,48.10-3 a. Hãy viết biểu thức của định luật tốc độ phản ứng cho phản ứng trên và tính hằng số tốc độ phản ứng. Chỉ dẫn cách tính cụ thể. b. Tính thời gian cần để nồng độ N2O5 giảm từ 0,150M xuống còn 0,050M. c. Tốc độ đầu của phản ứng khi nồng độ N2O5 bằng 0,150M là 2,37.10-3 mol.l-1.phút-1 tại 400C. Xác định năng lượng hoạt hoá của phản ứng. d. Cho biết cơ chế của phản ứng phân huỷ N2O5 theo sơ đồ sau: k

1 N2O5  → NO2 + NO3

k'

1 → N2 O5 NO2 + NO3 

k

2 NO2 + NO3  → NO2 + NO + O2

k

3 NO + N2O5  → 3NO2

Áp dụng nguyên lí nồng độ ổn định đối với NO3 và NO, hãy thiết lập biểu thức của tốc độ

d[N 2 O5 ] . dt

 → N2O4 (k) ở 250C 2. Cho cân bằng sau trong bình kín: 2NO2 (k) ←  (màu nâu đỏ) (không màu) a) Khi giảm nhiệt độ thì tỉ khối của hỗn hợp khí so với H2 tăng lên. Vậy khi hạ nhiệt độ của bình thì màu nâu đỏ đậm lên hay nhạt đi? Phản ứng thuận toả nhiệt hay thu nhiệt? Tại sao? b) Cho 18,4 gam N2O4 vào bình dung tích 5,904 lít ở 270C. Lúc cân bằng, áp suất của hỗn hợp khí trong bình là 1 atm (nhiệt độ 270C).

 → 2NO2 (k) ( Cho N = 14; O = 16) Tính Kp ; KC của cân bằng N2O4 (k) ←  Câu 2: (2,0 điểm) Cân bằng trong dung dịch điện li 1. Tính pH của dung dịch NaHA 0,1 M. Biết H2A có Ka1=10-6,35; Ka2=10-10,33. 2. Cho biết TCdS = 3,6.10-29 ; TNiS = 3,0.10-21. a) Tính độ tan của CdS trong nước (bỏ qua phản ứng của các ion với nước) b) Tính độ tan của CdS trong nước (bỏ qua phản ứng của ion Cd2+ với nước nhưng có có tính đến phản ứng của ion S2- với nước. Biết Kb, S2- = 3.10-2 ; Kb, HS- = 1.10-7 )


c) Một dung dịch chứa ion Cd2+ và Ni2+ đều có nồng độ là 0,020M. Khi thêm ion sunfua thì chất nào kết tủa trước (CdS hay NiS). Ngay trước khi ion kim loại thứ hai kết tủa, nồng độ của ion kim loại kết tủa trước còn lại là bao nhiêu? Nhận xét kết quả. Câu 3: (2,0 điểm) Điện hóa học Dung dịch A gồm Fe(NO3)3 0,05 M; Pb(NO3)2 0,10 M; Zn(NO3)2 0,01 M. 1. Tính pH của dung dịch A. 2. Sục khí H2S vào dung dịch A đến bão hoà ([H2S] = 0,10 M), thu được hỗn hợp B. Những kết tủa nào tách ra từ hỗn hợp B? 3. Thiết lập sơ đồ pin bao gồm điện cực chì nhúng trong hỗn hợp B và điện cực platin nhúng trong dung dịch CH3COONH4 1 M được bão hoà bởi khí hiđro nguyên chất ở áp suất 1,03 atm. Viết phản ứng xảy ra trên từng điện cực và phản ứng trong pin khi pin làm việc.

Cho: Fe3+ + H2O ⇌ FeOH2+ +

H+

lg*β1 = -2,17

Pb2+ + H2O ⇌ PbOH+

H+

lg*β2 = -7,80

+

lg*β3 = -8,96

Zn

2+

+ H2O ⇌ ZnOH

+

+

+ H

RT ln = 0,0592lg F pKS(PbS) = 26,6; pKS(ZnS) = 21,6; pKS(FeS) = 17,2. (pKS = - lgKS, với KS là tích số tan). E0

Fe3+ /Fe2+

0 = 0,771 V; ES/H = 0,141 V; E 0 2S

Pb2+ /Pb

pK a1(H 2S) = 7,02; pK a2(H2S) = 12,90; pK

a(NH + 4)

= -0,126 V ; ở 25 oC: 2,303

= 9,24; pK a(CH3COOH) = 4,76

Câu 4: (2,0 điểm) N - P, C – Si và hợp chất Một nguyên tố X, nguyên tử có ba lớp electron (K, L, M) có các giá trị năng lượng ion hoá I (tính theo kJ/mol) như sau: I1 I2 I3 I4 I5 I6 1012 1903 2910 4956 6278 22230 1. Viết cấu hình electron của X. 2. Xác định công thức phân tử , viết tên gọi các chất trong sơ đồ và viết PTHH thực hiện mỗi chuyển hóa theo sơ đồ sau:

(1)

(2)

(4)

(E) (3) (5) (10) (12) (14) (H) (6) (7) (9) (F) (G) (11) (13) (8) (15) (C) X (A)

(B)

(D)

Biết: Trong các hợp chất A, B, C, D, E, F, G, H thì X có mức oxi hóa cao nhất. B, D, E tạo dung dịch làm đỏ quỳ tím. Dung dịch F, G phản ứng với axit mạnh và bazơ mạnh. Các chất F, G, H khi đốt cho ngọn lửa màu tím. 3. Cho 0,1mol mỗi axit H3XO2 và H3XO3 tác dụng với dung dịch KOH dư thì thu được hai muối có khối lượng lần lượt là 10,408g và 15,816g. Xác định công thức cấu tạo và gọi tên hai phân tử axit trên. Câu 5: (2,0 điểm) Phức chất, trắc quang 1. a) Thêm CN− vào dung dịch Ni2+, đầu tiên xuất hiện kết tủa màu xanh, khi CN− dư thì kết tủa tan, tạo thành dung dịch trong suốt màu vàng, sau đó khi CN− rất dư thì dung dịch chuyển sang màu đỏ. Hãy giải thích các hiện tượng trên.


b) Nếu cho dung dịch màu vàng ở trên phản ứng với Na trong NH3 lỏng thì tạo thành một sản phẩm A có màu đỏ, nghịch từ, không bền trong không khí. Hãy cho biết A là hợp chất nào? 2. Chiếu một chùm tia đơn sắc (có bước sóng λ xác định) qua dung dịch mẫu chất nghiên cứu thì cường độ  I  của tia sáng tới Io giảm đi chỉ còn là I. Tỉ số T =   được gọi là độ truyền qua. T phụ thuộc vào nồng độ  I 0 λ

mol C (mol·L-1) của chất hấp thụ ánh sáng trong dung dịch, chiều dày lớp dung dịch l (cm) và hệ số hấp thụ mol ε (L·mol-1·cm-1) đặc trưng cho bản chất của chất hấp thụ (định luật Lambert-Beer): - lg T = εlC Để xác định giá trị Ka của một axit hữu cơ yếu HA, người ta đo độ truyền qua của một chùm tia đơn sắc (tại bước sóng λ xác định) với dung dịch axit HA 0,05 M đựng trong thiết bị đo với chiều dày lớp dung dịch l = 1 cm. Kết quả cho thấy 70% tia sáng tới bị hấp thụ. Giả thiết, chỉ có anion A- hấp thụ tia đơn sắc tại bước sóng này và hệ số hấp thụ mol ε của A- là 600 L·mol-1·cm-1. Tính giá trị Ka của HA trong điều kiện thí nghiệm. Câu 6 (2,0 điểm): Quan hệ giữa hiệu ứng cấu trúc và tính chất 1. Cho các dữ liệu pKa của một số axit cacboxylic như sau: Axit pKa1 pKa2 Axit oxalic 1,27 4,27 Axit malonic 2,86 5,70 Axit sucxinic 4,21 5,64 Axit glutaric 4,34 5,27 a) Tại sao các axit này mạnh hơn so với các axit monoankylcacboxylic tương ứng? b) Giải thích chiều biến đổi pKa1; pKa2 khi số nguyên tử cacbon tăng. 2. Sắp xếp (có giải thích) các chất trong mỗi dãy sau trình tự tăng dần tính bazơ: Dãy 1: NH2

NH2

A

B

NH

C

Dãy 2: H N

H N

H N H3C

O

D Dãy 3:

E

N H

F

NH

G

CH3


CH3 N N H3C

N CH3

H

J

N

K

Dãy 4:

N

N N

J

M

N

Câu 7 (2,0 điểm): Hidrocacbon 1. Người ta tách được từ dầu mỏ một hợp chất A. Khi cho A tác dụng với brom có chiếu sáng, thu được hợp chất B chứa 55,8%C và 7,01%H, còn lại là brom chỉ chiếm một nguyên tử trong phân tử. Cả A và B đều là những hợp chất bền, không làm mất màu dung dịch KMnO4 loãng và đều không quang hoạt. a) Xác định công thức phân tử của A và B. b) Viết công thức cấu tạo và công thức lập thể của A và B. c) Dự đoán trạng thái của A (lỏng hay rắn). Giải thích. d) Dự đoán khả năng phản ứng của B với dung dịch kiềm/ nước. Giải thích. 2. Hiđro hóa hiđrocacbon A (C10H14) thu được hiđrocacbon C10H18. Ozon hóa A rồi chế hóa tiếp với Zn/CH3COOH thu được O=CH[CH2]3CO-CO[CH2]3CH=O. a) Viết hai công thức cấu tạo có thể có của A. b) Hiđrocacbon A phản ứng với anhiđrit maleic cho sản phẩm cộng – đóng vòng Đinxơ – Anđơ B. Viết công thức cấu tạo chính xác của A và B. c) Viết công thức cấu tạo của các sản phẩm cộng và cơ chế phản ứng xảy ra khi cho A phản ứng với HCl theo tỉ lệ mol 1 :1. Câu 8 (2,0 điểm): Xác định cấu trúc, đồng phân lập thể, danh pháp 1. Anetol có phân tử khối là 148,2 và hàm lượng các nguyên tố: 81,04% C; 8,16% H; 10,8% O. Hãy: a. Xác định công thức phân tử của anetol. b. Viết công thức cấu trúc của anetol dựa vào các thông tin sau: - Anetol làm mất màu nước brom; - Anetol có hai đồng phân hình học; - Sự oxi hóa anetol tạo ra axit metoxibenzoic (M) và sự nitro hóa M chỉ cho duy nhất axit metoxinitrobenzoic. c. Viết phương trình của các phản ứng: (1) anetol với brom trong nước; (2) oxi hóa anetol thành axit metoxibenzoic; (3) nitro hóa M thành axit metoxinitrobenzoic. Viết tên của anetol và tất cả các sản phẩm hữu cơ nêu trên theo danh pháp IUPAC. d. Vẽ cấu trúc hai đồng phân hình học của anetol.


2. Viết công thức cấu tạo của các chất A và B trong sơ đồ điều chế nhựa melamin sau:

NH2 Xianogen clorua

A

NH3

N H2N

CH2O

N N

NH2

Xianuramit (melamin) Câu 9 (2,0 điểm): Cơ chế phản ứng 1. Cho sơ đồ phản ứng sau:

Hãy chỉ chỉ rõ mỗi bước phản ứng và cơ chế của mỗi bước. 2. Viết cơ chế của phản ứng sau ( phản ứng Mannich)

3. Viết cơ chế của phản ứng sau:

4. Hoàn thành các phản ứng sau và cho biết cơ chế phản ứng: O MgBr A H2O

H+

B

H2 Ni

C C 24H36

Câu 10 (2,0 điểm): Tổng hợp các hợp chất hữu cơ ( Dạng sơ đồ phản ứng). Hoàn thành dãy phản ứng chuyển hóa sau:

B


1.

H 3C H 3C

O +

O

COOH

A

COOH

2. H3C-CH=CH2 + Cl2 + CH3OH

A

o

t , -H2O

Mg,ete

B Mg,ete

ArCHO, AcOH

B C6H8O4

C D

CO2 CO2

C

, EtOH

piperi®in

D

Kh«ng ph¶n øng E

NaHCO3, to I J C6H13NO3

PCl5

F

Br2

CH3OH, HCl

G

NH3 (3 mol)

H

------ HẾT ------

KỲ THI HỌC SINH GIỎI CÁC TRƯỜNG THPT CHUYÊN KHU VỰC DUYÊN HẢI VÀ ĐỒNG BẰNG BẮC BỘ LẦN THỨ X, NĂM HỌC 2016 – 2017 TRƯỜNG THPT CHUYÊN THÁI NGUYÊN – TỈNH THÁI NGUYÊN ĐỀ THI ĐỀ XUẤT MÔN: HÓA HỌC 11 Thời gian: 180 phút (Không kể thời gian giao đề) Câu 1: (2,0 điểm) Động học (Có cơ chế) – Cân bằng hóa học 1. Cho phản ứng: 2N2O5(k) → 4NO2(k) + O2(k) Giá trị tốc độ đầu của N2O5 tại 250C được cho trong bảng dưới đây: [N2O5], M 0,150 0,350 0,650 -1 -1 -4 -4 Tốc độ, mol.l .phút 3,42.10 7,98.10 1,48.10-3 a. Hãy viết biểu thức của định luật tốc độ phản ứng cho phản ứng trên và tính hằng số tốc độ phản ứng. Chỉ dẫn cách tính cụ thể. b. Tính thời gian cần để nồng độ N2O5 giảm từ 0,150M xuống còn 0,050M. c. Tốc độ đầu của phản ứng khi nồng độ N2O5 bằng 0,150M là 2,37.10-3 mol.l-1.phút-1 tại 400C. Xác định năng lượng hoạt hoá của phản ứng. d. Cho biết cơ chế của phản ứng phân huỷ N2O5 theo sơ đồ sau: k

1 → NO2 + NO3 N2O5 


k'

1 NO2 + NO3  → N2 O5

k

2 NO2 + NO3  → NO2 + NO + O2

k

3 NO + N2O5  → 3NO2

Áp dụng nguyên lí nồng độ ổn định đối với NO3 và NO, hãy thiết lập biểu thức của tốc độ

d[N 2 O5 ] . dt

Hướng dẫn chấm NỘI DUNG ĐI Ể M 1. Cho phản ứng: 2N2O5(k) → 4NO2(k) + O2(k) 0,25 a. Biểu thức của định luật tốc độ phản ứng cho phản ứng trên và tính hằng số tốc độ phản ứng: v = k.[N2O5]x Dựa vào số liệu cho suy ra x = 1 hay v = k.[N2O5] Tính k của các thí nghiệm suy ra k trung bình k = 2,28.10-3 (phút-1) b. Thời gian cần để nồng độ N2O5 giảm từ 0,150M xuống còn 0,050M. 0,25 Áp dụng biểu thức của động học bậc nhất: kt = ln

[N 2O5 ]0 0,150 = ln = 2, 28.10−3.t [N 2O5 ] 0,050

t = 481 phút

c. Tốc độ đầu của phản ứng khi nồng độ N2O5 bằng 0,150M là 2,37.10-3, mol.l-1.phút-1 0,25 tại 400C. Năng lượng hoạt hoá của phản ứng. Tại 400C có k2 = 2,37.10-3 : 0,150 = 1,58.10-2 (phút-1) Áp dụng phương trình Arrhenus:

k  E ln  2  = a  k1  R

1 1  1, 58.10−2  E  1 1  = a   -  . Thay các số liệu: ln   −3   2, 28.10  8, 314  298 313   T1 T2 

⇒Ea = 1,00.105 (J/mol)

d. Thiết lập biểu thức của tốc độ phản ứng

d[N 2 O5 ] . dt

k

1 → NO2 + NO3 N2O5 

k'

1 NO2 + NO3  → N2 O5

k

2 → NO2 + NO + O2 NO2 + NO3 

k

3 → 3NO2 NO + N2O5  Áp dụng nguyên lí nồng độ ổn định đối với NO3 và NO: d[NO 3 ] = k1.[N2O5] - k1' .[NO2].[NO3] – k2.[NO2].[NO3] = 0 (1) dt d[NO] = k2.[NO2].[NO3] – k3.[NO].[N2O5] = 0 (2) dt d[N 2 O5 ] = - (k1.[N2O5] + k3.[NO].[N2O5] ) + k1' .[NO2].[NO3] dt

Từ (1) và (2) suy ra:

k1.[N2O5] = ( k1' + k2).[NO2].[NO3] k3.[NO].[N2O5] = k2.[NO2].[NO3]

0,25


k2 k = 3 [NO ] k + k 2 k1 ' 1

[NO ] =

k1k 2 k3 (k1' + k 2 )

k3 .[NO].[N2O5] k2

[NO2].[NO3] =

d[N 2 O5 ] k = - k1.[N2O5] - k3.[NO].[N2O5] + k1' . 3 .[NO].[N2O5] dt k2

= k1.[N2O5].( -1 -

k1' k2 + ) k1' + k 2 k1' + k2

 → N2O4 (k) ở 250C 2. Cho cân bằng sau trong bình kín: 2NO2 (k) ←  (không màu) (màu nâu đỏ) a) Khi giảm nhiệt độ thì tỉ khối của hỗn hợp khí so với H2 tăng lên. Vậy khi hạ nhiệt độ của bình thì màu nâu đỏ đậm lên hay nhạt đi? Phản ứng thuậ n toả nhiệt hay thu nhiệt? Tại sao? b) Cho 18,4 gam N2O4 vào bình dung tích 5,904 lít ở 270C. Lúc cân bằng, áp suất của hỗn hợp khí trong bình là 1 atm (nhiệt độ 270C).

 → 2NO2 (k) ( Cho N = 14; O = 16) Tính Kp ; KC của cân bằng N2O4 (k) ←  Hướng dẫn chấm NỘI DUNG 2. a) Khi giảm nhiệt độ thì tỉ khối của hỗn hợp khí so với H2 tăng lên, suy ra cân bằng trên chuyển dịch về phía có số mol khí ít hơn ( chiều thuận). Do đó phản ứng thuận là phản ứng toả nhiệt. Vậy khi hạ nhiệt độ của bình thì màu nâu đỏ nhạt đi.

ĐI Ể M 0,25

b) Số mol ban đầu N2O4 = 0,2

0,25

N2O4 (k) ⇌ 2 NO2 (k) [ ]

Tống số mol lúc CB = 0,2 + x =

0,2 – x

PV 1 × 5,904 = = 0,24 → x = 0,04 mol RT 0,082 × 300

số mol NO2 = 0,08 ; N2O4 = 0,16 →

PNO

2

PN O 2

→ Kp =

( PNO ) 2

PN O 2

4

2

=

2x

4

=

nNO

2

nN O 2

4

=

1 2

0,25

1 6

Ở TTCB : [NO2] = 0,01355M ; [N2O4] = 0,0271M ; KC = 6,775.10-3 hoặc KP = KC.RT

0,25

Câu 2: (2,0 điểm) Cân bằng trong dung dịch điện li 1. Tính pH của dung dịch NaHA 0,1 M. Biết H2A có Ka1=10-6,35; Ka2=10-10,33. Hướng dẫn chấm: NỘI DUNG -Mô tả 4 cân bằng trong dung dịch :

ĐI Ể M 0,25


 → Na + + HA− NaHA ←   → H + + OH − HOH ←   → H + + A2 − HA− ←   → H2 A HA− + H + ← 

K w = [H + ][OH - ] (1) [H + ][A 2- ] (2) [HA - ] [H 2 A] K a−11 = (3) [HA - ].[H + ]

Ka2 =

- Viết điều kiện proton : [H+] = [OH-] + [A2-] – [H2A] (4) Biến đổi: [H + ]= Rút ra : [H + ]=

K W K a 2 .[HA - ] [H + ].[HA - ] + − (5) [H + ] [H + ] K a1

K W + K a 2 .[HA - ] 1+K -1a1.[HA - ]

(6)

Trong đa số trường hợp HA- phân li rất yếu nên có thể coi [HA-] = C mol/l của mối ban đầu.:

[H + ]=

0,25

K W + K a 2 .C 1+K -1a1.C

0,25

(7)

Đáp số: [ H+]=4,576.10-9. pH = 8,34 (Nếu KW << Ka2.C; 1<< Ka1-1.C hay Ka1 <<C thì

1 [H + ]= K a1.K a 2 ⇒ pH = ( pK a1 + pK a 2 ) ) 2 -29 2. Cho biết TCdS = 3,6.10 ; TNiS = 3,0.10-21. a) Tính độ tan của CdS trong nước (bỏ qua phản ứng của các ion với nước) b) Tính độ tan của CdS trong nước (bỏ qua phản ứng của ion Cd2+ với nước nhưng có có tính đến phản ứng của ion S2- với nước. Biết Kb, S2- = 3.10-2 ; Kb, HS- = 1.10-7 ) c) Một dung dịch chứa ion Cd2+ và Ni2+ đều có nồng độ là 0,020M. Khi thêm ion sunfua thì chất nào kết tủa trước (CdS hay NiS). Ngay trước khi ion kim loại thứ hai kết tủa, nồng độ của ion kim loại kết tủa trước còn lại là bao nhiêu? Nhận xét kết quả. NỘI DUNG 2. a) Gọi độ tan của CdS là S (mol/l)

ĐI Ể M 0,25

 → Cd 2 + + S 2 − CdS ←  []

S 2+

S 2−

TCdS = [Cd ].[S ]=S.S=3,6.10-29 ⇔ S = 6.10−15 (mol / l ) b) Gọi độ tan của CdS là S (mol/l)

 → Cd 2+ + S 2− CdS ←   → HS − + OH − ; K 2− = [HS ].[OH ] = 3.10−2 (1) S2- + HOH ←  b,S [S2- ]

[H 2S- ].[OH - ] −  → HS − + HOH ← H S + OH ; K = = 1.10−7 (2)  2 b , HS − [HS ] Do độ tan của CdS khá nhỏ => lượng OH- do phản ứng thuỷ phân tạo ra rất nhỏ, còn nhỏ hơn lượng OH- do nước phân li ra. Do đó [OH - ] ≈ 10-7 M

0,25


Từ (1) suy ra: [HS-] / [S2-] = 3.105 => [S2-] << [HS-] => Bỏ qua [S2-] Từ (1) suy ra: [HS-] / [H2S] = 1 => [HS-] = [H2S] S = [S2-] +[HS-] + [H2S] = 0 +[HS-] + [H2S] = 2[HS-] => [HS-] = S/2 0,25

S [HS- ] S = 22- = 3.105 ⇒ [S2- ]= 2[S ] [S ] 6.105  → Cd 2 + + S 2− CdS ←  S 6.105 S TCdS = [Cd 2 + ].[S 2− ]=S. =3,6.10-29 ⇔ S = 4, 65.10−12 (mol / l ) 5 6.10 c) Chất kết tủa trước là chất có tích số ion vượt qua tích số tan trước. –Để CdS kết tủa ta phải có: [Cd2+].[S2-] > 3,6.10-29 => [S2-] > 1,8.10-27 –Để NiS kết tủa ta phải có: [Ni2+].[S2-] > 3,0.10-21 => [S2-] > 1,5.10-19 - Vậy CdS kết tủa trước. -Ở thời điểm NiS bắt đầu kết tủa thì [Ni2+].[S2-] = 3,0.10-21 => [S2-] = 1,5.10-19 [Cd2+].[S2-] = 3,6.10-29 => [Cd2+] = 2,4.10-10 < 10-6M nên có thể coi ion Cd2+ đã tách hết ra khỏi dung dịch. Như vậy có thể tách Cd2+ và Ni2+ ra khỏi dung dịch bằng cách khống chế chính xác nồng độ ion S2-. []

S

0,25

0,25

Câu 3: (2,0 điểm) Điện hóa học Dung dịch A gồm Fe(NO3)3 0,05 M; Pb(NO3)2 0,10 M; Zn(NO3)2 0,01 M. 1. Tính pH của dung dịch A. 2. Sục khí H2S vào dung dịch A đến bão hoà ([H2S] = 0,10 M), thu được hỗn hợp B. Những kết tủa nào tách ra từ hỗn hợp B? 3. Thiết lập sơ đồ pin bao gồm điện cực chì nhúng trong hỗn hợp B và điện cực platin nhúng trong dung dịch CH3COONH4 1 M được bão hoà bởi khí hiđro nguyên chất ở áp suất 1,03 atm. Viết phản ứng xảy ra trên từng điện cực và phản ứng trong pin khi pin làm việc. Cho: Fe3+ + H2O ⇌ FeOH2+ +

H+

lg*β1 = -2,17

Pb2+ + H2O ⇌ PbOH+

+

H+

lg*β2 = -7,80

Zn2+ + H2O ⇌ ZnOH+

+ H+

lg*β3 = -8,96

RT ln = 0,0592lg Fe /Fe Pb /Pb F pKS(PbS) = 26,6; pKS(ZnS) = 21,6; pKS(FeS) = 17,2. (pKS = - lgKS, với KS là tích số tan). E0

3+

2+ =

0 0,771 V; ES/H = 0,141 V; E 0 2S

pK a1(H 2S) = 7,02; pK a2(H2S) = 12,90; pK

a(NH + 4)

2+

= -0,126 V ; ở 25 oC: 2,303

= 9,24; pK a(CH3COOH) = 4,76

Hướng dẫn giải: NỘI DUNG 1.

Fe3+ + H2O

⇌ FeOH2+

+

H+

*β1 = 10-2,17

(1)

ĐI Ể M 0,25


Pb2+ + H2O Zn

2+

⇌ PbOH+

+ H2 O

⇌ ZnOH

H2 O

+

OH-

+

H+

*β2 = 10-7,80

(2)

+

+

-8,96

(3)

+

H H+

*β3 = 10

Kw = 10-14

(4)

So sánh (1) → (4): *β1. CFe3+ >> *β2. CPb2+ >> *β3. CZn 2+ >> Kw → tính pHA theo (1): Fe3+ + H2O C []

0,05 0,05 - x

⇌ FeOH2+

+

H+

*β1 = 10-2,17

(1)

0,25

x x [H ] = x = 0,0153 M → pHA = 1,82. +

0,25

0 2. Do E 0Fe3+ /Fe2+ = 0,771 V > ES/H = 0,141 V nên: 2S

1/ 2Fe3+ + H2S  → 2Fe2+ + S↓ + 2H+ ⇌

K1 = 1021,28

0,05 0,05 2+ 2/ Pb + H2S  → PbS↓ + 2H+ ⇌

K2 = 106,68

0,10 -

0,05

0,05 0,25

3/ Zn2+ + H2S ⇌ ZnS↓

+ 2H+

K3 = 101,68

4/ Fe2+

+ 2H+

K4 = 10-2,72

+ H2S ⇌ FeS↓

K3 và K4 nhỏ, do đó cần phải kiểm tra điều kiện kết tủa của ZnS và FeS: Vì môi trường axit → C'Zn 2+ = C Zn 2+ = 0,010 M; C'Fe2+ = CFe2+ = CFe3+ = 0,050 M. Đối với H2S, do Ka2 << Ka1 = 10-7,02 nhỏ → khả năng phân li của H2S trong môi trường 0,25

axit không đáng kể, do đó chấp nhận [H+] = CH+ = 0,25 M → tính CS' 2- theo cân bằng: H2 S

CS' 2- = Ka1.Ka2

S2-

+ 2H+

Ka1.Ka2 = 10-19,92

0,1 [ H 2S] -19,92 -19,72 = 10 . (0,25) 2 = 10 [H + ]2

Ta có: C'Zn 2+ . CS' 2- < KS(ZnS) → ZnS không xuất hiện;

0,25

C'Fe2+ . CS' 2- < KS(FeS) → FeS không tách ra. Như vậy trong hỗn hợp B, ngoài S, chỉ có PbS kết tủa.

0,0592 lg [Pb2+] 2 0,0592 K S(PbS) = - 0,126 + lg = - 0,33 V 2 [S2- ]

3. E PbS/Pb = E Pb2+ /Pb = E 0Pb2+ /Pb +

EPt = E + = 2H /H 2

0,0592 [H + ]2 , trong đó [H+] được tính như sau: lg 2 p H2 CH3COONH4

+

→ NH 4 + CH3COO-

0,25


1 +

NH 4

+ H+

⇌ NH3

1 Ka = 10-9,24

CH3COO- + H2 O ⇌ CH3COOH + OH-

Kb = 10-9,24 +

Do Ka = Kb và C + = C → pH = 7,00 → [H ] = 10 NH CH COO4

(5) (6) -7

3

+

(có thể tính [H ] theo điều kiện proton hoặc tổ hợp 2 cân bằng (5) và (6)) Vậy: E

+

2H /H 2

=

0,0592 [H + ]2 0,0592 10-14 lg = lg = -0,415 V < EPbS/Pb = - 0,33 V 2 pH2 2 1,03

0,25

→ điện cực chì là catot, điện cực platin là anot. Sơ đồ pin: +

(-)Pt(H2)│CH3COO- 1M; NH 4 1M ║S; PbS; H2S 1M; H+ 0,25M; Fe2+ 0,05M; Zn2+ 0,01M │Pb (+) (p = 1,03 atm) Trên catot: Trên anot :

PbS + 2H+ + 2e → Pb↓ + H2S → 2H+ + 2e H2 2x H+ + CH3COO- → CH3COOH H2 + 2CH3COO- → 2CH3COOH + 2e

0,25

Phản ứng trong pin: PbS + H2 + 2H+ + 2CH3COO- → Pb↓ + H2S + 2CH3COOH

Câu 4: (2,0 điểm) N - P, C – Si và hợp chất Một nguyên tố X, nguyên tử có ba lớp electron (K, L, M) có các giá trị năng lượng ion hoá I (tính theo kJ/mol) như sau: I1 I2 I3 I4 I5 I6 1012 1903 2910 4956 6278 22230 1. Viết cấu hình electron của X. 2. Xác định công thức phân tử , viết tên gọi các chất trong sơ đồ và viết PTHH thực hiện mỗi chuyển hóa theo sơ đồ sau:

(A) (6) X

(1) (7) (8)

(2) (B)

(10) (3)

(F)

(9) (11)

(C)

(D) (12) (13)

(4) (5) (G)

(E) (14) (H) (15)

Biết: Trong các hợp chất A, B, C, D, E, F, G, H thì X có mức oxi hóa cao nhất. B, D, E tạo dung dịch làm đỏ quỳ tím. Dung dịch F, G phản ứng với axit mạnh và bazơ mạnh. Các chất F, G, H khi đốt cho ngọn lửa màu tím. 3. Cho 0,1mol mỗi axit H3XO2 và H3XO3 tác dụng với dung dịch KOH dư thì thu được hai muối có khối lượng lần lượt là 10,408g và 15,816g. Xác định công thức cấu tạo và gọi tên hai phân tử axit trên. Hướng dẫn chấm: NỘI DUNG ĐI Ể M 1. Ta thấy có bước nhảy đột ngột về năng lượng ion hóa sau khi tất cả các e hóa trị đã bị 0,25


tách ra. Ở đây sau I5 có sự tăng đột biến, như vậy X có 5 e hóa trị, do đó thuộc nhóm VA, X có 3 lớp e nên thuộc chu kì 3. Vậy X là photpho (P). Cấu hình e của X: 1s22s22p63s23p3. 2. Lập luận xác định được kết quả như sau: 0,25

A Ca3(PO4)2 Canxi photphat

B H3PO4 Axit photphoric Axi orthophotphoric

C P2O5 Photpho(V) oxit Anhidrit photphoric

D H4P2O7 Axit điphotphoric

E HPO3 Axit metaphotphoric

F KH2PO4 Kali đihidrophotphat

G K2HPO4 Kali hidrophotphat

H K3PO4 Kali hotphat

-Các PTHH:

0,75

(1) Ca3(PO4)2 + 3H2SO4 → 3CaSO4↓ + 2H3PO4 0

t (2) 2H3PO4  → H4P2O7 + H2O

(3) H4P2O7 + H2O → 2H3PO4 0

t (4) H4P2O7  → 2HPO3 + H2O

(5) 2HPO3 + H2O → H4P2O7 0

1200 C (6) Ca3(PO4)2 + 3SiO2 + 5C  → 3CaSiO3 + 2P + 5CO

(7) P + 5HNO3 (đặc) → H3PO 4 + 5NO2 + H2O 0

t (8) 4 P + 5 O2  → 2 P2O5

(9) P2O5 + 3H2O → 2H3PO4. (10) H3PO4 + KOH → KH2PO4 + H2O (11) P2O5 + 2KOH + H2O → 2KH2PO4 (12) KH2PO4 + KOH → K2HPO4 + H2O (13) P2O5 + 4KOH → 2K2HPO4 + H2O (14) K2HPO4 + KOH → K3PO4 + H2O (15) P2O5 + 6KOH → 2K3PO4 + 3H2O 3. Từ 0,1 mol H3PO2 phản ứng với KOH tạo ra 0,1 mol muối

0,25

⇒ M muối = 10,408/ 0,1 mol = 104,08g/mol KxH3-xPO2 có M = 39,09 x + 1, 008 (3-x) + 30,97 + 32 = 104,08 M = 38,08 x + 65,994 = 104, 08 ⇒ x = 1 Công thức của muối là KH2PO2 → phân tử axit có 1 nguyên tử H có tính axit Từ 0,1 mol H3PO3 → 0,1 mol muối KyH3-y PO3 ⇒ khối lượng muối = 15,86g ⇒ M muối = 158,16g/mol 39,09 y + 1, 008 (3-y) + 30,97 + 48 = 158,16 38,08 y + 81,994 = 158, 16 ⇒ 38,08 y = 76,166 ⇒ y = 2 Công thức của muối là K2HPO3 → phân tử axit có 2 nguyên tử H axit

0,25


Các nguyên tử H axit phải liên kết với O để bị phân cực mạnh nên hai axit có công thức cấu tạo: O H3PO2 H O

H

P H

axit hypophotphorơ

0,25

O

P H O axit photphorơ H

H 3O H3PO

Câu 5: (2,0 điểm) Phức chất, trắc quang 1. a) Thêm CN− vào dung dịch Ni2+, đầu tiên xuất hiện kết tủa màu xanh, khi CN− dư thì kết tủa tan, tạo thành dung dịch trong suốt màu vàng, sau đó khi CN− rất dư thì dung dịch chuyển sang màu đỏ. Hãy giải thích các hiện tượng trên. b) Nếu cho dung dịch màu vàng ở trên phản ứng với Na trong NH3 lỏng thì tạo thành một sản phẩm A có màu đỏ, nghịch từ, không bền trong không khí. Hãy cho biết A là hợp chất nào? 2. Chiếu một chùm tia đơn sắc (có bước sóng λ xác định) qua dung dịch mẫu chất nghiên cứu thì cường độ  I  của tia sáng tới Io giảm đi chỉ còn là I. Tỉ số T =   được gọi là độ truyền qua. T phụ thuộc vào nồng độ  I 0 λ mol C (mol·L-1) của chất hấp thụ ánh sáng trong dung dịch, chiều dày lớp dung dịch l (cm) và hệ số hấp thụ mol ε (L·mol-1·cm-1) đặc trưng cho bản chất của chất hấp thụ (định luật Lambert-Beer): - lg T = εlC Để xác định giá trị Ka của một axit hữu cơ yếu HA, người ta đo độ truyền qua của một chùm tia đơn sắc (tại bước sóng λ xác định) với dung dịch axit HA 0,05 M đựng trong thiết bị đo với chiều dày lớp dung dịch l = 1 cm. Kết quả cho thấy 70% tia sáng tới bị hấp thụ. Giả thiết, chỉ có anion A- hấp thụ tia đơn sắc tại bước sóng này và hệ số hấp thụ mol ε của A- là 600 L·mol-1·cm-1. Tính giá trị Ka của HA trong điều kiện thí nghiệm. Hướng dẫn chấm NỘI DUNG ĐI Ể M 2+ − 1. a) Ni tạo thành với CN các hợp chất có thành phần khác nhau tùy theo tỉ lệ các chất 0,75 phản ứng: Ni2+ + 2CN− + xH2O → Ni(CN)2.xH2O ↓ (màu xanh) Ni(CN)2.xH2O + 2CN− → [Ni(CN)4]2− (tan, màu vàng) [Ni(CN)4]2− + CN− → [Ni(CN)5]3− (tan, màu đỏ) b) Khi cho dung dịch [Ni(CN)4]2− phản ứng với Na trong amoniac lỏng sẽ xảy ra phản 0,25 ứng khử Ni2+: 2[Ni(CN)4]2− + 2Na → [Ni2(CN)6]4− + 2Na+ + 2CN− Hoặc [Ni(CN)4]2− + 2Na → [Ni(CN)4]4− + 2Na+ [Ni2(CN)6]4− và [Ni(CN)4]4− đều nghịch từ (CN− là phối tử trường mạnh), dễ bị oxi hóa 0,25 lên Ni2+. 2. Gọi cường độ ánh sáng ban đầu là Io, cường độ ánh sáng sau khi đi qua dung dịch là I. 0,5 Theo đầu bài, cường độ ánh sáng sau khi đi qua dung dịch có giá trị: I = Io – 70%Io = 30%Io

 30% I o  Từ định luật Lambert-Beer ta có: D = 600.1. C A− = − lg   = 0,5229  Io  Từ đó, nồng độ của A- tại cân bằng là: 8,715.10-4 (M);


Xét cân bằng: HA

0,5 H

[ ] 0,05 – 8,715.10

-4

+

8,715.10

(8, 715.10 )

+ -4

A

-

Ka

8,715.10

-4

−4 2

Từ đó: K a =

(0, 05 − 8, 715.10−4 )

= 1, 55.10−5

Vậy, hằng số phân li của axit HA là Ka = 1,55.10-5. Câu 6 (2,0 điểm): Quan hệ giữa hiệu ứng cấu trúc và tính chất 1. Cho các dữ liệu pKa của một số axit cacboxylic như sau: Axit pKa1 pKa2 Axit oxalic 1,27 4,27 Axit malonic 2,86 5,70 Axit sucxinic 4,21 5,64 Axit glutaric 4,34 5,27 a) Tại sao các axit này mạnh hơn so với các axit monoankylcacboxylic tương ứng? b) Giải thích chiều biến đổi pKa1; pKa2 khi số nguyên tử cacbon tăng. 2. Sắp xếp (có giải thích) các chất trong mỗi dãy sau trình tự tăng dần tính bazơ: Dãy 1: NH2

NH2

A

B

NH

C

Dãy 2: H N

H N

H N H3C

O

D

N H

E

F

CH3

NH

G

Dãy 3: CH3 N N H3C

H

Dãy 4:

N CH3

J

N

K


N

N N

J

M

N

Hướng dẫn chấm: NỘI DUNG 1. a) Nhóm –COOH thứ hai đóng vai trò hút electron làm tăng độ mạnh tính axit của nhóm – COOH thứ nhất. b) pKa1 tăng dần (tính axit giảm dần) do ảnh hưởng hút electron của nhóm –COOH giảm khi độ dài mạch liên kết tăng. pKa2 của axit oxalic giảm bất thường là do tạo được bazơ liên hợp đặc biệt bền vững. pKa2 từ axit malonic giảm dần (tính axit tăng dần) do bazơ liên hợp tăng độ bền khi tương tác đẩy giữa 2 nhóm –COO- giảm (do khoảng cách tăng dần)

ĐIỂM 0,25

2.

0,25

0,25 0,25 0,25

0,25

0,25

,025

Câu 7 (2,0 điểm): Hidrocacbon 1. Người ta tách được từ dầu mỏ một hợp chất A. Khi cho A tác dụng với brom có chiếu sáng, thu được hợp chất B chứa 55,8%C và 7,01%H, còn lại là brom chỉ chiếm một nguyên tử trong phân tử. Cả A và B đều là những hợp chất bền, không làm mất màu dung dịch KMnO4 loãng và đều không quang hoạt. a) Xác định công thức phân tử của A và B. b) Viết công thức cấu tạo và công thức lập thể của A và B. c) Dự đoán trạng thái của A (lỏng hay rắn). Giải thích. d) Dự đoán khả năng phản ứng của B với dung dịch kiềm/ nước. Giải thích.


2. Hiđro hóa hiđrocacbon A (C10H14) thu được hiđrocacbon C10H18. Ozon hóa A rồi chế hóa tiếp với Zn/CH3COOH thu được O=CH[CH2]3CO-CO[CH2]3CH=O. a) Viết hai công thức cấu tạo có thể có của A. b) Hiđrocacbon A phản ứng với anhiđrit maleic cho sản phẩm cộng – đóng vòng Đinxơ – Anđơ B. Viết công thức cấu tạo chính xác của A và B. c) Viết công thức cấu tạo của các sản phẩm cộng và cơ chế phản ứng xảy ra khi cho A phản ứng với HCl theo tỉ lệ mol 1 :1. Hướng dẫn chấm: NỘI DUNG ĐIỂ M 1. a) Từ %m tìm được CTPT của A là C10H16, B là C10H16Br. 0,5 b) Công thức cấu tạo và công thức lập thể : 0,25

c) A là chất rắn, có nhiệt độ nóng chảy cao, vì có cấu trúc đối xứng cao. 0,25 d) B không phản ứng với kiềm nước, dù SN1 ( vì cacbocation không bền và không phẳng) hay là 0,25 SN2 (không thể xảy ra vì sự án ngữ không gian). 2. a) A có 2 liên kết C=C, hai vòng (vì sản phẩm thủy phân ozonit vẫn giữ nguyên số nguyên tử 0,25 cacbon). A có thể là :

b) A2 là công thức chính xác của A, vì chỉ A2 mới có khả năng chuyển thành cấu dạng s-cis là cấu 0,25 dạng đien tham gia phản ứng cộng đóng vòng Đinxơ – Anđơ :

0,25

Câu 8 (2,0 điểm): Xác định cấu trúc, đồng phân lập thể, danh pháp 1. Anetol có phân tử khối là 148,2 và hàm lượng các nguyên tố: 81,04% C; 8,16% H; 10,8% O. Hãy: a. Xác định công thức phân tử của anetol. b. Viết công thức cấu trúc của anetol dựa vào các thông tin sau:


- Anetol làm mất màu nước brom; - Anetol có hai đồng phân hình học; - Sự oxi hóa anetol tạo ra axit metoxibenzoic (M) và sự nitro hóa M chỉ cho duy nhất axit metoxinitrobenzoic. c. Viết phương trình của các phản ứng: (1) anetol với brom trong nước; (2) oxi hóa anetol thành axit metoxibenzoic; (3) nitro hóa M thành axit metoxinitrobenzoic. Viết tên của anetol và tất cả các sản phẩm hữu cơ nêu trên theo danh pháp IUPAC. d. Vẽ cấu trúc hai đồng phân hình học của anetol. 2. Viết công thức cấu tạo của các chất A và B trong sơ đồ điều chế nhựa melamin sau:

NH2 Xianogen clorua

NH3

A

N

N N

H2N

CH2O

B

NH2

Xianuramit (melamin) Hướng dẫn giải: NỘI DUNG 1. a. Xác định công thức phân tử: C = (81,04/12,00) = 6,75 ; H = (8,16/1,01) = 8,08 ; O = (10,8/16,0 = 0,675 C = 6,75/0,675 = 10 ; H = (8,08/0,675 ) = 12 ; O = 1 C10H12O b. Viết công thức cấu trúc của anetol: Anetol làm mất màu nước brôm nên có liên kết đôi; vì tồn tại ở dạng hai đồng phân hình học (liên kết đôi, π) và khi oxi hóa cho axit nên có liên kết đôi ở mạch nhánh; vì chỉ cho 1 sản phẩm sau khi nitro hóa nên nhóm metoxi ở vị trí 4 (COOH- nhóm thế loại 2, metoxi nhóm thế loại 1). Đó là axit 4-metoxi-3nitrobenzoic. Vậy anetol là:

H3 C O

0,25

CH CH CH3

c. Các phương trình phản ứng: (1) anetol với brom trong nước:

H3 C O

ĐIỂM 0,25

0,25

CH CH CH3

CH3 Br CH CH OH

Br2/H2O

CH3 Br CH CH Br

+ (2) H3CO

H3CO

(2) oxi hóa anetol thành axit metoxibenzoic:

0,25 + o

H 3C O

CH CH CH3

KMnO4/H3O , t

(3) nitro hóa M thành axit metoxinitrobenzoic:

H3CO

COOH + CH3COOH (3)

0,25


H3CO

O2N

HNO3/H2SO4

COOH

H3CO

COOH (4)

Tên của anetol và tất cả các sản phẩm hữu cơ nêu trên theo danh pháp IUPAC: (2) 2-Brom-1-(4-metoxiphenyl)-1-propanol; (3) Axit 4-metoxibenzoic; (4) Axit 4-metoxi-3-nitrobenzoic; d. Hai đồng phân hình học của anetol: H3CO

0,25

0,25

H3CO +

H CH3

H

H

CH3 H

(E) -1-metoxi-4-(1-propenyl)benzen (Z) -1-metoxi-4-(1-propenyl)benzen; hoặc (E)-1-(4-metoxiphenyl)-1-propen (Z)-1-(4-metoxiphenyl)-1-propen Cl

2. N

Cl C N Cl

N N A

NH3

Cl

H2N

N

0,25

HN CH2

H 2N N

N

HCHO

N

NH2 Melamin

H2C HN

Câu 9 (2,0 điểm): Cơ chế phản ứng 1. Cho sơ đồ phản ứng sau:

Hãy chỉ chỉ rõ mỗi bước phản ứng và cơ chế của mỗi bước. 2. Viết cơ chế của phản ứng sau ( phản ứng Mannich)

3. Viết cơ chế của phản ứng sau:

4. Hoàn thành các phản ứng sau và cho biết cơ chế phản ứng:

N N B

NH CH2

n


O MgBr A H2O

H+

B

H2 Ni

C C24H36

Hướng dẫn chấm: NỘI DUNG 1.

ĐIỂM 0,5

2.

0,5

3.

0,5


4.

0,5

Câu 10 (2,0 điểm): Tổng hợp các hợp chất hữu cơ ( Dạng sơ đồ phản ứng). Hoàn thành dãy phản ứng chuyển hóa sau: H 3C

1.

O +

H 3C

O

COOH

o

A t , -H2O B ArCHO, AcOH C6H8O4

COOH A

2. H3C-CH=CH2 + Cl2 + CH3OH

Mg,ete

C

CO2

, EtOH

C

piperi®in

D

Kh«ng ph¶n øng

CO2

B Mg,ete

D

J C6H13NO3

NaHCO3, to

PCl5

E I

Br2

F

G

NH3 (3 mol)

CH3OH, HCl

H

Hướng dẫn chấm: 1(1,0 điểm). Mỗi chất 0,25 điểm. 2(1,0 điểm). Mỗi chất 0,1 điểm. O O

1.

C

O

COOH

OH O B

A 2.

Cl

O O

O

O O

H3CO

Cl

Cl

H3CO

B

C COCl

G Br

H3CO

H3CO

MgCl

H3CO

D

CONH2 H NH3+Br-

Ar

O

OO

O

C

O

H2C=CH-CH3 A

Ar

O

CO2MgCl

D

H3CO F

E

H3CO I

COCl

COOCH3

H3CO

NH3+Cl

H3 C

------------Hết------------

Người ra đề: Nguyễn Văn Luyện – ĐT: 0915 589 398

COOCH3 J

NH2


SỞ GD & ĐT HẢI PHÒNG ĐỀ THI HSG DUYÊN HẢI – ĐỒNG BẰNG BẮC BỘ

NĂM HỌC 2016 - 2017 MÔN: HOÁ HỌC; Khối: 11 Ngày thi:………………….. HƯỚNG DẪN CHẤM

Thời gian làm bài: 180 phút, không kể thời gian phát đề

ĐỀ GIỚI THIỆU

Câu 1. (2 điểm) Tốc độ phản ứng – Cân bằng hóa học Thực nghiệm cho biết sự nhiệt phân ở pha khí N2O5

0

t  → NO 2 + O2 (*)

là phản ứng một chiều bậc nhất. Cơ chế được thừa nhận rộng rãi của phản ứng này là N2O5

k1  →

NO 2 + NO3

(1)

NO 2 + NO3

k −1  →

N2O5

(2)

NO 2 + NO3

k2  →

NO

N2O5 + NO

k3  →

+ NO 2 + O2

3 NO 2

(3) (4).

a) Áp dụng sự gần đúng trạng thái dừng cho NO, NO3 ở cơ chế trên, hãy thiết lập biểu thức tốc độ của (*). Kết quả đó có phù hợp với thực nghiệm không? b) Giả thiết rằng năng lượng hoạt hóa của (2) bằng không, của (3) bằng 41,570 kJ.mol-1. Dựa vào đặc điểm cấu tạo phân tử khi xét cơ chế ở trên, phân tích cụ thể để đưa ra biểu thức tính k-1/ k2 và hãy cho biết trị số đó tại 350 K.

Hướng dẫn chấm: 1.a) Xét d[NO3]/dt = k1[N2O5] – k -1[NO2][NO3] – k2[NO2][NO3] ≈ 0 → [NO3] = k1[N2O5] / {(k -1 + k2)[NO2]}

Xét d[NO]/dt = k2[NO2][NO3] - k3[NO][N2O5]

(a) (b).

≈0

(c)

→ [NO] = k2[NO2][NO3] / k3[N2O5] / {(k -1 + k2)[NO2]}

(d).

Thế (b) vào (d) ta được [NO] = k1k2 / k3(k -1 + k2)

(d).


Xét d[N2O5]/dt = - k1[N2O5] + k -1[NO2][NO3] - k3[NO][N2O5]

(e).

Thế (b), (d) vào (e) và biến đổi thích hợp, ta được d[N2O5]/dt = { - k1 + (k -1 – k2)/ (k -1 + k2)}[N2O5] = k`[N2O5]

(f).

b) Trong (2) do sự va chạm giữa NO2 với NO3 nên N2O5 ≡ O2NONO2 được tái tạo, tức là có sự va chạm của 1 N với 1 O. Ta gọi đây là trường hợp 1. Trong (3) NO được tạo ra do 1 O bị tách khỏi NO2; NO2 được tạo ra từ sự tách 1O khỏi NO3. Sau đó 2 O kết hợp tạo ra O2. Ta gọi đây là trường hợp 2. Như vậy ở đây số va chạm giữa các phân tử áng chừng gấp 2 so với trường hợp 1 trên. Phương trình Areniuxơ được viết cụ thể cho mỗi phản ứng đã xét: P.ư (2): k -1 = A2e − E2 / RT (*);

P.ư (3): k2 = A3e − E3 / RT (**)

Theo lập luận trên và ý nghĩa của đại lượng A trong pt Areniuxơ đặc trưng cho số va chạm dẫn tới phản ứng, ta thấy A3 = 2A2. Ta qui ước A 2 = 1 thì A3 = 2. Theo đề bài

E2 = 0; E3 = 41,570 kJ.mol -1; T = 350.

Thay số thích hợp, ta có:

k -1/ k2 = ½ e E3 / RT = ½ e 41,578 / 8,314.10

−3

.350

≈ 8.105(lần).

Câu 2. (2 điểm) Cân bằng trong dung dịch điện li Trộn 10,00 ml dung dịch KI 0,12 M với 10,00 ml dung dịch A chứa Fe(ClO4)2 0,01 M; Fe(ClO4)3 0,14 M và Na2H2Y 0,30 M thu được dung dịch B, pH trong các dung dịch đều duy trì bằng 9,00. Tính cân bằng trong dung dịch B (ở 25oC). Bỏ qua các phức hiđroxo bậc cao và đa nhân. Cho biết: E 0Fe3+ / Fe2+ = 0, 7710(v) ;

E 0I− / 3I− = 0,5355(v) ; 3

lg β FeY− = 25,10 ;

lg β FeY2− = 14,33 ;

lg* β Fe(OH)2+ = −2,17 ;

lg* β Fe(OH)+ = −5,92 ,

pK a1(H4 Y) = 2, 00 , pK a2(H4 Y) = 2,67 , pK a3(H4 Y) = 6,16 , pK a4(H 4 Y) = 10,26 .

Hướng dẫn chấm Nồng độ ban đầu: KI 0,06 M; Fe(ClO4)2 0,005 M; Fe(ClO4)3 0,07 M và Na2H2Y 0,15 M. Vì hằng số bền của FeY2- và FeY- khá lớn, nồng độ EDTA > tổng nồng độ Fe nên có thể coi cả hai dạng Fe2+ và Fe3+ đều tồn tại chủ yếu dưới dạng phức với EDTA.

Tính hằng số cân bằng điều kiện của cân bằng : 2Fe3+ + 3I-

↔ 2Fe2+ + I3-

Trong đó có các quá trình phụ : Fe3+ + H2O Fe2+ + H2O

↔ Fe(OH)+ + H+

Fe3+ + Y4Fe2+ + Y4-

↔ Fe(OH)2+ + H+ ↔ FeY-

↔ FeY2-

Theo định luật bảo toàn nồng độ ban đầu :


(

)

(

)

CFe3+ =  Fe3+  +  Fe(OH)2+  +  FeY −  =  Fe3+  1 + *βFeOH 2+ .h −1 + βFeY −  Y 4−            1 ⇔  Fe3+  = CFe3+ .   1 + *βFeOH 2+ .h −1 + βFeY −  Y 4−   

CFe2+ =  Fe 2+  +  Fe(OH)+  +  FeY 2−  =  Fe2+  1 + *βFeOH + .h −1 + βFeY 2−  Y 4−            ⇔  Fe 2+  = CFe2+ .   1 + *β

1 .h

FeOH +

−1

+ βFeY 2−

 Y 4−   

(

)

; CY 4− = CoY 4− − CFe3+ + CFe2+ =

 Y 4−  = C 4− .α 4− = C 4− K 4 = (015 − 0, 07 − 0, 005).0, 05209 = 3,907.10−3 ; Y Y Y h+K   4 0 + 0,0592 lg E 0'Fe3+ / Fe2+ = E Fe 3+ / Fe2+

CFe3+ (1+* β FeOH+ .h −1 + β FeY2− [Y 4- ]) [Fe3+ ] 0 = + E 0,0592 lg . Fe3+ / Fe2+ [Fe2+ ] CFe2+ (1+* β FeOH2+ .h −1 + β FeY− [Y 4- ])

=

E 0'Fe3+ / Fe2+ = 0,20(v) 2(E 0 ' 3+ Fe

2Fe

3+

0,07

-

2+

0,06

0,005

+ 3I ↔ 2Fe

+ I3

0,07-2x 0,06-3x 0,005+2x

-

K ' = 10

/ Fe2+

− E 0−

0,0592

I3 / 3 I −

)

= 10 −11,33

x

[I3-] = 2,89.10-12; [I-] = 0,06; [Fe3+] = 0,07.α = 0,07.2,03.10-23

= 1,42.10-24 M;

[FeOH2+] = 6,92.10-18M; [FeY-] = 0,07M [Fe2+] = 0,005.α = 0,005.1,197.10-12 = 5,99.10-15 M; [FeOH+] = 1,808.10-11; [FeY2-] = 0,005M

Câu 3. (2 điểm) Điện hóa học Một pin điện hóa làm việc dựa trên phản ứng oxi hóa khí propan bằng khí O2 khi có mặt dung dịch KOH 5M với điện cực bằng kim loại platin. Cho biết giá trị nhiệt động của một số phân tử và ion ở trạng thái tiêu chuẩn tại 250C như sau: C3H8 (k)

O2 (k)

H2O(l)

CO32-(aq)

OH-(aq)

∆ H s0 (kJ.mol-1)

- 103,85

0

- 285,83

- 677,14

- 229,99

S0(J.K-1.mol-1)

269,91

205,14

69,91

- 56,90

- 10,75

1. Hãy viết các nửa phản ứng ở anot, ở catot và phản ứng xảy ra trong pin. 2. Hãy biểu diễn sơ đồ pin và tính sức điện động của pin ở 250C. Hướng dẫn chấm 1. Tại anot: C3H8 + 26OH- → 3 CO23 (aq) + 17H2O(l) + 20e Tại catot: O2 + 2H2O + 4e → 4OH-(aq)


C3H8 (k) + 5O2 (k) + 6OH-(aq) → 3 CO32-(aq) + 7H2O(l)

(1)

2. (-) Pt|(C3H8)|OH-|| CO32 − |(O2)|Pt (+) ∆H0(1) = [3∆ H s0 ( CO32-(aq) ) + 7∆ H s0 (H2O)] - [∆ H s0 (C3H8) + 6∆ H s0 (OH-)] = -2548,44 kJ.mol-1 0 0 0 0 -1 -1 ∆S0(1) = [3 S0( CO23 (aq) )+7 S (H2O)] - [S (C3H8)+5S (O2)+6S (OH )] = - 912,43 J.K .mol

→∆G0(1) = ∆H0(1) - T∆S0(1) = - 2276399 J.mol-1 → E0 = −

∆G 0 (−2276399) =− = 1,18(v) nF 20.96500

→ Epin = E0 +

0, 0592 0, 0592 lg[OH-]6 = 1,18 + lg56 = 1,19 (v) 20 20

Câu 4. (2 điểm) Nhóm phi kim Để xác định hàm lượng nitơ (N3-) có trong một thanh thép người ta tiến hành các thí nghiệm sau:

Thí nghiệm 1: Hòa tan 10 gam thép trong dung dịch HCl dư thu được dung dịch X. Cho dung dịch NaOH đặc dư vào X đun nóng, khí Y thoát ra sau phản ứng cho hấp thụ hoàn toàn bằng 20 ml dung dịch H2SO4 5.10-3M thu được dung dịch Z. Thí nghiệm 2: Cho lượng dư KI và KIO3 vào trong dịch Z có dư H2SO4. Iot giải phóng ra được chuẩn độ bằng dung dịch Na2S2O3 1,2.10-2M và đã dùng hết 10,28 ml.

1. Viết các phương trình phản ứng xảy ra dưới dạng ion thu gọn. 2. Xác định hàm lượng nitơ có trong thép. Hướng dẫn chấm 1. N3- + 4H+ → NH +4

(1)

NH +4 + OH- → NH3 + H2O

(2)

2NH3 + H2SO4 → (NH4)2SO4

(3)

IO 3− + 5I- + 6H+ → 3I2 + 3H2O

(4)

I2 + 2 S2 O32 − → 2I- + S4 O 62 −

(5)

2. Theo bài:

n S2 O32 − = 1,2336.10-4 mol → nI2 = 6,168.10-5 mol và nH+ = 1,2336.10-4 mol nH+ = 2nH2SO4 = 2. 10-4 mol

Theo phản ứng: → nN3- = nNH3 = nH+(p.ư) = 2.10-4 – 1,2336.10-4 = 7,664.10-5 → %N =

7,664.10−5.15 .100 = 1,073.10 −2% 10

Câu 5. (2 điểm) Phức chất


1. Với thành phần [Cr(H2O)2(NH3)2Br2]+, ion này có 5 đồng phân hình học; trong đó một đồng phân hình học lại có hai đồng phân quang học; tất cả các dạng đồng phân trên đều có cấu tạo bát diện (tám mặt) đều. Hãy viết, vẽ công thức cấu tạo của mỗi đồng phân trên.

2. Tổng hợp một hợp chất của crom. Sự phân tích nguyên tố cho thấy rằng thành phần có Cr (27,1%); C (25,2%), H(4,25%) theo khối lượng, còn lại là oxy. a) Tìm công thức thực nghiệm của hợp chất này. b) Nếu công thức thực nghiệm gồm một phân tử nước, ligand kia là gì? Mức oxy hóa của Cr là bao nhiêu?

Hướng dẫn chấm

1. a) 5

ng phân hình h c c a ph c ch t

ól :

OH2

OH2 Br

Br

NH3

OH2 OH2

Br

Cr

Cr H3N

Br

Br

Cr NH3

Br

NH3

OH2

A: trans-diamin-trans-diaquatrans-dibrom Crom(III)

B: cis-diamin-cis-diaquacis-dibrom Crom(III)

Trong 5 sau:

C: cis-diamin-trans-diaquacis-dibrom Crom(III)

Br OH2

H3 N

Cr Br

NH3 OH2

NH3 Br

NH3

OH2 Cr

OH2

H3 N

OH2

NH3

Br

D: trans-diamin-cis-diaquacis-dibrom Crom(III)

D: cis-diamin-cis-diaquatrans-dibrom Crom(III)

ng phân hình h c trên thì B có hai

ng phân quang h c có c u t o B1, B2 nh


OH2

OH2 Br

OH2

OH2

Br Cr

Cr Br

H3 N

NH3

Br NH3

NH3

B2

B1 2. a) Công thức thực nghiệm CrC H O . 4

8

5

b) Từ công thức thực nghiệm CrC H O , hợp chất là [Cr(CH COO) (H O)]. Như vậy, ligand là các nhóm 4

8

5

3

2

2

-

axetat. Do nhóm (CH COO ) có điện tích –1 nên mức oxy hóa của Cr là +2. 3

Câu 6. (2 điểm) Quan hệ cấu trúc – tính chất

Có 3 hợp chất A, B, C:

HO

HO

C

C

C

O

O

O CH3

(A)

(B)

CH3

OH

(C)

CH3

1. Hãy so sánh tính axit của A và B. Giải thích? 2. Hãy so sánh nhiệt độ sôi và độ hòa tan trong dung môi không phân cực của B và C. Giải thích? 3. Cho biết số đồng phân lập thể có thể có của A, B và C. 2

N

CH2CHNH2COOH N1

4. Cho hợp chất E

(E)

Trong số hai nguyên tử N dị vòng của E, nguyên tử nào có tính bazo mạnh hơn. Hướng dẫn chấm 1.

Tính axit (A) > (B) - (A) có hiệu ứng (-C) và (-I); (B) có hiệu ứng (-I)

2.

Nhiệt độ sôi: (C) < (B); Độ tan trong dung môi không phân cực: (C) > (B) - (C) có liên kết hiđro nội phân tử; (B) có liên kết hiđro liên phân tử

3.

- (A), (B): có hai tâm bất đối → có 4 đông phân lập thể - (C) có 4 tâm bất đối → có 16 đồng phân lập thể.


4. Nguyên tử N1 tương tự như nguyên tử nitơ của pirole (cặp e của N tham gia vào hệ liên hợp thơm) nên

không có tính bazo Nguyên tử N2 tương tự như nguyên tử nitơ của piridin (cặp e của N không tham gia vào hệ liên hợp thơm) nên có thể hiện tính bazo So sánh tính bazo: N2 > N1. Câu 7. (2 điểm) Hiđrocacbon 1. Inđen C9H8 được tách từ nhựa than đá, có phản ứng với KMnO4 và làm mất màu dung dịch Br2

trong CCl4. Tiến hành hiđro hoá có xúc tác trong điều kiện êm dịu sẽ nhận được Inđan và trong điều kiện mạnh hơn thì được bixiclo [4,3,0] nonan. Khi oxi hoá Inđen sẽ thu được axit phtalic. Viết công thức cấu trúc của Inđen, Inđan và bixiclo [4,3,0] nonan. 2. Một ankin A quang hoạt có %C = 89,5% và 10,4%H. Sau khi đã hydro hóa hoàn toàn bằng Pd/C

thu được 1 – metyl – 4 – propylxiclohexan. Khi cho A phản ứng với CH3MgBr ta không thu được sản phẩm khí. Hydro hóa A trên xúc tác Lindlar tiếp theo là ozon phân và phản ứng với KMnO4 sinh ra B, có nhóm CO. Sản phẩm B phản ứng với I2/NaOH cho ra kết tủa màu vàng. Lọc kết tủa, axit hóa dịch lọc thu được sản phẩm C quang hoạt. Xác định công thức cấu tạo A, B, C. Hướng dẫn chấm 1. - Inden có CTPT C9H8 cho thấy phân tử có độ bất bảo hòa ∆= 6. Có phản ứng với KMnO4 và làm mất

màu dung dịch Br2 trong CCl4, chứng tỏ trong phân tử Inden có chứa liên kết bội kém bền - Khi hidro hoá Inden trong điều kiện êm diệu thu được Indan (C9H10) còn trong điều kiện mạnh hơn thì được bixiclo [4,3,0] nonan. Như vậy phân tử Inden có chứa một liên kết π kém bền, 2 vòng và 3 liên kết π

bền vững hơn (vì ∆= 6) - Công thức cấu tạo của các chất: Inden: 2.

Indan:

bixiclo [4,3,0] nonan:


Câu 8. (2 điểm) Xác định cấu trúc Trong quá trình tổng hợp chất hữu cơ X (C20H21NO4), người ta clometyl hóa 1,2-đimethoxybenzen bằng fomalđehit và axit clohiđric để được chất hữu cơ Y, sau đó cho chất Y tác dụng với natri xyanua để được chất hữu cơ Z. Sản phẩm Z một phần được thủy phân thu được chất hữu cơ M, phần khác được hiđro

hóa có xúc tác niken - Raney để được chất hữu cơ N. Hai chất M và N cho ngưng tụ với nhau ở khoảng nhiệt độ 1700C đến 1800C cho amit P, chất này được đóng vòng bằng POCl3 cho chất hữu cơ Q, tiếp đó đề hiđro hóa có xúc tác niken-Raney trong đecalin ở 1800C cho chất hữu cơ X. Xác định công thức cấu tạo của X, Y, Z, M, N, P và Q.

Hướng dẫn chấm

H3CO H3OC

CH2Cl

(Y)

H3CO H3OC

CH2CN H3CO H3OC

(Z)

CH2COOH

(M) CH3O

H3CO

CH2CH2NH2 H3CO

H3OC

CH2

O NH

(N)

H3CO

(P)

OCH3


CH3O

H3CO

CH3O

OCH3

CH2

H3CO

CH2

N H3CO

(Q)

Câu 9. (2 điểm) Cơ chế

Hãy đề nghị cơ chế phản ứng tạo thành (các) sản phẩm sau: a)

b)

Hướng dẫn chấm: a)

OCH3

N H3CO

(X)


b)

Câu 10. (2 điểm) Tổng hợp hữu cơ

Cho sơ đồ chuyển hóa sau:

0

H2SO4ñ, 170 C + H2/Pd 1. BrZnCH2COOC2H5 (Z) (X) (Y) + 2. H3O (C16H26O3) (C16H24O2) (saûn phaåm chính)

+ metylvinylxeton

+ LiAlH4 1. NaH (R) 2. CH3I (C34H40O2)

(Q) (xeton)

(CrO3/piridin)

(P)

1. B2H6

+ (C6H5)3CCl/piridin

(N) 2. H2O2/OH- (C H O) 33 38

COOC2H5

(M)

a. Xác định các công thức cấu trúc của các chất X, Y, Z, M, N, P, Q, R. b. Viết thêm các công thức cấu trúc của 3 sản phẩm phụ là đồng phân của X và các đồng phân phụ

của Z có thể sinh ra cùng với Z. Hướng dẫn chấm a.

OH (X)

(Y) O HO

(N)

b.

COOC2H5

(Z)

O

COOC2H5

(M)

O

OC(C6H5)3 OC(C6H5)3 (R) (Q) (P) (Viết sản phẩm khác đúng theo sản phẩm phụ của X thì chỉ trừ điểm chất X) OC(C6H5)3

OH

OC(C5H5)3


O (X)

(X1) O

O

O

(X3)

(X2)

COOC2H5 (Z)

COOC2H5

(Z1)

(Z2)

COOC2H5

********** Hết **********

(Z3)

(Z4) COOC2H5 C2H5OOC


TRƯỜNG THPT CHUYÊN BẮC NINH

SỞ GD&ĐT BẮC NINH

ĐỀ THI OLYMPIC KHU VỰC DHBB NĂM HỌC 2016 - 2017 Môn: Hóa học – lớp 11

(Đề thi đề xuất)

(Thời gian: 180 phút – không kể thời gian giao đề)

Câu 1: Trong bối cảnh tác hại xấu của sự biến đổi khí hậu ngày càng thể hiện rõ rệt trên phạm vi toàn cầu, hoá học khí quyển ngày càng có tầm quan trọng to lớn. Một trong những phản ứng được quan tâm nhiều của lĩnh vực này là sự phân huỷ NO trong pha khí:

2NO (k)

N2 (k) + O2(k)

Phản ứng này có cơ chế được thừa nhận rộng rãi là: 2NO

k1

O + NO

k2

O2 + N

N + NO

k3

N2 + O

2O + M

k4

N2 + 2O

O2 + M

k-4 a. Hãy lập biểu thức định luật tốc độ phân huỷ NO theo các giả định: Nồng độ N đạt giá trị dừng, tốc độ giai đoạn phát triển mạch vượt trội so với tốc độ giai đoạn khơi mào (khởi đầu) và cân bằng giữa oxi nguyên tử và oxi phân tử được thiết lập. ∗ b. Biểu diễn năng lượng hoạt hoá hiệu dụng /01ạ3 01á của phản ứng chung theo năng lượng hoạt hoá của các giai đoạn thích hợp trong cơ chế trên. Câu 2: Cho từ từ bột Na2CO3 vào 100 mL dung dịch A gồm Mg(NO3)2 0,1 M và Ca(NO3)2 0,2 cho đến khi tổng nồng độ cho vào là 0,3 M, thu được dung dịch hỗn hợp B (coi thể tích dung dịch không đổi sau khi thêm Na2CO3).

a) Kết tủa nào xuất hiện trước? b) Tính nồng độ cân bằng của các ion trong dung dịch B. c) Tính số mol Na2CO3 cần cho vào 100 mL dung dịch A để có thể tách hoàn toàn Ca2+ và Mg2+ ra khỏi dung dịch, biết rằng các ion được coi là tách hoàn toàn ra khỏi dung dịch nếu nồng độ còn lại trong dung dịch ≤10-5 M (coi thể tích dung dịch là không đổi).


Cho biết: Ks(MgCO3) = 10-5; Ks(CaCO3) = 10-8,3; H2CO3 có pKa1 = 6,35; pKa2 = 10,33.

Câu 3: Pin điện hoá sau đây dựa trên phản ứng ở pha rắn và hoạt động thuận nghịch ở 1000K dưới dòng khí O2. Các ion F- khuếch tán thông qua CaF2 (r) ở 1000 K. (-) MgF2 (r), MgO (r) | CaF2 (r)| MgF2 (r) , MgAl2O4(r), Al2O3(r) (+) Các nửa phản ứng: Ở điện cực âm: MgO(r) + 2 F- MgF2 (r) + ½ O2 (k) +2e Ở điện cực dương: MgF2 (r) + Al2O3(r) +½ O2 (k) +2e MgAl2O4(r) + 2Fa. Viết phương trình Nernst cho mỗi nửa pin. Viết phương trình phản ứng tổng cộng khi pin hoạt động và phương trình Nernst của nó. Coi áp suất O2(k) là như nhau ở 2 điện cực. Nồng độ F- là bằng nhau ở 2 điện cực và được duy trì bởi dòng khuếch tán ion F- thông qua CaF2 (r). b. Tính ∆G0 của phản ứng (ở 1000 K). Biết rằng, E0 (ở 1000 K) của phản ứng là 0,1529V. c. Sức điện động chuẩn của pin trong khoảng nhiệt độ từ 900 K đến 1250 K là: E0 (V) = 0,1223 + 3,06. 10-5T. Giả thiết ∆H0, ∆S0 là hằng số, tính các giá trị này. Câu 4: Cho dư bột Cu vào 10ml dung dịch HNO3 đặc thấy thu được 740 ml khí A ở điều kiện tiêu chuẩn, làm lạnh A tới -200C thấy thu được chất lỏng màu lam. Mặt khác nếu pha loãng 10 lần HNO3 rồi thêm dư bột Pb thì thu được 560ml khí. a. Viết các phương trình phản ứng xảy ra và xác định nồng độ HNO3. b. Hãy giải thích sự tạo thành sản phẩm khi hoá lỏng khí A ở -200C, nếu nhiệt độ hoá lỏng khí lớn hơn hoặc nhỏ hơn -200C thì sản phẩm của sự hoá lỏng là gì? Giải thích?

Câu 5: Hoà tan 1,092 g A là oxit kim loại X trong dung dịch HCl thu được khí màu vàng lục và một lượng 1,890g chất B màu tím là muối Clo của X, số mol khí bằng số mol muối. Phản ứng của B với Mg trong khí quyển CO (200atm ) tạo thành 2,772 g phức chất màu vàng C (% khối lượng Mg là 5,195% và không có chứa Cl). Khi cho C tác dụng với HCl thấy tạo thành H2 và 2,628 g hợp chất D màu xanh. Nung D trong khí quyển trơ thu được kim loại X. Ở nhiệt độ thấp, hợp chất D có thể tạo thành dạng đime 1. Xác định kim loại X. 2. Xác định các chất A,B,C,D. Viết các PUHH xảy ra. 3. Hãy dùng thuyết VB để giải thích cấu trúc của C và D. Câu 6: 1. Cho các hợp chất và ion sau:

G

H

I

Chỉ rõ những hợp chất và ion thơm? Giải thích. 2. Viết công thức cấu tạo và giải thích các hiện tượng sau: a. Xiclopropenon có momen lưỡng cực rất lớn b. X có momen lưỡng cực lớn hơn Y.


c. 3,4-Đibrôm-1,2,3,4-tetraphenylxiclobut-1-en tác dụng AgBF4 sinh ra hợp chất rắn màu đỏ. d. Xicloocta-1,3,5,7-tetraen tác dụng với K tạo hợp chất rắn. 3. Cho các axitsau: F: axit fumaric S: axit squaric C: axit croconic M: axit maleic D: axit dentic

a. Giải thích: - Vì sao pKa1 của M < F nhưng pKa2 M >F - Vì sao pKa1 của S xấp xỉ M nhưng pKa2 S nhỏ hơn nhiều M b. So sánh pKa của D, S với pKa của C. c. Viết phương trình phản ứng khi cho S và D tác dụng lượng dư NaHCO3 4. Bốn hợp chất có trong thiên nhiên đều không chứa nhóm cacboxyl xong đều có tính axit là: A: axit ascobic pKa=4,2 T: axit tereic B: axit bacbituric pKa≅ 4 U: axit uric (pKa= 3,9)

Giải thích vì sao các chất trên có tính axit. Câu 7: 1. Hidrocacbon A quang hoạt có công thức phân tử C8H12. Khi thực hiện phản ứng hidro hóa A bằng xúc tác Pt, đun nóng thu được B[C8H18] không quang hoạt, nhưng nếu dùng xúc tác Lindlar lại thu được C[C8H14] quang hoạt. Cho A phản ứng Na trong NH3 lỏng thu được hợp chất D không quang hoạt có công thức phân tử C8H14. Xác định cấu trúc các chất A, B, C, D 2. Hidrocacbon A chứa 1 nguyên tử C bất đối và có những tính chất sau: - 0,345 g A làm mất màu hoàn toàn 5 ml dung dịch brom 0,5M trong CHCl3 - Oxi hóa A bằng dung dịch KMnO4/H+ cho sản phẩm hữu cơ duy nhất B. Đun nhẹ B với I2 trong dung dịch kiềm thu được muối của axit hữu cơ C và chất rắn màu vàng có thành phần nguyên tố C là 3,1 %; H= 0,3 %; còn lại là iot. Có thể điều chế C từ phản ứng của phenol với propilen có mặt H2SO4, sau đó hidro hóa bằng H2/Ni rồi oxi hóa bằng KMnO4/H+.


- Cho A cộng hợp HBr cho sản phẩm không chứa C*. Hidro hóa A thu được hidrocacbon no là D, có thể điều chế D bằng cách hidro hóa L (L là một trong những sản phẩm đime hóa đóng vòng 2+4 của isopren). Xác định cấu tạo và tên gọi A, B, C, D, L. Câu 8: Hai lacton thơm A, B có công thức phân tử [C10H10O4] đều tan trong dung dịch NaOH loãng nhưng không tan trong dung dịch NaHCO3. Cả A và B cho phản ứng màu với dung dịch FeCl3. Khi cho A phản ứng CH3I/K2CO3 tạo ra chất C[C11H12O4]. Biết C chứa ba nhóm metyl không giống nhau, trong đó có một nhóm metyl liên kết vòng thơm. Xử lý C với BCl3 để tách loại một nhóm metyl tạo ra D là một đồng phân mới của A. D có một nhóm hiđroxi tạo liên kết hiđro nội phân tử. Cho chất E (2-metyl-1,3-đihiđroxibenzen) phản ứng MeI/K2CO3 tạo F[C9H12O2], F được khử bằng Li/NH3 lỏng có mặt 2 -metylpropan-2-ol cho một đien đối xứng và không liên hợp G. Tiếp tục cho G phản ứng KNH2/NH3 lỏng tạo ra một sản phẩm H. Thực hiện phản ứng ozon phân H rồi xử lý tiếp thu được nhiều sản phẩm trong đó có metyl 2-oxopropanoat. Thực hiện phản ứng giữa H với đimetylbut-2-inđioat, đun nóng tạo K[C15H20O6], tiếp tục đun nóng K để loại eten tạo được một este thơm L. Thủy phân L trong môi trường bazo rồi axit hóa dung dịch tạo thành M[C11H12O6], đun nóng M trong chân không tạo ra N[C11H10O5]. Khử N bằng NaBH4 trong DMF(N,N-đimetylformamit) tạo C và một lacton P đồng phân, P cũng có thể thu được nhờ metyl hóa B. Hãy biện luận xác định cấu tạo các chất từ A đến P. Câu 9: Bằng cơ chế phản ứng, hãy giải thích sự hình thành các sản phẩm từ các quá trình sau:

Câu 10: a. Cadien [C15H24]được tìm thấy trong tinh dầu một số loại thực vật, đề hidro hóa cadien bằng S tạo thành cadalen [C15H18]. Hoàn thành sơ đồ tổng hợp Cadalen từ Cavone theo dãy sau:


Biết rằng B không làm mất màu dung dịch brom. b. Hoàn thành dãy chuyển hóa sau:

Vũ Thị Len (hữu cơ): 0942119129 Đỗ Thu Hiền (vô cơ): 0989261865


Sáťž GD&Ä?T BẎC NINH

HĆŻáťšNG DẪN CHẤM Ä?ᝀ THI OLYMPIC KHU Váť°C DHBB NÄ‚M HáťŒC 2016 - 2017 MĂ´n: HĂła háť?c – láť›p 11

TRĆŻáťœNG THPT CHUYĂŠN BẎC NINH

(Ä?áť thi Ä‘áť xuẼt) Câu 1. ( 2 Ä‘iáťƒm) Ă? 1 (1,25Ä‘)

N᝙i dung

Ä?iáťƒm

Giải: a. Theo Ä‘áť‹nh luáş­t táť‘c Ä‘áť™, ta cĂł:

6[89] 63

= −2; [<=] − ; [<=][=] − ; [<=][<]

(1)

0,25

Náť“ng Ä‘áť™ cᝧa N Ä‘ất trấng thĂĄi dᝍng:

= ; [<=][=] − ; [<=][<] = 0

6[8] 63

⇒ ; [<=][<] = ; [<=][=] ⇒ [<] =

> [9] >"

0.25

(2)

Tháşż (2) vĂ o (1) ta cĂł:

6[89] 63

= −2; [<=] − 2; [<=][=]

0,25

Theo Ä‘iáť u kiᝇn táť‘c Ä‘áť™ giai Ä‘oấn phĂĄt triáťƒn mấch vưᝣt tráť™i so váť›i táť‘c Ä‘áť™ giai Ä‘oấn khĆĄi mĂ o:

; [<=][=]>>2; [<=] ⇒

6[89] 63

= −2; [<=][=]

(4)

Theo Ä‘iáť u kiᝇn cân báşąng giᝯa oxi nguyĂŞn táť­ vĂ oxi phân táť­ Ä‘ưᝣc thiáşżt láş­p: ⇒ [=] = ?

;' [=] = ; ' [= ]

>% [9 ] >

(5) 0,25

Tháşż (5) vĂ o (4) ta cĂł 6[89] 63

= −2; [<=]?

>% [9 ] >

(6)

Ký hiᝇu:

; = −2; ? >% >

0,25

(7)


Phương trình động học được viết lại: @[AB]

2 (0,75 đ)

@C

= D[AB]E[BF ]

b./ Theo phương trình Arrhenius, liên hệ giữa hằng số tốc độ phản ứng và năng lượng hoạt hoá E* là:

Với phản ứng đang xét: ; = G. H

; = G. H

%I∗LMạN LMá JK

%I∗ JK

0,25 %I∗Q

và ;P = GP . H JK

∗ Trong đó /01ạ3 01á là năng lượng hoạt hoá chung, Ei là năng lượng hoạt hoá thứ i trong cơ chế trên, vậy biểu thức (7) được viết lại là:

; = G. H

%I∗LMạN LMá JK

Với G = −2. G ?

R%

∗ SLMạN LMá

TU

R

= −2. G . H

= G. H

%I∗ JK

%I∗%

%I∗ G ' . H JK H JK JK . O O . = G. H ∗ %I% %I∗% G' . H JK H JK

I∗ I∗ I∗ % JK .JK .JK

0,25

% = − TU + .TU − .TU ⇒ W∗XYạC XYá = W∗F + F W∗ [ − W∗[

S∗

S∗

S∗

Câu 2 Ý A (0,5đ)

%I∗%

0,25

Z

Nội dung

a. C’ (CO32-) = C0. α(CO32-) = C0. C’ (Mg2+) =

\]). \]

0 \]) .0 \]) \]

Điểm = 0,25

C’ (Ca2+) = 0.25

b. (0,75)

b. ^ 9" % = ^_` a + ^ a

Ks-1 >>1 nên phản ứng 1 xảy ra hoàn toàn. Thành phần giới hạn của hệ là MgCO3 và


CaCO3. Các cân bằng diễn ra: MgCO3

Mg2+ + CO32-

CaCO3

Ca2+ + CO32- KS2 = 10-8,3 (2)

KS1 = 10-5 (1)

Nhận xét: KS1>>KS2 do đó sự đóng góp của CO32- từ cân bằng (2) là không đáng kể. Vậy có các cân bằng sau: MgCO3

Mg2+ + CO32-

CO32- + H2O

OH- + HCO3-

-

Trong đó, [ CO32-] = \b c

+ ?de

-3

\b

[_` a ]

=

\b

⇒ S= 3,573. 10 .

c

KS1 = 10-5 Kb1 = 10-3,67

0,25

\b c

c

Để Ca2+ kết tủa hoàn toàn thì [ CO32-] = 10-3,3M

c.

0,25

2-

S = [HCO3 ] + [ CO3 ]

⇒ S=

0,25

0,25

Để Mg2+ kết tủa hoàn toàn thì [ CO32-] = 1M.

(0,75 )

Điều kiện để kết tủa hoàn toàn Ca2+ và Mg2+ thì nồng độ [ CO32-] = 1M.

0,25

CO3 1

2-

+ H2O

-

-

OH +

HCO3 Kb1 = 10

x

x

-3,67

M

0,25

⇒ X = 0,015M ⇒ ΣfAgF hBi = ( 0,015 + 1+0,1+0,2) .0,1 = 0,1315 mol

Câu 3: Ý 1 (1,0đ)

Nội dung

a. Các nửa phản ứng theo đầu bài MgF2 (r) + Al2O3(r) +½ O2 (k) +2e

Điểm MgAl2O4(r) + 2F-

MgO(r) + 2 FMgF2 (r) + ½ O2 (k) +2e Phản ứng tổng cộng: Al2O3(r) + MgO(r) MgAl2O4(r) Phương trình Nernst cho mỗi nửa tế bào và phương trình tổng cộng khi pin hoạt động:

0,25

0.25


/ = / =

/

+

TU

j

k^

)

lm

[j % ] )

lm / + j k^ [j % ] TU

Ký hiệu (1), (2) chỉ các nửa phản ứng ở các điện cực (+) và (-) tương ứng trong pin. Phương trình Nernst cho phản ứng tổng cộng: E = / - / = =

2 (0,5 đ)

/

)

lm / + k^ % ] j [j TU

)

lm TU / + k^ % j [j ]

x

)

0,25

lm / + k^ % ] j [j

[j % ] ) lm

TU

= / − /

Do nồng độ của F- và áp suất của O2 là như nhau cho 2 điện cực của pin, nên từ phương trình trên, ta có: E (pin) = Wn − Wn = E0 (pin) = 0,1529 V.

a. ∆G0 = -nFE0 = -2.96500. 0,1529 = -29510 (J/mol) = -29,51 (kJ/mol) b. ∆G0 = ∆H0 - T.∆S0 = -nF(0,1223 + 3,06.10-5.T) = -nF.0,1223 – nF.3,06.10-5.T = -nF.0,1223 – T.nF.3,06.10-5.

∆H0 = -nF. 0,1223 = -2.96500.0,1223 = -23604J/mol = -23,604(kJ/mol) ∆S0 = nF.3,06.10-5 = 2.96500.3,06.10-5 = 5,906 (J/mol.K)

Câu 4: Ý

Nội dung

1 (1,0đ)

a. Khi pha loãng dung dịch HNO3 rồi cho Pb vào thì khí thu được là NO. Phương trình hoá học như sau: 3 Pb + 8 HNO3 3 Pb(NO3)2 + 2 NO + 4 H2O (1) nNO (1) = 0,025 mol ⇒ n HNO3 = 0,025. 4 = 0,1 mol ⇒ C0HNO3 ban đầu = 0,1/ 0,01 = 10M

0,25

0,5 0,25 0,25

Điểm

0,25

0.25

Mặt khác số mol khí A là nA= 0,74/ 22,4 = 3,3. 10-2 mol ⇒ khi cho Cu dư vào dung dịch HNO3 đặc trên thu được hỗn hợp khí A gồm 2 khí là NO2 và NO. ⇒ Các phương trình phản ứng xảy ra khi cho các kim loại vào dung dịch HNO3 là:

Cu

0,25

+ 4 HNO3 Cu(NO3)2 + 2 NO2 + 2 H2O

3 Cu + 8 HNO3 3 Cu(NO3)2 + 2 NO + 4 H2O 3 Pb + 8 HNO3 3 Pb(NO3)2 + 2 NO + 4 H2O Khi hạ nhiệt độ hỗn hợp khí A thì thu được chất lỏng màu lam là N2O3 (vì N2O2 và N2O4 không có màu)

-20

0,25


NO + NO2

2 (1,0 Ä‘)

N2O3

b. Giải thích s᝹ tấo thà nh N2O3 khi hoå l�ng A

0,5

' ∗ CẼu hĂŹnh e cᝧa NO lĂ : [kk] Ďƒ p Ďƒâˆ— p Ďƒq Ď€# Ď€#

CẼu hÏnh e cᝧa NO2 là : [kk]

QuĂĄ trĂŹnh káşżt hᝣp cᝧa 2 phân táť­ nĂ y lĂ sáťą káşżt hᝣp cᝧa 2 gáť‘c táťą do nĂŞn xảy ra rẼt dáť… dĂ ng, áť&#x; năng lưᝣng thẼp. Náşżu nâng nhiᝇt Ä‘áť™ thĂŹ sáş˝ diáť…n ra quĂĄ trĂŹnh phân huᝡ N2O3 thĂ nh 2 khĂ­ ban Ä‘ầu. QuĂĄ trĂŹnh Ä‘ime hoĂĄ NO2 xảy ra dáť… dĂ ng hĆĄn NO, vĂŹ sáťą nháş­n e vĂ o MO Ďƒo (cĂł năng lưᝣng thẼp hĆĄn Ď€* ) sáş˝ toả nhiáť u nhiᝇt hĆĄn quĂĄ trĂŹnh nháş­n e vĂ o MO Ď€*. Do Ä‘Ăł quĂĄ trĂŹnh Ä‘ime hoĂĄ NO2 sáş˝ thuáş­n lᝣi váť mạt năng lưᝣng hĆĄn, hay quĂĄ trĂŹnh phân huᝡ N2O4 thĂ nh NO2 xảy ra áť&#x; nhiᝇt Ä‘áť™ cao hĆĄn N2O2.

0,25

0,25

áť&#x; nhiᝇt Ä‘áť™ láť›n hĆĄn -200C sáş˝ diáť…n ra sáťą Ä‘ime hoĂĄ NO2,

VĂŹ váş­y,

áť&#x; nhiᝇt Ä‘áť™ nháť? hĆĄn -200C sáş˝ diáť…n ra sáťą Ä‘ime hoĂĄ NO

<-20

NO + NO 2NO2 Câu 5:

N2O4

>-20

Ă? 1 (0,75Ä‘)

N2O2

N᝙i dung

Ä?iáťƒm

Gáť?i cĂ´ng thᝊc cᝧa oxit lĂ X2On PhĆ°ĆĄng trĂŹnh phản ᝊng: X2On + HCl 2 XClx Sáť‘ mol khĂ­ báşąng sáť‘ mol muáť‘i nĂŞn n-x =2 CĂł

,

s. .t.

.2 =

⇒ CĂł bảng sau

+ (n-x) Cl2 + H2O 0,25

,u

s , .#

x

1

2

X

8,263

21,37

3 51

0.5

⇒ X là Vanadi.

b (0,75 Ä‘)

b. Oxit A cᝧa X lĂ V2O5 Muáť‘i B cần tĂŹm lĂ VCl3. Sáť‘ mol cᝧa B lĂ 0,012mol VCl3 + Mg+ CO C Giả sáť­ C cĂł 1 Mg thĂŹ MC = 462 Ä‘vC

0,25


⇒ ⇒ ⇒ ⇒

nC = 2,772/ 462 = 0,06 (mol) = ½ . nB C có 2 Vanadi C tạo bởi 12 CO Công thức phân tử của C là Mg[V(CO)6]2

0,25

Cho C tác dung với HCl thì thu được D +H2 mà mD= 2,628g ⇒ D là V(CO)6 0,25

Phương trình phản ứng xảy ra với D là: V(CO)6 V + 06 CO

t

c (0,5 đ)

t0

2V(CO)6 V2(CO)12 c. Dùng thuyết VB để giải thích cấu trúc của C và D D là V(CO)6 V có cấu hình e 3d34s2, sự phân bố e vào AO dưới tác dụng của trường phối tử CO như sau: 3d5 ⇒ V lai hoá trong d2sp3 0,25

0,25 3d5 4s 4p 6 cặp e nhận từ 6 CO V trong D còn 1 e độc thân nên D dễ đime hoá để tạo thành V2(CO)12. Hay trong D mới có 17 e hoá trị, dễ đime hoá để đạt cấu trúc 18 e bền vững. C là Mg[V(CO)6]2

Trong C thì V có số oxi hoá -1, có cấu hình e của V-1 là d6, nên V-1 lai hoá trong d2sp3 dưới tác dụng của trường phối tử CO, như sau:

3d6

4s

4p

6 cặp e nhận từ 6 CO

Câu 6: Ý 1 (0,5đ)

Nội dung Theo Huckel, hợp chất thơm có cấu trúc vòng phẳng, liên hợp kín và số eπ,n tham gia liên hợp là 4k+2.Vậy nên: A: không thơm(ko có liên hợp kín) E: thơm(k=2) B: không thơm(không có k thỏa mãn) F: không thơm(vòng không phẳng, 2H gần nhau nên đẩy nhau) C: Không thơm (không liên hợp kín) D: thơm (k=0) G, H, I là những hệ dung hợp nên mặc dù có 1 số đặc điểm không thõa mãn Huckel nhưng tính thơm lại được xét do sự tồn tại dạng ion thơm dung hợp G: thơm(k=2), dường như do 2 ion thơm dung hợp với nhau:

Điểm

0,25

0.25


H, I không có những đặc điểm trên nên không thơm. 2. (0,5)

a. Xiclopropennon có momen lưỡng cực rất lớn do phân tử rât phân cực , ở dạng ion lưỡng cực vòng 3 cạnh có tính thơm nên bền:

0,125

b. X có momen lưỡng cực lớn hơn vì dạng ion lưỡng cực là dạng bền do cả 2 ion tạo nên X có tính thơm (theo Huckel)

0,125

0,125

Vòng 4 cạnh có tính thơm, hệ liên hợp mạnh nên

0,125

có màu đỏ.

Vòng 8 cạnh có tính thơm 3. (0,5 đ)

a. pKa1 : M<F nấc 1 vì liên kết H nội phân tử làm cho M dễ phân li ra thành monoanion bề n

pKa2 : M > F vì đianion của M kém bền( 2 điện tích âm cạnh nhau đẩy nhau mạnh) do đó 0,25 sự phân li của monoanion thành đianion trở nên khó khăn. 0,25

- pKa1 của S có giá trị nhỏ tương đương và nhỏ hơn pKa1 của M vì có sự liên hợp làm tăng độ bền của monoanion S, còn pKa2 của S nhỏ hơn nhiều pKa2 của M vì sự liên hợp ở đianion của S làm cho nó trở thành 1 ion thơm bền vững 0,125


0,125

b. Nhờ có 3 nhóm C=O hút e mạnh, độ bền anion tạo càng lớn nên C có pKa1, pKa2 nhỏ nhất trong số 3 axit trên.( pKa1 của C=0,8 và pKa2=2,2) c.

4. (0,5)

Bốn hợp chất A, B, T, U không chứa nhóm cacboxyl song vẫn biểu hiện tính axit vì có sự liên hợp tương tự nhóm cacboxyl là

0,25

0,25

Câu 7: Ý

Nội dung

1 (0,75đ)

Me

Me

Me

Me

B

hoac

A

Me Me

C Me

Me

hoac

0,25 cho mỗi chất

Me Me

Me

Me

2 (1,25 đ)

Điểm

Me

Me Me

Me

D

Me

Me

* Công thức phân tử A: A có t nối đôi+ vòng 3 cạnh nên MA= Khối lượng mol A ứng với mỗi nối đôi hoặc vòng 3 cạnh là: 0,25


A có t nối đôi+ vòng 3 cạnh nên MA= 138t. Giá trị này tương ứng với công thức (C10H18)t. Vì A là đime của isopren cho cùng 1 hidrocacbon D khi bị hidro hóa, nên t=1 và CTPT A : C10H18. * Cấu tạo C: OH

OH

OH propen

H2/Ni

KMnO4

H+

t0

H+

Phenol

0,25 0,375

COOH COOH

C

* Cấu tạo A, B: C có nhánh isopropyl; C là sản phẩm oxi hóa cắt mạch bởi KMnO4, phản ứng idofom nên A là hidroacacbon mạch vòng chứa 1 nối đôi, 2 mạch nhánh Me, CH(Me)2. Để cho A tạo B, C có 2 giả thiết cấu tạo A là A1, A2:

0,375

Rõ ràng là A có cấu tạo A1, B có cấu tạo B1 * Cấu tạo D D là sản phẩm hidro hóa A, cũng là sản phẩm đi me hóa 2+4 của isopren:

Vậy cấu tạo và tên gọi A,L, D, B, C:

Câu 8:


Ý (2 đ)

Nội dung -A, B [C10H10O4] có π+v=6 là một lacton vòng thơm, tan trong NaOH, không tác dụng NaHCO3, phản ứng màu FeCl3 nên có -OH phenol -A phản ứng MeI/K2CO3 tạo C có C11H12O4, vậy C đã thêm một nhóm metyl nên -OH phenol chuyển thành -OMe. C có ba nhóm metyl không giống nhau, có một nhóm liên kết trực tiếp vòng benzen. Phản ứng ozon phân H thu nhiều sản phẩm trong đó có metyl-2-oxo-propanoat nên H là

Điểm

0,25

0.5 Xét phản ứng E:

C loại một nhóm metyl tạo D, D có một nhóm hidroxi tạo liên kết hidro nội nên C có nhóm cacbonyl cạnh nhóm -OMe. Do đó C là (II), P là (I), từ đó xác định cấu tạo B, D

Metyl hóa B được P nên có 2 cấu tạo B thỏa mãn là B1,B2

A là đồng phân của D nên A khác D ở vị trí nhóm hiđroxi và metoxi, cấu tạo phù hợp A:


Câu 9: Ý

Nội dung

Điểm

a. (0,5đ) 0,25

0.25

b. (0,5 đ)

0,25

0,25

c (0,5 đ) 0,5

d. (0,5)

0,5

Câu 10: Ý

Nội dung

Điểm


a. (1,0đ) 0,1 cho mỗi chất

b (1,0 đ)

SỞ GD&ĐT QUẢNG NAM TRƯỜNG THPT CHUYÊN NGUYỄN BỈNH KHIÊM

0,1 cho mỗi chất

ĐỀ THI CHỌN HSG VÙNG DUYÊN HẢI BẮC BỘ 2017 MÔN: HOÁ HỌC LỚP 11 Thời gian làm bài 180 phút

HƯỚNG DẪN CHẤM GV ra đề: Hồ Ngọc Quốc SĐT: 0917.18.09.81 Bài 1: (2 điểm) Động học (có cơ chế) - Cân bằng hóa học


Phương trình phân hủy ozon: 2O3 3O2. k1 O3 O + O2 (1) k-1

k2

O + O3

2O2 (2)

Với k1, k-1 và k2 là các hằng số tốc độ. 1.1 Dựa vào cơ chế trên viết phương trình tốc độ hình thành (hay tốc độ tiêu thụ) O3, O2 và O ở thời điểm t ở dạng vi phân, giả sử bước 2 của cơ chế là không thuận nghịch. 1.2 Ta có thể nhận được phương trình động học có dạng đơn giản hơn bằng cách lập các tính chất thích hợp. Giả sử rằng tốc độ của O tiến đến cân bằng rất nhanh nên nồng độ của nó có thể gán cho giá trị là hằng số của phản ứng (1). Bước thứ hai là bước xác định tốc dộ phản ứng. Dưới điều kiện gần đúng của cân bằng đã thiết lập trên hãy viết phương trình tốc độ tiêu thụ O3 (dạng vi phân) phụ thuộc vào nồng độ O2 và O. 1.3 Một phương pháp gần đúng thông dụng hơn là giả sử tốc độ phản ứng hình thành O là một hằng số (trạng thái dừng). Ở điều kiện này thì d[O]/dt = 0. Hãy chứng minh phương trình tốc độ phản ứng là

-

2k1k2[O3]2 k-1[O2] + k2[O3]

d[O3] = dt

Một cách khác làm phân hủy ozon ở trên tầng cao của khí quyển được xúc tác bởi Freon. Khi đưa CCl2F2 (Freon-12) lên tầng cao của khí quyển thì tia tử ngoại sẽ quang phân CCl2F2 thành nguyên tử Cl theo phản ứng: hv CClF2 + Cl (3) CCl2F2 1.4 Nguyên tử Cl có thể đóng vai trò như là một chất xúc tác trong phản ứng phân hủy ozon. Giai đoạn chậm đầu tiên của phản ứng phân hủy ozon dưới tác dụng của xúc tác clo là: Cl (k) + O3 (k) ClO (k) + O2 (k). Giả sử cơ chế gồm 2 bước, hãy viết bước thứ 2 của cơ chế. 1.5 Năng lượng hoạt hóa của phản ứng phân hủy ozon dưới tác dụng của xúc tác clo là 2,1 kJ/mol trong khi không có xúc tác thì năng lượng hoạt hóa là 14 kJ/mol. Tính tỉ số k(xúc tác)/k(không xúc tác) ở 25oC. Giả sử thừa số tấn số A là như nhau đối với mỗi phản ứng. STT NỘI DUNG ĐI Ể M 1.1 0,5 - d[O3] = k1[O3] - k-1[O][O2] + k2[O3][O] dt

1.2

-

d[O2] = -k1[O3] + k-1[O][O2] - k2[O3][O] dt

-

d[O] dt = -k1[O3] + k-1[O][O2] -+ k2[O3][O]

Hằng số cân bằng K được xác định bới: [O][O2] k1 = K= [O3] k-1 [O]=

-

k1[O3] k-1[O2]

d[O3] = k2[O3][O] = dt

k1k2[O3]2 k-1[O2]

0,5


1.3

-

0,5

d[O] dt = 0

-k1[O3] + k-1[O][O2] -+ k2[O3][O] = 0 Từ đó ta có được:

1.4 1.5

d[O3] = 2k2[O3][O] = dt

2k1k2[O3]2 k-1[O2] + k2[O3]

ClO(k) + O3 (k) Cl(K) + 2O2 (k) Dựa vào phương trình k = A.e-Ea/RT Ta rút ra được kxt / kkhông xt = e(14-2,1).1000/(8,314.298) = 122

0,25 0,25

Câu 2. (2,0 điểm): Dung dịch – Chuẩn độ 2.1 Dung dịch A gồm HCOONa 0,1M và Na2SO3 có pHA = 10,4. a. Tính nồng độ của SO32− . b. Thêm 14,2 ml HCl 0,6M vào 20 ml dung dịch A được dung dịch B. Tính pHB. Cho HCOOH có pK a = 3,75; H2SO3 có pK a1 = 1,76; pK a 2 = 7, 21 STT 2.1a

NỘI DUNG 2− 3

− 3

SO + H 2 O HSO + OH K b1 = 10 −

−6,79

ĐI Ể M 0.5

(1)

HCOO + H 2O HCOOH + OH K b = 10 −10,25 (2)

HSO3− + H 2O H 2SO3 + OH − K b2 = 10−12,24 (3) H 2O H + + OH − K w = 10 −14

(4)

[HCOOH] [H + ] 10 −10,4 pHA =10,4 nên [H ] <[OH ] . Khi đó = = −3,75 < 1 [HCOO − ] Ka 10 Kb2<<Kb1 nên cân bằng (1) là chính. Từ đó tính được CSO2 − = 0, 389 ( M ) +

-

3

2.1b

Thêm HCl thì:

CH + = 0, 249(M);CSO2− = 0, 228(M);CHCOO− = 0,0585(M) 3

2− 3

+

Phản ứng: SO + H → HSO3− 0,228 0,249 (M) 0,021 0,249(M)

HCOO − + H + → HCOOH 0,0585 0,021(M) 0,0375 0,021(M) Vậy dung dịch B: HCOOH (0,021M); HCOO- 0,0375M); HSO32− (0,249M)

HCOOH H + + HCOO − K a = 10 −3,75 (1) HSO3− H + + SO32−

K a 2 =10−7,21 (2)

H 2O H + + OH −

K w = 10−14 (3)

HCOO − + H 2 O HCOOH + OH − K b = 10−10,25 (4) HSO3− + H 2O H 2SO3 + OH −

K b2 = 10 −12,24 (5)

Bỏ qua (2) và (3) so với (1), bỏ qua (5) nên tính pH dung dịch B theo (1) và

0.5


(4)

pH B = 3,75 + lg

0,0375 =4 0,021

pH = 4 suy ra [OH-] =10-10 << 10-4 <<Ca, Cb

[ H 2SO3 ] = H +  =  HSO3− 

Ka1

10 −4 << 1 => [ H 2SO3 ] <<  HSO3−  −1,76 10

Vậy, bỏ qua (5) là hợp lý.

2.2 Thêm từ từ Na2CO3 rắn vào dung dịch A cho đến pH = 4,0 thu được dung dịch B. Tính số mol Na2CO3 đã thêm vào.

Cho biết:

H3PO4 có pK1 = 2,15; pK2 = 7,21; pK3 = 12,32; CH3COOH có pK = 4,76; CO2 + H2O có pK1 = 6,35; pK2 = 10,33.

STT 2.2

Từ (1) suy ra:

[H

NỘI DUNG

]

ĐIỂM 1

−2 ,15

K1 PO 4 10 = = = 101,85 = 70,8 + −4 [H 3 PO 4 ] H 10 2

[ ]

⇒ [H2PO4-] = 70,8.[H3PO4]

[HPO ] = K = 10 [H PO ] [H ] 10 2−

Từ (2) suy ra:

4

2

− 4,0

= 10-3,21

4

⇒ [HPO42-] << [H2PO4-]

[PO ] 3−

Từ (3) suy ra:

−7 , 21

2 +

(7)

K3

[HPO ] [H ] 4

2−

=

+

=

4

⇒ [PO43-] << [HPO42-]

10 −12,32 = 10-8,32 10 − 4, 0

(8)

Từ (7) và (8) suy ra, H3PO4 ban đầu tồn tại chủ yếu ở dạng H3PO4 và -

H2PO4 . C(H3PO4) = [H3PO4] + [H2PO4-] = 0,196 (M)

⇒ [H3PO4] + 70,8.[H3PO4] = 0,196 ⇒ [H3PO4] = 0,003 (M) ⇒ [H2PO4-] = 0,196 – 0,003 = 0,193 (M)

[CH COO ] -

Từ (4) suy ra:

3

[CH 3 COOH]

=

K4

[H ] +

=

(0.25đ)

10 −4 ,76 = 10-0,76 = 0,174 − 4, 0 10

⇒ [CH3COO-] = 0,174. [CH3COOH] C(CH3COOH) = [CH3COOH] + [CH3COO-] = 0,2/2 = 0,1 (M)

⇒ [CH3COOH] + 0,174.[CH3COOH] = 0,1 ⇒ [CH3COOH] = 0,085 M


⇒ [CH3COO-] = 0,1 – 0,085 = 0,015 (M) CO2 + H2O H+ + HCO3-

K5 = 10-6,35 (9)

HCO3- H+ + CO32-

[HCO ] = −

Từ (9) suy ra:

3

[CO 2 ]

K6 = 10-10,33 (10)

K5

[H ] = 10

-6,35

+

⇒ [HCO3-] << [CO2]

/10-4 = 10-2,35

(11)

[CO ] = K = 10 Từ (10) suy ra: [HCO ] [H ] 2−

3

6 +

-10,33

/10-4 = 10-6,33

3

⇒ [CO32-] << [HCO3-]

(12)

Từ (11) và (12) suy ra : [CO2] >> [HCO3-] >> [CO32-] Do đó, ion CO32- ban đầu chủ yếu tồn tại ở dạng CO2 Số mol H+ do H3PO4 và CH3COOH nhường ra là: (0.25đ) 0,2.[H2PO4-] + 0,2.[CH3COO-] = 0,2.0,193 + 0,2.0,015 = 0,0416 (mol) CO32- + 2H+ → CO2 + H2O

⇒ n(CO32-) = ½ n(H+) = 0,0208 mol

m(Na2CO3) = 0,0208.106 = 2,2048 (gam)

Câu 3. (2,0 điểm): Điện hóa học Pin chì-axit hay còn gọi là ắc quy chì gồm anot là tấm lưới chì phủ kín bởi chì xốp, catot là tấm lưới chì phủ kín bởi PbO2 xốp. Cả 2 điện cực được nhúng trong dung dịch điện li H2SO4.

3.1. Hãy viết các quá trình xảy ra trên từng điện cực, phản ứng trong pin khi pin chì-axit phóng điện và biểu diễn sơ đồ của pin. Cho biết: E0 của Pb2+/Pb là -0,126V; của PbO2/Pb2+ = 1,455V; HSO4- có pKa = 2,00; PbSO4 có pKS = 7,66; ở 250C.

3.2. Tính E0 của PbSO4/Pb; PbO2/PbSO4 và tính điện áp V của ắc quy chì, nếu CH2SO4 ≈ 1,8M. STT

NỘI DUNG

ĐIỂM

3.1 Catot:

+

PbO2 + 4 H

+ 2e

Pb HSO4-

Pb2+ + SO42-

2+

+ 2 H2O SO42- + H+ PbSO4

2.1,455 10 0,0592

10-2

0,25

107,66

Phản ứng trên catot: PbO2 + HSO4- + 3H+ + 2e ⇌ PbSO4 + 2 H2O

K1


(*)

0,25 Anot:

Pb2+ + 2e

Pb

HSO4-

−2.( −0,126) 10 0,0592

SO42- + H+

Pb2+ + SO42

PbSO4

Phản ứng trên anot: Pb + HSO4

0,25

10-2 107,66

PbSO4 + H+ + 2e

K2

(**)

Phản ứng xảy ra trong pin khi pin hoạt động: PbO2 + Pb + 2 HSO4- + 2 H+

(***)

0,25

(a) Pb│PbSO4, H+, HSO4-│PbO2 (Pb) (c)

Sơ đồ pin:

3.2

2 PbSO4 + 2 H2O

2.E 0PbO 2 /PbSO 4 0,0592

Từ (*): 10

= K1 =

2.1,455 10 0,0592

.10-2. 107,66 → E 0PbO /PbSO = 1,62 2 4

(V)

0,25 −2.E 0PbSO4 /Pb

Từ (**): 10

0,0592

−2.( −0,126) = K2 = 10 0,0592 .10-2. 107,66

→ E 0PbSO4 /Pb = - 0,29 (V) Ta có: V = E(c) – E(a) = E PbO 0 V= E PbO

2 /PbSO4

+

V = E 0PbO

2 /PbSO4

Trong đó

[HSO 4 ],

[]

- E PbSO

0,25 4 /Pb

0, 0592 0,0592 [H + ] .log([HSO -4 ].[H + ]3 ) - E 0PbSO /Pb − .log 4 2 2 [HSO-4 ]

- E 0PbSO

-

2 /PbSO4

4 /Pb

+

0,0592 .log([HSO-4 ]2 .[H + ]2 ) 2

[H ] được tính theo cân bằng:

HSO 4

H+

1,8 – x

1,8 + x

+

2-

0,25

Ka = 10-2

SO 4

x

[ SO 4 ] = x = 9,89.10-3 (M) → [H+] = 1,81 (M); [ HSO 4 ] = 1,79 (M) 2-

V = 1,62 + 0,29 +

0,25

+

-

0,0592 2

2

2

log{(1,79) .(1,81) } = 1,94 (V)


Câu 4 (2,0 điểm): Nhóm N-P; C – Si 4.1 Chất rắn màu đỏ (A) khi được nung trong môi trường trơ (không có không khí) bay hơi sau đó ngưng tụ thành chất sáp (B). (A) không phản ứng được với không khí ở nhiệt độ phòng nhưng (B) có thể tự bốc cháy tạo ra khói trắng là các hạt chất rắn (C). (C) tan trong nước tỏa nhiều nhiệt tạo dung dịch của axit 3 lần axit (D). (B) phản ứng với lượng thiếu khí clo tạo thành chất lỏng không màu dễ bốc khói (E), chất này dễ phản ứng tiếp với clo tạo chất rắn màu trắng (F). Khi hòa tan (F) vào nước thu được hỗn hợp gồm (D) và axit clohidric. Khi cho (E) vào nước, (E) tạo ra axit 2 lần axit (G) và axit clohidric. a/ Xác định công thức các chất từ A tới G và viết phản ứng xảy ra. b/ Cho biết trạng thái lai hóa của nguyên tử trung tâm và dạng hình học của phân tử F

4.2 Nung 109,6 gam Bari kim loại với một lượng vừa đủ NH4NO3 trong một bình kín, thu được hỗn hợp sản phẩm chỉ chứa 3 hợp chất của Bari (hỗn hợp A). Hòa tan hỗn hợp A trong một lượng nước dư, thu được hỗn hợp khí B và dung dịch C. a/ Giải thích và viết phương trình phản ứng xảy ra. b/ Cho khí B vào bình kín dung tích không đổi, khi áp suất ổn định (đạt tới trạng thái cân bằng) thấy áp suất tăng 10% so với áp suất ban đầu. Tính % thể tích các khí ở trạng thái cân bằng.

STT 4.1

NỘI DUNG

ĐI Ể M

a/ Phản ứng: (1) 4 P (đỏ) P4 (trắng). (A) (B) (2) P4 (trắng) + 5 O2 → P4O10. (B) (C) (3) P4O10 + 3 H2O → 2 H3PO4 (C) (D) (4) P4 + 6 Cl2 → 4 PCl3 (B) (E) (5) PCl3 + Cl2 → PCl5 (E) (F) (6) PCl5 + 4 H2O → H3PO4 + 5 HCl (F) (D) (7) PCl3 + 3 H2O → H3PO3 + 3 HCl b/ F là photpho pentaclorua, nguyên tử trung tâm P ở trạng thái lai hóa sp3d, phân tử có cấu trúc lưỡng tháp đáy tam giác.

t 0 cao 4.2 1/ 8Ba + NH4NO3  → 3BaO + Ba3N2 + 2 BaH2

0.75

0.25

0.5


BaO + H2O  → Ba(OH)2 Ba3N2 + 6H2O  → 3Ba(OH)2 + 2NH3 ↑ BaH2 + 2H2O  → Ba(OH)2 +2H2 ↑ b. Theo đầu bài n Ba =

109,6 1 =0,8mol;n NH3 =0,8. .2=0,2mol;n H 2 =0,4mol 137 8

Khi cho khí vào bình kín

2NH3 ⇌ N2 + 3H2

Trước phản ứng

0,2 mol

Phản ứng

2x

Cân bằng

0,2 – 2x

0

x

0,4 mol 0,4+3x

Theo đầu bài áp suất bình tăng 10% nên số mol khí sau phản ứng bằng 1,1 lần số mol trước phản ứng 0,2 – 2x + x + 0,4 + 3x = 1,1.0,6

0.5

 → x = 0,03 mol

Vậy ở trạng thái cân bằng thành phần số mol mỗi khí là 0,14 mol NH3 (21,21%); 0,03 mol N2 (4,55%); 0,49 mol H2 (74,24%) Bài 5: (2 điểm) Phức chất , trắc quang Phân tích nguyên tố một hợp chất của crom (A) cho thấy thành phần: Cr (27,1%); C (25,2%) và H (4,25%) theo khối lượng, còn lại là oxi. 5.1 Tìm công thức thực nghiệm của hợp chất A? 5.2 Nếu công thức thực nghiệm của A gồm một phân tử nước, thì phối tử còn lại là gì? Mức oxi hóa của Cr là bao nhiêu? 5.3 Khảo sát từ tính cho thấy hợp chất A này là nghịch từ, phải giải thích từ tính của A như thế nào? Vẽ thử cấu tạo phù hợp của chất A này? STT NỘI DUNG ĐI Ể M 5.1 Công thức thực nghiệm: CrC4H8O5 0,5 5.2 Từ công thức thực nghiệm CrC4H8O5 hợp chất 0,5 [Cr(H2O)(CH3COO)2]. Vậy phối tử còn lại là nhóm axetat. Mức oxi hóa +2 5.3 Ion Cr2+ là hệ d4, có 4 e độc thân phải thuộc loại spin cao, vì do phối Lí tử yếu [Cr(H2O)(CH3COO)2] có tính thuận từ. Tuy nhiên thực luận:0,5 nghiệm là nghịch từ, có thể giải thích là do hợp chất A này ở dạng đime hóa, cấu tạo như sau: Vẽ cấu trúc:0,5


CH3

CH3

C

C

O O H2O Cr

Liên kết

O O Cr OH2

O O

O O C

C

CH3

CH3

Bài 6: (2 điểm) Quan hệ giữa hiệu ứng cấu trúc và tính chất 6.1 (0,5 điểm) Sắp xếp các cacbocation sau theo chiều tăng dần về độ bền. Giải thích. OCH3 +

+

(A)

(B)

+

+

(C)

(D)

+

(E)

6.2 (0,75 điểm) Hãy sắp xếp các chất sau theo chiều tăng dần lực axit Bronsted. Giải thích và vẽ các công thức cộng hưởng của anion bazơ liên hợp.

6.3 (0,75 điểm) Hợp chất A và B là 2 lacton. Hãy giải thích vì sao hợp chất A khó tách proton α hơn so với B?


STT 6.1

Nội dung Thứ tự tăng dần về độ bền cacbocation: C < E < A < D < B Giải thích: 0,25đ C là cacbocation bậc 1, điện tích dương được giải tỏa bởi 1 nhóm +I E là cacbocation bậc 2, điện tích dương được giải tỏa bởi 2 nhóm +I A là cacbocation bậc 3, điện tích dương được giải tỏa bởi 3 nhóm +I D là cacbocation bậc 3, điện tích dương được giải tỏa bởi 2 nhóm +I và 1 nhóm +C1 B là cacbocation bậc 3, điện tích dương được giải tỏa bởi 2 nhóm +I và 1 nhóm +C2 (trong đó +C2 > +C1, nhờ cặp e tự do ở oxy của nhóm –OCH3). Ghi chú: HS có thể giải thích cacbocation B > D do số công thức cộng hưởng của B (3 công thức) nhiều hơn D (2 công thức).

6.2

Thứ tự lực axit tăng dần là: B < C < A < D Giải thích: 0,25đ

Điểm 0,25đ 0,25đ

0,25đ 0,25đ

Trong đó: -I1 > -I3 > -I2 Viết các công thức cộng hưởng: 0,25đ, gồm: A (3CT); B (2CT); C (2CT) và D (4CT) 0,25đ

6.3

Anion do A tạo ra có cặp electron tự do trực giao với liên kết pi của nhóm cacbonyl do 0,75đ tính chất hình học cứng nhắc của hệ thống vòng bicyclo (theo quy tắc Bredt). Do đó cặp electron tự do không thể xen phủ với liên kết pi của nhóm cacbonyl và không tạo công thức cộng hưởng. 0,25đ Trong khi đó, cặp electron tự do của anion do B tạo ra có thể xen phủ với liên kết pi của nhóm cacbonyl và tạo ra 2 công thức cộng hưởng, do vậy anion của B bền hơn anion của A nên hợp chất A dễ tách proton α hơn so với B. 0,25đ


(0,25đ)

Bài 7: (2 điểm) Hidrocacbon 7.1 (0,25 điểm) Vẽ cấu trúc hóa lập thể của các sản phẩm đồng phân tạo thành phản ứng sau. Cho biết sản phẩm nào là chính, giải thích?

1. BH3 2. H2O2

STT 7.2

NỘI DUNG

BH2

1. BH3

ĐIỂM 0,25

OH

2. H2O2

(SPC)

BH2

OH

2. H2O2

(SPP)

7.2 (0,5 điểm) Viết công thức cấu tạo của A và B trong sơ đồ chuyển hóa sau. Hãy giải thích cơ chế tạo thành B từ A.

+ CrO3

CH3COOH t0

A

OHH+

B


STT

NỘI DUNG

COOH

HO A

ĐIỂM 0.25

B O O 0.5

* Cơ chế để giải thích sự tạo thành B: O OH-

A

COOH

OH

O

O

O

O

HO

HO

COOH

COO

H+

B

7.3 (0,5 điểm) Dưới tác dụng của HCl, α-Pinen bị chuyển vị thành một hợp chất bixiclic (A) có khung carbon khác với α-pinen. Tiếp tục chuyển hóa A thành D theo sơ đồ sau. Viết công thức cấu tạo từ A-D.

HCl

A

1. H 2O/HO 2. PCC

α-Pinen

m-CPBA B

C

LiAlH4

D


STT 7.3

NỘI DUNG

+

Cl-

ch. vi

H

ĐIỂM Mỗi chất được 0,25

Cl

(A)

Bài 8: (2 điểm)

Xác định cấu trúc, đồng phân lập thể, danh pháp

8.1 (0,5 điểm) Gọi tên theo danh pháp IUPAC đối với các chất hữu cơ sau: a)

b)

OEt

c)

d)

OH

8.2 (1,0 điểm) Hãy viết cấu trúc cho các sản phẩm tạo thành từ các phản ứng sau và chỉ ra sản phẩm nào là không có trung tâm bất đối (A); là hỗn hợp raxemic (R); là đồng phân meso (M) hay là đồng phân hoạt động quang học (C)? a)

c)

1. BH 3/THF 2. H2O2/NaOH

b)

1. CH3CO3H

d)

2.

H3O+,H2O

1. CHCl3,KOH

1. Hg(OAc) 2,H2O 2. NaBH4

(H 3C)3C Et

8.3 (0,5 điểm) Hãy cho biết các sản phẩm chính của quá trình đồng phân hóa của các chất sau đây. Nêu rõ ràng cấu trúc hình học của trạng thái chuyển tiếp.


a)

STT 8.1

8.2

b)

Nội dung

Điểm 0,5đ

1. 0,125đ/1 chất gọi đúng = 0,5đ a) axit (2S),5-đimetyloct-(5E)-enoic b) (3R)-propoxy xiclopent-1-en c) (6R)-etoxy hept-(2Z)-en-4-in-2-ol d) (1R, 6S)-7-oxabicyclo[4.1.0]hept-2-en 0,25đ/1 trường hợp đúng (gồm cấu trúc + chỉ ra đúng R/M/C/A)

R

a)

b)

c)

1,0đ

M

C

d) C

8.3

0,25đ/1 trường hợp đúng

a)

0,5đ


b) Bài 9: (2 điểm) Cơ chế phản ứng 9.1 (1,5 điểm) Hãy trình bày cơ chế của các phản ứng sau:

a)

b) Ph

O

OH

Ph

Me-CHO Me

+

H

N

NHMe

c)

+ H2O

Me

9.2 (0,5 điểm) Xác định công thức cấu tạo của chất X, Y trong sơ đồ phản ứng sau và trình bày cơ chế của phản ứng từ X tạo ra Y: ClOC

+ STT 9.1

AlCl 3

X

SnCl 4

Y Nội dung

0,5đ/1 cơ chế đúng a)

Điểm 1,5đ


b) Đây là cơ chế chuyển vị Favorskii

c)

9.2

0,25đ

X là

Y là

0,25đ

0,25đ 0,25đ


Bài 10: (2 điểm) Tổng hợp các hợp chất hữu cơ (dạng sơ đồ phản ứng) 10.1 Hoàn thành sơ đồ chuyển hóa sau. Bu3SnH I

CH3

AIBN,

STT 10.1

KMnO4/H2O A

B

H+

C

lanh

NỘI DUNG

ĐI Ể M 0,125*3=0,375

OH A

O

OH

B

C

10.2 (0,5 điểm) Viết công thức cấu tạo bền của sản phẩm thu được. C2H5ONa Etyl o-amino benzoat + Đietyl malonat ? STT NỘI DUNG 10.3

O

COOEt

NH2

ĐI Ể M 0.375

C O O Et

E tO O C

NH

O

OH

N

OH

10.3 Xác định công thức cấu tạo của các chất A, B, C, D, E, F, G, H, I và K trong sơ đồ chuyển hóa sau: O

CH3O

NaNO2 NH2

E

STT

chuyen vi [3,3]

HCl, 0oC

F

H+

COOC2H5

A

G

C2H5

B

OH-

KOH

dong vong

H

- NH3

NỘI DUNG

C

I

H2 O -CH3COOH

- H+

D

- H+

E

K (C13H15NO3)

ĐI Ể M


10.3

O

CH3O

NaNO2 NH2

CH3O

CH3O

COOC2H5 C2H5

HCl, 0oC

N

A

N

O

EtOOC

KOH

N

B

N Et

B

OH-

CH3O

O

EtOOC N

N

C

CH3O

H2O

HN

H+

H

- NH3

CH3O

G

CH3O I

COOEt

E

Et

COOEt

NH

F

COOEt HN NH

HN N D

COOEt

chuyen vi [3,3]

H+ COOEt

-CH3COOH

Et

CH3O E

OH

CH3O

NH2

dong vong

CH3O

COOEt

NH2

- H+

NH

H

CH3O

NH

COOEt NH

K (C13H15NO3)

NH3

0.125 *10 =1.25


Turn static files into dynamic content formats.

Create a flipbook
Issuu converts static files into: digital portfolios, online yearbooks, online catalogs, digital photo albums and more. Sign up and create your flipbook.